Final Exam

Réussis tes devoirs et examens dès maintenant avec Quizwiz!

Tx for gallstones: ERCP

ERCP (endoscopic retrograde cholangiopancreatography) is a combination of endoscopy and x rays. Use in conjunction with sphincterotomy/papillotomy (incision into the sphincter of Oddi) to pull stones out.

biliary colic is also called

gallbladder attack, gallstone attack

Which finding is the best indicator that the fluid resuscitation for a patient with hypovolemic shock has been effective? a. Hemoglobin is within normal limits. b. Urine output is 60 mL over the last hour. c. Central venous pressure (CVP) is normal. d. Mean arterial pressure (MAP) is 72 mm Hg.

ANS: B Assessment of end organ perfusion, such as an adequate urine output, is the best indicator that fluid resuscitation has been successful. The hemoglobin level, CVP, and MAP are useful in determining the effects of fluid administration, but they are not as useful as data indicating good organ perfusion.

A 28-year-old man with von Willebrand disease is admitted to the hospital for minor knee surgery. The nurse will review the coagulation survey to check the a. platelet count. b. bleeding time. c. thrombin time. d. prothrombin time.

ANS: B The bleeding time is affected by von Willebrand disease. Platelet count, prothrombin time, and thrombin time are normal in von Willebrand disease

30. Which action will be included in the care for a patient who has recently been diagnosed with asymptomatic nonalcoholic fatty liver disease (NAFLD)? a. Teach symptoms of variceal bleeding. b. Draw blood for hepatitis serology testing. c. Discuss the need to increase caloric intake. d. Review the patient's current medication list.

ANS: D Some medications can increase the risk for NAFLD, and they should be eliminated. NAFLD is not associated with hepatitis, weight loss is usually indicated, and variceal bleeding would not be a concern in a patient with asymptomatic NAFLD.

The nurse knows the patient with AKI has entered the diuretic phase when what assessments occur (select all that apply)? A.Dehydration B.Hypokalemia C.Hypernatremia D.BUN increases E.Serum Creatinine Increases

Dehydration, Hypokalemia Dehydration, hypokalemia, and hyponatremia occur in the diuretic phase of AKI because the nephrons can excrete wastes but not concentrate urine. Therefore the serum BUN and serum creatinine levels also begin to decrease.

A 34-year old patient with chronic hepatitis C infection has several medications prescribed. Which medication requires further discussion with the health care provider before administration? a. Ribavirin (Rebetol, Copegus) 600 mg PO bid b. Pegylated -interferon (PEG-Intron, Pegasys) SQ daily c. Diphenhydramine (Benadryl) 25 mg PO every 4 hours PRN itching d. Dimenhydrinate (Dramamine) 50 mg PO every 6 hours PRN nausea

b. Pegylated -interferon (PEG-Intron, Pegasys) SQ daily Pegylated -interferon is administered weekly. The other medications are appropriate for a patient with chronic hepatitis C infection.

32. When preparing a patient for discharge following fixation of a mandibular fracture, the nurse determines that teaching has been successful when the patient says what? a. "I can keep my mouth moist by sucking on hard candy." b. "I should cut the wires with scissors if I begin to vomit." c. "I may use a bulk-forming laxative if my liquid diet causes constipation." d. "I should use a moist swab to clean my mouth every time I eat something."

32. c. The low-bulk, high-carbohydrate liquid diet and intake of air through a straw required during mandibular fixation often lead to constipation and flatus, which may be relieved with bulk-forming laxatives, prune juice, or ambulation. Wires or rubber bands should be cut only in the case of cardiac or respiratory arrest and patients should be taught to clear their mouth of vomitus or secretions. The mouth should be thoroughly cleaned with water, saline, or alkaline mouthwashes or using a Water Pik as necessary to remove food debris. Hard candy should not be held in the mouth.

When caring for a patient with metastatic cancer, the nurse notes a hemoglobin level of 8.7 g/dL and hematocrit of 26%. What associated clinical manifestations does the nurse anticipate observing? A. Thirst B. Fatigue C. Headache D. Abdominal pain

B. Fatigue The patient with a low hemoglobin and hematocrit is anemic and would be most likely to experience fatigue. Fatigue develops because of the lowered oxygen-carrying capacity that leads to reduced tissue oxygenation to carry out cellular functions. Thirst, headache, and abdominal pain are not related to anemia.

Three days after experiencing a series of tick bites, a patient presents to the emergency department. Which manifestation would indicate the patient is experiencing tick paralysis? Respiratory distress Aggression and frequent falls Decreased level of consciousness Fever and necrosis at the bite sites

Respiratory distress A classic manifestation of tick paralysis is flaccid ascending paralysis, which develops over 1 to 2 days. Without tick removal, the patient dies as respiratory muscles become paralyzed. Aggression, decreased level of consciousness, fever, and necrosis at the bite sites are not characteristic of the problem.

A nurse is caring for a patient with acute respiratory distress syndrome (ARDS) who is receiving mechanical ventilation using synchronized intermittent mandatory ventilation (SIMV). The settings include fraction of inspired oxygen (FIO2) 80%, tidal volume 450, rate 16/minute, and positive end-expiratory pressure (PEEP) 5 cm. Which assessment finding is most important for the nurse to report to the health care provider? a. Oxygen saturation 99% b. Respiratory rate 22 breaths/minute c. Crackles audible at lung bases d. Heart rate 106 beats/minute

a. O2 Sat of 99%

To evaluate the effectiveness of ordered interventions for a patient with ventilatory failure, which diagnostic test will be most useful to the nurse? a. chest x ray b. oxygen saturation c. ABG analysis d. CVP monitoring

c. ABG analysis

Appropriate treatment modalities for the mgmt. of cardiogenic shock include (select all) A) dobutamine to increase myocardial contractility B) vasopressors to increase systemic vascular resistance C) circulatory assist devices such as an intraaortic balloon pump D) corticosteroids to stabilize the cell wall in the infarcted myocardium E) Trendelenburg positioning to facilitate venous return and increase preload

A) dobutamine to increase myocardial contractility C) circulatory assist devices such as an intraaortic balloon pump

The patient with a history of lung cancer and hepatitis C has developed liver failure and is considering liver transplantation. After the comprehensive evaluation, the nurse knows that which factor discovered may be a contraindication for liver transplantation? A) Has completed a college education B) Has been able to stop smoking cigarettes C) Has well-controlled type 1 diabetes mellitus D) The chest x-ray showed another lung cancer lesion.

D) Contraindications for liver transplant include severe extrahepatic disease, advanced hepatocellular carcinoma or other cancer, ongoing drug and/or alcohol abuse, and the inability to comprehend or comply with the rigorous post-transplant course.

A patient has renal failure. The nurse, reviewing the lab results, recognizes which finding as indicative of the diminished renal function associated with the diagnosis? 1 Hypokalemia 2 Increased serum urea and serum creatinine 3 Anemia and decreased blood urea nitrogen 4 Increased serum albumin and hyperkalemia

2 Renal failure, whether acute or chronic, causes an increase in serum urea, creatinine, and blood urea nitrogen. Renal failure may also cause hyperkalemia and anemia and decrease serum albumin. However, it does not cause decreased blood urea nitrogen or increased serum albumin. Text Reference - p. 1102

The patient with leukemia has acute disseminated intravascular coagulation (DIC) and is bleeding. What diagnostic findings should the nurse expect to find? A. Elevated D-dimers B. Elevated fibrinogen C. Reduced prothrombin time (PT) D. Reduced fibrin degradation products (FDPs)

A. Elevated D-dimers The D-dimer is a specific marker for the degree of fibrinolysis and is elevated with DIC. FDP is elevated as the breakdown products from fibrinogen and fibrin are formed. Fibrinogen and platelets are reduced. PT, PTT, aPTT, and thrombin time are all prolonged.

Which action will the nurse implement for a patient who arrives for a calcium-scoring CT scan? a. Insert an IV catheter. b. Administer oral sedative medications. c. Teach the patient about the procedure. d. Confirm that the patient has been fasting.

ANS: C The nurse will need to teach the patient that the procedure is rapid and involves little risk. None of the other actions are necessary

Minority group at higher risk for cholelithiasis?

Native American, esp Navajo and Pima nations

The complete blood count (CBC) indicates that a patient is thrombocytopenic. Which action should the nurse include in the plan of care? a. Avoid intramuscular injections. b. Encourage increased oral fluids. c. Check temperature every 4 hours. d. Increase intake of iron-rich foods.

ANS: A Thrombocytopenia is a decreased number of platelets, which places the patient at high risk for bleeding. Neutropenic patients are at high risk for infection and sepsis and should be monitored frequently for signs of infection. Encouraging fluid intake and iron-rich food intake is not indicated in a patient with thrombocytopenia.

13. What is a disadvantage of open reduction and internal fixation of a fracture compared to closed reduction? a. Infection b. Skin irritation c. Nerve impairment d. Complications of immobility

13. a. Open reduction uses a surgical incision to correct bone alignment but infection is the main disadvantage, as well as anesthesia complications or the effect of preexisting medical conditions. Skin irritation and nerve impairment is most likely with skin traction. Prolonged immobility is possible with skeletal traction.

The nurse instructs the pt s/p AKA that the residual limb should not be routinely elevated because this position promotes a. hip flexion contracture b. clot formation at incision site c. skin irritation and breakdown d. increased risk of wound dehiscence

a. hip flexion contracture

reasons for critically ill ppl to get cholecystitis:

- bile stasis - fever, dehydration - no oral feeding -> no CCK -> CCK induces gallbladder contractions

Which collaborative problem will the nurse include in a care plan for a patient admitted to the hospital with idiopathic aplastic anemia? a. Potential complication: seizures b. Potential complication: infection c. Potential complication: neurogenic shock d. Potential complication: pulmonary edema

ANS: B Because the patient with aplastic anemia has pancytopenia, the patient is at risk for infection and bleeding. There is no increased risk for seizures, neurogenic shock, or pulmonary edema

A 23-year-old has been admitted with acute liver failure. Which assessment data are most important for the nurse to communicate to the health care provider? a. Asterixis and lethargy b. Jaundiced sclera and skin c. Elevated total bilirubin level d. Liver 3 cm below costal margin

a. Asterixis and lethargy The patient's findings of asterixis and lethargy are consistent with grade 2 hepatic encephalopathy. Patients with acute liver failure can deteriorate rapidly from grade 1 or 2 to grade 3 or 4 hepatic encephalopathy and need early transfer to a transplant center. The other findings are typical of patients with hepatic failure and would be reported but would not indicate a need for an immediate change in the therapeutic plan.

After receiving change-of-shift report for several patients *with neutropenia,* which patient should the nurse assess first? a. 56-year-old with frequent explosive diarrhea b. 33-year-old with a fever of 100.8° F (38.2° C) c. 66-year-old who has white pharyngeal lesions d. 23-year old who is complaining of severe fatigue

ANS: B Any fever in a neutropenic patient indicates infection and can quickly lead to sepsis and septic shock. Rapid assessment and (if prescribed) initiation of antibiotic therapy within 1 hour are needed. The other patients also need to be assessed but do not exhibit symptoms of potentially life-threatening problems

The patient has frostbite on the distal toes of both feet. The patient is scheduled for amputation of damaged tissue. Which assessment finding or diagnostic study is most objective in determining tissue viability? Arteriogram showing blood vessels Peripheral pulse palpation bilaterally Patches of black, indurated, cold tissue Bilateral pale, cool skin below the ankles

Arteriogram showing blood vessels Arteriography determines viable tissue for salvage based on blood flow observed in real time and is considered the gold standard for evaluating arterial perfusion. Only arteriography determines where tissue perfusion stops and amputation needs to occur. Bilateral peripheral pulse assessment and areas of black, indurated, cold, and pale skin indicate ischemia.

A patient arrives in the emergency department after being burned in a house fire. The patient's burns cover the face and the left forearm. What extent of burns does the patient most likely have? A) 13% B) 25% C) 9% D) 18%

Ans: D Feedback: When estimating the percentage of body area or burn surface area that has been burned, the Rule of Nines is used: the face is 9%, and the forearm is 9% for a total of 18% in this patient.

The nurse is caring for a 33-year-old patient who arrived in the emergency department with acute respiratory distress. Which assessment finding by the nurse requires the most rapid action? a. The patient's PaO2 is 45 mm Hg. b. The patient's PaCO2 is 33 mm Hg. c. The patient's respirations are shallow. d. The patient's respiratory rate is 32 breaths/minute.

a. the patient's PAO2 is 45 mm Hg

27. Which kind of hip fracture is usually repaired with a hip prosthesis? a. Intracapsular b. Extracapsular c. Subtrochanteric d. Intertrochanteric

27. a. A hip prosthesis is usually used for intracapsular fractures. The other options are all extracapsular fractures.

44. A nurse is considering which patient to admit to the same room as a patient who had a liver transplant 3 weeks ago and is now hospitalized with acute rejection. Which patient would be the best choice? a. Patient who is receiving chemotherapy for liver cancer b. Patient who is receiving treatment for acute hepatitis c. Patient who has a wound infection after cholecystectomy d. Patient who requires pain management for chronic pancreatitis

ANS: D The patient with chronic pancreatitis does not present an infection risk to the immunosuppressed patient who had a liver transplant. The other patients either are at risk for infection or currently have an infection, which will place the immunosuppressed patient at risk for infection.

A therapeutic measure used to prevent hypertrophic scarring during the rehabilitative phase of burn recovery is: a. applying pressure garments. b. repositioning the pt every 2 hours c. performing active ROM at least every 4 hours d. massaging the new tissue with water based moisturizers

a. applying pressure garments. Pressure can help keep a scar flat and reduce hypertrophic scarring. Gentle pressure can be maintained on the healed burn with custom-fitted pressure garments.

The critical care nurse recognizes that an ideal plan for caregiver involvement includes a. a caregiver at the bedside at all times b. allowing caregivers at the bedside at preset, brief intervals c. an individually devised plan to involves caregivers with care and comfort measures d. restriction of visiting in the ICU because the environment is overwhelming to caregivers

c. an individually devised plan to involves caregivers with care and comfort measures

Number the following in the order of the phases of exchange in PD. Begin with 1 and end with 3. a. Drain b. Dwell c. Inflow

a. 3; b. 2; c. 1

A male pt suffered full thickness burns to the chest and back and the nurse notes the pressure alarm on his mechanical ventilator is sounding every 5 minutes. What is the most relevant assessment to prevent respiratory complications in this pt? a. pH b. PaCO2 c. Breath sounds d. chest expansion

d. assessing the pt's chest expansion is initially the most important because a sever burn that includes the anterior and posterior thorax can restrict chest expansion from eschar or scar tissue.

A patient arrives in the emergency department after ingesting 8 g of acetaminophen (Tylenol). Which question is most important for the nurse to ask? A. Have you tried to commit suicide before? B. Do you feel like you have a fever? C. What time did you take the Tylenol? D. Are you experiencing any abdominal pain?

Answer: C. What time did you take the Tylenol? Rationale: Acetaminophen will bind to activated charcoal and pass through the gastrointestinal tract without being absorbed. Activated charcoal is most effective if administered within 1 hour of ingestion of acetaminophen and other select poisons. Ch. 69

The nurse would assess a patient with complaints of chest pain for which clinical manifestations associated with a myocardial infarction (MI) (select all that apply)? - Flushing - Ashen skin - Diaphoresis - Nausea and vomiting - S3 or S4 heart sounds

Ashen skin Diaphoresis Nausea and vomiting S3 or S4 heart sounds

The nurse instructs a patient with hyperphosphatemia to avoid what food item? 1 Yogurt 2 Soy sauce 3 Canned soup 4 Salad dressing

1 Yogurt is rich is phosphate and should be avoided by patients with hyperphosphatemia. The patient with hypernatremia and hypertension should avoid soy sauce, canned soups, and salad dressings because they are high in sodium. Text Reference - p. 1115

A triage nurse in the ED admits a 50 year old male client with second degree burns on the anterior and posterior portions of both legs. Based on the Rule of Nines, what percentage of his body is burned? Record your answer using a whole number.

36 The anterior and posterior portions of one leg are 18%, if both legs are burned, the total is 36%.

The nurse notes scleral jaundice in a patient being admitted with hemolytic anemia. The nurse will plan to check the laboratory results for the a. Schilling test. b. bilirubin level. c. stool occult blood test. d. gastric analysis testing.

ANS: B *Jaundice* is caused by the elevation of *bilirubin* level associated with red blood cell (RBC) hemolysis. The other tests would not be helpful in monitoring or treating a hemolytic anemia

A 61-year-old female patient admitted with pneumonia has a total serum calcium level of 13.3 mg/dL (3.3 mmol/L). The nurse will anticipate the need to teach the patient about testing for _____ levels. a. calcitonin b. catecholamine c. thyroid hormone d. parathyroid hormone

D

A patient who has acute glomerulonephritis is hospitalized with hyperkalemia. Which information will the nurse monitor to evaluate the effectiveness of the prescribed calcium gluconate IV? a. Urine volume b. Calcium level c. Cardiac rhythm d. Neurologic status

c. cardiac rhythm

What are the postrenal causes of acute kidney injury? Select all that apply. 1 Renal calculi 2 Renal trauma 3 Prostate cancer 4 Kidney ischemia 5 Myoglobin release

1, 2, 3 Renal calculi, trauma, and prostate cancer are postrenal causes of acute kidney injury. Intrarenal causes of acute kidney injury include kidney ischemia and myoglobin released from muscle cells. Text Reference - p. 1103

What glomerular filtration rate (GFR) would the nurse estimate for a 30-year-old patient with a creatinine clearance result of 60 mL/min? a. 60 mL/min b. 90 mL/min c. 120 mL/min d. 180 mL/min

ANS: A The creatinine clearance approximates the GFR. The other responses are not accurate.

27. A patient who is being admitted to the emergency department with intermittent chest pain gives the following list of medications to the nurse. Which medication has the most immediate implications for the patient's care? a. Sildenafil (Viagra) b. Furosemide (Lasix) c. Captopril (Capoten) d. Warfarin (Coumadin)

ANS: A The nurse will need to avoid giving nitrates to the patient because nitrate administration is contraindicated in patients who are using sildenafil because of the risk of severe hypotension caused by vasodilation. The other home medications also should be documented and reported to the health care provider but do not have as immediate an impact on decisions about the patient's treatment.

6. The nurse will instruct the patient with a fractured left radius that the cast will need to remain in place a. for several months. b. for at least 3 weeks. c. until swelling of the wrist has resolved. d. until x-rays show complete bony union.

ANS: B Bone healing starts immediately after the injury, but since ossification does not begin until 3 weeks postinjury, the cast will need to be worn for at least 3 weeks. Complete union may take up to a year. Resolution of swelling does not indicate bone healing.

To determine the effects of therapy for a patient who is being treated for heart failure, which laboratory result will the nurse plan to review? a. Troponin b. Homocysteine (Hcy) c. Low-density lipoprotein (LDL) d. B-type natriuretic peptide (BNP)

ANS: D Increased levels of BNP are a marker for heart failure. The other laboratory results would be used to assess for myocardial infarction (troponin) or risk for coronary artery disease (Hcy and LDL).

The nurse plans emergent care for four male pt's who have burns covering between 40-50% of the total body surface area. Rank these patients according to their risk for an inhalation injury beginning with the pt who has the highest risk. a. has posterior chemical burns from an exhibit at a parking lot b. has osteoporosis and electrical burns of the lower extremities c. has thermal burns of the right side and is a volunteer fireman d. has chronic bronchitis and thermal burns around the abdomen

d, c, a, b.

Priority Decision: What is the most appropriate snack for the nurse to offer a patient with stage 4 CKD? a. Raisins b. Ice cream c. Dill pickles d. Hard candy

d. A patient with CKD may have unlimited intake of sugars and starches (unless the patient is diabetic) and hard candy is an appropriate snack and may help to relieve the metallic and urine taste that is common in the mouth. Raisins are a high-potassium food. Ice cream contains protein and phosphate and counts as fluid. Pickled foods have high sodium content. Lewis, Sharon L.; Dirksen, Shannon Ruff; Bucher, Linda (2014-03-14). Study Guide for Medical-Surgical Nursing: Assessment and Management of Clinical Problems (Study Guide for Medical-Surgical Nursing: Assessment & Management of Clinical Problem) (Page 413). Elsevier Health Sciences. Kindle Edition.

The nurse admits a 73-year-old male patient with dementia for treatment of uncontrolled hypertension. The nurse will closely monitor for hypokalemia if the patient receives which medication? A Clonidine (Catapres) B Bumetanide (Bumex) C Amiloride (Midamor) D Spironolactone (Aldactone)

B Bumetanide (Bumex) Bumetanide is a loop diuretic. Hypokalemia is a common adverse effect of this medication. Amiloride is a potassium-sparing diuretic. Spironolactone is an aldosterone-receptor blocker. Hyperkalemia is an adverse effect of both amiloride and spironolactone. Clonidine is a central-acting α-adrenergic antagonist and does not cause electrolyte abnormalities.

The earliest signs of increased ICP the nurse should assess for include a. Cushing's triad b. unexpected vomiting c. decreasing level of consciousness (LOC) d. dilated pupil with sluggish response to light

C. One of the most sensitive signs of increased intracranial pressure (ICP) is a *decreasing LOC.* A decrease in LOC will occur before changes in vital signs, ocular signs, and projectile vomiting occur

In caring for the patient with angina, the patient said, "While I was having a bowel movement, I started having the worst chest pain ever, like before I was admitted. I called for a nurse, then the pain went away." What further assessment data should the nurse obtain from the patient? "What precipitated the pain?" "Has the pain changed this time?" "In what areas did you feel this pain?" "What is your pain level on a 0 to 10 scale?"

"In what areas did you feel this pain?" Using PQRST, the assessment data not volunteered by the patient is the radiation of pain, the area the patient felt the pain, and if it radiated. The precipitating event was going to the bathroom and having a bowel movement. The quality of the pain was "like before I was admitted," although a more specific description may be helpful. Severity of the pain was the "worst chest pain ever," although an actual number may be needed. Timing is supplied by the patient describing when the pain occurred and that he had previously had this pain.

Which statement by the nurse regarding continuous ambulatory peritoneal dialysis (CAPD) would be of highest priority when teaching a patient new to this procedure? A. "It is essential that you maintain aseptic technique to prevent peritonitis. B. "You will be allowed a more liberal protein diet once you complete CAPD." C."It is important for you to maintain a daily written record of blood pressure and weight." D."You will need to continue regular medical and nursing follow-up visits while performing CAPD."

"It is essential that you maintain aseptic technique to prevent peritonitis." Peritonitis is a potentially fatal complication of peritoneal dialysis, and thus it is imperative to teach the patient methods of preventing this from occurring. Although the other teaching statements are accurate, they do not have the potential for morbidity and mortality as does peritonitis, thus making that statement of highest priority.

Assessment findings of a patient with chronic kidney failure include a glomerular filtration rate (GFR) of 10 mL/min, numbness and burning sensation in the legs, and a blood urea nitrogen level (BUN) of 26 mg/dL. The nurse anticipates that which intervention will be included on the patient's plan of care? 1 Make a referral for dialysis. 2 Administer sodium polystyrene sulfonate. 3 Restrict sodium bicarbonate. 4 Provide a magnesium-containing antacid.

1 Numbness and burning sensation in the legs are manifestations of peripheral neuropathy caused by nitrogenous waste accumulation in the brain. A patient with a chronic kidney disease (CKD), increased blood urea nitrogen (BUN) levels, and a very low glomerular filtration rate of 10 mL/min should undergo dialysis to remove nitrogenous wastes and prevent fluid accumulation due to impaired excretion. Sodium polystyrene sulfonate treats hyperkalemia. Sodium bicarbonate treats metabolic acidosis. A patient with CKD must not take antacids containing magnesium or aluminum because they are excreted by the kidneys. Text Reference - p. 1110

While caring for a patient with acute kidney injury, the nurse observes that the patient has hand tremors while extending the wrist. The patient's laboratory report shows a blood urea nitrogen (BUN) level of 123 mg/dL. Which action by the patient does the nurse suspect as the cause of this symptom? 1 Eating protein-rich food 2 Eating sodium-rich food 3 Eating potassium-rich food 4 Eating carbohydrate-rich food

1 Patients with acute kidney injury have impaired renal excretion cannot eliminate nitrogenous wastes; this will result in increased blood urea nitrogen (BUN) levels. Eating protein-rich food will increase the level of BUN and cause neurologic changes such as asterixis, which is characterized by flapping tremor upon extension of the wrist. A normal level of BUN is 120 mg/dL. Because the patient has asterixis and a BUN level of 125 mg/dL, the nurse suspects the consumption of protein-rich food to be the cause of this symptom. Text Reference - p. 1104

A patient complains of pedal edema. The laboratory reports show 0.4 mL/kg/hr of urine output for the past 12 hours. The patient has a history of acute glomerulonephritis. Which method is the best to confirm acute glomerulonephritis as a cause of acute kidney injury in this patient? 1 Kidney biopsy 2 Kidney ultrasound 3 Computed tomographic scan 4 Magnetic resonance imaging

1 Pedal edema and urine output less than 0.5 mL/kg/hr for 12 hours indicate acute kidney injury. Glomerulonephritis is one of the intrarenal causes of acute kidney injury. A kidney biopsy is the best method to confirm intrarenal causes of kidney injury. A kidney ultrasound is the first diagnostic test used to establish acute kidney injury. A computed tomography scan is used to identify lesions, masses, lesions, and vascular anomalies. Magnetic resonance imaging is not advised in patients with renal failure unless necessary due to the development of nephrogenic systemic fibrosis. Text Reference - p. 1105

The nurse caring for a patient with heart failure notes the patient has decreased urine output of 200 mL/day. Which laboratory finding aids in the diagnosis of prerenal azotemia in this patient? 1 Elevated blood urea nitrogen (BUN) 2 Normal creatinine level 3 Decreased sodium level 4 Decreased potassium level

1 The patient with heart failure has a decreased circulating blood volume. This causes autoregulatory mechanisms to preserve blood flow to essential organs. Laboratory data for this patient will likely demonstrate an elevation in BUN, creatinine, and potassium. Prerenal azotemia results in a reduction in the excretion of sodium, increased sodium and water retention, and decreased urine output. Text Reference - p. 1102

The nurse is caring for a patient who is experiencing cardiogenic shock as a result of myocardial infarction. Which nursing assessment finding is most concerning? 1 PaO2 60 mm Hg 2 Blood pressure 100/56 mm Hg 3 Urine output 260 mL in eight hours 4 Heart rate 96 beats/minute

1 A PaO2 of 60 is below the normal 80 to 100 mm Hg. The patient experiencing cardiogenic shock will exhibit hypotension and tachycardia, and therefore a blood pressure of 100/56 mm Hg and heart rate of 96 would not apply. A urine output of 260 cc/8 hrs is borderline but not reportable without a continued trending pattern. Text Reference - p. 1633

What therapy is provided to a patient with acute respiratory distress syndrome (ARDS)? 1 Mechanical ventilation 2 Oxygen via a Venturi mask 3 Oxygen via a non-rebreather mask 4 Small volume nebulizer treatments

1 A patient with acute respiratory distress syndrome (ARDS) would be intubated and receive mechanical ventilation. Small volume nebulizer treatments would open airways; however, this intervention will not sufficiently treat ARDS. Oxygen via a Venturi mask or a non-rebreather mask would be insufficient to promote oxygenation and perfusion. Text Reference - p. 1651

What occurs when the inflammatory response is activated in a patient with systemic inflammatory response syndrome (SIRS)? 1 Release of mediators 2 Decrease in metabolism 3 Damage of the mesothelium 4 Decrease in vascular permeability

1 A release of mediators occurs when the inflammatory response is activated. Other changes that occur include an increase in metabolism or hypermetabolism, direct damage to the endothelium, and an increase in vascular permeability. Text Reference - p. 1649

How is systemic inflammatory response syndrome (SIRS) different from multiple organ dysfunction syndrome (MODS)? 1 Shock leads to SIRS, and SIRS causes MODS. 2 MODS is reversible, and SIRS has irreversible changes. 3 SIRS is caused by shock, and MODS is caused by perfusion deficits. 4 Homeostasis fails before SIRS, and homeostasis is maintained in MODS.

1 Any type of shock triggers the systemic inflammatory response. Generalized inflammation in organs remote from the initial shock is systemic inflammatory response syndrome (SIRS). Multiple organ dysfunction syndrome (MODS) results from SIRS. Both SIRS and MODS are reversible in the early stages. SIRS is caused by some kind of injury to the body such as sepsis, ischemia, infarction, and injury; SIRS can lead to MODS if not treated. In SIRS, a type of shock triggers a systemic inflammatory response, after which the body's homeostasis fails. In MODS, homeostasis fails and medical intervention is needed. Test-Taking Tip: Identify option components as correct or incorrect. This may help you identify a wrong answer. Text Reference - p. 1649

When obtaining a health history for the patient with chronic kidney disease, the nurse notes the following medications on the patient's medication list. The patient will need further education on which medication? 1 Ibuprofen 2 Acetaminophen 3 Calcium supplements 4 Calcium acetate

1 Ibuprofen, and other nonsteroidal antiinflammatory drugs (NSAIDS), will cause further damage to the kidneys. Chronic kidney disease (CKD) patients should be taking Tylenol as prescribed for pain. CKD patients also could be consuming calcium supplements and PhosLo tablets as prescribed by the health care provider. Text Reference - p. 1107

Metabolic acidosis occurs in the oliguric phase of AKI as a result of impairment of a. ammonia synthesis. b. excretion of sodium. c. excretion of bicarbonate. d. conservation of potassium.

a. Metabolic acidosis occurs in AKI because the kidneys cannot synthesize ammonia or excrete acid products of metabolism, resulting in an increased acid load. Sodium is lost in urine because the kidneys cannot conserve sodium. Impaired excretion of potassium results in hyperkalemia. Bicarbonate is normally generated and reabsorbed by the functioning kidney to maintain acidbase balance.

A patient has a PaO2 of 50 mm Hg and a PaCO2 of 42 mm Hg because of an intrapulmonary shunt. Which therapy is the patient most likely to respond best to? a. Positive pressure ventilation b. Oxygen administration at a FIO2 of 100% c. Administration of O2 per nasal cannula at 1 to 3 L/min d. Clearance of airway secretions with coughing and suctioning

a. Patients with a shunt are usually more hypoxemic than patients with a V/Q mismatch because the alveoli are filled with fluid, which prevents gas exchange. Hypoxemia resulting from an intrapulmonary shunt is usually not responsive to high O2 concentrations and the patient will usually require positive pressure ventilation. Hypoxemia associated with a V/Q mismatch usually responds favorably to O2 administration at 1 to 3 L/min by nasal cannula. Removal of secretions with coughing and suctioning is generally not effective in reversing an acute hypoxemia resulting from a shunt.

A patient with a massive hemothorax and pneumothorax has absent breath sounds in the right lung. To promote improved V/Q matching, how should the nurse position the patient? a. On the left side b. On the right side c. In a reclining chair bed d. Supine with the head of the bed elevated

a. When there is impaired function of one lung, the patient should be positioned with the unaffected lung in the dependent position to promote perfusion to the functioning tissue. If the diseased lung is positioned dependently, more V/Q mismatch would occur. The head of the bed may be elevated or a reclining chair may be used, with the patient positioned on the unaffected side, to maximize thoracic expansion if the patient has increased work of breathing. *good lung down*

A pt with a humeral Fx is returning for a 4 week check. The nurse explains that initial evidence of healing on x-ray is indicated by a. formation of callus b. complete bony union c. hematoma at Fx site d. presence of granulation tissue

a. formation of callus

While caring for a patient who has been admitted with a pulmonary embolism, the nurse notes a change in the patient's oxygen saturation from 94% to 88%. Which action should the nurse take next? a. increase oxygen flow rate b. suction the patient's oropharynx c. instruct the patient to cough and deep breathe d. help patient sit in a more upright position

a. increase oxygen flow rate

Which information about a patient who is receiving cisatracurium (Nimbex) to prevent asynchronous breathing with the positive pressure ventilator requires immediate action by the nurse? a. Only continuous IV opioids have been ordered. b. The patient does not respond to verbal stimulation. c. There is no cough or gag when the patient is suctioned. d. The patient's oxygen saturation fluctuates between 90% to 93%.

a. only continuous IV opioids have been ordered

The nurse is caring for a client who suffered an inhalation injury from a wood stove. The carbon monoxide blood report reveals a level of 12%. Based on this level, the nurse would anticipate which of the following signs in the client? a. coma b. flushing c. dizziness d. tachycardia

b. 11-20% - signs include flushing, headache, decreased visual acuity, decreased cerebral functioning, and slight breathlessness. 21-40% - signs include nausea, vomiting, dizziness, tinnitus, vertigo, confusion, drowsiness, pale to reddish-purple skin, tachycardia; levels of 41-60% result in seizure and come and levels higher than 60% result in death

Which physiologic mechanism of hypoxemia occurs with pulmonary fibrosis? a. Anatomic shunt b. Diffusion limitation c. Intrapulmonary shunt d. V/Q mismatch ratio of less than 1

b. Diffusion limitation in pulmonary fibrosis is caused by thickened alveolar-capillary interface, which slows gas transport.

Which patient with the following manifestations is most likely to develop hypercapnic respiratory failure? a. Rapid, deep respirations in response to pneumonia b. Slow, shallow respirations as a result of sedative overdose c. Large airway resistance as a result of severe bronchospasm d. Poorly ventilated areas of the lung caused by pulmonary edema

b. Hypercapnic respiratory failure is associated with alveolar hypoventilation with increases in alveolar and arterial carbon dioxide (CO2) and often is caused by problems outside the lungs. A patient with slow, shallow respirations is not exchanging enough gas volume to eliminate CO2. Deep, rapid respirations reflect hyperventilation and often accompany lung problems that cause hypoxemic respiratory failure. Pulmonary edema and large airway resistance cause obstruction of oxygenation and result in a V/Q mismatch or shunt typical of hypoxemic respiratory failure.

A pt who has an inhalation injury is receiving albuterol (Ventolin) for bronchospasm. What is the most important adverse effect of this medication for the nurse to manage? a. GI distress b. tachycardia c. restlessness d. hypokalemia

b. albuterol stimulates beta adrenergic receptors in the lungs to cause bronchodilation and is nonselective and also causes receptors in the heart to increase heart rate.

A 42-year-old patient admitted with acute kidney injury due to dehydration has oliguria, anemia, and hyperkalemia. Which prescribed actions should the nurse take first? a. Insert a urinary retention catheter. b. Place the patient on a cardiac monitor. c. Administer epoetin alfa (Epogen, Procrit). d. Give sodium polystyrene sulfonate (Kayexalate)

b. place the patient on a cardiac monitor

Before administration of captopril (Capoten) to a patient with stage 2 chronic kidney disease (CKD), the nurse will check the patient's a. glucose. b. potassium. c. creatinine. d. phosphate.

b. potassium

Complication of acalculous cholecystitis

bacteria notice there is a party happening and decide to crash it, causing secondary infection and potentially perforation usual suspects include E coli, Enterococcus faecalis, Klebsiella, Pseudomonas, Proteus

The critical care nurse recognizes that an ideal plan for caregiver involvement includes a. a caregiver at the bedside at all times b. allowing caregivers at the bedside at preset, brief intervals c. an individually devised plan to involves caregivers with care and comfort measures d. restriction of visiting in the ICU because the environment is overwhelming to caregivers

c. an individually devised plan t involves caregivers with care and comfort measures

The nurse is caring for a client following an autograft and grafting to a burn wound on the right knee. Which of the following would the nurse anticipate to be prescribed for the client? a. out of bed b. brp c. Immobilization of the affected leg d. placing the affected leg in a dependent position

c. autografts placed over joints or on the lower extremities after surgery often are elevated and immobilized for 3-7 days. this period allows the autograft time to adhere to the wound bed.

The nurse suspects a neurovascular problem based on assessment of a. exaggerated strength with movement b. increased redness and heat below the injury c. decreased sensation distal to the Fx site d. purulent drainage at the site of an open Fx

c. decreased sensation distal to the Fx site

What is the best method for preventing hypovolemic shock in a client admitted with severe burns? a. administering dopamine b. applying medical antishock trousers c. infusing i.v. fluids d. infusing fresh frozen plasma

c. during the early postburn period, large amounts of plasma fluid extravasates into interstitial spaces. Restoring the fluid loss is necessary to prevent hypovolemic shock; this is best accomplished with crystalloid and colloid solutions.

Which assessment finding may indicate that a patient is experiencing adverse effects to a corticosteroid prescribed after kidney transplantation? a. Postural hypotension b. Recurrent tachycardia c. Knee and hip joint pain d. Increased serum creatinine

c. knee and hip joint pain

The nursing management of a patient with an artificial airway includes a. maintaining ET tube cuff pressure at 30 cm H2O b. routine suctioning of the tube at least every 2 hours c. observing for cardiac dysrhythmias during suctioning d. preventing tube dislodgement by limiting mouth care to lubrication of the lips

c. observing for cardiac dysrhythmias during suctioning

The nurse will teach a patient with chronic pancreatitis to take the prescribed pancrelipase (Viokase) a. at bedtime. b. in the morning. c. with each meal. d. for abdominal pain.

c. with each meal. Pancreatic enzymes are used to help with digestion of nutrients and should be taken with every meal.

The nursing management of the patient with cholecystitis associated with cholelithiasis is based on the knowledge that a. shock-wave therapy should be tried initially. b. once gallstones are removed, they tend not to recur. c. the disorder can be successfully treated with oral bile salts that dissolve gallstones. d. laparoscopic cholecystectomy is the treatment of choice in most patients who are symptomatic. (Lewis 1042)

d Rationale: Laparoscopic cholecystectomy is the treatment of choice for symptomatic cholelithiasis.

The client arrives at the emergency department following a burn injury that occurred in the basement at home and an inhalation injury is suspected. Which of the following would the nurse anticipate to be prescribed for the client? a. 100% oxygen via an aerosol mask b. Oxygen via nasal cannula at 15L/min c. Oxygen via nasal cannula at 10L/min d. 100% oxygen via a tight fitting, non rebreather face mask

d.

Which finding indicates to the nurse that a patient's transjugular intrahepatic portosystemic shunt (TIPS) placed 3 months ago has been effective? a. Increased serum albumin level b. Decreased indirect bilirubin level c. Improved alertness and orientation d. Fewer episodes of bleeding varices

d. Fewer episodes of bleeding varices TIPS is used to lower pressure in the portal venous system and decrease the risk of bleeding from esophageal varices. Indirect bilirubin level and serum albumin levels are not affected by shunting procedures. TIPS will increase the risk for hepatic encephalopathy.

What indicates to the nurse that a patient with oliguria has prerenal oliguria? a. Urine testing reveals a low specific gravity. b. Causative factor is malignant hypertension. c. Urine testing reveals a high sodium concentration. d. Reversal of oliguria occurs with fluid replacement.

d. In prerenal oliguria, the oliguria is caused by a decrease in circulating blood volume and there is no damage yet to the renal tissue. It can be reversed by correcting the precipitating factor, such as fluid replacement for hypovolemia. Prerenal oliguria is characterized by urine with a high specific gravity and a low sodium concentration, whereas oliguria of intrarenal failure is characterized by urine with a low specific gravity and a high sodium concentration. Malignant hypertension causes damage to renal tissue and intrarenal oliguria.

When assessing a patient with chronic obstructive pulmonary disease (COPD), the nurse finds a new onset of agitation and confusion. Which action should the nurse take first? a. Notify the health care provider. b. Check pupils for reaction to light. c. Attempt to calm and reorient the patient. d. Assess oxygenation using pulse oximetry.

d. assess oxygenation using pulse oximetry

A pt is scheduled for total ankle replacement. The nurse should tell the pt that after surgery he should avoid a. lifting heavy objects b. sleeping on the back c. abduction exercises of the affected ankle d. bearing weight on the affected leg for 6 weeks

d. bearing weight on the affected leg for 6 weeks

labs for liver and pancreas?

liver: ALT, AST, alk phosphatase pancreas: amylase, lipase

post lap chole teaching

remove - gallbladder is gone report - s/s infection restrictions - no work for 1 week return - when is f/u resume - activity in 1 week

choledocholithiasis is?

stone in common bile duct, causing s/s of obstruction

26. The nurse suspects a fat embolism rather than a pulmonary embolism from a venous thrombosis when the patient with a fracture develops what? a. Tachycardia and dyspnea b. A sudden onset of chest pain c. Petechiae around the neck and upper chest d. Electrocardiographic (ECG) changes and decreased PaO2

26. c. Patients with fractures are at risk for both fat embolism and pulmonary embolism from venous thromboembolism but there is a difference in the time of occurrence, with fat embolism occurring shortly after the injury and thrombotic embolism occurring several days after immobilization. They both may cause pulmonary symptoms of chest pain, tachypnea, dyspnea, apprehension, tachycardia, and cyanosis. However, fat embolism may cause petechiae located around the neck, anterior chest wall, axilla, buccal membrane of the mouth, and conjunctiva of the eye, which differentiates it from thrombotic embolism.

The nurse reviews a plan of care for a patient with diagnosis of chronic kidney disease who is undergoing hemodialysis. Which part of the plan should the nurse question? 1 2-g sodium diet 2 Oxygen via nasal cannula at 4 L/min 3 Furosemide (Lasix) 40 mg PO twice a day 4 IV of 0.9% sodium chloride at 125 mL/hour

4 A patient with chronic kidney disease (CKD) should receive limited fluids because the kidneys are unable to remove excessive water. An IV solution of 0.9% sodium chloride at a rate of 125 mL/hr places this patient at high risk for complications such as fluid overload, electrolyte imbalance, and hypertension. A 2-g sodium diet, oxygen, and furosemide (Lasix) would be appropriate if prescribed for a patient with CKD. Text Reference - p. 1115

An ESRD patient receiving hemodialysis is considering asking a relative to donate a kidney for transplantation. In assisting the patient to make a decision about treatment, the nurse informs the patient that: A. successful transplantation usually provides a better quality of life than that offered by dialysis B. if rejection of the transplanted kidney occurs, no further treatment for the renal failure is available C. the immunosuppressive therapy that is required following transplantation causes fatal malignancies in many patients D. hemodialysis replaces the normal functioning of the kidneys and patients do not have to live with the continual fear of rejection

A

Despite a high dosage, a male patient who is taking nifedipine (Procardia XL) for antihypertensive therapy continues to have blood pressures over 140/90 mmHg. What should the nurse do next? A Assess his adherence to therapy. B Ask him to make an exercise plan. C Instruct him to use the DASH diet. D Request a prescription for a thiazide diuretic.

A Assess his adherence to therapy. A long-acting calcium-channel blocker such as nifedipine causes vascular smooth muscle relaxation resulting in decreased SVR and arterial BP and related side effects. The patient data the nurse has about this patient is very limited, so the nurse needs to assess his adherence to therapy.

The nurse is caring for a patient with hypertension who is scheduled to receive a dose of esmolol (Brevibloc). The nurse should withhold the dose and consult the prescribing physician for which vital sign taken just before administration? A Pulse 48 B Respirations 24 C Blood pressure 118/74 D Oxygen saturation 93%

A Pulse 48 Because esmolol is a β1-adrenergic blocking agent, it can cause hypotension and bradycardia as adverse effects. The nurse should withhold the dose and consult with the health care provider for parameters regarding pulse rate limits.

The patient with cirrhosis is being taught self-care. Which statement indicates the patient needs more teaching? A) "If I notice a fast heart rate or irregular beats, this is normal for cirrhosis." B) "I need to take good care of my belly and ankle skin where it is swollen." C) "A scrotal support may be more comfortable when I have scrotal edema." D) "I can use pillows to support my head to help me breathe when I am in bed."

A) If the patient with cirrhosis experiences a fast or irregular heart rate, it may be indicative of hypokalemia and should be reported to the health care provider, as this is not normal for cirrhosis. Edematous tissue is subject to breakdown and needs meticulous skin care. Pillows and a semi-Fowler's or Fowler's position will increase respiratory efficiency. A scrotal support may improve comfort if there is scrotal edema

The patient with cirrhosis has an increased abdominal girth from ascites. The nurse should know that this fluid gathers in the abdomen for which reasons (select all that apply)? A) There is decreased colloid oncotic pressure from the liver's inability to synthesize albumin. B) Hyperaldosteronism related to damaged hepatocytes increases sodium and fluid retention. C) Portal hypertension pushes proteins from the blood vessels, causing leaking into the peritoneal cavity. D) Osmoreceptors in the hypothalamus stimulate thirst, which causes the stimulation to take in fluids orally. E) Overactivity of the enlarged spleen results in increased removal of blood cells from the circulation, which decreases the vascular pressure.

A,B.C) The ascites related to cirrhosis are caused by decreased colloid oncotic pressure from the lack of albumin from liver inability to synthesize it and the portal hypertension that shifts the protein from the blood vessels to the peritoneal cavity, and hyperaldosteronism which increases sodium and fluid retention. The intake of fluids orally and the removal of blood cells by the spleen do not directly contribute to ascites.

During admission of a patient with a severe head injury to the ED, the nurse places highest priority on assessment for a. patency of of airway b. presence of a neck injury c. neurologic status with Glascow Coma Scale d. CSF leakage from ears and nose

A. Patency of airway is the #1 priority with all head injuries

Which action will the public health nurse take to reduce the incidence of epidemic encephalitis in a community? a. Encourage the use of effective insect repellents during mosquito season. b. Remind patients that most cases of viral encephalitis can be cared for at home. c. Teach about the importance of prophylactic antibiotics after exposure to encephalitis. d. Arrange for screening of school-age children for West Nile virus during the school year.

ANS: A Epidemic encephalitis is usually spread by mosquitoes and ticks. Use of insect repellent is effective in reducing risk. Encephalitis frequently requires that the patient be hospitalized in an intensive care unit during the initial stages. Antibiotic prophylaxis is not used to prevent encephalitis because most encephalitis is viral. West Nile virus is most common in adults over age 50 during the summer and early fall. DIF: Cognitive Level: Apply (application) REF: 1384 TOP: Nursing Process: Planning MSC: NCLEX: Health Promotion and Maintenance

A patient with septic shock has a urine output of 20 mL/hr for the past 3 hours. The pulse rate is 120 and the central venous pressure and pulmonary artery wedge pressure are low. Which of these orders by the health care provider will the nurse question? a. Give furosemide (Lasix) 40 mg IV. b. Increase normal saline infusion to 150 mL/hr. c. Administer hydrocortisone (SoluCortef) 100 mg IV. d. Prepare to give drotrecogin alpha (Xigris) 24 mcg/kg/hr.

ANS: A Furosemide will lower the filling pressures and renal perfusion further for the patient with septic shock. The other orders are appropriate. DIF: Cognitive Level: Application REF: 1724-1726 | 1731 | 1733 TOP: Nursing Process: Implementation MSC: NCLEX: Physiological Integrity

A patient has elevated blood urea nitrogen (BUN) and serum creatinine levels. Which bowel preparation order would the nurse question for this patient who is scheduled for a renal arteriogram? a. Fleet enema b. Tap-water enema c. Senna/docusate (Senokot-S) d. Bisacodyl (Dulcolax) tablets

ANS: A High-phosphate enemas, such as Fleet enemas, should be avoided in patients with elevated BUN and creatinine because phosphate cannot be excreted by patients with renal failure. The other medications for bowel evacuation are more appropriate.

6. A patient is diagnosed with hypertension and nadolol (Corgard) is prescribed. The nurse should consult with the health care provider before giving this medication upon finding a history of a. asthma. b. peptic ulcer disease. c. alcohol dependency. d. myocardial infarction (MI).

ANS: A Nonselective b-blockers block b1- and b2-adrenergic receptors and can cause bronchospasm, especially in patients with a history of asthma. b-blockers will have no effect on the patient's peptic ulcer disease or alcohol dependency. b-blocker therapy is recommended after MI.

A 19-year-old woman with immune thrombocytopenic purpura (ITP) has an order for a platelet transfusion. Which information indicates that the nurse should consult with the health care provider before obtaining and administering platelets? a. The platelet count is 42,000/mL. b. Petechiae are present on the chest. c. Blood pressure (BP) is 94/56 mm Hg. d. Blood is oozing from the venipuncture site.

ANS: A Platelet transfusions are not usually indicated until the platelet count is below 10,000 to 20,000/mL unless the patient is actively bleeding. Therefore the nurse should clarify the order with the health care provider before giving the transfusion. The other data all indicate that bleeding caused by ITP may be occurring and that the platelet transfusion is appropriate

The charge nurse observes the following actions being taken by a new nurse on the burn unit. Which action by the new nurse would require an intervention by the charge nurse? a. The new nurse uses clean latex gloves when applying antibacterial cream to a burn wound. b. The new nurse obtains burn cultures when the patient has a temperature of 95.2° F (35.1° C). c. The new nurse administers PRN fentanyl (Sublimaze) IV to a patient 5 minutes before a dressing change. d. The new nurse calls the health care provider for a possible insulin order when a nondiabetic patient's serum glucose is elevated.

ANS: A Sterile gloves should be worn when applying medications or dressings to a burn. Hypothermia is an indicator of possible sepsis, and cultures are appropriate. Nondiabetic patients may require insulin because stress and high calorie intake may lead to temporary hyperglycemia. Fentanyl peaks 5 minutes after IV administration, and should be used just before and during dressing changes for pain management. (Also, don't use latex on someone with large wounds.)

The nurse hears a murmur between the S1 and S2 heart sounds at the patient's left fifth intercostal space and midclavicular line. How will the nurse record this information? a. Systolic murmur heard at mitral area b. Systolic murmur heard at Erb's point c. Diastolic murmur heard at aortic area d. Diastolic murmur heard at the point of maximal impulse

ANS: A The S1 signifies the onset of ventricular systole. S2 signifies the onset of diastole. A murmur occurring between these two sounds is a systolic murmur. The mitral area is the intersection of the left fifth intercostal space and the midclavicular line. The other responses describe murmurs heard at different landmarks on the chest and/or during the diastolic phase of the cardiac cycle

During change-of-shift report, the nurse is told that a patient has been admitted with dehydration and hypotension after having vomiting and diarrhea for 4 days. Which finding is most important for the nurse to report to the health care provider? a. New onset of confusion b. Heart rate 112 beats/minute c. Decreased bowel sounds d. Pale, cool, and dry extremities

ANS: A The changes in mental status are indicative that the patient is in the progressive stage of shock and that rapid intervention is needed to prevent further deterioration. The other information is consistent with compensatory shock

Which patient information will the nurse plan to obtain in order to determine the effectiveness of the prescribed calcium carbonate (Caltrate) for a patient with chronic kidney disease (CKD)? a. Blood pressure b. Phosphate level c. Neurologic status d. Creatinine clearance

ANS: B Calcium carbonate is prescribed to bind phosphorus and prevent mineral and bone disease in patients with CKD. The other data will not be helpful in evaluating the effectiveness of calcium carbonate

37. Which assessment information will be most important for the nurse to report to the health care provider about a patient with acute cholecystitis? a. The patient's urine is bright yellow. b. The patient's stools are tan colored. c. The patient has increased pain after eating. d. The patient complains of chronic heartburn.

ANS: B Tan or grey stools indicate biliary obstruction, which requires rapid intervention to resolve. The other data are not unusual for a patient with this diagnosis, although the nurse would also report the other assessment information to the health care provider.

A patient with cardiogenic shock has the following vital signs: BP 86/50, pulse 126, respirations 30. The PAWP is increased and cardiac output is low. The nurse will anticipate a. infusion of 5% human albumin. b. administration of furosemide (Lasix) IV. c. titration of an epinephrine (Adrenalin) drip. d. administration of hydrocortisone (SoluCortef).

ANS: B The PAWP indicates that the patient's preload is elevated and furosemide is indicated to reduce the preload and improve cardiac output. Epinephrine would further increase heart rate and myocardial oxygen demand. Normal saline infusion would increase the PAWP further. Hydrocortisone might be used for septic or anaphylactic shock. DIF: Cognitive Level: Application REF: 1735 | 1736 TOP: Nursing Process: Planning MSC: NCLEX: Physiological Integrity

A registered nurse (RN) is observing a student nurse who is doing a physical assessment on a patient. The RN will need to intervene immediately if the student nurse a. presses on the skin over the tibia for 10 seconds to check for edema. b. palpates both carotid arteries simultaneously to compare pulse quality. c. documents a murmur heard along the right sternal border as a pulmonic murmur. d. places the patient in the left lateral position to check for the point of maximal impulse.

ANS: B The carotid pulses should never be palpated at the same time to avoid vagal stimulation, dysrhythmias, and decreased cerebral blood flow. The other assessment techniques also need to be corrected. However, they are not dangerous to the patient

28. Which assessment finding by the nurse caring for a patient who has had coronary artery bypass grafting using a right radial artery graft is most important to communicate to the health care provider? a. Complaints of incisional chest pain b. Pallor and weakness of the right hand c. Fine crackles heard at both lung bases d. Redness on both sides of the sternal incision

ANS: B The changes in the right hand indicate compromised blood flow, which requires immediate evaluation and actions such as prescribed calcium channel blockers or surgery. The other changes are expected and/or require nursing interventions.

When admitting a patient for a cardiac catheterization and coronary angiogram, which information about the patient is most important for the nurse to communicate to the health care provider? a. The patient's pedal pulses are +1. b. The patient is allergic to shellfish. c. The patient had a heart attack a year ago. d. The patient has not eaten anything today.

ANS: B The contrast dye used for the procedure is iodine based, so patients who have shellfish allergies will require treatment with medications such as corticosteroids and antihistamines before the angiogram. The other information is also communicated to the health care provider but will not require a change in the usual precardiac catheterization orders or medications

12. Which BP finding by the nurse indicates that no changes in therapy are needed for a patient with stage 1 hypertension who has a history of heart failure? a. 108/64 mm Hg b. 128/76 mm Hg c. 140/90 mm Hg d. 136/ 82 mm Hg

ANS: B The goal for antihypertensive therapy for a patient with hypertension and heart failure is a BP of <130/80 mm Hg. The BP of 108/64 may indicate overtreatment of the hypertension and an increased risk for adverse drug effects. The other two blood pressures indicate a need for modifications in the patient's treatment.

When assessing a patient who spilled hot oil on the right leg and foot, the nurse notes that the skin is dry, pale, hard skin. The patient states that the burn is not painful. What term would the nurse use to document the burn depth? a. First-degree skin destruction b. Full-thickness skin destruction c. Deep partial-thickness skin destruction d. Superficial partial-thickness skin destruction

ANS: B With full-thickness skin destruction, the appearance is pale and dry or leathery and the area is painless because of the associated nerve destruction. Erythema, swelling, and blisters point to a deep partial-thickness burn. With superficial partial-thickness burns, the area is red, but no blisters are present. First-degree burns exhibit erythema, blanching, and pain.

A patient with burns covering 40% total body surface area (TBSA) is in the acute phase of burn treatment. Which snack would be best for the nurse to offer to this patient? a. Bananas b. Orange gelatin c. Vanilla milkshake d. Whole grain bagel

ANS: C A patient with a burn injury needs high protein and calorie food intake, and the milkshake is the highest in these nutrients. The other choices are not as nutrient-dense as the milkshake. Gelatin is likely high in sugar. The bagel is a good carbohydrate choice, but low in protein. Bananas are a good source of potassium, but are not high in protein and calories.

A patient has just arrived on the unit after a thyroidectomy. Which action should the nurse take first? a. Observe the dressing for bleeding. b. Check the blood pressure and pulse. c. Assess the patient's respiratory effort. d. Support the patient's head with pillows.

ANS: C Airway obstruction is a possible complication after thyroidectomy because of swelling or bleeding at the site or tetany. The priority nursing action is to assess the airway. The other actions are also part of the standard nursing care postthyroidectomy but are not as high of a priority.

10. Which information will the nurse include when teaching a patient who is scheduled for a radiofrequency catheter ablation for treatment of atrial flutter? a. The procedure will prevent or minimize the risk for sudden cardiac death. b. The procedure will use cold therapy to stop the formation of the flutter waves. c. The procedure will use electrical energy to destroy areas of the conduction system. d. The procedure will stimulate the growth of new conduction pathways between the atria.

ANS: C Radiofrequency catheter ablation therapy uses electrical energy to "burn" or ablate areas of the conduction system as definitive treatment of atrial flutter (i.e., restore normal sinus rhythm) and tachydysrhythmias. All other statements regarding the procedure are incorrect. DIF: Cognitive Level: Apply (application) REF: 805 TOP: Nursing Process: Implementation MSC: NCLEX: Psychosocial Integrity

Which intervention will the nurse include in the plan of care for a patient who has cardiogenic shock? a. Avoid elevating head of bed. b. Check temperature every 2 hours. c. Monitor breath sounds frequently. d. Assess skin for flushing and itching.

ANS: C Since pulmonary congestion and dyspnea are characteristics of cardiogenic shock, the nurse should assess the breath sounds frequently. The head of the bed is usually elevated to decrease dyspnea. Elevated temperature and flushing or itching of the skin are not typical of cardiogenic shock. DIF: Cognitive Level: Application REF: 1721 TOP: Nursing Process: Implementation MSC: NCLEX: Physiological Integrity

Before administering sodium polystyrene sulfonate (Kayexalate) to a patient with hyperkalemia, the nurse should assess the a. blood urea nitrogen (BUN) and creatinine. b. blood glucose level. c. patient's bowel sounds. d. level of consciousness (LOC).

ANS: C Sodium polystyrene sulfonate (Kayexalate) should not be given to a patient with a paralytic ileus (as indicated by absent bowel sounds) because bowel necrosis can occur. The BUN and creatinine, blood glucose, and LOC would not affect the nurse's decision to give the medication.

The public health nurse is planning a program to decrease the incidence of meningitis in adolescents and young adults. Which action is most important? a. Encourage adolescents and young adults to avoid crowds in the winter. b. Vaccinate 11- and 12-year-old children against Haemophilus influenzae. c. Immunize adolescents and college freshman against Neisseria meningitides. d. Emphasize the importance of hand washing to prevent the spread of infection.

ANS: C The Neisseria meningitides vaccination is recommended for children ages 11 and 12, unvaccinated teens entering high school, and college freshmen. Hand washing may help decrease the spread of bacteria, but it is not as effective as immunization. Vaccination with Haemophilus influenzae is for infants and toddlers. Because adolescents and young adults are in school or the workplace, avoiding crowds is not realistic. DIF: Cognitive Level: Apply (application) REF: 1381 | 1383 TOP: Nursing Process: Implementation MSC: NCLEX: Health Promotion and Maintenance

Which finding about a patient with polycythemia vera is most important for the nurse to report to the health care provider? a. Hematocrit 55% b. Presence of plethora c. Calf swelling and pain d. Platelet count 450,000/mL

ANS: C The calf swelling and pain suggest that the patient may have developed a deep vein thrombosis, which will require diagnosis and treatment to avoid complications such as pulmonary embolus. The other findings will also be reported to the health care provider but are expected in a patient with this diagnosis

The nurse is reviewing the laboratory results for newly admitted patients on the cardiovascular unit. Which patient laboratory result is most important to communicate as soon as possible to the health care provider? a. Patient whose triglyceride level is high b. Patient who has very low homocysteine level c. Patient with increase in troponin T and troponin I level d. Patient with elevated high-sensitivity C-reactive protein level

ANS: C The elevation in troponin T and I indicates that the patient has had an acute myocardial infarction. Further assessment and interventions are indicated. The other laboratory results are indicative of increased risk for coronary artery disease but are not associated with acute cardiac problems that need immediate intervention

Which action will the nurse include in the plan of care for a patient who has thalassemia major? a. Teach the patient to use iron supplements. b. Avoid the use of intramuscular injections. c. Administer iron chelation therapy as needed. d. Notify health care provider of hemoglobin 11g/dL.

ANS: C The frequent transfusions used to treat thalassemia major lead to iron toxicity in patients unless iron chelation therapy is consistently used. Iron supplementation is avoided in patients with thalassemia. There is no need to avoid intramuscular injections. The goal for patients with thalassemia major is to maintain a hemoglobin of 10 g/dL or greater

18. In preparation for discharge, the nurse teaches a patient with chronic stable angina how to use the prescribed short-acting and long-acting nitrates. Which patient statement indicates that the teaching has been effective? a. "I will check my pulse rate before I take any nitroglycerin tablets." b. "I will put the nitroglycerin patch on as soon as I get any chest pain." c. "I will stop what I am doing and sit down before I put the nitroglycerin under my tongue." d. "I will be sure to remove the nitroglycerin patch before taking any sublingual nitroglycerin."

ANS: C The patient should sit down before taking the nitroglycerin to decrease cardiac workload and prevent orthostatic hypotension. Transdermal nitrates are used prophylactically rather than to treat acute pain and can be used concurrently with sublingual nitroglycerin. Although the nurse should check blood pressure before giving nitroglycerin, patients do not need to check the pulse rate before taking nitrates.

On admission to the burn unit, a patient with an approximate 25% total body surface area (TBSA) burn has the following initial laboratory results: Hct 58%, Hgb 18.2 mg/dL (172 g/L), serum K+ 4.9 mEq/L (4.8 mmol/L), and serum Na+ 135 mEq/L (135 mmol/L). Which action will the nurse anticipate taking now? a. Monitor urine output every 4 hours. b. Continue to monitor the laboratory results. c. Increase the rate of the ordered IV solution. d. Type and crossmatch for a blood transfusion.

ANS: C The patient's laboratory data show hemoconcentration, which may lead to a decrease in blood flow to the microcirculation unless fluid intake is increased. Because the hematocrit and hemoglobin are elevated, a transfusion is inappropriate, although transfusions may be needed after the emergent phase once the patient's fluid balance has been restored. On admission to a burn unit, the urine output would be monitored more often than every 4 hours; likely every1 hour.

5. A 19-year-old is brought to the emergency department (ED) with multiple lacerations and tissue avulsion of the left hand. When asked about tetanus immunization, the patient denies having any previous vaccinations. The nurse will anticipate giving a. tetanus immunoglobulin (TIG) only. b. TIG and tetanus-diphtheria toxoid (Td). c. tetanus-diphtheria toxoid and pertussis vaccine (Tdap) only. d. TIG and tetanus-diphtheria toxoid and pertussis vaccine (Tdap).

ANS: D For an adult with no previous tetanus immunizations, TIG and Tdap are recommended. The other immunizations are not sufficient for this patient. DIF: Cognitive Level: Apply (application) REF: 1681 TOP: Nursing Process: Planning MSC: NCLEX: Health Promotion and Maintenance

The nurse has identified a nursing diagnosis of acute pain related to inflammatory process for a patient with acute pericarditis. The priority intervention by the nurse for this problem is to a. teach the patient to take deep, slow breaths to control the pain. b. force fluids to 3000 mL/day to decrease fever and inflammation. c. remind the patient to request opioid pain medication every 4 hours. d. place the patient in Fowler's position, leaning forward on the overbed table.

ANS: D Sitting upright and leaning forward frequently will decrease the pain associated with pericarditis. Forcing fluids will not decrease the inflammation or pain. Taking deep breaths will tend to increase pericardial pain. Opioids are not very effective at controlling pain caused by acute inflammatory conditions and are usually ordered PRN. The patient would receive scheduled doses of a nonsteroidal antiinflammatory drug (NSAID).

2. During the primary survey of a patient with severe leg trauma, the nurse observes that the patient's left pedal pulse is absent and the leg is swollen. Which action will the nurse take next? a. Send blood to the lab for a complete blood count. b. Assess further for a cause of the decreased circulation. c. Finish the airway, breathing, circulation, disability survey. d. Start normal saline fluid infusion with a large-bore IV line.

ANS: D The assessment data indicate that the patient may have arterial trauma and hemorrhage. When a possibly life-threatening injury is found during the primary survey, the nurse should immediately start interventions before proceeding with the survey. Although a complete blood count is indicated, administration of IV fluids should be started first. Completion of the primary survey and further assessment should be completed after the IV fluids are initiated. DIF: Cognitive Level: Apply (application) REF: 1676 TOP: Nursing Process: Implementation MSC: NCLEX: Physiological Integrity

While assessing a patient who was admitted with heart failure, the nurse notes that the patient has jugular venous distention (JVD) when lying flat in bed. Which action should the nurse take next? a. Document this finding in the patient's record. b. Obtain vital signs, including oxygen saturation. c. Have the patient perform the Valsalva maneuver. d. Observe for JVD with the patient upright at 45 degrees.

ANS: D When the patient is lying flat, the jugular veins are at the level of the right atrium, so JVD is a common (but not a clinically significant) finding. Obtaining vital signs and oxygen saturation is not warranted at this point. JVD is an expected finding when a patient performs the Valsalva maneuver because right atrial pressure increases. JVD that persists when the patient is sitting at a 30- to 45-degree angle or greater is significant. The nurse will document the JVD in the medical record if it persists when the head is elevated

2. The nurse needs to quickly estimate the heart rate for a patient with a regular heart rhythm. Which method will be best to use? a. Count the number of large squares in the R-R interval and divide by 300. b. Print a 1-minute electrocardiogram (ECG) strip and count the number of QRS complexes. c. Calculate the number of small squares between one QRS complex and the next and divide into 1500. d. Use the 3-second markers to count the number of QRS complexes in 6 seconds and multiply by 10.

ANS: D *(6 second strip x 10)* This is the quickest way to determine the ventricular rate for a patient with a regular rhythm. All the other methods are accurate, but take longer. DIF: Cognitive Level: Apply (application) REF: 789-790 TOP: Nursing Process: Assessment MSC: NCLEX: Physiological Integrity

An 18-yr-old man who fell through the ice on a pond near his farm was admitted to the emergency department with somnolence. Vital signs are blood pressure of 82 mm Hg systolic with Doppler, respirations of 9 breaths/min, and core temperature of 90°F (32.2°C). The nurse should anticipate which intervention? Active core rewarming Immersion in a hot bath Rehydration and massage Passive external rewarming

Active core rewarming Active internal or core rewarming is used for moderate to severe hypothermia and involves the application of heat directly to the core. Immersion in a hot bath, rehydration, and massage are not appropriate interventions in the treatment of severe hypothermia. Passive rewarming is used in mild hypothermia.

A patient who is in the acute phase of recovery from a burn injury has yet to experience adequate pain control. What pain management strategy is most likely to meet this patient's needs? A) A patient-controlled analgesia (PCA) system B) Oral opioids supplemented by NSAIDs C) Distraction and relaxation techniques supplemented by NSAIDs D) A combination of benzodiazepines and topical anesthetics

Ans: A Feedback: The goal of treatment is to provide a long-acting analgesic that will provide even coverage for this long-term discomfort. It is helpful to use escalating doses when initiating the medication to reach the level of pain control that is acceptable to the patient. The use of patient-controlled analgesia (PCA) gives control to the patient and achieves this goal. Patients cannot normally achieve adequate pain control without the use of opioids, and parenteral administration is usually required.

A nurse is teaching a patient with a partial-thickness wound how to wear his elastic pressure garment. How would the nurse instruct the patient to wear this garment? A) 4 to 6 hours a day for 6 months B) During waking hours for 2 to 3 months after the injury C) Continuously D) At night while sleeping for a year after the injury

Ans: C Feedback: Elastic pressure garments are worn continuously (i.e., 23 hours a day).

A 35-year-old female patient with a possible pituitary adenoma is scheduled for a computed tomography (CT) scan with contrast media. Which patient information is most important for the nurse to communicate to the health care provider before the test? a. Bilateral poor peripheral vision b. Allergies to iodine and shellfish c. Recent weight loss of 20 pounds d. Complaint of ongoing headaches

B

When assessing the patient for orthostatic hypotension, after taking the blood pressure (BP) and pulse (P) in the supine position, what should the nurse do next? A Repeat BP and P in this position. B Take BP and P with patient sitting. C Record the BP and P measurements. D Take BP and P with patient standing.

B Take BP and P with patient sitting. When assessing for orthostatic changes in BP after measuring BP in the supine position, the patient is placed in a sitting position and BP is measured within 1 to 2 minutes and then repositioned to the standing position with BP measured again, within 1 to 2 minutes. The results are then recorded with a decrease of 20 mm Hg or more in SBP, a decrease of 10 mm Hg or more in DBP, and/or an increase in pulse of greater than or equal to 20 beats/minute from supine to standing indicating orthostatic hypotension.

During the oliguric phase of AKI, the nurse monitors the patient for (select all that apply): A. hypotension B. ECG changes C. hypernatremia D. pulmonary edema E. urine with high specific gravity

B, D

Vasogenic cerebral edema increases ICP by a. shifting fluid in gray matter b. altering the endothelial lining of cerebral capillaries c. leaking molecules from the intracellular fluid to the capillaries d. altering the osmotic gradient flow into the intravascular component

B. Altering the endothelial lining of cerebral capillaries

The nurse recognizes the presence of Cushing's triad in the patient with a. Increased pulse, irregular respiration, increased BP b. decreased pulse, irregular respiration, increased pulse pressure c. increased pulse, decreased respiration, increased pulse pressure d. decreased pulse, increased respiration, decreased systolic BP

B. Cushing's triad consists of three vital sign measures that reflect ICP and its effect on the medulla, the hypothalamus, the pons, and the thalamus. Because these structures are very deep, Cushing's triad is usually a late sign of ICP. The signs include an increasing systolic BP with a widening pulse pressure, a bradycardia with a full and bounding pulse, and irregular respirations.

To assess the patency of a newly places arteriovenous graft for dialysis, the nurse should: A. irrigate the graft daily with low-dose heparin B. monitor for any increase of BP in the affected arm C. listen with a stethoscope over the graft for presence of a bruit D. frequently monitor the pulses and neurovascular status distal to the graft

C

Which additional information will the nurse need to consider when reviewing the laboratory results for a patient's total calcium level? a. The blood glucose is elevated. b. The phosphate level is normal. c. The serum albumin level is low. d. The magnesium level is normal.

C

The condition of a patient who has cirrhosis of the liver has deteriorated. Which diagnostic study would help determine if the patient has developed liver cancer? A) Serum α-fetoprotein level B) Ventilation/perfusion scan C) Hepatic structure ultrasound D) Abdominal girth measurement

C) Hepatic structure ultrasound, CT, and MRI are used to screen and diagnose liver cancer. Serum α-fetoprotein level may be elevated with liver cancer or other liver problems. Ventilation/perfusion scans do not diagnose liver cancer. Abdominal girth measurement would not differentiate between cirrhosis and liver cancer

Increased ICP in the left cerebral cortex, caused by intracranial bleeding causes displacement of brain tissue to the right hemisphere beneath the falx cerebri. The nurse knows that this is referred to as a. uncal herniation b. tentorial herniation c. cingulate herniation d. temporal lobe herniation

C. Cingulate herniation- the dural structures that separate the two hemispheres and the cerebral hemispheres from the cerebellum influence the patterns of cerebral herniation. A cingulated herniation occurs where there is lateral displacement of brain tissue beneath the falx cerebri.

Postop lap chole: drain

May go home c T-tube No bag: flush daily or BID c NS Bag: drain bag, don't lie on that side

A patient is admitted to the hospital in hypertensive emergency (BP 244/142 mmHg). Sodium nitroprusside is started to treat the elevated BP. Which management strategy(ies) would be appropriate for this patient (select all that apply)? a. Measuring hourly urine output b. Decreasing the MAP by 50% within the first hour c. Continuous BP monitoring with an intraarterial line d. Maintaining bed rest and providing tranquilizers to lower the BP e. Assessing the patient for signs and symptoms of heart failure and changes in mental status

a, c, e

A patient is admitted to the emergency department with first- and second-degree burns after being involved in a house fire. Which of the following assessment findings would alert you to the presence of an inhalation injury (select all that apply)? A.Singed nasal hair B.Generalized pallor C.Painful swallowing D.Burns on the upper extremities E.History of being involved in a large fire

a, c. Reliable clues to the occurrence of inhalation injury is the presence of facial burns, singed nasal hair, hoarseness, painful swallowing, darkened oral and nasal membranes, carbonaceous sputum, history of being burned in an enclosed space, and cherry red skin color

When assessing a pt with a partial thickness burn, the nurse would expect to find (select all that apply) a. blisters b. exposed fascia c. exposed muscles d. intact nerve endings e. red, shiny, wet appearance

a, d, e

Which information will be included when the nurse is teaching self-management to a patient who is receiving peritoneal dialysis (select all that apply)? a. Avoid commercial salt substitutes. b. Drink 1500 to 2000 mL of fluids daily. c. Take phosphate-binders with each meal. d. Choose high-protein foods for most meals. e. Have several servings of dairy products daily.

a,c,d

Which complication of chronic kidney disease is treated with erythropoietin (EPO)? a. Anemia b. Hypertension c. Hyperkalemia d. Mineral and bone disorder

a. Erythropoietin is used to treat anemia, as it stimulates the bone marrow to produce red blood cells.

The nurse conducts a complete physical assessment on a patient admitted with infective endocarditis. Which finding is significant? A. Respiratory rate of 18 and heart rate of 90 B. Regurgitant murmur at the mitral valve area C. Heart rate of 94 and capillary refill time of 2 seconds D. Point of maximal impulse palpable in fourth intercostal space

B. Regurgitant murmur at the mitral valve area A regurgitant murmur of the aortic or mitral valves would indicate valvular disease, which is a complication of endocarditis. All the other findings are within normal limits.

The nurse reviews a patient's glycosylated hemoglobin (Hb A1C) results to evaluate a. fasting preprandial glucose levels. b. glucose levels 2 hours after a meal. c. glucose control over the past 90 days. d. hypoglycemic episodes in the past 3 months.

C

Which statement by a 50-year-old female patient indicates to the nurse that further assessment of thyroid function may be necessary? a. "I notice my breasts are tender lately." b. "I am so thirsty that I drink all day long." c. "I get up several times at night to urinate." d. "I feel a lump in my throat when I swallow."

D

A 54-year-old patient admitted with diabetes mellitus, malnutrition, osteomyelitis, and alcohol abuse has a serum amylase level of 280 U/L and a serum lipase level of 310 U/L. To which of the following diagnoses does the nurse attribute these findings? A. Malnutrition B. Osteomyelitis C. Alcohol abuse D. Diabetes mellitus

C) The patient with alcohol abuse could develop pancreatitis as a complication, which would increase the serum amylase (normal 30-122 U/L) and serum lipase (normal 31-186 U/L) levels as shown.

An 80-year-old patient with uncontrolled type 1 diabetes mellitus is diagnosed with aortic stenosis. When conservative therapy is no longer effective, the nurse knows that the patient will need to do or have what done? A. Aortic valve replacement B. Take nitroglycerin for chest pain. C. Open commissurotomy (valvulotomy) procedure D. Percutaneous transluminal balloon valvuloplasty (PTBV) procedure

D. Percutaneous transluminal balloon valvuloplasty (PTBV) procedure *(percutaneous = safer for ppl c chronic health problems eg poorly controlled DM)*

Which statement by a 62-year-old patient with stage 5 chronic kidney disease (CKD) indicates that the nurse's teaching about management of CKD has been effective? a. "I need to get most of my protein from low-fat dairy products." b. "I will increase my intake of fruits and vegetables to 5 per day." c. "I will measure my urinary output each day to help calculate the amount I can drink." d. "I need to take erythropoietin to boost my immune system and help prevent infection."

c. i will measure my urinary output each day to help calculate the amount I can drink

A patient with diabetes who has bacterial pneumonia is being treated with IV gentamicin (Garamycin) 60 mg IV BID. The nurse will monitor for adverse effects of the medication by evaluating the patient's a. blood glucose. b. urine osmolality. c. serum creatinine. d. serum potassium.

c. serum creatinine

Which assessment finding would the nurse need to report most quickly to the health care provider regarding a patient with acute pancreatitis? a. Nausea and vomiting b. Hypotonic bowel sounds c. Abdominal tenderness and guarding d. Muscle twitching and finger numbness

d. Muscle twitching and finger numbness Muscle twitching and finger numbness indicate hypocalcemia, which may lead to tetany unless calcium gluconate is administered. Although the other findings should also be reported to the health care provider, they do not indicate complications that require rapid action.

transhepatic cholangiography what is it and what do you need to check beforehand?

x-ray c iodinated contrast of bile ducts - contrast introduced through the liver creatinine - iodine is really hard on kidneys

Which continuous renal replacement therapy requires no fluid replacement? 1 Slow continuous ultrafiltration 2 Continuous venovenous hemodialysis 3 Continuous venovenous hemofiltration 4 Continuous venovenous hemodiafiltration

1 Slow continuous ultrafiltration is a simplified version of continuous venovenous hemofiltration. No fluid replacement is required in this process. Continuous venovenous hemodialysis removes both fluids and solutes and requires both dialysate and replacement fluid. Continuous venovenous hemofiltration removes both fluids and solutes and requires replacement fluid. Continuous venovenous hemodiafiltration removes both fluids and solutes and requires both dialysate and replacement fluid. Text Reference - p. 1123

The nurse performs discharge teaching for a patient with an implantable cardioverter-defibrillator (ICD). Which statement by the patient indicates to the nurse that further teaching is needed? "The device may set off the metal detectors in an airport." "My family needs to keep up to date on how to perform CPR." "I should not stand next to antitheft devices at the exit of stores." "I can expect redness and swelling of the incision site for a few days."

"I can expect redness and swelling of the incision site for a few days." Patients should be taught to report any signs of infection at incision site (e.g., redness, swelling, drainage) or fever to their primary care providers immediately. Teach patients to inform TSA airport security of presence of ICD because it may set off metal detectors. If a handheld screening wand is used, it should not be placed directly over the ICD. Teach patients to avoid standing near antitheft devices in doorways of stores and public buildings and to walk through them at a normal pace. Caregivers should learn cardiopulmonary resuscitation.

Which condition should the nurse suspect in a patient with chronic kidney disease (CKD) who develops osteomalacia? 1 Asterixis 2 Uremic frost 3 Gastroparesis 4 Uremic red eye

4 Chronic kidney disease mineral and bone disorder (CKD-MBD) is a common complication of CKD and results in both skeletal and extraskeletal complications. Osteomalacia is a skeletal complication. Calcium deposition in the eye may create irritation leading to uremic red eye, an extraskeletal complication. Asterixis (hand-flapping tremor) occurs due to motor neuropathy. Uremic frost is the crystallization of urea on the skin when blood urea nitrogen levels are elevated to 200 mg/dL. Gastroparesis (delayed gastric emptying) compounds the effect of malnutrition for patients with diabetes. Text Reference - p. 1111

25. Which action will the nurse include in the plan of care for a patient who was admitted with syncopal episodes of unknown origin? a. Instruct the patient to call for assistance before getting out of bed. b. Explain the association between various dysrhythmias and syncope. c. Educate the patient about the need to avoid caffeine and other stimulants. d. Tell the patient about the benefits of implantable cardioverter-defibrillators.

ANS: A A patient with fainting episodes is at risk for falls. The nurse will plan to minimize the risk by having assistance whenever the patient up. The other actions may be needed if dysrhythmias are found to be the cause of the patient's syncope, but are not appropriate for syncope of unknown origin. DIF: Cognitive Level: Apply (application) REF: 807 TOP: Nursing Process: Planning MSC: NCLEX: Physiological Integrity

42. Which information obtained by the nurse about a 29-year-old patient with a lumbar vertebral compression fracture is most important to report to the health care provider? a. Patient refuses to be turned due to back pain. b. Patient has been incontinent of urine and stool. c. Patient reports lumbar area tenderness to palpation. d. Patient frequently uses oral corticosteroids to treat asthma.

ANS: B Changes in bowel or bladder function indicate possible spinal cord compression and should be reported immediately because surgical intervention may be needed. The other findings are also pertinent but are consistent with the patient's diagnosis and do not require immediate intervention

39. When assessing for Tinel's sign in a patient with possible right-sided carpal tunnel syndrome, the nurse will ask the patient about a. weakness in the right little finger. b. tingling in the right thumb and fingers. c. burning in the right elbow and forearm. d. tremor when gripping with the right hand.

ANS: B Testing for Tinel's sign will cause tingling in the thumb and first three fingers of the affected hand in patients who have carpal tunnel syndrome. The median nerve does not innervate the right little finger or elbow and forearm. Tremor is not associated with carpal tunnel syndrome.

A patient with pancytopenia of unknown origin is scheduled for the following diagnostic tests. The nurse will provide a consent form to sign for which test? a. ABO blood typing b. Bone marrow biopsy c. Abdominal ultrasound d. Complete blood count (CBC)

ANS: B A bone marrow biopsy is a minor surgical procedure that requires the patient or guardian to sign a surgical consent form. The other procedures do not require a signed consent by the patient or guardian.

1. When teaching seniors at a community recreation center, which information will the nurse include about ways to prevent fractures? a. Tack down scatter rugs in the home. b. Most falls happen outside the home. c. Buy shoes that provide good support and are comfortable to wear. d. Range-of-motion exercises should be taught by a physical therapist.

ANS: C Comfortable shoes with good support will help decrease the risk for falls. Scatter rugs should be eliminated, not just tacked down. Activities of daily living provide range of motion exercise; these do not need to be taught by a physical therapist. Falls inside the home are responsible for many injuries.

25. A patient who slipped and fell in the shower at home has a proximal humerus fracture immobilized with a left-sided long-arm cast and a sling. Which nursing intervention will be included in the plan of care? a. Use surgical net dressing to hang the arm from an IV pole. b. Immobilize the fingers of the left hand with gauze dressings. c. Assess the left axilla and change absorbent dressings as needed. d. Assist the patient in passive range of motion (ROM) for the right arm.

ANS: C The axilla can become excoriated when a sling is used to support the arm, and the nurse should check the axilla and apply absorbent dressings to prevent this. A patient with a sling would not have traction applied by hanging. The patient will be encouraged to move the fingers on the injured arm to maintain function and to help decrease swelling. The patient will do active ROM on the uninjured side.

The health care provider's progress note for a patient states that the complete blood count (CBC) shows a "shift to the left." Which assessment finding will the nurse expect? a. Cool extremities b. Pallor and weakness c. Elevated temperature d. Low oxygen saturation

ANS: C The term shift to the left indicates that the number of immature polymorphonuclear neutrophils (bands) is elevated and that finding is a sign of infection. There is no indication that the patient is at risk for hypoxemia, pallor/weakness, or cool extremities.

Which patient information is most important for the nurse to monitor when evaluating the effectiveness of deferoxamine (Desferal) for a patient with hemochromatosis? a. Skin color b. Hematocrit c. Liver function d. Serum iron level

ANS: D Because iron chelating agents are used to lower serum iron levels, the most useful information will be the patient's iron level. The other parameters will also be monitored, but are not the most important to monitor when determining the effectiveness of deferoxamine

Which type of shock causes an absence of bowel sounds? 1 Cardiogenic shock 2 Neurogenic shock 3 Hypovolemic shock 4 Anaphylactic shock

Absence of bowel sounds is associated with hypovolemic shock. Decreased bowel sounds are seen with cardiogenic shock. Bowel dysfunction is associated with neurogenic shock. Abdominal pain, nausea and vomiting are seen with anaphylactic shock. Test-Taking Tip: Identifying content and what is being asked about that content is critical to your choosing the correct response. Be alert for words in the stem of the item that are the same or similar in nature to those in one or two of the options. Text Reference - p. 1635

A nurse on a burn unit is caring for a patient in the acute phase of burn care. While performing an assessment during this phase of burn care, the nurse recognizes that airway obstruction related to upper airway edema may occur up to how long after the burn injury? A) 2 days B) 3 days C) 5 days D) 1 week

Ans: A Feedback: Airway obstruction caused by upper airway edema can take as long as 48 hours to develop. Changes detected by x-ray and arterial blood gases may occur as the effects of resuscitative fluid and the chemical reaction of smoke ingredients with lung tissues become apparent.

The nurse is completing discharge teaching with an 80-yr-old male patient who is recovering from a right total hip arthroplasty by posterior approach. Which patient action indicates further instruction is needed? Uses an elevated toilet seat Sits with feet flat on the floor Maintains hip in adduction and internal rotation Verifies need to notify future caregivers about the prosthesis

Maintains hip in adduction and internal rotation The patient should not force hip into adduction or internal rotation because these movements could dislocate the hip prosthesis. Sitting with feet flat on the floor (avoiding crossing the legs), using an elevated toilet seat, and notifying future caregivers about the prosthesis indicate understanding of discharge teaching.

The nurse is caring for a patient placed in Buck's traction before open reduction and internal fixation of a left hip fracture. Which care can be delegated to the LPN/LVN? Assess skin integrity around the traction boot. Determine correct body alignment to enhance traction. Remove weights from traction when turning the patient. Monitor pain intensity and administer prescribed analgesics.

Monitor pain intensity and administer prescribed analgesics. The LPN/LVN can monitor pain intensity and administer analgesics. Assessment of skin integrity and determining correct alignment to enhance traction are within the RN scope of practice. Removing weights from the traction should not be delegated or done. Removal of weights can cause muscle spasms and bone misalignment, and should not be delegated or done.

The oxygen saturation (SpO2) for a patient with left lower lobe pneumonia is 90%. The patient has rhonchi, a weak cough effort, and complains of fatigue. Which action is a priority for the nurse to take? a. Position the patient on the left side. b. Assist the patient with staged coughing. c. Place a humidifier in the patient's room. d. Schedule a 2-hour rest period for the patient.

b. assist the patient with staged coughing

The nurse will ask a 64-year-old patient being admitted with acute pancreatitis specifically about a history of a. diabetes mellitus. b. high-protein diet. c. cigarette smoking. d. alcohol consumption.

d. alcohol consumption. Alcohol use is one of the most common risk factors for pancreatitis in the United States. Cigarette smoking, diabetes, and high-protein diets are not risk factors.

The injury that is least likely to result in a full thickness burn is: a. sunburn b. scald injury c. chemical burn d. electrical injury

a.

A patient has a core temperature of 90F. The most appropriate rewarming technique would be a. passive rewarming with warm blankets b. active internal rewarming using IV fluids c. passive rewarming using air-filled warming blankets d. active external rewarming by submersing in a warm bath

b. active internal rewarming using IV fluids

Knowing the most common causes of household fires, which of the following prevention strategies would the nurse focus on when teaching about fire safety? a. set hot water temp at 140 F b. use only hardwired smoke detectors c. encourage regular home fire exit drills d. never permit older adults to cook unattended.

c.

Which laboratory test result will the nurse monitor when evaluating the effects of therapy for a 62-year-old female patient who has acute pancreatitis? a. Calcium b. Bilirubin c. Amylase d. Potassium

c. Amylase Amylase is elevated in acute pancreatitis. Although changes in the other values may occur, they would not be useful in evaluating whether the prescribed therapies have been effective.

The charge nurse observes an inexperienced staff nurse who is caring for a patient who has had a craniotomy for a brain tumor. Which action by the inexperienced nurse requires the charge nurse to intervene? a. The staff nurse suctions the patient every 2 hours. b. The staff nurse assesses neurologic status every hour. c. The staff nurse elevates the head of the bed to 30 degrees. d. The staff nurse administers a mild analgesic before turning the patient.

ANS: A Suctioning increases intracranial pressure and is done only when the patient's respiratory condition indicates it is needed. The other actions by the staff nurse are appropriate. DIF: Cognitive Level: Application REF: 1430-1431

While admitting a patient with pericarditis, the nurse will assess for what manifestations of this disorder? A. Pulsus paradoxus B. Prolonged PR intervals C. Widened pulse pressure D. Clubbing of the fingers

A. Pulsus paradoxus Pericarditis can lead to cardiac tamponade, an emergency situation. Pulsus paradoxus greater than 10 mm Hg is a sign of cardiac tamponade that should be assessed at least every 4 hours in a patient with pericarditis. Prolonged PR intervals occur with first-degree AV block. Widened pulse pressure occurs with valvular heart disease. Clubbing of fingers may occur in subacute forms of infective endocarditis and valvular heart disease.

A nurse is caring for a patient in the emergent/resuscitative phase of burn injury. During this phase, the nurse should monitor for evidence of what alteration in laboratory values? A) Sodium deficit B) Decreased prothrombin time (PT) C) Potassium deficit D) Decreased hematocrit

ANS: A Feedback: Anticipated fluid and electrolyte changes that occur during the emergent/resuscitative phase of burn injury include sodium deficit, potassium excess, base-bicarbonate deficit, and elevated hematocrit. PT does not typically decrease.

The nurse identifies the nursing diagnosis of decreased cardiac output related to valvular insufficiency for the patient with infective endocarditis (IE) based on which assessment finding(s)? a. Fever, chills, and diaphoresis b. Urine output less than 30 mL/hr c. Petechiae on the inside of the mouth and conjunctiva d. Increase in heart rate of 15 beats/minute with walking

ANS: B Decreased renal perfusion caused by inadequate cardiac output will lead to decreased urine output. Petechiae, fever, chills, and diaphoresis are symptoms of IE, but are not caused by decreased cardiac output. An increase in pulse rate of 15 beats/minute is normal with exercise.

Which assessment finding in a patient who is admitted with infective endocarditis (IE) is most important to communicate to the health care provider? a. Generalized muscle aching b. Sudden onset right flank pain c. Janeway's lesions on the palms d. Temperature 100.7° F (38.1° C)

ANS: B Sudden onset of flank pain indicates possible embolization to the kidney and may require diagnostic testing such as a renal arteriogram and interventions to improve renal perfusion. The other findings are typically found in IE, but do not require any new interventions.

The nurse admitting a patient who has a right frontal lobe tumor would expect the patient may have a. expressive aphasia. b. impaired judgment. c. right-sided weakness. d. difficulty swallowing.

ANS: B The frontal lobe controls intellectual activities such as judgment. Speech is controlled in the parietal lobe. Weakness and hemiplegia occur on the contralateral side from the tumor. Swallowing is controlled by the brainstem. DIF: Cognitive Level: Apply (application) REF: 1376 TOP: Nursing Process: Assessment MSC: NCLEX: Physiological Integrity

13. When titrating IV nitroglycerin (Tridil) for a patient with a myocardial infarction (MI), which action will the nurse take to evaluate the effectiveness of the medication? a. Monitor heart rate. b. Ask about chest pain. c. Check blood pressure. d. Observe for dysrhythmias.

ANS: B The goal of IV nitroglycerin administration in MI is relief of chest pain by improving the balance between myocardial oxygen supply and demand. The nurse also will monitor heart rate and blood pressure (BP) and observe for dysrhythmias, but these parameters will not indicate whether the medication is effective.

A patient's complete blood count (CBC) shows a hemoglobin of 19 g/dL and a hematocrit of 54%. Which question should the nurse ask to determine possible causes of this finding? a. "Have you had a recent weight loss?" b. "Do you have any history of lung disease?" c. "Have you noticed any dark or bloody stools?" d. "What is your dietary intake of meats and protein?"

ANS: B The hemoglobin and hematocrit results indicate polycythemia, which can be associated with chronic obstructive pulmonary disease (COPD). The other questions would be appropriate for patients who are anemic.

A patient recovering from heart surgery develops pericarditis and complains of level 6 (0 to 10 scale) chest pain with deep breathing. Which ordered PRN medication will be the most appropriate for the nurse to give? a. Fentanyl 1 mg IV b. IV morphine sulfate 4 mg c. Oral ibuprofen (Motrin) 600 mg d. Oral acetaminophen (Tylenol) 650 mg

ANS: C The pain associated with pericarditis is caused by inflammation, so nonsteroidal antiinflammatory drugs (NSAIDs) (e.g., ibuprofen) are most effective. Opioid analgesics are usually not used for the pain associated with pericarditis. *-itis = inflammation, so they get NSAIDs*

Postop lap chole care

Comfort, monitor for bleeds, prepare for discharge Common complaint: referred shoulder pain d/t CO2 used to inflate abd cavity. Can irritate diaphragm -> dyspnea L lateral Sims' can help move gas pocket. Deep breathe, move around. NSAIDs or codeine prn.

A patient is brought to the ED by paramedics, who report that the patient has partial-thickness burns on the chest and legs. The patient has also suffered smoke inhalation. What is the priority in the care of a patient who has been burned and suffered smoke inhalation? A) Pain B) Fluid balance C) Anxiety and fear D) Airway management

Ans: D Feedback: Systemic threats from a burn are the greatest threat to life. The ABCs of all trauma care apply during the early postburn period. While all options should be addressed, pain, fluid balance, and anxiety and fear do not take precedence over airway management.

The nurse is attending to a patient who is receiving hemodialysis for chronic kidney disease. For which complications should the nurse be observant in the patient? Select all that apply. 1 Hypotension 2 Renal calculi 3 Hepatitis type B 4 Bladder infection 5 Muscle cramps

1, 3, 5 The patient on hemodialysis may have decreased blood pressure due to rapid removal of blood. Hepatitis type B is a blood-borne infection, and hemodialysis poses a high risk for transmission of hepatitis B. Muscle cramps are a common complication of hemodialysis. Factors associated with the development of muscle cramps in hemodialysis include hypotension, hypovolemia, a high ultrafiltration rate (large interdialytic weight gain), and low-sodium dialysis solution. Hemodialysis does not increase the risk of development of renal calculi; people who are on bed rest or have low urine output may be at risk. Bladder infection is not related to dialysis. Text Reference - p. 1122

The nurse receives a physician's order to transfuse fresh frozen plasma to a patient with acute blood loss. Which procedure is most appropriate for infusing this blood product? A. Infuse the fresh frozen plasma as rapidly as the patient will tolerate. B. Hang the fresh frozen plasma as a piggyback to the primary IV solution. C. Infuse the fresh frozen plasma as a piggyback to a primary solution of normal saline. D. Hang the fresh frozen plasma as a piggyback to a new bag of primary IV solution without KCl.

A. Infuse the fresh frozen plasma as rapidly as the patient will tolerate. The fresh frozen plasma should be administered as rapidly as possible and should be used within 24 hours of thawing to avoid a decrease in factors V and VIII. Fresh frozen plasma is infused using any straight-line infusion set. Any existing IV should be interrupted while the fresh frozen plasma is infused, unless a second IV line has been started for the transfusion.

When caring for a patient with infective endocarditis, the nurse will assess the patient for which vascular manifestations (select all that apply)? A. Osler's nodes B. Janeway's lesions C. Splinter hemorrhages D. Subcutaneous nodules E. Erythema marginatum lesions

A. Osler's nodes B. Janeway's lesions C. Splinter hemorrhages Osler's nodes, Janeway's lesions, and splinter hemorrhages are all vascular manifestations of infective endocarditis. Subcutaneous nodules and erythema marginatum lesions occur with rheumatic fever.

Medication risk for cholelithiasis

OCPs in younger women, and HRT in postmenopausal

When planning emergent care for a patient with a suspected myocardial infarction (MI), what should the nurse anticipate administrating? Oxygen, nitroglycerin, aspirin, and morphine Aspirin, nitroprusside, dopamine, and oxygen Oxygen, furosemide (Lasix), nitroglycerin, and meperidine Nitroglycerin, lorazepam (Ativan), oxygen, and warfarin (Coumadin)

Oxygen, nitroglycerin, aspirin, and morphine The American Heart Association's guidelines for emergency care of the patient with chest pain include the administration of oxygen, nitroglycerin, aspirin, and morphine. These interventions serve to relieve chest pain, improve oxygenation, decrease myocardial workload, and prevent further platelet aggregation. The other medications may be used later in the patient's treatment.

A 64-yr-old woman is admitted to the emergency department vomiting bright red blood. The patient's vital signs are blood pressure of 78/58 mm Hg, pulse of 124 beats/min, respirations of 28 breaths/min, and temperature of 97.2°F (36.2°C). Which physician order should the nurse complete first? Obtain a 12-lead ECG and arterial blood gases. Rapidly administer 1000 mL normal saline solution IV. Administer norepinephrine (Levophed) by continuous IV infusion. Carefully insert a nasogastric tube and an indwelling bladder catheter.

Rapidly administer 1000 mL normal saline solution IV. Isotonic crystalloids, such as normal saline solution, should be used in the initial resuscitation of hypovolemic shock. Vasopressor drugs (e.g., norepinephrine) may be considered if the patient does not respond to fluid resuscitation and blood products. Other orders (e.g., insertion of nasogastric tube and indwelling bladder catheter and obtaining the diagnostic studies) can be initiated after fluid resuscitation is initiated.

A pt in the emergent phase of burn care for thermal burns on 20% of the total body surface area is unconscious. Which assessment data is the most important for the nurse's evaluation of the pt's injuries? a. condition of the oropharynx b. percentage of TBSA affected c. location of the pt in the fire d. comorbidities of the pt

a. the pt is likely to have suffered a smoke inhalation injury because thermal burns are caused by flames that emit smoke and because the pt is unconscious.

An 83-year-old female patient was found lying on the bathroom floor. She said she fell 2 days ago and has not been able to take her heart medicine or eat or drink anything since then. What conditions could be causing prerenal AKI in this patient (select all that apply)? a. Anaphylaxis b. Renal calculi c. Hypovolemia d. Nephrotoxic drugs e. Decreased cardiac output

c, e. Because the patient has had nothing to eat or drink for 2 days, she is probably dehydrated and hypovolemic. Decreased cardiac output (CO) is most likely because she is older and takes heart medicine, which is probably for heart failure or hypertension. Both hypovolemia and decreased CO cause prerenal AKI. Anaphylaxis is also a cause of prerenal AKI but is not likely in this situation. Nephrotoxic drugs would contribute to intrarenal causes of AKI and renal calculi would be a postrenal cause of AKI.

When monitoring initial fluid replacement for the patient with 40% TBSA deep partial-thickness and full-thickness burns, which finding is of most concern to the nurse? a) Serum K+ of 4.5 mEq/L b) Urine output of 35 mL/hr c) Decreased bowel sounds d) Blood pressure of 86/72 mm Hg

d Rationale: Adequacy of fluid replacement is assessed by urine output and cardiac parameters. Urine output should be 0.5 to 1 mL/kg/hr. Mean arterial pressure should be >65 mm Hg, systolic BP >90 mm Hg, and heart rate <120 beats/min. A blood pressure of 86/72 indicates inadequate fluid replacement. However, the MAP is calculated at 77 mm Hg.

The nurse completes an admission history for a 73-yr-old man with osteoarthritis scheduled for total knee arthroplasty. Which response is expected when asking the patient the reason for admission? Recent knee trauma Debilitating joint pain Repeated knee infections Onset of frozen knee joint

Debilitating joint pain The most common reason for knee arthroplasty is debilitating joint pain despite exercise, weight management, and drug therapy. Recent knee trauma, repeated knee infections, and onset of frozen knee joint are not primary indicators for a knee arthroplasty.

The nurse is caring for a 72-yr-old man in cardiogenic shock after an acute myocardial infarction. Which clinical manifestations would be most concerning? Restlessness, heart rate of 124 beats/min, and hypoactive bowel sounds Mean arterial pressure of 54 mm Hg; increased jaundice; and cold, clammy skin PaO2 of 38 mm Hg, serum lactate level of 46.5 mcg/dL, and puncture site bleeding Agitation, respiratory rate of 32 breaths/min, and serum creatinine of 2.6 mg/dL

PaO2 of 38 mm Hg, serum lactate level of 46.5 mcg/dL, and puncture site bleeding Severe hypoxemia, lactic acidosis, and bleeding are clinical manifestations of the irreversible state of shock. Recovery from this stage is not likely because of multiple organ system failure. Restlessness, tachycardia, and hypoactive bowel sounds are clinical manifestations that occur during the compensatory stage of shock. Decreased mean arterial pressure, jaundice, cold and clammy skin, agitation, tachypnea, and increased serum creatinine are clinical manifestations of the progressive stage of shock.

To maintain a positive nitrogen balance in a major burn, the patient must a. increase normal caloric intake by about 3 times b. eat a high-protein, low-fat, high-carbohydrate diet. c. eat at least 1500 calories per day in small, frequent meals. d. eat rice and whole wheat for the chemical effect on nitrogen balance

b. eat a high-protein, low-fat, high-carbohydrate diet. The patient should be encouraged to eat high-protein, high-carbohydrate foods to meet increased caloric needs. Massive catabolism can occur and is characterized by protein breakdown and increased gluconeogenesis. Failure to supply adequate calories and protein leads to malnutrition and delayed healing.

A patient with chronic obstructive pulmonary disease (COPD) arrives in the emergency department complaining of shortness of breath and dyspnea on minimal exertion. Which assessment finding by the nurse is most important to report to the health care provider? a. The patient has bibasilar lung crackles. b. The patient is sitting in the tripod position. c. The patient's respirations have decreased from 30 to 10 breaths/minute. d. The patient's pulse oximetry indicates an O2 saturation of 91%.

c. the patient's respirations have decreased from 30 to 10 breaths/minute

The nurse is caring for a patient with partial- and full-thickness burns to 65% of the body. When planning nutritional interventions for this patient, the nurse should implement which of the following dietary choices? A. Full liquids only B. Whatever the patient requests C. High-protein and low-sodium foods D. High-calorie and high-protein foods

d. A hypermetabolic state occurs proportional to the size of the burn area. Massive catabolism can occur and is characterized by protein breakdown and increases gluconeogenesis. Caloric needs are often in the 5000-kcal range. Failure to supply adequate calories and protein leads to malnutrition and delayed healing.

What indicates to the nurse that a patient with AKI is in the recovery phase? a. A return to normal weight b. A urine output of 3700 mL/day c. Decreasing sodium and potassium levels d. Decreasing blood urea nitrogen (BUN) and creatinine levels

d. The blood urea nitrogen (BUN) and creatinine levels remain high during the oliguric and diuretic phases of AKI. The recovery phase begins when the glomerular filtration returns to a rate at which BUN and creatinine stabilize and then decrease. Urinary output of 3 to 5 L/ day, decreasing sodium and potassium levels, and fluid weight loss are characteristic of the diuretic phase of AKI.

A nurse is caring for a patient with ARDS who is being treated with mechanical ventilation and high levels of positive end-expiratory pressure (PEEP). Which assessment finding by the nurse indicates that the PEEP may need to be reduced? a. The patient's PaO2 is 50 mm Hg and the SaO2 is 88%. b. The patient has subcutaneous emphysema on the upper thorax. c. The patient has bronchial breath sounds in both the lung fields. d. The patient has a first-degree atrioventricular heart block with a rate of 58.

b. the patient has subcutaneous emphysema on the upper thorax

Which intervention is most likely to prevent or limit barotrauma in the pt c ARDS who is mechanically ventilated a. Decreasing PEEP b. Increasing tidal volume c. Use of permissive hypercapnia d. Use of PPV

c. Use of permissive hypercapnia - Decreasing TV can help prevent barotrauma by dec peak airway pressures but this can mean allowing higher CO2 levels. Hypercapnia isn't awesome but it won't kill you like hypoxemia will.

The nurse manager is observing a new nursing graduate caring for a burn client in protective isolation. The nurse manager intervenes if the new nursing graduate planned to implement which incorrect component of protective isolation technique? a. using sterile sheets and linens b. performing strict hand washing technique c. wearing gloves and gown only when giving direct care to the client d. wearing protective garb, including a mask, gloves, cap, shoe covers, gowns, and plastic apron

c. ppe should be worn whenever entering the client's room

Fluid and electrolyte shifts that occur during the early emergent phase include a. adherence of albumin to vascular walls b. movement of potassium into the vascular space c. sequestering of sodium and water in the interstitial fluid. d. hemolysis of red blood cells from large volumes of rapidly administered fluid.

c. sequestering of sodium and water in the interstitial fluid.

why do gallstones hurt?

stones migrating to cystic or common bile duct cause pain can lodge and obstruct

post chole - discharge and activity?

- d/c same-day or day after sx - don't forget to TCDB - start c clear liquids - may resume normal activities in a week

Teaching for pt who has not had cholecystectomy yet:

- low fat diet - fat soluble vitamin supplements - monitor for changes in appearance of urine/stool - monitor for s/s jaundice, pruritus - importance of continued medical followup

who is more likely to get cholecystitis without obstruction

(AKA acalculous cholecystitis) - older adults - really ill people - immobile people - people who are fasting - being on TPN for a long time - DM Basically it's the pneumonia of the biliary area

Acute care of cholecystitis/lithiasis:

- Aggressive pain management - Comfort measures - clean bed, positioning, oral care - I&Os, maintain suction as ordered - Oral care c N/V, remove emesis from pt view - Skin care prn pruritus

Nutrition considerations: prevention

- Small frequent meals c some fat @ each to promote gallbladder emptying - Lose weight prn, but losing weight TOO fast can promote stone formation - Eat plant fats, not animal fats

Post chole meds - long term

- fat soluble vitamins - bile salts - Questran (mix c milk, juice - just NOT carbonated) - medical dissolution therapy (UDCA, Actigall, CDCA)

contraindications for laparoscopic chole include

- infection: peritonitis, cholangitis, gangrene - perforation of gallbladder - portal HTN - serious bleeding disorder

bile obstruction s/s

- jaundice - dark, foamy urine - clay colored poop - pruritus (bile salts deposited in skin) - unable to tolerate fatty food (unable to digest) - tendency to bleed - fatty, greasy stool (undigested fat) - fever, chills, cholangitis (infection)

post open chole care

- ventilation and respiratory - T-tube care - patency, drainage - hospital for 2-3 days - activity restriction 4-6 weeks

Which statement made by a nursing student indicates effective learning about what should be included on a plan of care for a patient with chronic kidney disease that is taking ferrous gluconate and calcium acetate? 1 Administer a stool softener. 2 Obtain consent for immediate dialysis. 3 Give both drugs at the same time. 4 Administer sodium polystyrene sulfonate

1 A patient with chronic kidney disease who is taking oral iron salts, such as ferrous gluconate, and phosphate binders, such calcium acetate, may develop constipation and need to take a stool softener. Dialysis does not provide relief from constipation in patients with chronic kidney disease. Oral iron supplements should not be given at the same time as calcium-containing phosphate binders because they prevent iron absorption. Sodium polystyrene sulfonate helps to treat hyperkalemia but does not provide relief from constipation. Text Reference - p. 1110

Which finding indicates oliguria? 1 Urinary output of 350 mL/day 2 Urinary output of 450 mL/day 3 Urinary output of 550 mL/day 4 Urinary output of 650 mL/day

1 A urinary output rate of less than 400 mL/day indicates oliguria; thus a urinary output of 350 mL/day suggests oliguria. Urine outputs of 450, 550, or 650 mL/day are considered normal. Test-Taking Tip: Have confidence in your initial response to an item because it more than likely is the correct answer. Text Reference - p. 1103

11. A patient is brought to the emergency department with an injured lower left leg following a fall while rock climbing. The nurse identifies the presence of a fracture based on what cardinal sign of fracture? a. Muscle spasms b. Obvious deformity c. Edema and swelling d. Pain and tenderness

11. b. Deformity is the cardinal sign of fracture but may not be apparent in all fractures. Other supporting signs include edema and swelling, localized pain and tenderness, muscle spasm, ecchymosis, loss of function, crepitation, and an inability to bear weight.

42. Following a knee arthroplasty, a patient has a continuous passive motion machine for the affected joint. The nurse explains to the patient that this device is used for what purpose? a. To relieve edema and pain at the incision site b. To promote early joint mobility and increase knee flexion c. To prevent venous stasis and the formation of a deep venous thrombosis d. To improve arterial circulation to the affected extremity to promote healing

42. b. Continuous passive motion machines are frequently used following knee surgery to promote earlier joint mobility. Because joint dislocation is not a problem with knee replacements, early exercise with straight leg raises and gentle ROM is also encouraged postoperatively.

A 22-year-old patient is being seen in the clinic with increased secretion of the anterior pituitary hormones. The nurse would expect the laboratory results to show a. increased urinary cortisol. b. decreased serum thyroxine. c. elevated serum aldosterone levels. d. low urinary catecholamines excretion.

A

Patients with chronic kidney disease experience an increased incidence of cardiovascular disease related to (select all that apply): A. hypertension B. vascular calcifications C. a genetic predisposition D. hyperinsulinemia causing dyslipiemia E. increased high-density lipoprotein levels

A, B, D, E

The patient is being dismissed from the hospital after acute coronary syndrome and will be attending rehabilitation. What information would be taught in the early recovery phase of rehabilitation? Therapeutic lifestyle changes should become lifelong habits. Physical activity is always started in the hospital and continued at home. Attention will focus on management of chest pain, anxiety, dysrhythmias, and other complications. Activity level is gradually increased under cardiac rehabilitation team supervision and with electrocardiographic (ECG) monitoring.

Activity level is gradually increased under cardiac rehabilitation team supervision and with electrocardiographic (ECG) monitoring. In the early recovery phase after the patient is dismissed from the hospital, the activity level is gradually increased under supervision and with ECG monitoring. The late recovery phase includes therapeutic lifestyle changes that become lifelong habits. In the first phase of recovery, activity is dependent on the severity of the angina or myocardial infarction, and attention is focused on the management of chest pain, anxiety, dysrhythmias, and other complications. With early recovery phase, the cardiac rehabilitation team may suggest that physical activity be initiated at home, but this is not always done.

A patient is in acute respiratory distress syndrome (ARDS) as a result of sepsis. Which measure would be implemented to maintain cardiac output? Administer crystalloid fluids. Position the patient in the Trendelenburg position. Place the patient on fluid restriction and administer diuretics. Perform chest physiotherapy and assist with staged coughing.

Administer crystalloid fluids. Low cardiac output may necessitate crystalloid fluids in addition to lowering positive end-expiratory pressure (PEEP) or administering inotropes. The Trendelenburg position (not recommended to treat hypotension) and chest physiotherapy are unlikely to relieve decreased cardiac output, and fluid restriction and diuresis would be inappropriate interventions.

When providing discharge teaching for the patient after a laparoscopic cholecystectomy, what information should the nurse include? A) A lower-fat diet may be better tolerated for several weeks. B) Do not return to work or normal activities for 3 weeks. C) Bile-colored drainage will probably drain from the incision. D) Keep the bandages on and the puncture site dry until it heals.

Although the usual diet can be resumed, a low-fat diet is usually better tolerated for several weeks following surgery. Normal activities can be gradually resumed as the patient tolerates. Bile-colored drainage or pus, redness, swelling, severe pain, and fever may all indicate infection. The bandage may be removed the day after surgery, and the patient can shower.

A mass casualty incident was identified on a nearby freeway. Which patient would likely be designated "red" during triage at the site? An individual who is distraught at the violence of the incident An individual who has experienced an open arm fracture from falling debris An individual who is not expected to survive a crushing head and neck wound An individual whose femoral artery has been severed and is bleeding profusely

An individual whose femoral artery has been severed and is bleeding profusely Red indicates a life-threatening injury requiring immediate intervention, such as severe bleeding. Emotional trauma would not warrant a "red" designation, and a fracture would likely be deemed "yellow," urgent but not life threatening. Those not expected to survive are categorized "blue." "Black" identifies the dead.

A burn patient is transitioning from the acute phase of the injury to the rehabilitation phase. The patient tells the nurse, ìI can't wait to have surgery to reconstruct my face so I look normal again.î What would be the nurse's best response? A) That's something that you and your doctor will likely talk about after your scars mature. B) That is something for you to talk to your doctor about because it's not a nursing responsibility. C) I know this is really important to you, but you have to realize that no one can make you look like you used to. D) Unfortunately, it's likely that you will have most of these scars for the rest of your life.

Ans: A Feedback: Burn reconstruction is a treatment option after all scars have matured and is discussed within the first few years after injury. Even though this is not a nursing responsibility, the nurse should still respond appropriately to the patient's query. It is true that the patient will not realistically look like he or she used to, but this does not instill hope.

A 78-year-old patient has Stage 3 CKD and is being taught about a low potassium diet. The nurse knows the patient understands the diet when the patient selects which foods to eat? A.Apple, green beans, and a roast beef sandwich B.Granola made with dried fruits, nuts, and seeds C.Watermelon and ice cream with chocolate sauce D.Bran cereal with ½ banana and milk and orange juice

Apple, green beans, and a roast beef sandwich When the patient selects an apple, green beans, and a roast beef sandwich, the patient demonstrates understanding of the low potassium diet. Granola, dried fruits, nuts and seeds, milk products, chocolate sauce, bran cereal, banana, and orange juice all have elevated levels of potassium, at or above 200 mg per 1/2 cup.

If a patient is in the diuretic phase of AKI, the nurse must monitor for which serum electrolyte imbalance? A. hyperkalemia and hyponatremia B. hyperkalemia and hypernatremia C. hypokalemia and hyponatremia D. hypokalemia and hypernatremia

C

A patient with cholelithiasis needs to have the gallbladder removed. Which patient assessment is a contraindication for a cholecystectomy? A) Low-grade fever of 100° F and dehydration B) Abscess in the right upper quadrant of the abdomen C) Activated partial thromboplastin time (aPTT) of 54 seconds D) Multiple obstructions in the cystic and common bile duct

C) An aPTT of 54 seconds is above normal and indicates insufficient clotting ability. If the patient had surgery, significant bleeding complications postoperatively are very likely. Fluids can be given to eliminate the dehydration; the abscess can be assessed, and the obstructions in the cystic and common bile duct would be relieved with the cholecystectomy.

10. The patient with increased ICP from a brain tumor is being monitored with a ventriculostomy. What nursing intervention is the priority in caring for this patient? A. Administer IV mannitol (Osmitrol). B. Ventilator use to hyperoxygenate the patient C. Use strict aseptic technique with dressing changes. D. Be aware of changes in ICP related to leaking CSF.

C. The priority nursing intervention is to use strict aseptic technique with dressing changes and any handling of the insertion site to prevent the serious complication of infection. IV mannitol (Osmitrol) or hypertonic saline will be administered as ordered. Ventilators may be used to maintain oxygenation. CSF leaks may cause inaccurate ICP readings, or CSF may be drained to decrease ICP, but strict aseptic technique to prevent infection is the nurse's priority of care.

An unconscious patient with increased ICP is on ventilatory support. The nurse notifies the health care provider when arterial blood gas measurement results reveal a a. pH of 7.43 b. SaO2 of 94% c. PaO2 of 50 mm Hg d. PaCO2 of 30 mm Hg

C. PaO2 of 50 mm Hg- A PaO2 of 50 mm Hg reflects a hypoxemia that may lead to further decreased cerebral perfusion and hypoxia and must be corrected. The pH of SaO2 are within normal range, and a PaCO2 of 30 mm Hg reflects acceptable value for the patient with increased ICP

The postanesthesia care unit (PACU) has several patients with endotracheal tubes. Which patient should receive the least amount of endotracheal suctioning? Transplantation of a kidney Replacement of aortic valve Cerebral aneurysm resection Formation of an ileal conduit

Cerebral aneurysm resection The nurse should avoid suctioning the patient after a craniotomy until it is necessary because suctioning will increase this patient's intracranial pressure. The patients with a kidney transplantation, aortic valve replacement, or formation of an ileal conduit will not be negatively affected by suctioning, although it should only be done when needed, not routinely.

The physician has decided to use renal replacement therapy to remove large volumes of fluid from a patient who is hemodynamically unstable in the intensive care unit. The nurse should expect which treatment to be used for this patient? A.Hemodialysis (HD) 3 times per week B.Automated peritoneal dialysis (APD) C.Continuous venovenous hemofiltration (CVVH) D. Continuous ambulatory peritoneal dialysis (CAPD)

Continuous venovenous hemofiltration (CVVH) CVVH removes large volumes of water and solutes from the patient over a longer period of time by using ultrafiltration and convection. HD 3 times per week would not be used for this patient because fluid and solutes build up and then are rapidly removed. With APD (used at night instead of during the day) fluid and solutes build up during the day and would not benefit this patient as much. CAPD will not as rapidly remove large amounts of fluid as CVVH can do.

When planning care for a patient with cirrhosis, the nurse will give highest priority to which of the following nursing diagnoses? A. Imbalanced nutrition: less than body requirements B. Impaired skin integrity related to edema, ascites, and pruritus C. Excess fluid volume related to portal hypertension and hyperaldosteronism D. Ineffective breathing pattern related to pressure on diaphragm and reduced lung volume

D) Although all of these nursing diagnoses are appropriate and important in the care of a patient with cirrhosis, airway and breathing are always the highest priorities.

The ED nurse has inspected, auscultated, and palpated the abdomen with no obvious abnormalities, except pain. When the nurse palpates the abdomen for rebound tenderness, there is severe pain. The nurse should know that this could indicate what problem? A) Hepatic cirrhosis B) Hypersplenomegaly C) Gall bladder distention D) Peritoneal inflammation

D) When palpating for rebound tenderness, the problem area of the abdomen will produce pain and severe muscle spasm when there is peritoneal inflammation. Hepatic cirrhosis, hypersplenomegaly, and gall bladder distention do not manifest with rebound tenderness.

11. A male patient suffered a diffuse axonal injury from a traumatic brain injury (TBI). He has been maintained on IV fluids for 2 days. The nurse seeks enteral feeding for this patient based on what rationale? A. Free water should be avoided. B. Sodium restrictions can be managed. C. Dehydration can be better avoided with feedings. D. Malnutrition promotes continued cerebral edema.

D. A patient with diffuse axonal injury is unconscious and, with increased ICP, is in a hypermetabolic, hypercatabolic state that increases the need for fuel for healing. Malnutrition promotes continued cerebral edema, and early feeding may improve outcomes when begun within 3 days after injury. Fluid and electrolytes will be monitored to maintain balance with the enteral feedings.

A patient with ICP monitoring has pressure of 12 mm Hg. The nurse understand that this pressure reflects a. a severe decrease in cerebral perfusion pressure b. an alteration in the production of CSF c. the loss of autoregulatory control of ICP d. a normal balance between brain tissue, blood, and CSF

D. A normal balance between brain tissue, blood, and CSF- normal is 10- 15 mm Hg

Successful achievement of patient outcomes for the patient with cranial surgery would be best indicated by the a. ability to return home in 6 days b. ability to meet all self-care needs c. acceptance of residual neurologic deficits d. absence of signs and symptoms of increased ICP

D. Absence of signs and symptoms of increased ICP- The primary goal after cranial surgery is prevention of increased ICP, and interventions to prevent ICP and infection postoperatively are nursing priorities. The residual deficits, rehabilitation potential, and ultimate function of the patient depend on the reason for surgery, the postoperative course, and the patient's general state of health

Multiple patients arrive in the emergency department from a house fire. Which patient is a priority? A. Patient with erythremic, dry burns over the arms and a history of taking prednisone B. Patient with moist blisters over the chest and who reports pain as 10 C. Patient with dry, black skin on one hand and a history of diabetes mellitus D. Patient with multiple reddened skin areas on the chest and with high-pitched respiratory sounds

D. Airway injury is a priority, and stridor results from a narrowing of the airway caused by edema. A history of prednisone use or diabetes is a concern for long-term infection risk, but the airway is always first.

The nurse suspects the presence of an arterial epidural hematoma in the patient who experiences a. failure to regain consciousness following a head injury b. a rapid deterioration of neurologic function within 24 to 48 hours following a head injury c. nonspecific, nonlocalizing progression of alteration in LOC occurring over weeks or months d. unconsciousness at the time of a head injury with a brief period of consciousness followed by a decrease in LOC

D. An arterial epidural hematoma is the most acute neurologic emergency, and the typical symptoms include unconsciousness at the scene, with a brief lucid interval followed by a decrease in LOC. An acute subdural hematoma manifests signs within 48 hours of an injury; a chronic subdural hematoma develops over weeks or months

The patient has pulmonary fibrosis and experiences hypoxemia during exercise but not at rest. To plan patient care, the nurse should know the patient is experiencing which physiologic mechanism of respiratory failure? Diffusion limitation Intrapulmonary shunt Alveolar hypoventilation Ventilation-perfusion mismatch

Diffusion limitation The patient with pulmonary fibrosis has a thickened alveolar-capillary interface that slows gas transport, and hypoxemia is more likely during exercise than at rest. Intrapulmonary shunt occurs when alveoli fill with fluid (e.g., acute respiratory distress syndrome, pneumonia). Alveolar hypoventilation occurs when there is a generalized decrease in ventilation (e.g., restrictive lung disease, central nervous system diseases, neuromuscular diseases). Ventilation-perfusion mismatch occurs when the amount of air does not match the amount of blood that the lung receives (e.g., chronic obstructive pulmonary disease, pulmonary embolus).

A 70-yr-old patient in the intensive care unit (ICU) has become agitated and inattentive since his heart surgery. The nurse knows that ICU delirium frequently occurs in individuals with preexisting dementia, history of alcohol abuse, and severe disease. What interventions should the nurse provide to improve the patient's cognition (select all that apply.)? Select all that apply. Improve oxygenation. Initiate early mobilization. Provide a small amount of beer. Have the family stay with the patient. Enable the patient to sleep on a schedule with dim lights. Encourage conversation in the patient's room to help reorient.

Improve oxygenation. Initiate early mobilization. Have the family stay with the patient. Enable the patient to sleep on a schedule with dim lights. ICU delirium is common in ICU patients. Improving oxygenation, enabling the patient to sleep, early mobilization, and decreasing sensory overload along with orientation is all helpful in improving the patient's cognition. The beer may or may not be allowed for this patient, and the nurse should not assume that it will help. Having a family member stay with the patient to reorient the patient is helpful, but the family group may increase sensory overload with conversations not involving the patient.

Tx for gallstones: laparoscopic cholecystectomy

Lap chole - *treatment of choice if symptomatic*

A patient is recovering in the intensive care unit (ICU) after receiving a kidney transplant approximately 24 hours ago. What is an expected assessment finding for this patient during this early stage of recovery? A.Hypokalemia B. Hyponatremia C.Large urine output D. Leukocytosis with cloudy urine output

Large urine output Patients frequently experience diuresis in the hours and days immediately following a kidney transplant. Electrolyte imbalances and signs of infection are unexpected findings that warrant prompt intervention.

The nurse is caring for a patient with osteoarthritis scheduled for total left knee arthroplasty. Preoperatively, the nurse assesses for which contraindication to surgery? Pain Left knee stiffness Left knee infection Left knee instability

Left knee infection The patient must be free of infection before total knee arthroplasty. An infection in the joint could lead to even greater pain and joint instability, requiring more extensive surgery. The nurse must assess the patient for signs of infection, such as redness, swelling, fever, and elevated white blood cell count. Pain, knee stiffness, or instability are typical of osteoarthritis.

The nurse is caring for a patient who is intubated and receiving positive pressure ventilation to treat acute respiratory distress syndrome (ARDS). Which finding is most important to report to the health care provider? a. Blood urea nitrogen (BUN) level 32 mg/dL b. Red-brown drainage from orogastric tube c. Scattered coarse crackles heard throughout lungs d. Arterial blood gases: pH 7.31, PaCO2 50, PaO2 68

b. red-brown drainage from orogastric tube

The nurse is caring for a patient intubated and on a mechanical ventilator for several days. Which weaning parameter would tell the nurse if the patient has enough muscle strength to breathe without assistance? Tidal volume Minute ventilation Forced vital capacity Negative inspiratory force

Negative inspiratory force The negative inspiratory force measures inspiratory muscle strength. Tidal volume and minute ventilation assess the patient's respiratory endurance. Forced vital capacity is not used as a measure to determine weaning from a ventilator.

When caring for a patient with acute respiratory distress syndrome (ARDS), which finding indicates therapy is appropriate? pH is 7.32. PaO2 is greater than or equal to 60 mm Hg. PEEP increased to 20 cm H2O caused BP to fall to 80/40. No change in PaO2 when patient is turned from supine to prone position

PaO2 is greater than or equal to 60 mm Hg. The overall goal in caring for the patient with ARDS is for the PaO2 to be greater than or equal to 60 mm Hg with adequate lung ventilation to maintain a normal pH of 7.35 to 7.45. PEEP is usually increased for ARDS patients, but a dramatic reduction in BP indicates a complication of decreased cardiac output. A positive occurrence is a marked improvement in PaO2 from perfusion better matching ventilation when the anterior air-filled, nonatelectatic alveoli become dependent in the prone position.

The nurse is examining the electrocardiogram (ECG) of a patient just admitted with a suspected MI. Which ECG change is most indicative of prolonged or complete coronary occlusion? Sinus tachycardia Pathologic Q wave Fibrillatory P waves Prolonged PR interval

Pathologic Q wave The presence of a pathologic Q wave, as often accompanies STEMI, is indicative of complete coronary occlusion. Sinus tachycardia, fibrillatory P waves (e.g., atrial fibrillation), or a prolonged PR interval (first-degree heart block) are not direct indicators of extensive occlusion.

A 52-yr-old male patient has received a bolus dose and an infusion of alteplase (Activase) for an ST-segment elevation myocardial infarction (STEMI). Which patient assessment would determine the effectiveness of the medication? Presence of chest pain Blood in the urine or stool Tachycardia with hypotension Decreased level of consciousness

Presence of chest pain Alteplase is a fibrinolytic agent that is administered to patients who have had an STEMI. If the medication is effective, the patient's chest pain will resolve because the medication dissolves the thrombus in the coronary artery and results in reperfusion of the myocardium. Bleeding is a major complication of fibrinolytic therapy. Signs of major bleeding include decreased level of consciousness, blood in the urine or stool, and increased heart rate with decreased blood pressure.

When planning care for a patient on a mechanical ventilator, the nurse understands that the application of positive end-expiratory pressure (PEEP) to the ventilator settings has which therapeutic effect? Increased inflation of the lungs Prevention of barotrauma to the lung tissue Prevention of alveolar collapse during expiration Increased fraction of inspired oxygen concentration (FIO2) administration

Prevention of alveolar collapse during expiration PEEP is positive pressure that is applied to the airway during exhalation. This positive pressure prevents the alveoli from collapsing, improving oxygenation and enabling a reduced FIO2 requirement. PEEP does not cause increased inflation of the lungs or prevent barotrauma. Actually auto-PEEP resulting from inadequate exhalation time may contribute to barotrauma.

The patient has atrial fibrillation with a rapid ventricular response. What electrical treatment option does the nurse prepare the patient for? Defibrillation Synchronized cardioversion Automatic external defibrillator (AED) Implantable cardioverter-defibrillator (ICD)

Synchronized cardioversion Synchronized cardioversion is planned for a patient with supraventricular tachydysrhythmias (atrial fibrillation with a rapid ventricular response). Defibrillation or AEDs are the treatment of choice to end ventricular fibrillation and pulseless ventricular tachycardia (VT). An ICD is used with patients who have survived sudden cardiac death, have spontaneous sustained VT, and are at high risk for future life-threatening dysrhythmias.

The patient with CKD is receiving dialysis, and the nurse observes excoriations on the patient's skin. What pathophysiologic changes in CKD can contribute to this finding (select all that apply)? a. Dry skin b. Sensory neuropathy c. Vascular calcifications d. Calcium-phosphate skin deposits e. Uremic crystallization from high BUN

a, b, d. Pruritus is common in patients receiving dialysis. It causes scratching from dry skin, sensory neuropathy, and calcium-phosphate deposition in the skin. Vascular calcifications contribute to cardiovascular disease, not to itching skin. Uremic frost rarely occurs without BUN levels greater than 200 mg/dL, which should not occur in a patient on dialysis; urea crystallizes on the skin and also causes pruritis.

Although ARDS may result from direct lung injury or indirect lung injury as a result of systemic inflammatory response syndrome (SIRS), the nurse is aware that ARDS is most likely to occur in the patient with a host insult resulting from a. sepsis. b. oxygen toxicity. c. prolonged hypotension. d. cardiopulmonary bypass.

a. Although ARDS may occur in the patient who has virtually any severe illness and may be both a cause and a result of systemic inflammatory response syndrome (SIRS), the most common precipitating insults of ARDS are sepsis, gastric aspiration, and severe massive trauma.

A client's burn is infected and mafenide (Sulfamylon) is prescribed. The nurse's knowledge about this medication would indicate that which organism is involved? a. pseudomonas aeruginosa b. tubercle bacillus c. Methicillin resistant staphylococcus aureus (MRSA) d. Candida albicans

a. Mafenide is useful in treatment of partial and full thickness burns to prevent septicemia caused by organisms such as Pseudomonas aeruginosa.

A male burn pt who was struck by lightning arrives at the emergency department with full thickness burns to the arms and chest and with a cervical collar in place. Which assessment finding is the nurse's priority? a. serum K+ of 5.6 mEq/L b. Arterial blood ph of 7.35 c. Cervical spine fracture d. hemoglobin 18g/dL

a. The pt's potassium level puts them at risk for life threatening cardiac dysrhythmias.

A nurse is caring for a patient who is orally intubated and receiving mechanical ventilation. To decrease the risk for ventilator-associated pneumonia, which action will the nurse include in the plan of care? a. Elevate head of bed to 30 to 45 degrees. b. Suction the endotracheal tube every 2 to 4 hours. c. Limit the use of positive end-expiratory pressure. d. Give enteral feedings at no more than 10 mL/hr.

a. elevate head of bed to 30 to 45 degrees

What are effective interventions to decrease absorption or increase elimination of an ingested poison (SATA)? a. hemodialysis b. milk dilution c. eye irrigation d. gastric lavage e. activated charcoal

a. hemodialysis d. gastric lavage e. activated charcoal

meds for chole procedures

abx (may be prophylaxis) N/V - meds (or NGT) anticholinergics for secretions pain meds

A 48-year-old patient with stage 2 chronic kidney disease (CKD) is scheduled for an intravenous pyelogram (IVP). Which order for the patient will the nurse question? a. NPO for 6 hours before procedure b. Ibuprofen (Advil) 400 mg PO PRN for pain c. Dulcolax suppository 4 hours before procedure d. Normal saline 500 mL IV infused before procedure

b. ibuprofen 400 mg PO PRN for pain

The nurse in the dialysis clinic is reviewing the home medications of a patient with chronic kidney disease (CKD). Which medication reported by the patient indicates that patient teaching is required? a. Multivitamin with iron b. Magnesium hydroxide c. Acetaminophen (Tylenol) d. Calcium phosphate (PhosLo)

b. magnesium hydroxide

Which descriptions are characteristic of hypoxemic respiratory failure (select all that apply)? a. Referred to as ventilatory failure b. Primary problem is inadequate O2 transfer c. Risk of inadequate O2 saturation of hemoglobin exists d. Body is unable to compensate for acidemia of increased PaCO2 e. Most often caused by ventilation-perfusion (V/Q) mismatch and shunt f. Exists when PaO2 is 60 mm Hg or less, even when O2 is administered at 60%

b, c, e, f. Hypoxemic respiratory failure is often caused by ventilation-perfusion (V/Q) mismatch and shunt. It is called oxygenation failure because the primary problem is inadequate oxygen transfer. There is a risk of inadequate oxygen saturation of hemoglobin and it exists when PaO2 is 60 mm Hg or less, even when oxygen is administered at 60%. Ventilatory failure is hypercapnic respiratory failure. Hypercapnic respiratory failure results from an imbalance between ventilatory supply and ventilatory demand and the body is unable to compensate for the acidemia of increased PaCO2

Which goal has the highest priority in the plan of care for a 26-year-old homeless patient admitted with viral hepatitis who has severe anorexia and fatigue? a. Increase activity level. b. Maintain adequate nutrition. c. Establish a stable environment. d. Identify sources of hepatitis exposure.

b. Maintain adequate nutrition. The highest priority outcome is to maintain nutrition because adequate nutrition is needed for hepatocyte regeneration. Finding a home for the patient and identifying the source of the infection would be appropriate activities, but they do not have as high a priority as ensuring adequate nutrition. Although the patient's activity level will be gradually increased, rest is indicated during the acute phase of hepatitis.

The nurse suspects that a patient with PEEP is experiencing negative effects of this ventilatory maneuver when which of the following is assessed? a. Increasing PaO2 b. Decreasing blood pressure c. Decreasing heart rate (HR) d. Increasing central venous pressure (CVP)

b. PEEP increases intrathoracic and intrapulmonic pressures, compresses the pulmonary capillary bed, and reduces blood return to both the right and left sides of the heart. Increased PaO2 is an expected effect of PEEP. Preload (CVP) and cardiac output (CO) are decreased, often with a dramatic decrease in BP.

A patient has 25% TBSA burned from a car fire. His wounds have been debrided and covered with a silver-impregnated dressing. Your priority intervention for wound care is to: a. reapply a new dressing without disturbing the wound bed b. observe the wound for signs of infection during dressing changes c. apply cool compresses for pain relief in between dressing changes d. wash the wound aggressively with soap and water three times each day.

b. observe the wound for signs of infection during dressing changes Infection is the most serious threat for further tissue injury and possible sepsis

The nurse is caring for a patient with superficial partial-thickness burns of the face sustained within the last 12 hours. Upon assessment, the nurse would expect to find which of the following symptoms? A. Blisters B. Reddening of the skin C. Destruction of all skin layers D. Damage to sebaceous glands

b. The clinical appearance of superficial partial-thickness burns includes erythema, blanching with pressure, and pain and minimal swelling with no vesicles or blistering during the first 24 hours. *like a sunburn or a scald*

A patient has arrived for a scheduled hemodialysis session. Which nursing action is most appropriate for the registered nurse (RN) to delegate to a dialysis technician? a. Teach the patient about fluid restrictions. b. Check blood pressure before starting dialysis. c. Assess for causes of an increase in predialysis weight. d. Determine the ultrafiltration rate for the hemodialysis.

b. check blood pressure before starting dialysis

The nurse should expect to apply which type of ordered antiseptic to a client with a burn wound, once the area has been cleansed with sterile saline? a. copper containing b. silver containing c. biguanide d. acetic acid

b. silver sulfadiazine is a metallic type of antiseptic that is widely used on burns. The silver in the solution is toxic to bacteria, and prevents them from reproducing.

The patient with chronic kidney disease is considering whether to use peritoneal dialysis (PD) or hemodialysis (HD). What are advantages of PD when compared to HD (select all that apply)? a. Less protein loss b. Rapid fluid removal c. Less cardiovascular stress d. Decreased hyperlipidemia e. Requires fewer dietary restrictions

c, e. Peritoneal dialysis is less stressful for the cardiovascular system and requires fewer dietary restrictions. Peritoneal dialysis actually contributes to more protein loss and increased hyperlipidemia. The fluid and creatinine removal are slower with peritoneal dialysis than hemodialysis.

if bile can't get out of the gallbladder because of an obstruction, this can happen

cholecystitis!

After receiving change-of-shift report on a medical unit, which patient should the nurse assess first? a. A patient with cystic fibrosis who has thick, green-colored sputum b. A patient with pneumonia who has crackles bilaterally in the lung bases c. A patient with emphysema who has an oxygen saturation of 90% to 92% d. A patient with septicemia who has intercostal and suprasternal retractions

d. a patient with septicemia who has intercostal and suprasternal retractions

In caring for the patient with AKI, what should the nurse be aware of? a. The most common cause of death in AKI is irreversible metabolic acidosis. b. During the oliguric phase of AKI, daily fluid intake is limited to 1000 mL plus the prior day's measured fluid loss. c. Dietary sodium and potassium during the oliguric phase of AKI are managed according to the patient's urinary output. d. One of the most important nursing measures in managing fluid balance in the patient with AKI is taking accurate daily weights.

d. Measuring daily weights with the same scale at the same time each day allows for the evaluation and detection of excessive body fluid gains or losses. Infection is the leading cause of death in AKI, so meticulous aseptic technique is critical. The fluid limitation in the oliguric phase is 600 mL plus the prior day's measured fluid loss. Dietary sodium and potassium intake are managed according to the plasma levels.

The nurse is caring for a client who sustained superficial partial thickness burns on the anterior lower legs and anterior thorax. Which of the following does the nurse expect to note during the resuscitation/emergent phase of the burn injury? a. decreased heart rate b. increased urinary output c. increased blood pressure d. elevated hematocrit levels

d. during the resuscitation/emergent phase, the hematocrit level increases to above normal because of hemoconcentration from the large fluid shift.

The home care nurse visits a 34-year-old woman receiving peritoneal dialysis. Which statement, if made by the patient, indicates a need for immediate follow-up by the nurse? A. "Drain time is faster if I rub my abdomen." B."The fluid draining from the catheter is cloudy." C."The drainage is bloody when I have my period." D."I wash around the catheter with soap and water."

"The fluid draining from the catheter is cloudy." The primary clinical manifestation of peritonitis is a cloudy peritoneal effluent. Blood may be present in the effluent of women who are menstruating, and no intervention is indicated. Daily catheter care may include washing around the catheter with soap and water. Drain time may be facilitated by gently massaging the abdomen.

Which types of shock may cause reduced urinary output in a patient? Select all that apply. 1 Septic shock 2 Hypovolemic shock 3 Obstructive shock 4 Neurogenic shock 5 Anaphylactic shock 6 Cardiogenic shock.

1, 2, 3, 6 Decreased urine output is a clinical manifestation of septic, hypovolemic, obstructive and cardiogenic shock. Neurogenic shock is associated with bladder dysfunction. Anaphylactic shock is associated with urinary incontinence. Test-Taking Tip: Make certain that the answer you select is reasonable and obtainable under ordinary circumstances and that the action can be carried out in the given situation. Text Reference - p. 1635

Hemodialysis is planned for a patient who has end-stage kidney disease. The patient is scheduled for the creation of an internal arteriovenous fistula and the placement of an external arteriovenous shunt to be used until the fistula heals. What postoperative nursing care is appropriate for this patient? Select all that apply. 1 Regularly check the positioning of the external shunt. 2 Check for signs and symptoms of respiratory complications. 3 Ensure that intravenous fluids are not infused in the arm with the shunt. 4 Cover the ends of the shunt cannula with a dressing. 5 Do not take blood pressure on the extremity with the shunt

1, 3, 5 The external shunt may come apart, external temperatures make clotting a potential hazard, and frequent handling increases the risk of infection. Infusions should not be in the extremity with the shunt or the fistula to avoid pressure from the tourniquet and to lessen the chance of phlebitis. Blood pressure readings should not be obtained in the extremity that has a shunt or fistula because of the pressure exerted on the circulatory system during the procedure. There are no respiratory complications of this procedure. The ends of the shunt cannula should be left exposed for rapid reconnection in the event of disruption. Test-Taking Tip: Being prepared reduces your stress or tension level and helps you maintain a positive attitude. Text Reference - p. 1120

A nurse is delivering a lecture on organ donation. She is explaining about the selection criteria for kidney donors. What are the donor characteristics that the nurse should discuss with the group? Select all that apply. 1 Donors should not have diabetes. 2 Donors should be a first-degree relative of a recipient. 3 Donors should be approximately the same body size as the recipient. 4 Donors must have ABO compatibility with the recipient. 5 The donor and recipient should have matching leukocyte antigen complexes.

1, 4, 5 Diabetes is a major predisposing factor for development of kidney disease; hence, the donor should not be a diabetic. ABO compatibility is necessary for being a donor, although the exact blood type is not necessary. Human leukocyte antigen compatibility provides the most specific predictions of the body's tendency to accept or reject foreign tissue. Being a member of the same family is unsafe unless the family member has matching leukocyte antigen complexes. Being a member of the same family may increase the possibility of a match, but there is no guarantee that a family member will match. Differences in body size do not cause problems. Text Reference - p. 1124

Which type of shock can be treated by minimizing spinal cord trauma with stabilization? 1 Septic shock 2 Neurogenic shock 3 Anaphylactic shock 4 Hypovolemic shock

2 Neurogenic shock is caused by severe injury to the spinal cord and results in loss of sympathetic stimulation of blood vessels. Apart from administering vasoconstrictor agents, minimizing the spinal cord trauma with stabilization is a supporting therapy for neurogenic shock. Septic shock occurs in response to a systemic infection. Obtaining the cultures before starting antibiotics is appropriate care for septic shock. A life-threatening allergic reaction to a sensitizing substance causes anaphylactic shock. Avoiding exposure to allergens is supportive therapy for anaphylactic shock. Excessive loss of intravascular fluid causes hypovolemic shock. Besides restoring fluid volume, correcting the cause of fluid loss is supportive therapy. Test-Taking Tip: If the question asks for an immediate action or response, all of the answers may be correct, so base your selection on identified priorities for action. Text Reference - p. 1645

Routine urinalysis for a diabetic patient reveals moderate proteinuria. What further tests help to identify decreased kidney function at an early stage? Select all that apply. 1 Serum creatinine 2 Glomerular filtration rate (GFR) 3 Renal ultrasound 4 Evaluation of microalbuminuria 5 Magnetic resonance angiography (MRA)

2, 3, 4 If routine urinalysis indicates moderate to severe proteinuria, the preferred way of determining kidney functions is by assessing the GFR. An ultrasound of the kidneys is usually done to detect any obstructions and to determine the size of the kidneys. A patient with diabetes needs to have a further examination of the urine for microalbuminuria. The patient may not have an increase in serum creatinine until there is a decrease of 50% or more in kidney function. MRA study with the contrast media gadolinium is generally not advised unless the ultrasound or computed tomography (CT) does not provide the information needed. Test-Taking Tip: Become familiar with reading questions on a computer screen. Familiarity reduces anxiety and decreases errors. Text Reference - p. 1104

A nurse has to determine the volume of fluid that must be administered to the patient with acute renal failure who is in the oliguric phase. The total urine output of the patient the previous day was 250 mL. What should be the fluid allocation for this patient on this day? Record your answer using a whole number. __ mL

850 The patient is at a risk of developing hypovolemia, and to prevent this, adequate fluid resuscitation should be done. To determine the volume for fluid resuscitation, the nurse adds together all losses during the previous 24 hours (e.g., urine, diarrhea, emesis, blood) and adds 600 mL for insensible losses (e.g., respiration, diaphoresis). Text Reference - p. 1105

1. The nurse is caring for a patient admitted for evaluation and surgical removal of a brain tumor. The nurse will plan interventions for this patient based on knowledge that brain tumors can lead to which complications (select all that apply)? A. Vision loss B. Cerebral edema C. Pituitary dysfunction D. Parathyroid dysfunction E. Focal neurologic deficits

A,B,C,E Brain tumors can manifest themselves in a wide variety of symptoms depending on location, including vision loss and focal neurologic deficits. Tumors that put pressure on the pituitary can lead to dysfunction of the gland. As the tumor grows, clinical manifestations of increased intracranial pressure (ICP) and cerebral edema appear. The parathyroid gland is not regulated by the cerebral cortex or the pituitary gland.

While doing an admission assessment, the nurse notes clubbing of the patient's fingers. Based on this finding, the nurse will question the patient about which disease process? A. Endocarditis B. Acute kidney injury C. Myocardial infarction D. Chronic thrombophlebitis

A. Endocarditis Clubbing of the fingers is a loss of the normal angle between the base of the nail and the skin. This finding can be found in endocarditis, congenital defects, and/or prolonged oxygen deficiency. Clinical manifestations of acute kidney injury, myocardial infarction, and chronic thrombophlebitis will not include clubbing of the fingers.

A patient is suspected of having a cranial tumor. The signs and symptoms include memory deficits, visual disturbances, weakness of right upper and lower extremities, and personality changes. The nurse recognizes that the tumor is most likely located in the a. frontal lobe b. parietal lobe c. occipital lobe d. temporal lobe

A. Frontal lobe

When a patient is admitted to the emergency department following a head injury, the nurse's first priority in management of the patient once a patent airway is confirmed is a. maintaining cervical spine precautions b. determining the presence of increased ICP c. monitoring for changes in neurologic status d. establishing IV access with a large-bore catheter

A. In addition to monitoring for a patent airway during emergency care of the patient with a head injury, the nurse must always assume that a patient with a head injury may have a cervical spine injury. Maintaining cervical spine precautions in all assessment and treatment activities with the patient is essential to prevent additional neurologic damage.

A patient with cancer is having chemotherapy treatments and has now developed neutropenia. What care should the nurse expect to provide and teach the patient about (select all that apply.)? A. Strict hand washing B. Daily nasal swabs for culture C. Monitor temperature every hour. D. Daily skin care and oral hygiene E. Encourage eating all foods to increase nutrients. F. Private room with a high-efficiency particulate air (HEPA) filter

A. Strict hand washing D. Daily skin care and oral hygiene F. Private room with a high-efficiency particulate air (HEPA) filter Strict hand washing and daily skin and oral hygiene must be done with neutropenia, because the patient is predisposed to infection from the normal body flora; other people; and uncooked meats, seafood, and eggs; unwashed fruits and vegetables; and fresh flowers or plants. The private room with HEPA filtration reduces the aerosolized pathogens in the patient's room. Blood cultures and antibiotic treatment are used when the patient has a temperature of 100.4°F or more, but temperature is not monitored every hour.

A public health nurse has reviewed local data about the incidence and prevalence of burn injuries in the community. These data are likely to support what health promotion effort? A) Education about home safety B) Education about safe storage of chemicals C) Education about workplace health threats D) Education about safe driving

Ans: A Feedback: A large majority of burns occur in the home setting; educational interventions should address this epidemiologic trend.

During the physical examination of a 36-year-old female, the nurse finds that the patient's thyroid gland cannot be palpated. The most appropriate action by the nurse is to a. palpate the patient's neck more deeply. b. document that the thyroid was nonpalpable. c. notify the health care provider immediately. d. teach the patient about thyroid hormone testing.

B

The nurse performs discharge teaching for a 68-year-old man who is newly diagnosed with infective endocarditis with a history of IV substance abuse. Which statement by the patient indicates to the nurse that teaching was successful? A."I will need antibiotics before having any invasive procedure or surgery." B. "I will inform my dentist about my hospitalization for infective endocarditis." C."I should not be alarmed if I have difficulty breathing or pink-tinged sputum." D. "An elevated temperature is expected and can be managed by taking acetaminophen."

B. "I will inform my dentist about my hospitalization for infective endocarditis." Patients with infective endocarditis should inform their dental providers of their health history. Antibiotic prophylaxis is recommended for patients with a history of infective endocarditis who have certain dental procedures performed. Antibiotics are not indicated before genitourinary or gastrointestinal procedures unless an infection is present. Patients should immediately report the presence of fever or clinical manifestations indicating heart failure to their health care provider.

A 42-year-old man who is scheduled for an arthrocentesis arrives at the outpatient surgery unit and states, "I do not want this procedure done today." Which response by the nurse is most appropriate? A. "When would you like to reschedule the procedure?" B. "Tell me what your concerns are about this procedure." C. "The procedure is safe, so why should you be worried?" D. "The procedure is not painful because an anesthetic is used."

B. "Tell me what your concerns are about this procedure." The nurse should use therapeutic communication to determine the patient's concern about the procedure. The nurse should not provide false reassurance. It is not appropriate for the nurse to assume the patient is concerned about pain or to assume the patient is asking to reschedule the procedure.

A patient has been diagnosed with acute myelogenous leukemia (AML). What should the nurse educate the patient that care will focus on? A. Leukopheresis B. Attaining remission C. One chemotherapy agent D. Waiting with active supportive care

B. Attaining remission Attaining remission is the initial goal of care for leukemia. The methods to do this are decided based on age and cytogenetic analysis. The treatments include leukopheresis or hydroxyurea to reduce the white blood cell count and risk of leukemia-cell-induced thrombosis. A combination of chemotherapy agents will be used for aggressive treatment to destroy leukemic cells in tissues, peripheral blood, and bone marrow and minimize drug toxicity. In nonsymptomatic patients with chronic lymphocytic leukemia, waiting may be done to attain remission, but not with AML.

A patient with left knee pain is diagnosed with bursitis. The nurse will explain that bursitis is an inflammation of a. the synovial membrane that lines the joint. b. a small, fluid-filled sac found at some joints. c. the fibrocartilage that acts as a shock absorber in the knee joint. d. any connective tissue that is found supporting the joints of the body.

B. Bursae are fluid-filled sacs that cushion joints and bony prominences. Fibrocartilage is a solid tissue that cushions some joints. Bursae are a specific type of connective tissue. The synovial membrane lines many joints but is not a bursa.

A nursing measure that is indicated to reduce the potential for seizures and increased ICP in the patient with bacterial meningitis is a. administering codeine for relief of head and neck pain b. controlling fever with prescribed drugs and cooling techniques c. keeping the room darkened and quiet to minimize environmental stimulation d. maintaining the patient on strict bed rest with the HOB slightly elevated

B. Controlling fever with prescribed drugs and cooling techniques

Assisting the family to understand what is happening to the patient is an especially important role of the nurse when the patient has a tumor of the a. ventricles b. frontal lobe c. parietal lobe d. occipital lobe

B. Frontal lobe- frontal lobe tumors often lead to loss of emotional control, confusion, memory loss, disorientation, and personality changes that are very disturbing and frightening to the family. Physical symptoms, such as blindness, disturbances in sensation and perception, and even seizures, that occur with other tumors are more likely to be understood and accepted by the family

In caring for a patient admitted with poorly controlled hypertension, which laboratory test result should the nurse understand as indicating the presence of target organ damage? A BUN of 15 mg/dL B Serum uric acid of 3.8 mg/dL C Serum creatinine of 2.6 mg/dL D Serum potassium of 3.5 mEq/L

C Serum creatinine of 2.6 mg/dL The normal serum creatinine level is 0.6-1.3 mg/dL. This elevated level indicates target organ damage to the kidneys. The other lab results are within normal limits.

The nurse is assigned to care for several patients on a medical unit. Which patient should the nurse check on first? A. A 60-yr-old patient with a blood pressure of 92/64 mm Hg and hemoglobin of 9.8 g/dL B. A 50-yr-old patient with a respiratory rate of 26 breaths/minute and an elevated D-dimer C. A 40-yr-old patient with a temperature of 100.8oF (38.2oC) and a neutrophil count of 256/μL D. A 30-yr-old patient with a pulse of 112 beats/min and a white blood cell count of 14,000/μL

C. A 40-yr-old patient with a temperature of 100.8oF (38.2oC) and a neutrophil count of 256/μL A low-grade fever greater than 100.4°F (38°C) in a patient with a neutrophil count below 500/μL is a medical emergency and may indicate an infection. An infection in a neutropenic patient could lead to septic shock and possible death if not treated immediately.

The patient sustained a full-thickness burn encompassing the entire right arm. What is the best indicator an escharotomy achieved its desired effect? A. Patient rates the pain at less than 4. B. Blood pressure remains above 120/80 mm Hg. C. Right fingers blanch with a 2-second refill. D. Patient maintains full range of motion for the right arm.

C. Circulation to the extremities can be severely impaired by deep circumferential burns and subsequent edema that impairs the blood supply. An escharotomy (electrocautery incision through the full-thickness eschar) is performed to restore circulation. Normal refill is less than 2 seconds.

The nurse knows that hemolytic anemia can be caused by which extrinsic factors? A. Trauma or splenic sequestration crisis B. Abnormal hemoglobin or enzyme deficiency C. Macroangiopathic or microangiopathic factors D. Chronic diseases or medications and chemicals

C. Macroangiopathic or microangiopathic factors Macroangiopathic or microangiopathic extrinsic factors lead to acquired hemolytic anemias. Trauma or splenic sequestration crisis can lead to anemia from acute blood loss. Abnormal hemoglobin or enzyme deficiency are intrinsic factors that lead to hereditary hemolytic anemias. Chronic diseases or medications and chemicals can decrease the number of red blood cell (RBC) precursors which reduce RBC production.

A 72-year-old man with a history of aortic stenosis is admitted to the emergency department. He reports severe left-sided chest pressure radiating to the jaw. Which medication, if ordered by the health care provider, should the nurse question? A. Aspirin B. Oxygen C. Nitroglycerin D. Morphine sulfate

C. Nitroglycerin Aspirin, oxygen, nitroglycerin, and morphine sulfate are all commonly used to treat acute chest pain suspected to be caused by myocardial ischemia. However, nitroglycerin should be used cautiously or avoided in patients with aortic stenosis as a significant reduction in blood pressure may occur. Chest pain can worsen because of a drop in blood pressure.

What nursing action should the nurse prioritize during the care of a patient who has recently recovered from rheumatic fever? A. Teach the patient how to manage his or her physical activity. B. Teach the patient about the need for ongoing anticoagulation. C. Teach the patient about the need for continuous antibiotic prophylaxis. D. Teach the patient about the need to maintain standard infection control procedures.

C. Teach the patient about the need for continuous antibiotic prophylaxis. Patients with a history of rheumatic fever frequently require ongoing antibiotic prophylaxis, an intervention that necessitates education. This consideration is more important than activity management in preventing recurrence. Anticoagulation is not indicated in this patient population. Standard precautions are indicated for all patients.

CN III originating in the midbrain is assessed by the nurse for an early indication of pressure on the brainstem by a. assessing for nystagmus b. testing the corneal reflex c. testing pupillary reaction to light d. testing for oculocephalic (doll's eye) reflex

C. Testing pupillary reaction to light- One of the functions of CN III, the oculomotor nerve, is *pupillary constriction*, and testing for pupillary constriction is important to identify patients at risk for brainstem herniation caused by increased ICP. The corneal reflex is used to assess the functions of CN V and VII, and the oculocephalic reflex tests all cranial nerves involved with eye movement. Nystagmus is commonly associatted with specific lesions or chemical toxicities and is not a definitive sign of ICP

The nurse is performing a focused abdominal assessment of a patient who has been recently admitted. In order to palpate the patient's liver, where should the nurse palpate the patient's abdomen? A) Left lower quadrant B) Left upper quadrant C) Right lower quadrant D) Right upper quadrant

D) Although the left lobe of the liver is located in the left upper quadrant of the abdomen, the bulk of the liver is located in the right upper quadrant.

A pt. has a spinal cord injury at T4. VS include a falling BP with bradycardia. The nurse recognizes that the pt. is experiencing A) a relative hypervolemia B) an absolute hypovolemia C) Neurogenic shock from low blood flow D) Neurogenic shock from massive vasodilation

D) Neurogenic shock from massive vasodilation

A patient with leukemia is admitted for severe hypovolemia after prolonged diarrhea has a platelet count of 43,000/µL. It is most important for the nurse to take which action? A. Insert two 18-gauge IV catheters. B. Administer prescribed enoxaparin. C. Monitor the patient's temperature every 2 hours. D. Check stools for presence of frank or occult blood.

D. Check stools for presence of frank or occult blood. A platelet count below 150,000/µL indicates thrombocytopenia. Prolonged bleeding from trauma or injury does not usually occur until the platelet counts are below 50,000/µL. Bleeding precautions (e.g., check all secretions for frank and occult blood) are indicated for patients with thrombocytopenia. Injections (including IVs) should be avoided; however, when needed for critical fluids and medications, IV access should be provided through the smallest bore devices that are feasible. Enoxaparin, an anticoagulant administered subcutaneously, is contraindicated in patients with thrombocytopenia. Monitoring temperature would be indicated in a patient with leukopenia.

A 55-year-old female patient develops acute pericarditis after a myocardial infarction. It is most important for the nurse to assess for which clinical manifestation of a possible complication? A. Presence of a pericardial friction rub B. Distant and muffled apical heart sounds C. Increased chest pain with deep breathing D. Decreased blood pressure with tachycardia

D. Decreased blood pressure with tachycardia Cardiac tamponade is a serious complication of acute pericarditis. Signs and symptoms indicating cardiac tamponade include narrowed pulse pressure, tachypnea, tachycardia, a decreased cardiac output, and decreased blood pressure. The other symptoms are consistent with acute pericarditis.

The patient has a potassium level of 2.9 mEq/L, and the nurse obtains the following measurements on the rhythm strip: Heart rate of 86 with a regular rhythm, the P wave is 0.06 seconds (sec) and normal shape, the PR interval is 0.24 sec, and the QRS is 0.09 sec. How should the nurse document this rhythm? First-degree AV block Second-degree AV block Premature atrial contraction (PAC) Premature ventricular contraction (PVC)

First-degree AV block In first-degree atrioventricular (AV) block, there is prolonged duration of AV conduction that lengthens the PR interval above 0.20 sec. In type I second-degree AV block, the PR interval continues to increase in duration until a QRS complex is blocked. In type II, the PR interval may be normal or prolonged, the ventricular rhythm may be irregular, and the QRS is usually greater than 0.12 sec. PACs cause an irregular rhythm with a different-shaped P wave than the rest of the beats, and the PR interval may be shorter or longer. PVCs cause an irregular rhythm, and the QRS complex is wide and distorted in shape.

Which hematologic problem significantly increases the risks associated with pulmonary artery (PA) catheter insertion? Leukocytosis Hypovolemia Hemolytic anemia Thrombocytopenia

Thrombocytopenia PA catheter insertion carries a significant risk of bleeding, which is exacerbated when the patient has low levels of platelets. Leukocytosis, hypovolemia, and anemia are less likely to directly increase the risks associated with PA insertion.

An injured soldier underwent left leg amputation 2 weeks ago, but now reports shooting pain and heaviness in the left leg. What action by the nurse is supported by research findings? Use mirror therapy. Give opioid analgesics. Rebandage the residual limb. Show the patient the leg is gone.

Use mirror therapy. Mirror therapy has been shown to reduce phantom limb pain in some patients. Opioid analgesics, rebandaging the residual limb, and showing the patient that the leg is gone may not decrease phantom limb pain.

What are the appropriate nursing interventions for the patient with delirium in the ICU? SATA. a. use clocks and calendars to maintain orientation b. encourage round-the-clock presence of caregivers at the bedside c. silence all alarms, reduce overhead paging, and avoid conversations around the patient d. sedate the patient with appropriate drugs to protect the patient from harmful behaviors e. identify physiologic factors that may be contributing to the patient's confusion and irritability

a. use clocks and calendars to maintain orientation d. sedate the patient with appropriate drugs to protect the patient from harmful behaviors e. identify physiologic factors that may be contributing to the patient's confusion and irritability

To prevent muscle atrophy, the nurse teaches the patient with a leg immobilized in traction to perform (select all that apply) a.flexion contractions. b.tetanic contractions. c.isotonic contractions. d.isometric contractions. e.extension contractions.

d.isometric contractions Isometric contractions increase the tension within a muscle but do not produce movement. Repeated isometric contractions make muscles grow larger and stronger. Muscular atrophy (i.e., decrease in size) occurs with the absence of contraction that results from immobility.

cholecystitis most commonly associated c..

gallstone obstruction biliary sludge

Post chole diet

low fat low cal avoid dairy small frequent meals

This morning a 21-yr-old male patient had a long leg cast applied, and he asks to crutch walk before dinner. Which statement explains why the nurse will decline the patient's request? "No one is available to assist and accompany the patient." "The cast is not dry yet, and it may be damaged while using crutches." "Rest, ice, compression, and elevation are in process to decrease pain." "Excess edema and complications are prevented when the leg is elevated for 24 hours."

"Excess edema and complications are prevented when the leg is elevated for 24 hours." For the first 24 hours after a lower extremity cast is applied, the leg should be elevated on pillows above heart level to avoid excessive edema and compartment syndrome. A plaster cast will also be drying during this 24-hour period. RICE is used for soft tissue injuries, not with long leg casts.

A nurse performs discharge teaching for a 58-yr-old woman after a left hip arthroplasty using the posterior approach. Which statement by the patient indicates teaching is successful? "Leg-raising exercises are necessary for several months." "I should not try to drive a motor vehicle for 2 to 3 weeks." "I will not have any restrictions now on hip and leg movements." "Blood tests will be done weekly while taking enoxaparin (Lovenox)."

"Leg-raising exercises are necessary for several months." Exercises designed to restore strength and muscle tone will be done for months after surgery. The exercises include leg raises in supine and prone positions. Driving a car is not allowed for 4 to 6 weeks. In the posterior approach hip arthroplasties, extremes of internal rotation and 90-degree flexion of the hip must be avoided for 4 to 6 weeks postoperatively. The knees must be kept apart. The patient should never cross the legs or twist to reach behind. To prevent thromboembolism, enoxaparin is administered subcutaneously and can be given at home. Enoxaparin does not require monitoring of the patient's coagulation status.

The nurse instructs a 50-year-old woman about cholestyramine to reduce pruritis caused by gallbladder disease. Which statement by the patient to the nurse indicates she understands the instructions? "This medication will help me digest fats and fat-soluble vitamins." "I will apply the medicated lotion sparingly to the areas where I itch." "The medication is a powder and needs to be mixed with milk or juice." "I should take this medication on an empty stomach at the same time each day."

"The medication is a powder and needs to be mixed with milk or juice." For treatment of pruritus, cholestyramine may provide relief. This is a resin that binds bile salts in the intestine, increasing their excretion in the feces. Cholestyramine is in powder form and should be mixed with milk or juice before oral administration.

While providing postoperative care for a live kidney donor, the nurse monitors the hematocrit levels. What rationale does the nurse provide to the patient for this action? 1 To assess for bleeding 2 To assess for impairment 3 To assess for hypokalemia 4 To assess for hyponatremia

1 Patients who have donated their kidney should be monitored for hematocrit levels to assess for bleeding. The nurse should monitor renal function to assess for impairment. The nurse should monitor for electrolytes to assess for hypokalemia and hyponatremia in kidney recipients. Text Reference - p. 1127

Nutrition considerations - acute chole

- NPO c NGT in - d/t problems c fat absorption, need to supplement fat-soluble vitamins

Which condition does the nurse suspect in a patient with a glomerular filtration rate (GFR) of 10 mL/minute, a blood urea nitrogen (BUN) level of 23 mg/dL, a potassium level of 4 mEq/L, and a serum bicarbonate level of 20 mEq/L? 1 Uremia 2 Hypertension 3 Dysrhythmias 4 Metabolic acidosis

1 A patient with glomerular filtration rate (GFR) of 10 mL/minute has chronic kidney disease (CKD), causing the BUN to rise above the normal BUN level of 6 to 20 mg/dL, which may be a result of uremia. Hypertension may result from hypernatremia in the patient with CKD; there is no sodium level information given. Dysrhythmias may occur in the patient with CKD due to hyperkalemia due to decreased excretion of potassium by the kidneys; however, this patient's potassium level of 4 mEq/L is a normal finding. An inability of the kidneys to excrete ammonia and generate bicarbonates leads to metabolic acidosis, indicated by a serum bicarbonate level lower than 15 mEq/L.; the patient's level is greater than 15 mEq/L. Text Reference - p. 1108

The nurse is planning an education program on chronic kidney disease. Which ethnic group would the nurse target for promoting this event? 1 African Americans 2 Asian descent 3 Caucasian males 4 Hispanics

1 African Americans are at the greatest risk for develop kidney disease. Those of Asian descent, Caucasian males, and Hispanics are not at as great a risk. Text Reference - p. 1108

Which is a clinical manifestation of acute kidney injury? 1 Oliguria 2 Uremia 3 Anemia 4 Pruritus

1 Oliguria is a sign of acute kidney injury. Uremia, anemia, and pruritus are signs of chronic kidney injury. Text Reference - p. 1103

What are the complications of peritoneal dialysis? Select all that apply. 1 Hernias 2 Hepatitis 3 Peritonitis 4 Hypotension 5 Exit site infection

1, 3, 5 Peritoneal dialysis is removal of waste products from the body when kidneys no longer work adequately. The complications of peritoneal dialysis include hernias, peritonitis, and exit site infection. Hernias are caused by increased intraabdominal pressure secondary to the dialysate infusion. Peritonitis results from contamination or from progression of an exit site or tunnel infection. Exit site infection is caused by infection of the peritoneal catheter. Hepatitis and hypotension are complications of hemodialysis. Text Reference - p. 1128

The patient is in the diuretic phase of acute kidney injury. What education should the nurse provide to the patient regarding this phase? Select all that apply. 1 Urine output is increased. 2 The kidney has become fully functional. 3 The electrolyte imbalance will be normalized. 4 This phase will last no more than three weeks 5 There is a possibility that the fluid volume will be reduced in the body

1, 4, 5 During the diuretic phase of acute kidney injury, daily urine output is usually around 1 to 3 L but may reach 5 L or more. Hypovolemia and hypotension can occur from massive fluid losses. The diuretic phase may last one to three weeks. Near the end of this phase, the patient's acid-base, electrolyte, and waste product (blood urea nitrogen, creatinine) values begin to normalize. Although urine output is increasing, the nephrons are still not fully functional. The high urine volume is caused by osmotic diuresis from the high urea concentration in the glomerular filtrate and the inability of the tubules to concentrate the urine. In this phase the kidneys have recovered their ability to excrete wastes, but not to concentrate the urine. Because of the large losses of fluid and electrolytes, the patient must be monitored for hyponatremia, hypokalemia, and dehydration. Text Reference - p. 1104

A nurse caring for a patient with multiple organ dysfunction syndrome understands that the patient may be at increased risk of bleeding. What nursing interventions should the nurse perform to manage this patient? Select all that apply. 1 Observe bleeding sites. 2 Decrease fluid intake. 3 Provide enteral feedings. 4 Administer platelets and clotting factors. 5 Minimize traumatic interventions.

1, 4, 5 The patient with multiple organ dysfunction syndrome is at a risk of bleeding due to increased bleeding time, thrombocytopenia, and dysfunctional clotting process. The nursing interventions should be aimed at preventing potential bleeding and replacing factors being lost. The patient should be observed for frank or occult bleeding from potential sites. The factors like platelets and clotting factors should be replaced if deficient. Traumatic interventions such as intramuscular injections or multiple venipunctures should be avoided. Decreasing the fluid intake and providing enteral feedings will not help in minimizing hematologic complications. Text Reference - p. 1650

Which laboratory value should the nurse review to determine whether a patient's hypothyroidism is caused by a problem with the anterior pituitary gland or with the thyroid gland? a. Thyroxine (T4) level b. Triiodothyronine (T3) level c. Thyroid-stimulating hormone (TSH) level d. Thyrotropin-releasing hormone (TRH) level

C

When computing a heart rate from the electrocardiography (ECG) tracing, the nurse counts 15 of the small blocks between the R waves of a patient whose rhythm is regular. What does the nurse calculate the patient's heart rate to be? 60 beats/min 75 beats/min 100 beats/min 150 beats/min

100 beats/min Because each small block on the ECG paper represents 0.04 seconds, 1500 of these blocks represents 1 minute. By dividing the number of small blocks (15, in this case) into 1500, the nurse can calculate the heart rate in a patient whose rhythm is regular (in this case, 100).

15. A patient is admitted with an open fracture of the tibia following a bicycle accident. During assessment of the patient, what specifically should the nurse question the patient about? a. Any previous injuries to the leg b. The status of tetanus immunization c. The use of antibiotics in the last month d. Whether the injury was exposed to dirt or gravel

15. b. Infection is the greatest risk with an open fracture and all open fractures are considered contaminated. Tetanus prevention is always indicated if the patient has not been immunized or does not have current boosters. Prophylactic antibiotics are often used in management of open fractures but recent antibiotic therapy is not relevant, nor is previous injury to the site.

The nurse teaches safety measures to a patient with chronic kidney disease (CKD) who is experiencing constipation. The patient's blood pressure is 145/95 mm Hg. Which statement made by the patient indicates effective learning? 1 "I should eat three bananas after every meal." 2 "I should monitor my blood pressure regularly at home." 3 "I should rest in a prone position while recording my blood pressure." 4 "I should take magnesium-containing laxatives if I am experiencing constipation."

2 A patient with chronic kidney disease (CKD) and hypertension has to monitor blood pressure at home regularly. Controlling blood pressure helps to slow the incidence of atherosclerosis that further impairs kidney function. The patient with CKD has an elevated level of serum potassium and ingestion of bananas may aggravate the condition and lead to fatal dysrhythmias. The patient should be in supine position while measuring blood pressure, not in prone position. The patient may develop hypermagnesemia from taking magnesium-containing laxatives. Text Reference - p. 1113

The nurse has the following tasks to perform. Which is an appropriate task to delegate to the unlicensed assistive personnel (UAP)? 1 Document intake and output on the patient performing bedside peritoneal dialysis 2 Obtain a finger stick blood sugar on the patient receiving hemodialysis 3 Ambulate the patient who is postoperative day one following a right-sided nephrectomy 4 Report the patient's potassium level of 5.2 to the primary health care provider

2 It is within the scope of practice of the UAP to obtain a finger stick blood glucose level. It is not within the UAP scope of practice to assess the intake and output during a peritoneal dialysis exchange. The patient postoperative day one will need a nursing assessment on his or her ability to ambulate, as well as a pain assessment. UAP do not report any results to health care providers. Text Reference - p. 1117

What causes prerenal acute kidney injury? 1 Release of nephrotoxins 2 Reduced renal blood flow 3 Urine reflux into renal pelvis 4 Presence of extrarenal tumors

2 Prerenal acute kidney injury can be caused by a reduced flow of blood to the kidneys. A release of nephrotoxins is an intrarenal cause of acute kidney injury. Urine reflux into the renal pelvis and the presence of extrarenal tumors are postrenal causes of acute kidney injury. Text Reference - p. 1103

When examining a patient in the progressive stage of shock, which factors related to the gastrointestinal (GI) system should the nurse consider? Select all that apply. 1 Increased motility and peristalsis 2 Increased likelihood of GI ulcers 3 Increased ability to absorb nutrients 4 Increased risk of GI bleeding 5 Increased risk of bacterial migration from the GI tract to the bloodstream.

2, 4, 5 In the progressive stage of shock, the GI system gets affected by prolonged decreased tissue perfusion. As the blood supply to the GI tract is decreased, the normally protective mucosal barrier becomes ischemic. This ischemia predisposes the patient to ulcers and GI bleeding. It also increases the risk of bacterial migration from the GI tract to the blood. The decreased perfusion to the GI tract also results in a decreased ability to absorb nutrients, decreased motility, and slowed peristalsis. Text Reference - p. 1639

A patient who has been on hemodialysis for several weeks asks the nurse what substances are being removed by the dialysis. What substances should the nurse tell the patient are passing through the membrane during hemodialysis? Select all that apply. 1 Red blood cells (RBCs) 2 Creatinine 3 Glucose 4 Bacteria 5 Sodium

2, 5 Creatinine, urea, uric acid, and electrolytes such as sodium and potassium are filtered by the semipermeable membrane during hemodialysis. RBCs do not pass through the semipermeable membrane during hemodialysis because of their molecular weight. Glucose does not pass through the semipermeable membrane during hemodialysis due to the osmotic difference of the dialysate. Bacteria do not pass through the semipermeable membrane during hemodialysis due to their high molecular weight. Text Reference - p. 1117

What is the correct order of the pathophysiologic steps involved in systemic inflammatory response syndrome (SIRS)? 1. Activation of coagulation cascade 2. Release of mediators 3. Increase in vascular permeability 4. Leakage of mediators and proteins into interstitial space 5. Digestion of foreign debris by white blood cells

2. Release of mediators 3. Increase in vascular permeability 4. Leakage of mediators and proteins into interstitial space 5. Digestion of foreign debris by white blood cells 1. Activation of coagulation cascade Release of mediators occurs in a patient with systemic inflammatory response syndrome (SIRS) when the inflammatory response is activated. This release increases the vascular permeability. As a result, mediators and proteins leak out of the blood vessels into the interstitial space. White blood cells digest the foreign debris and the coagulation cascade is activated. Text Reference - p. 1649

20. What surgical treatment will the nurse prepare the patient for in the presence of compartment syndrome? a. Fasciotomy b. Amputation c. Internal fixation d. Release of tendons

20. a. Soft tissue edema in the area of the injury may cause an increase of pressure within the closed spaces of the tissue compartments formed by the nonelastic fascia, creating compartment syndrome. If symptoms occur, it may be necessary to incise the fascia surgically, a procedure known as a fasciotomy. Amputation is usually necessary only if the limb becomes septic because of untreated compartment syndrome.

21. Which type of fracture occurred when there is radial nerve and brachial artery damage and the fracture is reduced with a hanging arm cast? a. Fractured tibia b. Colles' fracture c. Fractured humerus d. Femoral shaft fracture

21. c. The fractured humerus may cause radial nerve and brachial artery damage and it may be reduced nonsurgically with a hanging arm cast. A fractured tibia and femoral shaft are in the leg. The Colles' fracture is in the wrist and manifests with pronounced swelling and obvious deformity of the wrist; it is treated with closed manipulation and immobilization.

29. A patient with an extracapsular hip fracture is admitted to the orthopedic unit and placed in Buck's traction. The nurse explains to the patient that the purpose of the traction is to do what? a. Pull bone fragments back into alignment b. Immobilize the leg until healing is complete c. Reduce pain and muscle spasms before surgery d. Prevent damage to the blood vessels at the fracture site

29. c. Although surgical repair is the preferred method of managing intracapsular and extracapsular hip fractures, initially patients frequently may be treated with skin traction, such as Buck's traction or Russell's traction, to immobilize the limb temporarily and to relieve the painful muscle spasms before surgery is performed. Prolonged traction would be required to reduce the fracture or immobilize it for healing, creating a very high risk for complications of immobility.

The nursing instructor asks the student nurse about fluid and electrolyte changes that occur in a patient with an acute kidney injury. Which statement by the student nurse indicates effective learning? 1 "The patient will have hypokalemia." 2 "The patient will have hypernatremia." 3 "The patient will have increased serum creatinine levels." 4 "The patient will have decreased levels of blood urea nitrogen."

3 Creatinine is a waste product of muscle catabolism. Patients with acute kidney injury cannot remove body waste and it accumulates in the blood, which raises the serum creatinine level. Acute kidney injury is associated with an increased level of potassium, a decreased level of sodium, and a decreased level of blood urea nitrogen. Thus, the statements that the patient will have hypokalemia, hypernatremia, and decreased levels of blood urea nitrogen are incorrect. Text Reference - p. 1104

The nurse is administering oxygen therapy to a patient in septic shock. What are the possible factors that directly affect oxygen delivery in the patient and should be monitored? Select all that apply. 1 Urine output 2 White blood cells 3 Cardiac output 4 Available hemoglobin 5 Arterial oxygen saturation

3, 4, 5 Oxygen delivery depends on cardiac output, available hemoglobin, and arterial oxygen saturation (SaO2). The amount of blood that the heart pumps to the body may decide the amount of oxygen delivered to the tissues. Hemoglobin, the protein content in red blood cells, is responsible for carrying oxygen molecules. Low hemoglobin means a low oxygen supply to the tissues. Arterial oxygen saturation indicates the total oxygen carried by the blood in the arteries and implies the level of tissue oxygenation. The urine output and white blood cells are also important parameters to be monitored in a patient who suffers from a shock. However, these do not impact the oxygen delivery directly. Text Reference - p. 1641

3. The patient asks, "What does the doctor mean when he says that I have an avulsion fracture in my leg? I thought I had a sprain!" What is the best response by the nurse? a. "It is a fracture with more than two fragments." b. "It means that a ligament pulled a bone fragment loose." c. "The line of the fracture is twisted along the shaft of the bone." d. "The line of the fracture is at right angles to the longitudinal axis of the bone."

3. b. An avulsion fracture occurs when a ligament pulls a bone fragment loose, with pain similar to a sprain. A fracture with two or more fragments is a comminuted fracture. It is a spiral fracture when it is twisted around a bone shaft. It is a transverse fracture when the line of fracture is at right angles to the longitudinal axis.

33. Priority Decision: Twenty-four hours after a below-the-knee amputation, a patient uses the call system to tell the nurse that his dressing (a compression bandage) has fallen off. What is the first action that the nurse should take? a. Apply ice to the site. b. Cover the incision with dry gauze. c. Reapply the compression dressing. d. Elevate the extremity on a couple of pillows.

33. c. The compression dressing or bandage supports the soft tissues, reduces edema, hastens healing, minimizes pain, and promotes residual limb shrinkage. If the dressing is left off, edema will form quickly and may delay rehabilitation. Elevation and ice will not be as effective at preventing the edema that will form. Dressing the incision with dry gauze will not provide the benefits of a compression dressing.

The nurse recalls that the reason that patients with chronic kidney disease experience arterial stiffness is what? 1 Excessive sodium retention 2 Decrease in the sodium bicarbonate level 3 Increase in nitrogenous waste products 4 Excessive calcium deposition in vascular smooth layer

4 A patient with chronic kidney disease (CKD) may have arterial stiffness due to calcium deposition in the vascular smooth layer of the blood vessels. Excessive sodium retention causes extracellular fluid accumulation that leads to hypertension and edema. Decrease in the sodium bicarbonate level in the body leads to metabolic acidosis. Accumulation of the nitrogenous waste products leads to neurologic complications. Text Reference - p. 1110

What laboratory finding correlates with a medical diagnosis of cardiogenic shock? 1 Decreased liver enzymes 2 Increased white blood cells 3 Decreased red blood cells, hemoglobin, and hematocrit 4 Increased blood urea nitrogen (BUN) and serum creatinine levels

4 The renal hypoperfusion that accompanies cardiogenic shock results in increased BUN and creatinine levels. Impaired perfusion of the liver results in increased liver enzymes, whereas white blood cell levels typically do not increase in cardiogenic shock. Red blood cell indices are typically normal because of relative hypovolemia. Test-Taking Tip: Bring to your test prep with a positive attitude about yourself, your nursing knowledge, and your test-taking abilities. A positive attitude is achieved through self-confidence gained by effective study. This means (a) answering questions (assessment), (b) organizing study time (planning), (c) reading and further study (implementation), and (d) answering questions (evaluation). Text Reference - p. 1633

Which sign of neurologic dysfunction is commonly seen in both systemic inflammatory response syndrome (SIRS) and multiple organ dysfunction syndrome (MODS)? 1 Increased heart rate 2 Increased liver enzymes 3 Difficulty breathing 4 Confusion, agitation, and lethargy

4 The sign of neurologic dysfunction commonly seen in systemic inflammatory response syndrome (SIRS) and multiple organ dysfunction syndrome (MODS) is change in mental status, which may cause the patient to become confused, agitated, and lethargic. The patient's heart rate increases due to changes in the cardiovascular system. The patient's liver enzymes increase due to dysfunction in the hepatic system and finally the dysfunction results in hepatic encephalopathy. The patient's dyspnea is caused by changes in the respiratory system by inflammatory mediators. Test-Taking Tip: Identify option components as correct or incorrect. This may help you identify a wrong answer. Text Reference - p. 1649

The nurse reviews the plan of care for a patient with multisystem organ dysfunction syndrome. What is the most desirable outcome for the patient? 1 The patient will be free of signs and symptoms of sepsis 2 The patient will maintain a balanced fluid intake and output 3 The patient will experience enhanced overall well-being and mental rest 4 The patient will demonstrate improved perfusion and oxygenation of organs

4 The underlying pathophysiology of multisystem organ dysfunction syndrome (MODS) is a lack of perfusion to organs, resulting in tissue and/or organ hypoxia. Interventions to improve perfusion with fluids or medications improve patient outcomes. The outcomes listed in the other answer options are appropriate and desirable for the patient with MODS, but they are secondary to improved perfusion and oxygenation. Text Reference - p. 1647

Which factor indicates that tracheostomy placement would be preferable to endotracheal intubation? The patient is unable to clear secretions. The patient is at high risk for aspiration. A long-term airway is probably necessary. An upper airway obstruction is impairing the patient's ventilation.

A long-term airway is probably necessary. A tracheostomy is indicated when the need for an artificial airway is expected to be long term. Aspiration risk, an inability to clear secretions, and upper airway obstruction are indications for an artificial airway, but these are not specific indications for tracheostomy.

Which question will provide the most useful information to a nurse who is interviewing a patient about a possible thyroid disorder? a. "What methods do you use to help cope with stress?" b. "Have you experienced any blurring or double vision?" c. "Have you had a recent unplanned weight gain or loss?" d. "Do you have to get up at night to empty your bladder?"

C

4. The patient with osteoporosis had a spontaneous hip fracture. How should the nurse document this before the x-ray results return? a. Open fracture b. Oblique fracture c. Pathologic fracture d. Greenstick fracture

4. c. A pathologic fracture is a spontaneous fracture at the site of bone disease, such as osteoporosis. An open fracture is when there is communication with the external environment. The oblique fracture has a slanted fracture line. A greenstick fracture is splintered on one side and the other side is bent.

41. When positioning the patient after a total hip arthroplasty with a posterior approach, it is important that the nurse maintain the affected extremity in what position? a. Adduction and flexion b. Abduction and extension c. Abduction and internal rotation d. Adduction and external rotation

41. b. Following a total hip arthroplasty with a posterior approach, during hospitalization an abduction pillow is placed between the legs to maintain abduction and the leg is extended. Extremes of internal rotation, adduction, and 90-degree flexion of the hip must be avoided for 4 to 6 weeks postoperatively to prevent dislocation of the prosthesis.

Which patient is at greatest risk for sudden cardiac death (SCD)? A 42-yr-old white woman with hypertension and dyslipidemia A 52-yr-old African American man with left ventricular failure A 62-yr-old obese man with diabetes mellitus and high cholesterol A 72-yr-old Native American woman with a family history of heart disease

A 52-yr-old African American man with left ventricular failure Patients with left ventricular dysfunction (ejection fraction <30%) and ventricular dysrhythmias after myocardial infarction are at greatest risk for SCD. Other risk factors for SCD include (1) male gender (especially African American men), (2) family history of premature atherosclerosis, (3) tobacco use, (4) diabetes mellitus, (5) hypercholesterolemia, (6) hypertension, and (7) cardiomyopathy.

The nurse is monitoring a patient for increased ICP following a head injury. Which of the following manifestations indicate an increased ICP (select all that apply) a. fever b. oriented to name only c. narrowing pulse pressure d. dilated right pupil > left pupil e. decorticate posturing to painful stimulus

A, B, D, E- The first sign of increased ICP is a change in LOC. Other manifestations are dilated ipsilateral pupil, changes in motor response such as posturing, and fever, which may indicate pressure on the hypothalamus. Changes in vital signs would be an increased systolic BP with widened pulse pressure and bradycardia

27. Which action by a new registered nurse (RN) who is orienting to the progressive care unit indicates a good understanding of the treatment of cardiac dysrhythmias? a. Injects IV adenosine (Adenocard) over 2 seconds to a patient with supraventricular tachycardia b. Obtains the defibrillator and quickly brings it to the bedside of a patient whose monitor shows asystole c. Turns the synchronizer switch to the "on" position before defibrillating a patient with ventricular fibrillation d. Gives the prescribed dose of diltiazem (Cardizem) to a patient with new-onset type II second degree AV block

ANS: A Adenosine must be given over 1 to 2 seconds to be effective. The other actions indicate a need for more education about treatment of cardiac dysrhythmias. The RN should hold the diltiazem until talking to the health care provider. The treatment for asystole is immediate CPR. The synchronizer switch should be "off" when defibrillating. DIF: Cognitive Level: Analyze (analysis) REF: 795 OBJ: Special Questions: Multiple Patients TOP: Nursing Process: Evaluation MSC: NCLEX: Safe and Effective Care Environment

15. A patient with a right lower leg fracture will be discharged home with an external fixation device in place. Which information will the nurse teach? a. "You will need to check and clean the pin insertion sites daily." b. "The external fixator can be removed for your bath or shower." c. "You will need to remain on bed rest until bone healing is complete." d. "Prophylactic antibiotics are used until the external fixator is removed."

ANS: A Pin insertion sites should be cleaned daily to decrease the risk for infection at the site. An external fixator allows the patient to be out of bed and avoid the risks of prolonged immobility. The device is surgically placed and is not removed until the bone is stable. Prophylactic antibiotics are not routinely given when an external fixator is used.

3. The occupational health nurse will teach the patient whose job involves many hours of typing about the need to a. obtain a keyboard pad to support the wrist. b. do stretching exercises before starting work. c. wrap the wrists with compression bandages every morning. d. avoid using nonsteroidal antiinflammatory drugs (NSAIDs) for pain.

ANS: A Repetitive strain injuries caused by prolonged times working at a keyboard can be prevented by the use of a pad that will keep the wrists in a straight position. Stretching exercises during the day may be helpful, but these would not be needed before starting. Use of a compression bandage is not needed, although a splint may be used for carpal tunnel syndrome. NSAIDs are appropriate to use to decrease swelling.

30. Which nursing action for a patient who has had right hip replacement surgery can the nurse delegate to experienced unlicensed assistive personnel (UAP)? a. Reposition the patient every 1 to 2 hours. b. Assess for skin irritation on the patient's back. c. Teach the patient quadriceps-setting exercises. d. Determine the patient's pain level and tolerance.

ANS: A Repositioning of orthopedic patients is within the scope of practice of UAP (after they have been trained and evaluated in this skill). The other actions should be done by licensed nursing staff members. (B and D are assessments, C is teaching.)

A 62-year old man with chronic anemia is experiencing increased fatigue and occasional palpitations at rest. The nurse would expect the patient's laboratory findings to include a. a hematocrit (Hct) of 38%. b. an RBC count of 4,500,000/mL. c. normal red blood cell (RBC) indices. d. a hemoglobin (Hgb) of 8.6 g/dL (86 g/L).

ANS: D The patient's clinical manifestations indicate moderate anemia, which is consistent with a Hgb of 6 to 10 g/dL. The other values are all within the range of normal

The nurse is admitting a 45-yr-old patient with asthma in acute respiratory distress. The nurse auscultates the patient's lungs and notes cessation of the inspiratory wheezing. The patient has not yet received any medication. What should this finding suggest to the nurse? Spontaneous resolution of the acute asthma attack An acute development of bilateral pleural effusions Airway constriction requiring immediate interventions Overworked intercostal muscles resulting in poor air exchange

Airway constriction requiring immediate interventions When a patient in respiratory distress has inspiratory wheezing and then it ceases, it is an indication of airway obstruction. This finding requires emergency action to restore airway patency. Cessation of inspiratory wheezing does not indicate spontaneous resolution of the acute asthma attack, bilateral pleural effusion development, or overworked intercostal muscles in this asthmatic patient that is in acute respiratory distress.

The nurse is providing emergent care for a patient with a possible inhalation injury sustained in a house fire. The patient is anxious and disoriented, and the skin is a cherry red color. Which action should the nurse take first? A. Assist the patient to a high Fowler's position. B. Teach the patient deep breathing exercises. C. Allow the patient to verbalize feelings. D. Administer 100% humidified oxygen.

Answer: D. Administer 100% humidified oxygen. Rationale: Carbon monoxide (CO) poisoning may occur in house fires; CO displaces oxygen on the hemoglobin molecule resulting in hypoxia. High levels of CO in the blood result in a skin color that is described as cherry red. Hypoxia may cause anxious behaviors and altered mental status. Emergency treatment for inhalation injury and CO poisoning includes the immediate administration of 100% humidified oxygen. The other interventions are appropriate for inhalation injury but are not as emergent as oxygen administration. Ch. 25

When caring for older adult patients with respiratory failure, the nurse will add which intervention to individualize care? Position the patient in the supine position primarily. Assess frequently for signs and symptoms of delirium. Provide early endotracheal intubation to reduce complications. Delay activity and ambulation to provide additional healing time.

Assess frequently for signs and symptoms of delirium. Older adult patients are more predisposed to factors such as delirium, health care associated infections, and polypharmacy. Individualizing the older patient's care plan to address these factors will improve care. Older adult patients are not required to remain in a supine position only and should increase activity as soon as stability is determined. Endotracheal intubation is not provided early, and noninvasive positive pressure ventilation may be considered as an alternative. The nurse should consider that the aging process leads to decreased lung elastic recoil, weakened lung muscles and reduced gas exchange, which may make the patient difficult to wean from the ventilator.

A 44-year-old patient is admitted with tetany. Which laboratory value should the nurse monitor? a. Total protein b. Blood glucose c. Ionized calcium d. Serum phosphate

C

Nursing management of a patient with a brain tumor includes (select all that apply) a. discussing with the patient methods to control appropriate behavior b. using diversion techniques to keep the patient stimulated and motivated c. assisting and supporting the family in understanding any changes in behavior d. limiting self care activities until the patient has regained maximal physical functioning e. plan for seizure precautions and teaching the patient and caregiver about antiseizure drugs.

C, E

When preparing to administer an ordered blood transfusion, which IV solution does the nurse use when priming the blood tubing? A. Lactated Ringer's B. 5% dextrose in water C. 0.9% sodium chloride D. 0.45% sodium chloride

C. 0.9% sodium chloride The blood set should be primed before the transfusion with 0.9% sodium chloride, also known as normal saline. It is also used to flush the blood tubing after the infusion is complete to ensure the patient receives blood that is left in the tubing when the bag is empty. Dextrose and lactated Ringer's solutions cannot be used with blood because they will cause RBC hemolysis.

The nurse is alerted to possible acute subdural hematoma in the patient who a. has a linear skull fracture crossing a major artery b. has focal symptoms of brain damage with no recollection of a head injury c. develops decreased LOC and a headache within 48 hours of head injury d. has an immediate loss of consciousness with a brief lucid interval followed by decreasing LOC

C. develops decreased LOC and a headache within 48 hours of head injury

When evaluating a patient's knowledge regarding a low-sodium, low-fat cardiac diet, the nurse recognizes additional teaching is needed when the patient selects which food? Baked flounder Angel food cake Baked potato with margarine Canned chicken noodle soup

Canned chicken noodle soup Canned soups are very high in sodium content. Patients need to be taught to read food labels for sodium and fat content.

A patient admitted to the emergency department 24 hours ago with complaints of chest pain was diagnosed with a ST-segment-elevation myocardial infarction (STEMI). What complication of myocardial infarction should the nurse anticipate? - Unstable angina - Cardiac tamponade - Sudden cardiac death - Cardiac dysrhythmias

Cardiac dysrhythmias *Dysrhythmias are present in 80% to 90% of patients after myocardial infarction (MI).* Unstable angina is considered a precursor to MI rather than a complication. Cardiac tamponade is a rare event, and sudden cardiac death is defined as an unexpected death from cardiac causes. Cardiac dysfunction in the period following an MI would not be characterized as sudden cardiac death.

RIFLE defines three stages of AKI based on changes in: A. blood pressure and urine osmolality B. fractional excretion of urinary sodium C. estimation of GFR with the MDRD equation D. serum creatinine or urine output from baseline

D. serum creatinine or urine output from baseline

The patient had tibia and fibula fractures repaired using open reduction internal fixation. A fiberglass cast is in place. She wants to know when she can resume exercise classes. To answer this question, the nurse must understand the stages of union occur in what order? (Answer with a letter followed by a comma and a space (e.g. A, B, C, D).) a. Ossification b. Granulation c. Remodeling d. Consolidation e. Callus formation f. Fracture hematoma

F, B, E, A, D, C A fracture hematoma occurs in the first 72 hours after the fracture injury. Granulation produces the basis for new bone substance 3 to 14 days after injury. Callus formation (composed of cartilage, osteoblasts, calcium, and phosphorus) appears by the end of the second week after injury. Ossification of the callus will prevent movement at the fracture when the bones are gently stressed and occurs from 3 weeks to 6 months after the fracture and continues until the fracture is healed. Consolidation occurs when the distance between bone fragments diminishes and there is radiologic evidence of union. Remodeling is the reabsorption of excess bone tissue in the final stage of bone healing; it occurs in response to gradually increased stress on the bone or weight bearing.

The home care nurse visits a 74-yr-old man diagnosed with Parkinson's disease who fell while walking this morning. What observation is of most concern to the nurse? 2 × 6 cm right calf abrasion with sanguineous drainage Left leg externally rotated and shorter than the right leg Stooped posture with a shuffling gait and slow movements Mild pain and minimal swelling of the right ankle and foot

Left leg externally rotated and shorter than the right leg Clinical manifestations of hip fracture include external rotation, muscle spasm, shortening of the affected extremity, and severe pain and tenderness in the region of the fracture site. Expected clinical manifestations of Parkinson's disease include a stooped posture, shuffling gait, and slow movements. An abrasion is a soft tissue injury. Mild pain and minimal swelling may occur with a sprain or strain.

In reviewing the chart, which patient assessment is likely to have the greatest impact on this patient's risk of death from the accident? - Male - Found floating face down after surfing accident - CPR done by rescuers - Sinus tachycardia with frequent premature ventricular contractions (PVCs) - Mechanical ventilation - Left pupil size 10 cm, not reactive to light - Pulmonary artery wedge pressure (PAWP) 16 mm Hg - PaO2 108 mm Hg, FIO2 50%, PEEP 5 cm - Cool extremities, weak peripheral pulses

Left pupil 10 cm, not reactive to light Unilateral pupil dilation without response to light can be a clinical indicator of tentorial herniation of the brain and can occur in a surfing accident as the surfboard and patient are forcefully tossed around in the waves. If the excessive intracranial pressure is allowed to continue, the patient is at a high risk for brainstem death. This finding merits emergency interventions to prevent death. The PAWP, sinus tachycardia with frequent PVCs, and cool extremities with weak peripheral pulses do not indicate imminent death.

A 74-yr-old man with a history of prostate cancer and hypertension is admitted to the emergency department with substernal chest pain. Which action will the nurse complete before administering sublingual nitroglycerin? Administer morphine sulfate IV. Auscultate heart and lung sounds. Obtain a 12-lead electrocardiogram (ECG). Assess for coronary artery disease risk factors.

Obtain a 12-lead electrocardiogram (ECG). If a patient has chest pain, the nurse should institute the following measures: (1) administer supplemental oxygen and position the patient in upright position unless contraindicated, (2) assess vital signs, (3) obtain a 12-lead ECG, (4) provide prompt pain relief first with a nitrate followed by an opioid analgesic if needed, and (5) auscultate heart sounds. Obtaining a 12-lead ECG during chest pain aids in the diagnosis.

A 72-yr-old woman with aspiration pneumonia develops severe respiratory distress. Her PaO2 is 42 mmHg and FIO2 is 80%. Which intervention should the nurse complete first? Stat portable chest radiography Administer lorazepam (Ativan) 1 mg IV push Place the patient in a prone position on a rotational bed Position the patient with arms supported away from the chest

Position the patient with arms supported away from the chest The nurse will first position the patient to facilitate ventilation. Additional oxygen support may be necessary. Refractory hypoxemia indicates the patient is not demonstrating acute lung injury but has now developed acute respiratory distress syndrome (ARDS). If the PaO2 is 42 mm Hg on 80% FIO2 (fraction of inspired oxygen; room air is 21% FIO2), then the PaO2/FIO2 ratio is 52.5, indicating ARDS (PaO2/FIO2 ratio<200). Stat portable chest radiography may show worsening infiltrates or "white lung." A rotational bed placing the patient in prone position would be a strategy to use for select patients with ARDS. This patient's age, diagnosis, and comorbidities may indicate appropriateness for this treatment. Administration of lorazepam (Ativan) 1 mg may be harmful to this patient's oxygenation status. Further assessment would be needed to determine safety.

The nurse observes ventricular tachycardia (VT) on the patient's monitor. What evaluation made by the nurse led to this interpretation? Unmeasurable rate and rhythm Rate 150 beats/min; inverted P wave Rate 200 beats/min; P wave not visible Rate 125 beats/min; normal QRS complex

Rate 200 beats/min; P wave not visible VT is associated with a rate of 150 to 250 beats/min; the P wave is not normally visible. Rate and rhythm are not measurable in ventricular fibrillation. P wave inversion and a normal QRS complex are not associated with VT.

The nurse is caring for a 68-year-old man who had coronary artery bypass surgery 3 weeks ago. If the patient is now is in the oliguric phase of acute kidney disease, which action would be appropriate to include in the plan of care? A.Provide foods high in potassium. B.Restrict fluids based on urine output. C.Monitor output from peritoneal dialysis D.Offer high protein snacks between meals.

Restrict fluids based on urine output. Fluid intake is monitored during the oliguric phase. Fluid intake is determined by adding all losses for the previous 24 hours plus 600 mL. Potassium and protein intake may be limited in the oliguric phase to avoid hyperkalemia and elevated urea nitrogen. Hemodialysis, not peritoneal dialysis, is indicated in acute kidney injury if dialysis is needed.

The nurse in collaboration with respiratory therapy is determining a patient's readiness to wean from the ventilator. Which finding indicates the patient is not a candidate for weaning (select all that apply.)? Select all that apply. Minute volume of 8 L/min Patient follow commands Serum hemoglobin of 6 g/dL Respirations of 28 breaths/min Mean arterial pressure (MAP) of 45 mm Hg Negative inspiratory force (NIF) of -15 cm H2O

Serum hemoglobin of 6 g/dL Mean arterial pressure (MAP) of 45 mm Hg Negative inspiratory force (NIF) of -15 cm H2O Findings that support readiness for weaning are minute volume of 8 L/min, patient is alert and follow commands, and respirations of 28 breaths/min. Findings that indicate the patient is not ready for weaning include serum hemoglobin of 6 g/dL, mean arterial pressure (MAP) of 45 mm Hg, and negative inspiratory force (NIF) of -15 cm H2O. Extubating a patient with severe anemia, poor perfusion, and weakened breathing effort will likely result in poor outcomes such as worsening of condition and reintubation.

Which statement best describes the electrical activity of the heart represented by measuring the PR interval on the electrocardiogram (ECG)? The length of time it takes to depolarize the atrium The length of time it takes for the atria to depolarize and repolarize The length of time for the electrical impulse to travel from the sinoatrial (SA) node to the Purkinje fibers The length of time it takes for the electrical impulse to travel from the sinoatrial (SA) node to the atrioventricular (AV) node

The length of time for the electrical impulse to travel from the sinoatrial (SA) node to the Purkinje fibers The electrical impulse in the heart must travel from the SA node through the AV node and into the Purkinje fibers in order for synchronous atrial and ventricular contraction to occur. When measuring the PR interval (the time from the beginning of the P wave to the beginning of the QRS), the nurse is identifying the length of time it takes for the electrical impulse to travel from the SA node to the Purkinje fibers. The P wave represents the length of time it takes for the impulse to travel from the SA node through the atrium, causing depolarization of the atria (atrial contraction). Atrial repolarization occurs during ventricular depolarization and is hidden by the QRS complex. The length of time it takes for the electrical impulse to travel from the SA node to the AV node is the flat line between the end of the P wave and the beginning of the Q wave on the ECG and is not usually measured.

When providing nutritional counseling for patients at risk for coronary artery disease (CAD), which foods would the nurse encourage patients to include in their diet (select all that apply.)? Select all that apply. - Tofu - Walnuts - Tuna fish - Whole milk - Orange juice

Tofu Walnuts Tuna fish *"next time on Chopped!"*

Tx for gallstones: Trans-hepatic biliary catheter

Trans-hepatic biliary catheter - percutaneous catheter to allow free drainage either before or instead of cholecystectomy. - Catheters changed q 4-6 weeks under imaging. - Risk of access site infection - Replace draining fluids c gatorade/pedialyte

difference between *cholecystitis* and *cholelithiasis*? which one is more common?

*cholecystitis* - chole = gall, bile cyst = bladder itis = inflammation *cholelithiasis* - more common chole = gall, bile lith = stone iasis = condition of

Management of cholecystitis

- pain control - opioids - antispasmodics - anticholinergics - antiemetics - abx if possible infection - NGT/decompression if severe N/V - cholecystostomy to drain purulence

If the patient in shock is to receive 1000 mL of normal saline in two hours, at what rate should the infusion pump be set? Fill in the blank. ___mL/hour

500 For the 1000 mL of normal saline to be infused in two hours, the infusion pump should be set at 500 mL per hour (1000 mL divided by two hours). Test-Taking Tip: When taking the NCLEX exam, an on-screen calculator will be available for you to determine your response, which you will then type in the provided space. Text Reference - p. 1641

6. The athlete comes to the clinic with bursitis. What does the nurse know happens to the tissue to cause pain when bursitis occurs? a. Tearing of a ligament b. Stretching of muscle and fascia sheath c. Inflammation of synovial membrane sac at friction sites d. Incomplete separation of articular surfaces of joint caused by ligament injury

6. c. Bursitis is inflammation of synovial membrane sac at friction sites. Tearing of a ligament is a sprain. Stretching of muscle and fascia sheath is a strain. Incomplete separation of articular surfaces of joints caused by ligament injury is subluxation.

7. Application of RICE (rest, ice, compression, and elevation) is indicated for initial management of which type of injury? a. Muscle spasms b. Sprains and strains c. Repetitive strain injury d. Dislocations and subluxations

7. b. Application of cold, compression, and elevation are indicated to prevent edema resulting from sprain and some strain injuries. Muscle spasms are usually treated with heat applications and massage and repetitive strain injuries require cessation of the precipitating activity and physical therapy. Dislocations or subluxations require immediate reduction and immobilization to prevent vascular impairment and bone cell death.

The nurse teaches a patient with hypertension that uncontrolled hypertension may damage organs in the body primarily by which mechanism? A Hypertension promotes atherosclerosis and damage to the walls of the arteries. Hypertension causes direct pressure on organs, Bresulting in necrosis and replacement of cells with scar tissue. C Hypertension causes thickening of the capillary membranes, leading to hypoxia of organ systems. D Hypertension increases blood viscosity, which contributes to intravascular coagulation and tissue necrosis distal to occlusions.

A Hypertension promotes atherosclerosis and damage to the walls of the arteries. Hypertension is a major risk factor for the development of atherosclerosis by mechanisms not yet fully known. However, once atherosclerosis develops, it damages the walls of arteries and reduces circulation to target organs and tissues.

Which action will the nurse include in the plan of care for a 72-year-old woman admitted with multiple myeloma? a. Monitor fluid intake and output. b. Administer calcium supplements. c. Assess lymph nodes for enlargement. d. Limit weight bearing and ambulation.

A high fluid intake and urine output helps prevent the complications of kidney stones caused by hypercalcemia and renal failure caused by deposition of Bence-Jones protein in the renal tubules. Weight bearing and ambulation are encouraged to help bone retain calcium. Lymph nodes are not enlarged with multiple myeloma. Calcium supplements will further increase the patient's calcium level and are not used

Which patient would most benefit from noninvasive positive pressure ventilation (NIPPV) to promote oxygenation? A patient whose cardiac output and blood pressure are unstable A patient whose respiratory failure is due to a head injury with loss of consciousness A patient with a diagnosis of cystic fibrosis and who is currently producing copious secretions A patient who is experiencing respiratory failure as a result of the progression of myasthenia gravis

A patient who is experiencing respiratory failure as a result of the progression of myasthenia gravis NIPPV such as continuous positive airway pressure (CPAP) is most effective in treating patients with respiratory failure resulting from chest wall and neuromuscular disease. It is not recommended in patients who are experiencing hemodynamic instability, decreased level of consciousness, or excessive secretions.

14. A 20-year-old has a mandatory electrocardiogram (ECG) before participating on a college soccer team and is found to have sinus bradycardia, rate 52. Blood pressure (BP) is 114/54, and the student denies any health problems. What action by the nurse is most appropriate? a. Allow the student to participate on the soccer team. b. Refer the student to a cardiologist for further diagnostic testing. c. Tell the student to stop playing immediately if any dyspnea occurs. d. Obtain more detailed information about the student's family health history.

ANS: A In an aerobically trained individual, sinus bradycardia is normal. The student's normal BP and negative health history indicate that there is no need for a cardiology referral or for more detailed information about the family's health history. Dyspnea during an aerobic activity such as soccer is normal. DIF: Cognitive Level: Apply (application) REF: 793 TOP: Nursing Process: Implementation MSC: NCLEX: Physiological Integrity

27. After being hospitalized for 3 days with a right femur fracture, a 32-year-old patient suddenly develops shortness of breath and tachypnea. The patient tells the nurse, "I feel like I am going to die!" Which action should the nurse take first? a. Stay with the patient and offer reassurance. b. Administer the prescribed PRN oxygen at 4 L/min. c. Check the patient's legs for swelling or tenderness. d. Notify the health care provider about the symptoms.

ANS: B The patient's clinical manifestations and history are consistent with a pulmonary embolus, and the nurse's first action should be to ensure adequate oxygenation. The nurse should offer reassurance to the patient, but meeting the physiologic need for oxygen is a higher priority. The health care provider should be notified after the oxygen is started and pulse oximetry and assessment for fat embolus or venous thromboembolism (VTE) are obtained.

45. When caring for a patient who is using Buck's traction after a hip fracture, which action can the nurse delegate to unlicensed assistive personnel (UAP)? a. Monitor the skin under the traction boot for redness. b. Ensure that the weight for the traction is off the floor. c. Check for intact sensation and movement in the affected leg. d. Offer reassurance that hip and leg pain are normal after hip fracture.

ANS: B UAP can be responsible for maintaining the integrity of the traction once it has been established. Assessment of skin integrity and circulation should be done by the registered nurse (RN). UAP should notify the RN if the patient experiences hip and leg pain because pain and effectiveness of pain relief measures should be assessed by the RN.

33. When admitting a patient with a non-ST-segment-elevation myocardial infarction (NSTEMI) to the intensive care unit, which action should the nurse perform first? a. Obtain the blood pressure. b. Attach the cardiac monitor. c. Assess the peripheral pulses. d. Auscultate the breath sounds.

ANS: B Because dysrhythmias are the most common complication of myocardial infarction (MI), the first action should be to place the patient on a cardiac monitor. The other actions also are important and should be accomplished as quickly as possible.

39. To improve the physical activity level for a mildly obese 71-year-old patient, which action should the nurse plan to take? a. Stress that weight loss is a major benefit of increased exercise. b. Determine what kind of physical activities the patient usually enjoys. c. Tell the patient that older adults should exercise for no more than 20 minutes at a time. d. Teach the patient to include a short warm-up period at the beginning of physical activity.

ANS: B Because patients are more likely to continue physical activities that they already enjoy, the nurse will plan to ask the patient about preferred activities. The goal for older adults is 30 minutes of moderate activity on most days. Older adults should plan for a longer warm-up period. Benefits of exercises, such as improved activity tolerance, should be emphasized rather than aiming for significant weight loss in older mildly obese adults.

20. When caring for a patient who is recovering from a sudden cardiac death (SCD) event and has no evidence of an acute myocardial infarction (AMI), the nurse will anticipate teaching the patient that a. sudden cardiac death events rarely reoccur. b. additional diagnostic testing will be required. c. long-term anticoagulation therapy will be needed. d. limited physical activity after discharge will be needed to prevent future events.

ANS: B Diagnostic testing (e.g., stress test, Holter monitor, electrophysiologic studies, cardiac catheterization) is used to determine the possible cause of the SCD and treatment options. SCD is likely to recur. Anticoagulation therapy will not have any effect on the incidence of SCD, and SCD can occur even when the patient is resting.

Which information obtained by the nurse caring for a patient with thrombocytopenia should be immediately communicated to the health care provider? a. The platelet count is 52,000/µL. b. The patient is difficult to arouse. c. There are purpura on the oral mucosa. d. There are large bruises on the patient's back.

ANS: B Difficulty in arousing the patient may indicate a cerebral hemorrhage, which is life threatening and requires immediate action. The other information should be documented and reported but would not be unusual in a patient with thrombocytopenia

When assessing a newly admitted patient, the nurse notes a murmur along the left sternal border. To document more information about the murmur, which action will the nurse take next? a. Find the point of maximal impulse. b. Determine the timing of the murmur. c. Compare the apical and radial pulse rates. d. Palpate the quality of the peripheral pulses.

ANS: B Murmurs are caused by turbulent blood flow, such as occurs when blood flows through a damaged valve. Relevant information includes the position in which the murmur is heard best (e.g., sitting and leaning forward), the timing of the murmur in relation to the cardiac cycle (e.g., systole, diastole), and where on the thorax the murmur is heard best. The other information is also important in the cardiac assessment but will not provide information that is relevant to the murmur

The nurse is assessing a patient with myocarditis before administering the scheduled dose of digoxin (Lanoxin). Which finding is most important for the nurse to communicate to the health care provider? a. Leukocytosis b. Irregular pulse c. Generalized myalgia d. Complaint of fatigue

ANS: B Myocarditis predisposes the heart to digoxin-associated dysrhythmias and toxicity. The other findings are common symptoms of myocarditis and there is no urgent need to report these.

10. The nurse will suspect that the patient with stable angina is experiencing a side effect of the prescribed metoprolol (Lopressor) if the a. patient is restless and agitated. b. blood pressure is 90/54 mm Hg. c. patient complains about feeling anxious. d. cardiac monitor shows a heart rate of 61 beats/minute.

ANS: B Patients taking β-adrenergic blockers should be monitored for hypotension and bradycardia. Because this class of medication inhibits the sympathetic nervous system, restlessness, agitation, hypertension, and anxiety will not be side effects.

9. Diltiazem (Cardizem) is ordered for a patient with newly diagnosed Prinzmetal's (variant) angina. When teaching the patient, the nurse will include the information that diltiazem will a. reduce heart palpitations. b. decrease spasm of the coronary arteries. c. increase the force of the heart contractions. d. help prevent plaque from forming in the coronary arteries.

ANS: B Prinzmetal's angina is caused by coronary artery spasm. Calcium channel blockers (e.g., diltiazem, amlodipine [Norvasc

A patient with septicemia develops prolonged bleeding from venipuncture sites and blood in the stools. Which action is most important for the nurse to take? a. Avoid venipunctures. b. Notify the patient's physician. c. Apply sterile dressings to the sites. d. Give prescribed proton-pump inhibitors.

ANS: B The patient's new onset of bleeding and diagnosis of sepsis suggest that disseminated intravascular coagulation (DIC) may have developed, which will require collaborative actions such as diagnostic testing, blood product administration, and heparin administration. The other actions also are appropriate, but the most important action should be to notify the physician so that DIC treatment can be initiated rapidly

2. Administration of hepatitis B vaccine to a healthy 18-year-old patient has been effective when a specimen of the patient's blood reveals a. HBsAg. b. anti-HBs. c. anti-HBc IgG. d. anti-HBc IgM.

ANS: B The presence of surface antibody to HBV (anti-HBs) is a marker of a positive response to the vaccine. The other laboratory values indicate current infection with HBV.

The nurse is planning to administer a transfusion of packed red blood cells (PRBCs) to a patient with blood loss from gastrointestinal hemorrhage. Which action can the nurse delegate to unlicensed assistive personnel (UAP)? a. Verify the patient identification (ID) according to hospital policy. b. Obtain the temperature, blood pressure, and pulse before the transfusion. c. Double-check the product numbers on the PRBCs with the patient ID band. d. Monitor the patient for shortness of breath or chest pain during the transfusion.

ANS: B UAP education includes measurement of vital signs. UAP would report the vital signs to the registered nurse (RN). The other actions require more education and a larger scope of practice and should be done by licensed nursing staff members

36. A 42-year-old patient is admitted to the emergency department with a left femur fracture. Which information obtained by the nurse is most important to report to the health care provider? a. Ecchymosis of the left thigh b. Complaints of severe thigh pain c. Slow capillary refill of the left foot d. Outward pointing toes on the left foot

ANS: C Prolonged capillary refill may indicate complications such as arterial damage or compartment syndrome. The other findings are typical with a left femur fracture

1. A patient who arrives at the emergency department experiencing severe left knee pain is diagnosed with a patellar dislocation. The initial patient teaching by the nurse will focus on the need for a. a knee immobilizer. b. gentle knee flexion. c. monitored anesthesia care. d. physical activity restrictions.

ANS: C The first goal of collaborative management is realignment of the knee to its original anatomic position, which will require anesthesia or monitored anesthesia care (MAC), formerly called conscious sedation. Immobilization, gentle range-of-motion (ROM) exercises, and discussion about activity restrictions will be implemented after the knee is realigned.

A patient has just arrived in the emergency department after an electrical burn from exposure to a high-voltage current. What is the priority nursing assessment? a. Oral temperature b. Peripheral pulses c. Extremity movement d. Pupil reaction to light

ANS: C All patients with electrical burns should be considered at risk for cervical spine injury, and assessments of extremity movement will provide baseline data. The other assessment data are also necessary but not as essential as determining the cervical spine status.

When a patient with splenomegaly is scheduled for splenectomy, which action will the nurse include in the preoperative plan of care? a. Discourage deep breathing to reduce risk for splenic rupture. b. Teach the patient to use ibuprofen (Advil) for left upper quadrant pain. c. Schedule immunization with the pneumococcal vaccine (Pneumovax). d. Avoid the use of acetaminophen (Tylenol) for 2 weeks prior to surgery.

ANS: C Asplenic patients are at high risk for infection with Pneumococcus and immunization reduces this risk. There is no need to avoid acetaminophen use before surgery, but nonsteroidal antiinflammatory drugs (NSAIDs) may increase bleeding risk and should be avoided. The enlarged spleen may decrease respiratory depth and the patient should be encouraged to take deep breaths

25. A 49-year-old female patient with cirrhosis and esophageal varices has a new prescription for propranolol (Inderal). Which finding is the best indicator that the medication has been effective? a. The patient reports no chest pain. b. Blood pressure is 140/90 mm Hg. c. Stools test negative for occult blood. d. The apical pulse rate is 68 beats/minute.

ANS: C Because the purpose of β-blocker therapy for patients with esophageal varices is to decrease the risk for bleeding from esophageal varices, the best indicator of the effectiveness for propranolol is the lack of blood in the stools. Although propranolol is used to treat hypertension, angina, and tachycardia, the purpose for use in this patient is to decrease the risk for bleeding from esophageal varices.

23. A patient who is on the progressive care unit develops atrial flutter, rate 150, with associated dyspnea and chest pain. Which action that is included in the hospital dysrhythmia protocol should the nurse do first? a. Obtain a 12-lead electrocardiogram (ECG). b. Notify the health care provider of the change in rhythm. c. Give supplemental O2 at 2 to 3 L/min via nasal cannula. d. Assess the patient's vital signs including oxygen saturation.

ANS: C Because this patient has dyspnea and chest pain in association with the new rhythm, the nurse's initial actions should be to address the patient's airway, breathing, and circulation (ABC) by starting with oxygen administration. The other actions also are important and should be implemented rapidly. DIF: Cognitive Level: Apply (application) REF: 793 OBJ: Special Questions: Prioritization TOP: Nursing Process: Implementation MSC: NCLEX: Physiological Integrity *1) O2 to keep them alive 2) VS, while those are going place stickers for 3) ECG-12 then 4) address pain (MS, NTG) after bc drugs can affect ECG*

When caring for a patient with infective endocarditis of the tricuspid valve, the nurse should monitor the patient for the development of a. flank pain. b. splenomegaly. c. shortness of breath. d. mental status changes.

ANS: C Embolization from the tricuspid valve would cause symptoms of pulmonary embolus. Flank pain, changes in mental status, and splenomegaly would be associated with embolization from the left-sided valves. *IE -> valve vegetations -> tricuspid valve is btwn R atrium & ventricle so if veg there breaks off it will go to the lungs*

Which instruction will the nurse plan to include in discharge teaching for the patient admitted with a sickle cell crisis? a. Take a daily multivitamin with iron. b. Limit fluids to 2 to 3 quarts per day. c. Avoid exposure to crowds when possible. d. Drink only two caffeinated beverages daily.

ANS: C Exposure to crowds increases the patient's risk for infection, the most common cause of sickle cell crisis. There is no restriction on caffeine use. Iron supplementation is generally not recommended. A high-fluid intake is recommended

The nurse assesses a patient with pernicious anemia. Which assessment finding would the nurse expect? a. Yellow-tinged sclerae b. Shiny, smooth tongue c. Numbness of the extremities d. Gum bleeding and tenderness

ANS: C Extremity numbness is associated with cobalamin (vitamin B12) deficiency or pernicious anemia. Loss of the papillae of the tongue occurs with chronic iron deficiency. Yellow-tinged sclera is associated with hemolytic anemia and the resulting jaundice. Gum bleeding and tenderness occur with thrombocytopenia or neutropenia.

Which parameter will be most important for the nurse to consider when titrating the IV fluid infusion rate immediately after a patient has had kidney transplantation? a. Heart rate b. Blood urea nitrogen (BUN) level c. Urine output d. Creatinine clearance

ANS: C Fluid volume is replaced based on urine output after transplant because the urine output can be as high as a liter an hour. The other data will be monitored but are not the most important determinants of fluid infusion rate.

When the charge nurse is evaluating the skills of a new RN, which action by the new RN indicates a need for more education in the care of patients with shock? a. Placing the pulse oximeter on the ear for a patient with septic shock b. Keeping the head of the bed flat for a patient with hypovolemic shock c. Decreasing the room temperature to 68° F for a patient with neurogenic shock d. Increasing the nitroprusside (Nipride) infusion rate for a patient with a high SVR

ANS: C Patients with *neurogenic* shock may have *poikilothermia*. The room temperature should be kept warm to avoid hypothermia. The other actions by the new RN are appropriate. DIF: Cognitive Level: Application REF: 1721-1722 | 1724 OBJ: Special Questions: Delegation TOP: Nursing Process: Evaluation MSC: NCLEX: Safe and Effective Care Environment

Which assessment is most important for the nurse to make in order to evaluate whether treatment of a patient with anaphylactic shock has been effective? a. Pulse rate b. Orientation c. Blood pressure d. Oxygen saturation

ANS: D Because the airway edema that is associated with anaphylaxis can affect airway and breathing, the oxygen saturation is the most critical assessment. Improvements in the other assessments also will be expected with effective treatment of anaphylactic shock. DIF: Cognitive Level: Application REF: 1724-1725 | 1732 TOP: Nursing Process: Evaluation MSC: NCLEX: Physiological Integrity

When the nurse is monitoring a patient who is undergoing exercise (stress) testing on a treadmill, which assessment finding requires the most rapid action by the nurse? a. Patient complaint of feeling tired b. Pulse change from 87 to 101 beats/minute c. Blood pressure (BP) increase from 134/68 to 150/80 mm Hg d. Newly inverted T waves on the electrocardiogram

ANS: D ECG changes associated with coronary ischemia (such as T-wave inversions and ST segment depression) indicate that the myocardium is not getting adequate oxygen delivery and that the exercise test should be terminated immediately. Increases in BP and heart rate (HR) are normal responses to aerobic exercise. Feeling tired is also normal as the intensity of exercise increases during the stress testing

A 32-year-old patient who is employed as a hairdresser and has a 15 pack-year history of cigarette smoking is scheduled for an annual physical examination. The nurse will plan to teach the patient about the increased risk for a. renal failure. b. kidney stones. c. pyelonephritis. d. bladder cancer.

ANS: D Exposure to the chemicals involved with working as a hairdresser and in smoking both increase the risk of bladder cancer, and the nurse should assess whether the patient understands this risk. The patient is not at increased risk for renal failure, pyelonephritis, or kidney stones.

A patient with cardiogenic shock is cool and clammy and hemodynamic monitoring indicates a high systemic vascular resistance (SVR). Which action will the nurse anticipate taking? a. Increase the rate for the prescribed dopamine (Intropin) infusion. b. Decrease the rate for the prescribed nitroglycerin (Tridil) infusion. c. Decrease the rate for the prescribed 5% dextrose in water (D5W) infusion. d. Increase the rate for the prescribed sodium nitroprusside (Nipride) infusion.

ANS: D Nitroprusside is an arterial vasodilator and will decrease the SVR and afterload, which will improve cardiac output. Changes in the D5W and nitroglycerin infusions will not directly increase SVR. Increasing the dopamine will tend to increase SVR. DIF: Cognitive Level: Application REF: 1733-1734 TOP: Nursing Process: Planning MSC: NCLEX: Physiological Integrity

The nurse is caring for a patient with a diagnosis of immune thrombocytopenic purpura (ITP). What is a priority nursing action in the care of this patient? Administration of packed red blood cells Administration of oral or IV corticosteroids Administration of clotting factors VIII and IX Maintenance of reverse isolation and application of standard precautions

Administration of oral or IV corticosteroids Common treatment modalities for ITP include corticosteroid therapy to suppress the phagocytic response of splenic macrophages. Blood transfusions, administration of clotting factors, and reverse isolation are not interventions that are indicated in the care of patients with ITP. Standard precautions are used with all patients.

A patient is admitted to the burn unit after being transported from a facility 1000 miles away. The patient has burns to the groin area and circumferential burns to both upper thighs. When assessing the patient's legs distal to the wound site, the nurse should be cognizant of the risk of what complication? A) Ischemia B) Referred pain C) Cellulitis D) Venous thromboembolism (VTE)

Ans: A Feedback: As edema increases, pressure on small blood vessels and nerves in the distal extremities causes an obstruction of blood flow and consequent ischemia. This complication is similar to compartment syndrome. Referred pain, cellulitis, and VTE are not noted complications that occur distal to the injury site.

A patient has been admitted to a burn intensive care unit with extensive full-thickness burns over 25% of the body. After ensuring cardiopulmonary stability, what would be the nurse's immediate, priority concern when planning this patient's care? A) Fluid status B) Risk of infection C) Nutritional status D) Psychosocial coping

Ans: A Feedback: During the early phase of burn care, the nurse is most concerned with fluid resuscitation, to correct large-volume fluid loss through the damaged skin. Infection control and early nutritional support are important, but fluid resuscitation is an immediate priority. Coping is a higher priority later in the recovery period.

A patient is admitted to the emergency department with cold exposure and a core body temperature of 86.6o F (30.3o C). Which action is most appropriate for the nurse to take? A. Immerse the extremities in a water bath (102° to 108° F) [38.9° to 42.2° C]) B. Place an air-filled warming blanket on the patient. C. Position patient under a radiant heat lamp. D. Administer warmed intravenous (IV) fluids.

Answer: D. Administer warmed intravenous (IV) fluids. Rationale: A patient with a core body temperature of 86.6o F (30.3o C) has moderate hypothermia. Active core rewarming is used for moderate to severe hypothermia and includes administration of warmed IV fluids (109.4° F [43° C]). Patients with moderate to severe hypothermia should have the core warmed before the extremities to prevent after drop (or further drop in core temperature). This occurs when cold peripheral blood returns to the central circulation. Use passive or active external rewarming for mild hypothermia. Active external rewarming involves fluid-filled warming blankets or radiant heat lamps. Immersion of extremities in a water bath is indicated for frostbite. Ch. 69

A 30-year-old patient seen in the emergency department for severe headache and acute confusion is found to have a serum sodium level of 118 mEq/L. The nurse will anticipate the need for which diagnostic test? a. Urinary 17-ketosteroids b. Antidiuretic hormone level c. Growth hormone stimulation test d. Adrenocorticotropic hormone level

B

A 40-year-old male patient has been newly diagnosed with type 2 diabetes mellitus. Which information about the patient will be most useful to the nurse who is helping the patient develop strategies for successful adaptation to this disease? a. Ideal weight b. Value system c. Activity level d. Visual changes

B

Classic symptoms of bacterial meningitis include a. papilledema and psychomotor seizures b. high fever, nuchal rigidity, and severe headache c. behavioral changes with memory loss and lethargy d. positive Kernig's and Brudzinski's signs and hemiparesis

B. High fever, severe headache, nuchal rigidity, and positive Brudzinski's and Kernig's signs are such classic symptoms of meningitis that they are usually considered diagnostic for meningitis. Other symptoms, such as papilledema, generalized seizures, hemiparesis, and decreased LOC, may occur as complications of increased ICP and cranial nerve dysfunction.

The nurse is caring for a 37-yr-old female patient with multiple musculoskeletal injuries who has developed acute respiratory distress syndrome (ARDS). Which intervention should the nurse initiate to prevent stress ulcers? Observe stools for frank bleeding and occult blood. Maintain head of the bed elevation at 30 to 45 degrees. Begin enteral feedings as soon as bowel sounds are present. Administer prescribed lorazepam (Ativan) to reduce anxiety.

Begin enteral feedings as soon as bowel sounds are present. Stress ulcers prevention includes early initiation of enteral nutrition to protect the gastrointestinal (GI) tract from mucosal damage. Antiulcer agents such as histamine (H2)-receptor antagonists, proton pump inhibitors, and mucosal protecting agents are also indicated to prevent stress ulcers. Monitoring for GI bleeding does not prevent stress ulcers. Ventilator-associated pneumonia related to aspiration is prevented by elevation of the head of bed to 30 to 45 degrees Stress ulcers are not caused by anxiety. Stress ulcers are related to GI ischemia from hypotension, shock, and acidosis.

A 29-year-old patient in the outpatient clinic will be scheduled for blood cortisol testing. Which instruction will the nurse provide? a. "Avoid adding any salt to your foods for 24 hours before the test." b. "You will need to lie down for 30 minutes before the blood is drawn." c. "Come to the laboratory to have the blood drawn early in the morning." d. "Do not have anything to eat or drink before the blood test is obtained."

C

A 60-year-old patient is taking spironolactone (Aldactone), a drug that blocks the action of aldosterone on the kidney, for hypertension. The nurse will monitor for a. increased serum sodium. b. decreased urinary output. c. elevated serum potassium. d. evidence of fluid overload.

C

The nurse is caring for a patient with microcytic, hypochromic anemia. What teaching should the nurse provide that would be beneficial to the patient? A. Take enteric-coated iron with each meal. B. Take cobalamin with green leafy vegetables. C. Take the iron with orange juice one hour before meals. D. Decrease the intake of the antiseizure medications to improve.

C. Take the iron with orange juice one hour before meals. With microcytic, hypochromic anemia may be caused by iron, vitamin B6, or copper deficiency; thalassemia; or lead poisoning. The iron prescribed should be taken with orange juice one hour before meals as it is best absorbed in an acid environment. Megaloblastic anemias occur with cobalamin (vitamin B12) and folic acid deficiencies. Vitamin B12 may help red blood cell (RBC) maturation if the patient has the intrinsic factor in the stomach. Green leafy vegetables provide folic acid for RBC maturation. Antiseizure drugs may contribute to aplastic anemia or folic acid deficiency, but the patient should not stop taking the medications. The health care provider will prescribe changes in medications.

A patient with hepatitis A is in the acute phase. The nurse plans care for the patient based on the knowledge that a. pruritus is a common problem with jaundice in this phase. b. the patient is most likely to transmit the disease during this phase. c. gastrointestinal symptoms are not as severe in hepatitis A as they are in hepatitis B. d. extrahepatic manifestations of glomerulonephritis and polyarteritis are common in this phase. (Lewis 1042)

Correct answer: a Rationale: The acute phase of jaundice may be icteric or anicteric. Jaundice results when bilirubin diffuses into the tissues. Pruritus sometimes accompanies jaundice. Pruritus is the result of an accumulation of bile salts beneath the skin.

A patient with acute hepatitis B is being discharged in 2 days. In the discharge teaching plan the nurse should include instructions to a. avoid alcohol for the first 3 weeks. b. use a condom during sexual intercourse. c. have family members get an injection of immunoglobulin. d. follow a low-protein, moderate-carbohydrate, moderate-fat diet. (Lewis 1042)

Correct answer: b Rationale: Hepatitis B virus may be transmitted by mucosal exposure to infected blood, blood products, or other body fluids (e.g., semen, vaginal secretions, saliva). Hepatitis B is a sexually transmitted disease that is acquired through unprotected sex with an infected person. Condom use should be taught to patients to prevent transmission of hepatitis B.

An 18-year-old male patient with a small stature is scheduled for a growth hormone stimulation test. In preparation for the test, the nurse will obtain a. ice in a basin. b. glargine insulin. c. a cardiac monitor. d. 50% dextrose solution.

D

When caring for a critically ill patient who is being mechanically ventilated, the nurse will monitor for which clinical manifestation of multiple organ dysfunction syndrome (MODS)? Increased serum albumin Decreased respiratory compliance Increased gastrointestinal (GI) motility Decreased blood urea nitrogen (BUN)/creatinine ratio

Decreased respiratory compliance Clinical manifestations of MODS include symptoms of respiratory distress, signs and symptoms of decreased renal perfusion, decreased serum albumin and prealbumin, decreased GI motility, acute neurologic changes, myocardial dysfunction, disseminated intravascular coagulation (DIC), and changes in glucose metabolism.

The nurse is providing teaching to a patient recovering from a myocardial infarction. How should resumption of sexual activity be discussed? Delegated to the primary care provider Discussed along with other physical activities Avoided because it is embarrassing to the patient Accomplished by providing the patient with written material

Discussed along with other physical activities Although some nurses may not feel comfortable discussing sexual activity with patients, it is a necessary component of patient teaching. It is helpful to consider sex as a physical activity and to discuss or explore feelings in this area when other physical activities are discussed. Although providing the patient with written material is appropriate, it should not replace a verbal dialogue that can address the individual patient's questions and concerns.

The patient was diagnosed with prerenal AKI. The nurse should know that what is most likely the cause of the patient's diagnosis? A.IV tobramycin (Nebcin) B.Incompatible blood transfusion C.Poststreptococcal glomerulonephritis D.Dissecting abdominal aortic aneurysm

Dissecting abdominal aortic aneurysm A dissecting abdominal aortic aneurysm is a prerenal cause of AKI because it can decrease renal artery perfusion and therefore the glomerular filtrate rate. Aminoglycoside antibiotic administration, a hemolytic blood transfusion reaction, and poststretpcoccal glomerulonephritis are intrarenal causes of AKI.

Which antilipemic medications should the nurse question for a patient with cirrhosis of the liver (select all that apply.)? Select all that apply. Niacin Cholestyramine Ezetimibe (Zetia) Gemfibrozil (Lopid) Atorvastatin (Lipitor)

Ezetimibe (Zetia) Gemfibrozil (Lopid) Atorvastatin (Lipitor) Ezetimibe (Zetia) should not be used by patients with liver impairment. Adverse effects of atorvastatin (Lipitor), a statin drug, include liver damage and myopathy. Liver enzymes must be monitored frequently and the medication stopped if these enzymes increase. Niacin's side effects subside with time, although decreased liver function may occur with high doses. Cholestyramine is safe for long-term use *it stays in the GI tract, binds to bile acids and gets pooped out w/o absorption, so it's not involving the liver*

Which factor should be considered when caring for a woman with suspected coronary artery disease? Fatigue may be the first symptom. Classic signs and symptoms are expected. Increased risk is present before menopause. Women are more likely to develop collateral circulation.

Fatigue may be the first symptom. Fatigue, rather than pain or shortness of breath, may be the first symptom of impaired cardiac circulation. Women may not exhibit the classic signs and symptoms of ischemia such as chest pain which radiates down the left arm. Neck, throat, or back pain may be symptoms experienced by women. Risk for coronary artery disease increases four times after menopause. Men are more likely to develop collateral circulation.

The nurse would recognize which clinical manifestation as suggestive of sepsis? Sudden diuresis unrelated to drug therapy Hyperglycemia in the absence of diabetes Respiratory rate of seven breaths per minute Bradycardia with sudden increase in blood pressure

Hyperglycemia in the absence of diabetes Hyperglycemia in patients with no history of diabetes is a diagnostic criterion for sepsis. Oliguria, not diuresis, typically accompanies sepsis along with tachypnea and tachycardia.

Which assessment finding is a consequence of the oliguric phase of AKI? A. Hypovolemia B. Hyperkalemia C. Hypernatremia D.Thrombocytopenia

Hyperkalemia In AKI the serum potassium levels increase because the normal ability of the kidneys to excrete potassium is impaired. Sodium levels are typically normal or diminished, whereas fluid volume is normally increased because of decreased urine output. Thrombocytopenia is not a consequence of AKI, although altered platelet function may occur in AKI.

A nurse in the emergency department is working triage. Which patient assessment findings would indicate immediate care is required? Shortened and externally rotated leg Inability to swallow and move the left arm Warm, edematous, reddened and painful calf Yellow sputum and pain with deep inspiration

Inability to swallow and move the left arm Inability to swallow and move the left arm suggests the patient is experiencing a stroke. A CT scan is indicated within 25 minutes of arrival to determine ischemic versus hemorrhagic origin, which will delineate available treatments. The warm, edematous, reddened and painful calf suggests deep vein thrombosis. Although not an immediate threat, there is a risk of pulmonary emboli. A shortened and externally rotated leg suggests a hip fracture. A patient with yellow sputum and pain with deep inspiration suggest a pneumonia that may require hospitalization or could be treated as an outpatient.

A 56-yr-old man with acute respiratory distress syndrome (ARDS) is on positive pressure ventilation (PPV). The patient's cardiac index is 1.4 L/min and pulmonary artery wedge pressure is 8 mm Hg. What order by the physician is important for the nurse to question? Initiate a dobutamine infusion at 3 mcg/kg/min. Administer 1 unit of packed red blood cells over the next 2 hours. Change the maintenance intravenous (IV) rate from 75 to 125 mL/hr. Increase positive end-expiratory pressure (PEEP) from 10 to 15 cm H2O.

Increase positive end-expiratory pressure (PEEP) from 10 to 15 cm H2O. Patients on PPV and PEEP frequently experience decreased cardiac output (CO) and cardiac index (CI). High levels of PEEP increase intrathoracic pressure and cause decreased venous return which results in decreased CO. Interventions to improve CO include lowering the PEEP, administering crystalloid fluids or colloid solutions, and use of inotropic drugs (e.g., dobutamine, dopamine). Packed red blood cells may also be administered to improve CO and oxygenation if the hemoglobin is less than 9 or 10 mg/dL.

Diffusion, osmosis, and ultrafiltration occur in both hemodialysis and peritoneal dialysis. The nurse should know that ultrafiltration in peritoneal dialysis is achieved by which method? A.Increasing the pressure gradient B.Increasing osmolality of the dialysate C.Decreasing the glucose in the dialysate D.Decreasing the concentration of the dialysate

Increasing osmolality of the dialysate Ultrafiltration in peritoneal dialysis is achieved by increasing the osmolality of the dialysate with additional glucose. In hemodialysis the increased pressure gradient from increased pressure in the blood compartment or decreased pressure in the dialysate compartment causes ultrafiltration. Decreasing the concentration of the dialysate in either peritoneal or hemodialysis will decrease the amount of fluid removed from the blood stream.

The patient has developed cardiogenic shock after a left anterior descending myocardial infection. Which circulatory-assist device should the nurse expect to use for this patient? Cardiopulmonary bypass Impedance cardiography (ICG) Intraaortic balloon pump (IABP) Central venous pressure (CVP) measurement

Intraaortic balloon pump (IABP) The most commonly used mechanical circulatory assist device is the IABP, and it is used to decrease ventricular workload, increase myocardial perfusion, and augment circulation. Cardiopulmonary bypass provides circulation during open heart surgery. It is not used as an assist device after surgery. ICG is a noninvasive method to obtain cardiac output and assess thoracic fluid status. CVP measurement is an invasive measurement of right ventricular preload and reflects fluid volume problems.

The nurse is caring for a 55-year-old man patient with acute pancreatitis resulting from gallstones. Which clinical manifestation would the nurse expect the patient to exhibit? Hematochezia Left upper abdominal pain Ascites and peripheral edema Temperature over 102o F (38.9o C)

Left upper abdominal pain Abdominal pain (usually in the left upper quadrant) is the predominant manifestation of acute pancreatitis. Other manifestations of acute pancreatitis include nausea and vomiting, low-grade fever, leukocytosis, hypotension, tachycardia, and jaundice. Abdominal tenderness with muscle guarding is common. Bowel sounds may be decreased or absent. Ileus may occur and causes marked abdominal distention. Areas of cyanosis or greenish to yellow-brown discoloration of the abdominal wall may occur. Other areas of ecchymoses are the flanks (Grey Turner's spots or sign, a bluish flank discoloration) and the periumbilical area (Cullen's sign, a bluish periumbilical discoloration).

A 42-yr-old man underwent amputation below the knee on the left leg after a recent heavy farm machinery accident. Which intervention should the nurse include in the plan of care? Sit in a chair for 1 to 2 hours three times each day. Dangle the residual limb for 20 to 30 minutes every 6 hours. Lie prone with hip extended for 30 minutes four times per day. Elevate the residual limb on a pillow for 4 to 5 days after surgery.

Lie prone with hip extended for 30 minutes four times per day. To prevent hip flexion contractures, the patient should lie on the abdomen for 30 minutes three or four times each day and position the hip in extension while prone. The patient should avoid sitting in a chair for more than 1 hour with hips flexed or having pillows under the surgical extremity. The patient should avoid dangling the residual limb over the bedside to minimize edema.

A male patient is brought into the emergency department with multiple stab wounds to the legs, one stab wound to the left abdomen, and gang tattoos on both arms. He refused to identify his attacker and then loses consciousness. Police identify him as the assailant in the fatal stabbing of another man. What is the nurse's priority? Guard locked access doors. Maintain patient safety from revenge. Maintain personal and work place safety. Attain open patient airway and breathing.

Maintain personal and work place safety. The nurse's priority is to maintain personal and work place safety. Violence can erupt in the emergency department when treating gang members if the rival gang seeks revenge, or the patient's gang members seek to protect the patient with their presence. Staff members can be victims of that violence, so they should maintain a safe work environment by seeking law enforcement and security assistance in maintaining safety for the staff and the patient. ABCs are the usual priority, but this situation does not show any problem with the patient's airway or breathing.

The nurse is caring for a 65-yr-old man with acute respiratory distress syndrome (ARDS) who is on pressure support ventilation (PSV), fraction of inspired oxygen (FIO2) at 80%, and positive end-expiratory pressure (PEEP) at 15 cm H2O. The patient weighs 72 kg. What finding would indicate that treatment is effective? PaO2 of 60 mm Hg Tidal volume of 700 mL Cardiac output of 2.7 L/min Inspiration to expiration ratio of 1:2

PaO2 of 60 mm Hg Severe hypoxemia (PaO2 less than 40 mm Hg) occurs with ARDS, and PEEP is increased to improve oxygenation and prevent oxygen toxicity by reducing FIO2. A PaO2 level of 60 mm Hg indicates that treatment is effective and oxygenation status has improved. Decreased cardiac output is a complication of PEEP. Normal cardiac output is 4 to 8 L/minute. Normal tidal volume is 6 to 10 mL/kg. PSV delivers a preset pressure but the tidal volume varies with each breath. I:E ratio is usually set at 1:2 to 1:1.5 and does not indicate patient improvement.

A male patient who has coronary artery disease (CAD) has serum lipid values of low-density lipoprotein (LDL) cholesterol of 98 mg/dL and high-density lipoprotein (HDL) cholesterol of 47 mg/dL. What should the nurse include in patient teaching? Consume a diet low in fats. Reduce total caloric intake. Increase intake of olive oil. The lipid levels are normal.

The lipid levels are normal. For men, the recommended LDL is less than 100 mg/dL, and the recommended level for HDL is greater than 40mg/dL. His normal lipid levels should be included in the patient teaching and encourage him to continue taking care of himself. Assessing his need for teaching related to diet should also be done.

The nurse determines there is artifact on the patient's telemetry monitor. Which factor should the nurse assess for that could correct this issue? - Disabled automaticity - Electrodes in the wrong lead - Too much hair under the electrodes - Stimulation of the vagus nerve fibers

Too much hair under the electrodes Artifact is caused by muscle activity, electrical interference, or insecure leads and electrodes that could be caused by excessive chest wall hair. Disabled automaticity would cause an atrial dysrhythmia. Electrodes in the wrong lead will measure electricity in a different plane of the heart and may have a different wave form than expected. Stimulation of the vagus nerve fibers causes a decrease in heart rate, not artifact.

A patient has been diagnosed with stage 1A Hodgkin's lymphoma. The nurse knows that which chemotherapy regimen is most likely to be prescribed for this patient? A. Brentuximab vedotin (Adcetris) B. Two to four cycles of ABVD: doxorubicin (Adriamycin), bleomycin, vinblastine, and dacarbazine C. Four to six cycles of ABVD: doxorubicin (Adriamycin), bleomycin, vinblastine, and dacarbazine D. BEACOPP: bleomycin, etoposide, doxorubicin (Adriamycin), cyclophosphamide, vincristine (Oncovin), procarbazine, and prednisone

Two to four cycles of ABVD: doxorubicin (Adriamycin), bleomycin, vinblastine, and dacarbazine The patient with a favorable prognosis early-stage Hodgkin's lymphoma (stage 1A) will receive two to four cycles of ABVD. The unfavorable prognostic featured (stage 1B) Hodgkin's lymphoma would be treated with four to six cycles of chemotherapy. Advanced-stage Hodgkin's lymphoma is treated more aggressively with more cycles or with BEACOPP. Brentuximab vedotin (Adcetris) is a newer agent that will be used to treat patients who have relapsed or refractory disease.Note: Some of acronyms for drug protocols use the brand/trade name of drugs (Adriamycin, Oncovin). These brand/trade names have been discontinued but the drugs are still available as generic drugs.

The nurse prepares to defibrillate a patient. Which dysrhythmia has the nurse observed in this patient? Ventricular fibrillation Third-degree AV block Uncontrolled atrial fibrillation Ventricular tachycardia with a pulse

Ventricular fibrillation Defibrillation is always indicated in the treatment of ventricular fibrillation. Drug treatments are normally used in the treatment of uncontrolled atrial fibrillation and for ventricular tachycardia with a pulse (if the patient is stable). Otherwise, synchronized cardioversion is used (as long as the patient has a pulse). Pacemakers are the treatment of choice for third-degree heart block.

When caring for a patient with a biliary obstruction, the nurse will anticipate administering which vitamin supplements (select all that apply)? Vitamin A Vitamin D Vitamin E Vitamin K Vitamin B

Vitamin A Correct Vitamin D Correct Vitamin E Correct Vitamin K Correct Biliary obstruction prevents bile from entering the small intestine and thus prevents the absorption of fat-soluble vitamins. Vitamins A, D, E, and K are all fat-soluble and thus would need to be supplemented in a patient with biliary obstruction.

The nurse is caring for a woman recently diagnosed with viral hepatitis A. Which individual should the nurse refer for an immunoglobin (IG) injection? a. A caregiver who lives in the same household with the patient b. A friend who delivers meals to the patient and family each week c. A relative with a history of hepatitis A who visits the patient daily d. A child living in the home who received the hepatitis A vaccine 3 months ago

a IG is recommended for persons who do not have anti-HAV antibodies and are exposed as a result of close contact with persons who have HAV or foodborne exposure. Persons who have received a dose of HAV vaccine more than 1 month previously or who have a history of HAV infection do not require IG.

In planning care for a patient with metastatic liver cancer, the nurse should include interventions that a. focus primarily on symptomatic and comfort measures. b. reassure the patient that chemotherapy offers a good prognosis. c. promote the patient's confidence that surgical excision of the tumor will be successful. d. provide information necessary for the patient to make decisions regarding liver transplantation. (Lewis 1042)

a Rationale: Nursing intervention for a patient with liver cancer focuses on keeping the patient as comfortable as possible. The prognosis for patients with liver cancer is poor. The cancer grows rapidly, and death may occur within 4 to 7 months as a result of hepatic encephalopathy or massive blood loss from gastrointestinal (GI) bleeding.

The nurse is planning care for a patient with partial- and full-thickness skin destruction related to burn injury of the lower extremities. Which of the following interventions would the nurse expect to include in this patient's care (select all that apply)? A.Escharotomy B.Administration of diuretics C.IV and oral pain medications D.Daily cleansing and debridement E.Application of topical antimicrobial agent

a, c, d, e. An escharotomy (a scalpel incision through full-thickness eschar) is frequently required to restore circulation to compromised extremities. Daily cleansing and debridement as well as application of an antimicrobial ointment are expected interventions used to minimize infection and enhance wound healing. With full-thickness burns, myoglobin and hemoglobin released into the bloodstream can occlude renal tubules. Adequate fluid replacement is used to prevent this occlusion. Pain control is essential in the care of a patient with a burn injury

Which BP-regulating mechanism(s) can result in the development of hypertension if defective (select all that apply)? a. Release of norepinephrine b. Secretion of prostaglandins c. Stimulation of the sympathetic nervous system d. Stimulation of the parasympathetic nervous system e. Activation of the renin-angiotensin-aldosterone system

a, c, e *A and C are basically same - SNS stimulates catecholamines; E is renal which is the system in charge of BP*

When admitting a patient with possible respiratory failure with a high PaCO2, which assessment information should be immediately reported to the health care provider? a. The patient is somnolent. b. The patient complains of weakness. c. The patient's blood pressure is 164/98. d. The patient's oxygen saturation is 90%.

a. the patient is somnolent

11. A patient with acute respiratory distress syndrome (ARDS) is placed in the prone position. When prone positioning is used, which information obtained by the nurse indicates that the positioning is effective? a. The patient's PaO2 is 89 mm Hg, and the SaO2 is 91%. b. Endotracheal suctioning results in clear mucous return. c. Sputum and blood cultures show no growth after 48 hours. d. The skin on the patient's back is intact and without redness.

a. the patient's PaO2 is 89 mm Hg and the SaO2 is 91%

A patient with tendonitis asks what the tendon does. The nurse's response is based on the knowledge that tendons a.connect bone to muscle. b.provide strength to muscle. c.lubricate joints with synovial fluid. d.relieve friction between moving parts.

a.connect bone to muscle Tendons are composed of dense, fibrous connective tissue that contains bundles of closely packed collagen fibers arranged in the same plane for additional strength. They connect the muscle sheath to adjacent bone.

subphrenic abscess

accumulation of infected fluid between the diaphragm, liver, and spleen - after surgery

Teaching in relation to home management after a laparoscopic cholecystectomy should include a. keeping the bandages on the puncture sites for 48 hours. b. reporting any bile-colored drainage or pus from any incision. c. using over-the-counter antiemetics if nausea and vomiting occur. d. emptying and measuring the contents of the bile bag from the T tube every day. (Lewis 1042)

b Rationale: The following discharge instructions are taught to the patient and caregiver after a laparoscopic cholecystectomy: First, remove the bandages on the puncture site the day after surgery and shower. Second, notify the surgeon if any of the following signs and symptoms occur: redness, swelling, bile-colored drainage or pus from any incision; and severe abdominal pain, nausea, vomiting, fever, or chills. Third, gradually resume normal activities. Fourth, return to work within 1 week of surgery. Fifth, resume a usual diet, but a low-fat diet is usually better tolerated for several weeks after surgery.

When caring for a client with extensive burns, the nurse anticipates that pain medication will be administered via which route? a. oral b. IV c. IM d. Subq

b.

The nurse is administering fluids intravenously as prescribed to a client who sustained superficial partial thickness burn injuries of the back and legs. In evaluating the adequacy of fluid resuscitation, the nurse understands that which of the following would provide the most reliable indicator for determining the adequacy? a. vital signs b. urine output c. mental status d. peripheral pulses

b. successful or adequate fluid resuscitation in the client is signaled by stable vital signs, adequate urine output, palpable peripheral pulses and clear sensorium.

The nurse plans care for a male pt who suffered thermal burns to the entire posterior aspect of his body when he fell on an outdoor grill. Which pt need is likely to be the primary problem of this pt in the emergent phase? a. maintain tissue oxygenation b. halt progression of the burn c. maintain intravascular volume d. prevent invasion of pathogens

b. the first priority is halting the severity of the burn, to limit the depth of the burn and quick action must be a priority.

Pain management for the burn patient is most effective when a. opioids are administered on a set schedule around the clock b. the patient has as much control over the management of the pain as possible. c. there is flexibility to administer opioids withing a dosage and frequency range d. painful dressing changes are delayed until the pt's pain is totally relieved.

b. the patient has as much control over the management of the pain as possible. The more control the patient has in managing the pain, the more successful the chosen strategies. Active patient participation has been found to be effective for some patients in anticipating and coping with treatment-induced pain.

A patient is to undergo skin grafting with the use of cultured epithelial autografts full-thickness burns. The nurse explains to the patient that this treatment involves a) Shaving a split-thickness layer of the patient's skin to cover the burn wound. b) Using epidermal growth factor to cultivate cadaver skin for temporary wound coverage. c) Growing small specimens of the patient's skin into sheets to use as permanent skin coverage. d) Exposing animal skin to growth factors to decrease antigenicity so it can be used for permanent wound coverage.

c Rationale: Cultured epithelial autograft (CEA) is a method of obtaining permanent skin from a person with limited available skin for harvesting. CEA is grown from biopsy specimens obtained from the patient's own unburned skin.

In reviewing the medical record shown in the accompanying figure for a patient admitted with acute pancreatitis, the nurse sees that the patient has a positive Cullen's sign. Indicate the area where the nurse will assess for this change. a. epigastric b. flanks c. periumbilical d. idk the ass cheeks, I don't have the paid subscription so I can't get the accompanying figures hopefully it's obvious that it's not d

c. 3 The area around the umbilicus should be indicated. Cullen's sign consists of ecchymosis around the umbilicus. Cullen's sign occurs because of seepage of bloody exudates from the inflamed pancreas and indicates severe acute pancreatitis.

A 64-year-old male patient who has had progressive chronic kidney disease (CKD) for several years has just begun regular hemodialysis. Which information about diet will the nurse include in patient teaching? a. Increased calories are needed bc glucose is lost during hemodialysis. b. Unlimited fluids are allowed bc retained fluid is removed during dialysis c. More protein is allowed bc urea and creatinine are removed by dialysis d. Dietary potassium is not restricted bc the level is normalized by dialysis

c. more protein is allowed because urea and creatinine are removed by dialysis Once the patient is started on dialysis and nitrogenous wastes are removed, more protein in the diet is encouraged. Fluids are still restricted to avoid excessive weight gain and complications such as shortness of breath. Glucose is not lost during hemodialysis. Sodium and potassium intake continues to be restricted to avoid the complications associated with high levels of these electrolytes.

The nurse documents the vital signs for a patient admitted 2 days ago with gram-negative sepsis: temperature 101.2° F, blood pressure 90/56 mm Hg, pulse 92, respirations 34. Which action should the nurse take next? a. Give the scheduled IV antibiotic. b. Give the PRN acetaminophen (Tylenol). c. Obtain oxygen saturation using pulse oximetry. d. Notify the health care provider of the patient's vital signs.

c. obtain O2Sat using pulse oximetry

After the insertion of an arteriovenous graft (AVG) in the right forearm, a 54-year-old patient complains of pain and coldness of the right fingers. Which action should the nurse take? a. Teach the patient about normal AVG function. b. Remind the patient to take a daily low-dose aspirin tablet. c. Report the patient's symptoms to the health care provider. d. Elevate the patient's arm on pillows to above the heart level.

c. report the patient's symptoms to the healthcare provider

The nurse monitors the patient with positive pressure mechanical ventilation for a. paralytic ileus because pressure on the abdominal contents affects bowel motility b. diuresis and sodium depletion because of increased release of atrial natriuretic peptide c. signs of cardiovascular insufficiency because pressure in the chest impedes venous return d. respiratory acidosis in a patient with COPD because of alveolar hyperventilation and increased PaO2 levels

c. signs of cardiovascular insufficiency because pressure in the chest impedes venous return

A patient who is admitted to a burn unit is hypovolemic. A new nurse asks an experienced nurse about the patient's condition. Which response if made by the experienced nurse is most appropriate? a) "Blood loss from burned tissue is the most likely cause of hypovolemia." b) "Third spacing of fluid into fluid-filled vesicles is usually the cause of hypovolemia." c) "The usual cause of hypovolemia is vaporation of fluid from denuded body surfaces." d) "Increased capillary permeability causes fluid shifts out of blood vessels and results in hypovolemia."

d Rationale: Hypovolemic shock is caused by a massive shift of fluids out of the blood vessels as a result of increased capillary permeability. Water, sodium, and plasma proteins move into interstitial spaces and other surrounding tissue.

A pt c comminuted Fx of the tibia is to have an ORIF. The nurse explains that ORIF is indicated when a. pt is unable to tolerate prolonged immobilization b. pt cannot tolerate the surgery for a closed reduction c. a temporary cast would be too unstable to provide normal mobility d. adequate alignment cannot be obtained by other, nonsurgical methods

d. adequate alignment cannot be obtained by other, nonsurgical methods

cholelithiasis risk factors - Fs

female fair (as in Caucasian) fat (BMI >30) forty (age => 40) fertile (multiparous)

Complications of ERCP c papillotomy

pancreatitis (pain, fever, elevated panc enzymes) perforation infection bleeding

bile peritonitis

perf - bile in abd cavity - s/s acute abdomen

cause of gallstones?

unknown but high cholesterol? most common stones are mixed cholesterol stones. also bile stasis r/t immobility, pregnancy, inflammatory disease = biliary sludge, which is the name of my next punk rock band

The nurse provides discharge instructions for a 64-year-old woman with ascites and peripheral edema related to cirrhosis. Which statement, if made by the patient, indicates teaching was effective? "It is safe to take acetaminophen up to four times a day for pain." "Lactulose (Cephulac) should be taken every day to prevent constipation." "Herbs and other spices should be used to season my foods instead of salt." "I will eat foods high in potassium while taking spironolactone (Aldactone)."

"Herbs and other spices should be used to season my foods instead of salt." A low-sodium diet is indicated for the patient with ascites and edema related to cirrhosis. Table salt is a well-known source of sodium and should be avoided. Alternatives to salt to season foods include the use of seasonings such as garlic, parsley, onion, lemon juice, and spices. Pain medications such as acetaminophen, aspirin, and ibuprofen should be avoided as these medications may be toxic to the liver. The patient should avoid potentially hepatotoxic over-the-counter drugs (e.g., acetaminophen) because the diseased liver is unable to metabolize these drugs. Spironolactone is a potassium-sparing diuretic. Lactulose results in the acidification of feces in bowel and trapping of ammonia, causing its elimination in feces.

After teaching a patient with chronic stable angina about nitroglycerin, the nurse recognizes the need for further teaching when the patient makes which statement? "I will replace my nitroglycerin supply every 6 months." "I can take up to five tablets every 3 minutes for relief of my chest pain." "I will take acetaminophen (Tylenol) to treat the headache caused by nitroglycerin." "I will take the nitroglycerin 10 minutes before planned activity that usually causes chest pain."

"I can take up to five tablets every 3 minutes for relief of my chest pain." The recommended dose of nitroglycerin is one tablet taken sublingually (SL) or one metered spray for symptoms of angina. If symptoms are unchanged or worse after 5 minutes, the patient should be instructed to activate the emergency medical services (EMS) system. If symptoms are improved, repeat the nitroglycerin every 5 minutes for a maximum of three doses and contact EMS if symptoms have not resolved completely.

A 21-yr-old soccer player has injured the anterior crucial ligament (ACL) and is having reconstructive surgery. Which patient statement indicates more teaching is required? "I probably won't be able to play soccer for 6 to 8 months." "They will have me do range of motion with my knee soon after surgery." "I can't wait to get this done now so I can play soccer for the next tournament." "I will need to wear an immobilizer and progressively bear weight on my knee."

"I can't wait to get this done now so I can play soccer for the next tournament." The patient does not understand the severity of ACL reconstructive surgery if planning to resume playing soccer soon; safe return will not occur for 6 to 8 months. Initial range of motion, immobilization, and progressive weight bearing will be overseen by a physical therapist.

The nurse is doing discharge teaching with the patient who received an implantable cardioverter-defibrillator (ICD) in the left side. Which statement by the patient indicates to the nurse that further teaching is required? "I will call the cardiologist if my ICD fires." "I cannot fly because it will damage the ICD." "I cannot move my left arm until it is approved." "I cannot drive until my cardiologist says it is okay."

"I cannot fly because it will damage the ICD." The patient statement that flying will damage the ICD indicates misunderstanding about flying. The patient should be taught that informing TSA security screening agents at the airport about the ICD should be done because it may set off the metal detector and if a hand-held screening wand is used, it should not be placed directly over the ICD. The other options indicate the patient understands the teaching.

The nurse instructs a 68-yr-old woman with hypercholesterolemia about natural lipid-lowering therapies. The nurse determines further teaching is necessary if the patient makes which statement? "Omega-3 fatty acids are helpful in reducing triglyceride levels." "I should check with my physician before I start taking any herbal products." "Herbal products do not go through as extensive testing as prescription drugs do." "I will take garlic instead of my prescription medication to reduce my cholesterol."

"I will take garlic instead of my prescription medication to reduce my cholesterol." Current evidence does not support using garlic in the treatment of elevated cholesterol. Strong evidence supports the use of omega-3 fatty acids for reduction of triglyceride levels. Many herbal products are not standardized and effects are not predictable. Patients should consult with their health care provider before starting herbal or natural therapies.

A patient develops third-degree heart block and reports feeling chest pressure and shortness of breath. Which instructions should the nurse provide to the patient before initiating emergency transcutaneous pacing? "The device will convert your heart rate and rhythm back to normal." "The device uses overdrive pacing to slow the heart to a normal rate." "The device is inserted through a large vein and threaded into your heart." "The device delivers a current through your skin that can be uncomfortable."

"The device delivers a current through your skin that can be uncomfortable." Before initiating transcutaneous pacing therapy, it is important to tell the patient what to expect. The nurse should explain that the muscle contractions created by the pacemaker when the current passes through the chest wall are uncomfortable. Pacing for complete heart block will not convert the heart rhythm to normal. Overdrive pacing is used for very fast heart rates. Transcutaneous pacing is delivered through pacing pads adhered to the skin.

The nursing instructor is teaching a student nurse about continuous renal replacement therapy (CRRT). Which statement by the student nurse indicates effective learning? 1 "CRRT is provided over approximately 24 hours." 2 "CRRT does not require the addition of an anticoagulant." 3 "CRRT cannot be used in conjunction with hemodialysis." 4 "CRRT has a faster blood flow rate than hemodialysis."

1 Continuous renal replacement therapy (CRRT) is a physiologic therapy that simulates kidney function day and night. CRRT is done either by cannulating an artery and a vein or by cannulating two veins. CRRT is provided continuously for approximately 24 hours. CRRT involves the flow of blood from the body through a filter and carries an increased risk of clotting; thus an anticoagulant must be added. CRRT can be performed along with hemodialysis. CRRT has a slower blood flow rate than intermittent hemodialysis. Text Reference - p. 1106

The nurse is preparing to perform peritoneal dialysis for a patient with chronic kidney disease. Which osmotic agent will the nurse obtain for the dialysis exchanges? 1 Dextrose 2 Normal saline 3 Icodextrin solution 4 Amino acid solution

1 Dextrose is the most commonly used osmotic agent used in peritoneal dialysis. Normal saline solution is not used in peritoneal dialysis. Icodextrin and amino acid solutions are used as alternatives to dextrose. Text Reference - p. 1118

A dialysis nurse is performing hemodialysis for a patient with chronic kidney disease. Which action by the nurse will prevent blood clotting during the procedure? 1 Addition of heparin to the blood 2 Addition of dextrose to the blood 3 Addition of icodextrin to the blood 4 Addition of saline solution to the blood

1 Heparin is added to the blood to prevent clotting when the patient's blood contacts a foreign substance. Dextrose and icodextrin are used as osmotic agents during dialysis. Saline solution is used to flush the dialyzer. Text Reference - p. 1121

The nursing instructor is teaching a student nurse about sodium polystyrene sulfonate. Which statement by the student nurse indicates the need for further teaching? 1 "The drug is effective in treating a paralytic ileus." 2 "It can be administered as an enema." 3 "The drug helps exchange potassium for sodium." 4 "It is mixed in water with sorbitol and then administered."

1 Sodium polystyrene sulfonate is used to correct hyperkalemia and is contraindicated in patients with a paralytic ileus because it causes bowel necrosis. Sodium polystyrene sulfonate can be administered in the form of an enema, which acts by exchanging potassium for sodium ions. It can also be administered after mixing it in water with sorbitol to facilitate the removal of potassium from the body. Text Reference - p. 1105

The nurse recognizes that which intervention would help a patient with stage 5 chronic kidney disease who experiences restless leg syndrome, altered mental ability, seizures, coma, and a blood urea nitrogen (BUN) level of 35 mg/dL? 1 Refer the patient for dialysis. 2 Administer calcium phosphate binders. 3 Recommend that the patient receive a blood transfusion. 4 Administer 10% calcium gluconate intravenously

1 The patient's symptoms of restless leg syndrome, altered mental ability, and irritability are manifestations of neurologic complications due to accumulation of nitrogenous wastes in the brain and nervous system. The patient has seizures and coma due to the high blood urea nitrogen (BUN) level of 35 mg/dL. Therefore, dialysis would improve central nervous system functions and slow the neuropathies. Calcium phosphate binders are administered in a patient with hyperphosphatemia. A blood transfusion is not preferred to treat anemia unless the patient experiences an acute blood loss or symptomatic anemia. Intravenous administration of 10% calcium gluconate helps to reduce hyperkalemia in a patient. Text Reference - p. 1110

The nurse just received an urgent laboratory value on a patient in renal failure. The potassium level is 6.3. The telemetry monitor is showing peaked T waves. Which prescription from the primary health care provider should be implemented first? 1 Administer regular insulin intravenously (IV) 2 Restrict dietary potassium intake to 40 meq daily 3 Administer kayexalate enema 4 Educate the patient on dietary restriction of potassium

1 This patient is showing signs of hyperkalemia, which could be fatal and lead to myocardial damage. Regular insulin IV is needed to quickly force potassium into the cells. The kayexalate enema will take too long to excrete the potassium. Restricting oral intake and educating the patient will be needed when the crisis has resolved. Text Reference - p. 1112

The intensive care unit nurse is caring for a patient who is ventilated mechanically. To prevent sepsis in this patient, which nursing intervention does the nurse include in the plan of care? 1 Provide oral care every two to four hours. 2 Turn patient from side to side every eight hours. 3 Position patient in a supine position every two hours. 4 Use clean gloves when suctioning the endotracheal tube.

1 Providing oral care every two to four hours is correct, because research has found that the oral flora of critically ill patients are predominately gram-negative organisms that can potentially cause ventilator-associated pneumonia. Oral care will help reduce the organisms. Turning the patient from side to side every eight hours is incorrect, because patients need to be turned at least every two hours to prevent accumulation of mucus, which could lead to pneumonia. Positioning the patient in a supine position is incorrect, because patients should have the head of the bed elevated during mechanical ventilation. The nurse should use sterile gloves when conducting endotracheal suctioning of the patient. Test-Taking Tip: Never leave a question unanswered. Even if answering is no more than an educated guess on your part, go ahead and mark an answer. You might be right, but if you leave it blank, you will certainly be wrong and lose precious points. Text Reference - p. 1648

The nurse reviews the medical record of a patient with pneumonia and notes that the patient has hypotension, hypothermia, leukocytosis, and hypoxemia. What should the nurse infer from these findings? 1 The patient has septic shock. 2 The patient has neurogenic shock. 3 The patient has cardiogenic shock. 4 The patient has hypovolemic shock.

1 Septic shock is most commonly found in the patient having gram-negative bacterial infections, such as pneumonia. Because it is characterized by hypertension, hypothermia, leukocytosis, and hypoxemia in patients with infections, the nurse concludes that the patient has septic shock. Neurogenic shock is most commonly seen in the patient who has an injury. Cardiogenic shock is caused by cardiovascular disorders, such as a myocardial infarction and cardiomyopathy. Hypovolemic shock is caused by hemorrhage or trauma. Test-Taking Tip: Multiple-choice questions can be challenging, because students think that they will recognize the right answer when they see it or that the right answer will somehow stand out from the other choices. This is a dangerous misconception. The more carefully the question is constructed, the more each of the choices will seem like the correct response. Text Reference - p. 1637

The primary health care provider prescribes antibiotics and vasopressors for a patient. Which type of shock does the nurse expect to be treating? 1 Septic shock 2 Cardiogenic shock 3 Neurogenic shock 4 Anaphylactic shock

1 Septic shock occurs in response to infection. Therefore, antibiotics are prescribed for a patient with septic shock. Cardiogenic shock occurs when systolic or diastolic function of the heart is impaired. Sympathomimetic drugs are used for the treatment of cardiogenic shock. Injury to the spinal cord at the fifth thoracic vertebra or above causes neurogenic shock. Vasconstricting medications are prescribed to prevent vasodilation for a patient in septic shock. Anaphylactic shock is a life-threatening allergic reaction to a sensitizing substance. Antihistamines, bronchodilators, and corticosteroids are used in the treatment of anaphylactic shock. Test-Taking Tip: Do not worry if you select the same numbered answer repeatedly, because there usually is no pattern to the answers. Text Reference - p. 1645

A patient is showing signs of anaphylactic shock from an insect sting. Which primary health care provider's prescription does the nurse implement first? 1 Epinephrine 1:1000, 0.5 mg subcutaneous (SQ) 2 Normal saline intravenous (IV) to run at 150 mL/hr 3 Diphenhydramine 50 mg IV 4 Oxygen via nasal cannula at 3 L

1 The patient in anaphylaxis experiences bronchial spasm and constriction. The administration of epinephrine is necessary to reverse this process and facilitate an open airway. Although administering normal saline, diphenhydramine, and oxygen are appropriate, they must be done after an airway has been established. Text Reference - p. 1646

A nurse is giving dietary advice to a patient who is on continuous ambulatory peritoneal dialysis for chronic renal failure. Which dietary instructions are appropriate for this patient? Select all that apply. 1 High-calorie foods 2 High-protein foods 3 High-potassium content 4 High-phosphorus content 5 High-fluid intake

1, 2 A chronic renal failure patient on continuous ambulatory peritoneal dialysis is encouraged to have a high-calorie diet to meet the increased demands of the body. A good amount of protein should be consumed to replace that lost during dialysis. Foods containing high amounts of potassium and phosphorus should be avoided in patients with chronic renal failure. High potassium can cause hyperkalemia and related complications, especially cardiac complications. High phosphorus may deteriorate bone health. Usually there is a modest restriction of fluids when the patient is on dialysis. Text Reference - p. 1115

A patient has end-stage kidney disease and is receiving hemodialysis. During dialysis the patient complains of nausea and a headache and appears confused. On examination, the nurse finds that the blood pressure is very low. What is the priority action by the nurse? Select all that apply. 1 Decrease the volume of fluids being removed. 2 Infuse 0.9% saline solution. 3 Infuse hypertonic glucose solution. 4 Avoid excess coagulation. 5 Transfuse blood, as ordered

1, 2 Hypotension is a complication of hemodialysis and may manifest as headache and nausea. The nurse should try to keep the intravascular volume adequate by decreasing the volume of fluids being removed and infusing 0.9% saline solution. Hypertonic glucose solutions are infused if the patient gets muscle cramps. Excess coagulation is avoided if the patient has blood loss. Blood is transfused if the patient has blood loss. Text Reference - p. 1122

The nurse is attending to a patient who is undergoing peritoneal dialysis. The nurse assesses the patient is developing symptoms of respiratory distress. What nursing interventions are necessary to prevent further respiratory complications? Select all that apply. 1 Auscultate the lungs. 2 Frequently reposition the patient. 3 Promote deep-breathing exercises. 4 Increase the rate of infusion of the dialysate. 5 Place the patient in a low Fowler's position.

1, 2, 3 Auscultation is very important to find the cause of respiratory distress. Decreased areas of ventilation suggest the presence of atelectasis, whereas adventitious sounds may suggest fluid overload, retained secretions, or infection. Frequent positioning will promote equal ventilation to all parts of the lungs. Deep-breathing exercises could help to promote proper expansion of lungs. Rapid infusion would cause more pressure on the diaphragm. The patient should be placed in the semi-Fowler's position for peritoneal dialysis; this allows inflow of fluid while not impinging on the thoracic cavity. Text Reference - p. 1119

A patient with end-stage kidney disease is to begin continuous ambulatory peritoneal dialysis (CAPD). What are the preparations to be done by the nurse before starting the catheter insertion for this patient? Select all that apply. 1 Ask patient to empty the bladder and bowel. 2 Note the patient's weight. 3 Obtain a signed consent form. 4 Monitor for abnormal cardiac signs and symptoms. 5 Monitor for abnormal respiratory signs and symptoms

1, 2, 3 Preparation of the patient for catheter insertion includes emptying the bladder and bowel, weighing the patient, and obtaining a signed consent form. The bladder should be emptied to prevent accidental puncture of the bladder by the needle. Weighing the patient before and after the procedure is important to determine the effectiveness of dialysis. Because it is an invasive procedure, the nurse should explain about the risks and benefits, and informed consent should be obtained. Other factors are not contraindications for CAPD. Monitoring of cardiac and respiratory signs is essential but does not directly affect the procedure. Text Reference - p. 1118

When examining a patient with cardiogenic shock, which signs of peripheral hypoperfusion does the nurse expect? Select all that apply. 1 Cyanosis 2 Cold skin 3 Weak pulse 4 Bradycardia 5 Hypertension

1, 2, 3 Cyanosis, cold skin, and a weak pulse are the signs of peripheral hypoperfusion in cardiogenic shock. Bradycardia and hypertension are not seen in cardiogenic shock; instead, tachycardia and low blood pressure are noted. Text Reference - p. 1633

The health care provider prescribes a dose of dobutamine for a patient in cardiogenic shock due to myocardial infarction. What appropriate actions should the nurse perform for safely administering the medication? Select all that apply. 1 Monitor heart rate and blood pressure. 2 Stop infusion if tachydysrhythmias develop. 3 Always administer with sodium bicarbonate. 4 Administer through a central line. 5 Use a glass bottle for infusion.

1, 2, 4 Dobutamine is a sympathomimetic medication. When used in therapy with dobutamine, the patient's heart rate and blood pressure should be continuously monitored, as they may worsen hypotension, requiring the addition of a vasopressor. The infusion should be stopped if tachydysrhythmias develop. The administration through a central line is recommended, because infiltration leads to tissue sloughing. The drug should not be administered with sodium bicarbonate, because it can get deactivated. Because dobutamine is not adsorbed in plastic containers, it is not necessary to administer the drug in glass bottles. Test-Taking Tip: Key words or phrases in the stem of the question such as first, primary, early, or best are important. Similarly, words such as only, always, never, and all in the alternatives are frequently evidence of a wrong response. As in life, no real absolutes exist in nursing; however, every rule has its exceptions, so answer with care. Text Reference - p. 1643

A patient with chronic kidney disease is advised to undergo peritoneal dialysis (PD). What advantages of PD over hemodialysis should the nurse explain to the patient? Select all that apply. 1 It is a simple procedure. 2 It is home-based. 3 It requires special water systems. 4 It needs a vascular access device. 5 Equipment setup is simple

1, 2, 5 PD has many advantages over hemodialysis. The procedure is simple and home-based, with easy equipment setup. The patient can perform peritoneal dialysis. Because the dialysis is done through the peritoneal membrane, PD does not require a special water system or a vascular access device, as in hemodialysis. Text Reference - p. 1120

A patient with end-stage kidney disease is receiving continuous ambulatory peritoneal dialysis. The patient has a fever and the nurse suspects that it is due to peritonitis. For what are other manifestations that the nurse should monitor the patient? Select all that apply. 1 Vomiting 2 Abdominal pain 3 Bloody stools 4 Weight loss 5 Cloudy peritoneal effluent

1, 2, 5 Peritonitis may manifest as vomiting due to the inflammatory process in the peritoneum. The patient may have pain in the abdomen due to peritoneal irritation caused by the inflammatory process in the peritoneum. The primary clinical manifestations of peritonitis are abdominal pain and cloudy peritoneal effluent with a white blood cell (WBC) count greater than 100 cells/μL (more than 50% neutrophils). An activated immune response may attract WBCs, and an elevated level of WBC in the peritoneal fluid indicates peritonitis. Bloody stool or weight loss is not associated with peritonitis. Peritonitis may not cause hemorrhage; therefore, bloody stools may not be present. Weight loss is usually caused by malnutrition or fluid loss and therefore may not be seen in peritonitis; weight gain may occur due to fluid retention. Text Reference - p. 1119

When examining a patient with septic shock, what symptoms would the nurse expect to find? Select all that apply. 1 Paralytic ileus 2 Gastrointestinal (GI) bleeding 3 Pulsus paradoxus 4 Distended jugular vein 5 Decreased urinary output

1, 2, 5 Patients suffering from septic shock may experience decreased tissue perfusion, which may result in a paralytic ileus, GI bleeding, and decreased urinary output. Pulsus paradoxus and jugular vein distension are found in obstructive shock and are mainly the result of compromised hemodynamics. Text Reference - p. 1637

The registered nurse is teaching a student nurse about physiologic changes in the diuretic phase of a patient with acute kidney disease. Which statement by the student nurse about the diuretic phase indicates effective learning? Select all that apply. 1 "The diuretic phase lasts for one to three weeks." 2 "Urine volume decreases in the diuretic phase." 3 "Hypovolemia occurs during the diuretic phase." 4 "The kidneys will have the ability to concentrate urine." 5 "The creatinine level increases drastically at the end of the diuretic phase."

1, 3 The diuretic phase lasts for one to three weeks and hypovolemia and hypotension occur due to increased urinary output. In the diuretic phase, urine output increases because of the renal tubules' inability to concentrate urine. At the end of the diuretic phase, the creatinine, blood urine nitrogen, and electrolyte levels return to normal. Text Reference - p. 1104

What are the complications of hemodialysis? Select all that apply. 1 Hepatitis 2 Hypertension 3 Muscle cramps 4 Light-headedness 5 Excess coagulation of blood

1, 3, 4 Hemodialysis is extracorporeal removal of waste products such as creatinine, urea, and free water from the blood during renal failure. The complications of hemodialysis include hepatitis, hypotension, muscle cramps, light-headedness and loss of blood. Hepatitis is common in patients who are undergoing dialysis due to the transmission of infection-causing organisms. Hypotension occurs due to rapid removal of vascular volume and decreased cardiac output. Muscle cramps are caused by hypotension, hypovolemia, or high ultrafiltration rate. Light-headedness is caused by a drop in blood pressure. Hemodialysis may cause hypotension and bleeding. Text Reference - p. 1122

10. The x-ray shows that the patient's fracture is at the remodeling stage. What characteristics of the fracture healing process are happening at this stage (select all that apply)? a. Radiologic union b. Absorption of excess cells c. Return to preinjury strength and shape d. Semisolid blood clot at the ends of fragments e. Deposition and absorption of bone in response to stress f. Unorganized network of bone woven around fracture parts

10. a, b, c, e. When the remodeling stage of healing occurs, radiologic union is present. Excess callus is reabsorbed, trabecular bone is laid, and the bone returns to its preinjury structure strength and shape. The osteoblasts and osteoclasts function normally in response to stress. The fracture hematoma stage is when the hematoma at the ends of the fragments becomes a semisolid blood clot. There is an unorganized network of bone composed of cartilage, osteoblasts, calcium, and phosphorus woven around fracture parts in the callus formation stage.

12. A patient with a fractured femur experiences the complication of malunion. The nurse recognizes that what happens with this complication? a. The fracture heals in an unsatisfactory position. b. The fracture fails to heal properly despite treatment. c. Fracture healing progresses more slowly than expected. d. Loss of bone substances occurs as a result of immobilization.

12. a. A malunion occurs when the bone heals in the expected time but in an unsatisfactory position, possibly resulting in deformity or dysfunction. Nonunion occurs when the fracture fails to heal properly despite treatment and delayed union is healing of the fracture at a slower rate than expected. In posttraumatic osteoporosis, the loss of bone substances occurs as a result of immobilization.

14. A young patient with a fractured femur has a hip spica cast applied. While the cast is drying, what should the nurse do? a. Elevate the legs above the level of the heart for 24 hours. b. Turn the patient to both sides and prone to supine every 2 hours. c. Cover the cast with a light blanket to avoid chilling from evaporation. d. Assess the patient frequently for abdominal pain, nausea, and vomiting.

14. d. Complaints of abdominal pain or pressure, nausea, and vomiting are signs of cast syndrome that occur when hip spica casts or body jacket braces are applied too tightly, causing compression of the superior mesenteric artery against the duodenum. The cast may need to be split or removed and the health care provider should be notified. Elevation is not indicated for a spica cast and the patient with a spica cast should not be placed in the prone position during the initial drying stage because the cast is so large and heavy it may break. A cast should never be covered with a blanket because heat builds up in the cast and may increase edema.

Priority Decision: A patient has fallen in the bathroom of the hospital room and reports pain in the upper right arm and elbow. Before splinting the injury, the nurse knows that the priority management of a possible fracture should include which action? a. Elevation of the arm b. Application of ice to the site c. Notification of the health care provider d. Neurovascular checks below the site of the injur

16. d. Pulses, sensation, and motor function distal to the injury should be checked before and after splinting to assess for nerve or vascular damage and documented to avoid doubts about whether a problem discovered later was missed during the original examination or was caused by the treatment. Elevation of the limb and application of ice should be instituted after the extremity is splinted.

17. To assess for neurologic status in a patient with a fractured humerus, what should the nurse ask the patient to do? a. Evert, invert, dorsiflex, and plantar flex the foot. b. Abduct, adduct, and oppose the fingers and pronate and supinate the hand. c. Assess the location, quality, and intensity of pain below the site of the injury. d. Assess the color, temperature, capillary refill, peripheral pulses, and presence of edema in the extremity.

17. b. Neurologic assessment includes evaluation of motor and sensory function and, in the upper extremity, includes abduction and adduction of the fingers, opposition of the fingers, and supination and pronation of the hands. It also includes sensory perception in the fingers. Evaluation of the feet would occur in lower extremity injuries. Assessment of color, temperature, capillary refill, peripheral pulses, and edema evaluates vascular status.

18. A patient is discharged from the outpatient clinic following application of a synthetic fiberglass long arm cast for a fractured ulna. Before discharge, the nurse instructs the patient to do what? a. Never get the cast wet. b. Move the shoulder and fingers frequently. c. Place tape petals around the edges of the cast when it is dry. d. Use a sling to support the arm at waist level for the first 48 hours.

18. b. A patient with any type of cast should exercise the joints above and below the cast frequently and moving the fingers frequently will improve circulation and help to prevent edema. Unlike plaster casts, thermoplastic resin or fiberglass casts are relatively waterproof and, if they become wet, can be dried with a hair dryer on low setting. Tape petals are used on plaster casts to protect the edges from breaking and crumbling but are not necessary for synthetic casts. After the cast is applied, the extremity should be elevated at about the level of the heart to promote venous return and ice may be used to prevent edema.

19. A patient with a fractured tibia accompanied by extensive soft tissue damage initially has a splint applied and held in place with an elastic bandage. What early sign should alert the nurse that the patient is developing compartment syndrome? a. Paralysis of the toes b. Absence of peripheral pulses c. Distal pain unrelieved by opioid analgesics d. Skin over the injury site is blanched when the bandage is removed

19. c. Pain that is distal to the injury and is unrelieved by opioid analgesics is the earliest sign of compartment syndrome. Paralysis and absence of peripheral pulses will eventually occur if it is not treated but these are late signs that often appear after permanent damage has occurred. The overlying skin may appear normal because the surface vessels are not occluded.

The nurse recognizes that which intervention will likely be included in a treatment plan for a patient with chronic kidney disease (CKD) who is undergoing peritoneal dialysis? 1 Restricting potassium intake 2 Encouraging consumption of 25 to 35 kcal/kg/day 3 Avoiding iron supplements when taking erythropoietin 4 Calculating fluid restrictions based on urine output

2 A patient with chronic kidney disease (CKD) must maintain good nutrition and should be referred to a dietitian for nutritional education and guidance. To avoid calorie-protein malnutrition, the patient should consume 25 to 35 kcal/kg/day, which includes calories from dialysate glucose absorption. Potassium is usually not restricted for a patient undergoing peritoneal dialysis. The needs to take iron supplements when on erythropoietin to prevent iron deficiency. Fluid intake in the patient who is undergoing peritoneal dialysis should be unrestricted if weight and blood pressure are in control. Therefore, it is not dependent upon urine output. Text Reference - p. 1114

The nurse recognizes that which recommendation is appropriate for a patient with chronic kidney disease (CKD)? 1 Eat prunes and raisins. 2 Take phosphate binders with meals. 3 Drink plenty of water. 4 Take calcium and iron supplements on an empty stomach.

2 A patient with chronic kidney disease who is prescribed phosphate binders, such as lanthanum carbonate, should take them with meals to reduce gastrointestinal side effects. The patient must avoid potassium-rich foods such as bananas, prunes, and raisins to prevent further aggravation of hyperkalemia. Patients with CKD have decreased urine output and fluid accumulation, so an appropriate fluid balance is important to prevent further complications such as edema and heart failure. The patient should take calcium supplements on an empty stomach for better absorption. Text Reference - p. 1116

A patient donated a kidney via a laparoscopic donor nephrectomy to a nonrelated recipient. The patient is experiencing a lot of pain and refuses to get up to walk. How should the nurse handle this situation? 1 Have the transplant psychologist convince the patient to walk. 2 Encourage even a short walk to avoid complications of surgery. 3 Tell the patient that no other patients have ever refused to walk. 4 Tell the patient that he or she is lucky it was not necessary to have an open nephrectomy.

2 Because ambulating will improve bowel, lung, and kidney function with improved circulation, even a short walk with assistance should be encouraged after pain medication. The transplant psychologist or social worker's role is to determine if the patient is emotionally stable enough to handle donating a kidney, whereas postoperative care is the nurse's role. Trying to shame the patient into walking by telling him or her that other patients have not refused and telling the patient he or she is lucky not to have had an open nephrectomy (implying how much more pain the patient would be having if it had been open) will not be beneficial to the patient or to the postoperative recovery. Text Reference - p. 1127

The nurse is caring for the patient receiving hemodialysis. What action by the nurse is a priority? 1 Checking the patient's skin condition 2 Recording the vital signs every 30 to 60 minutes 3 Recording the patient's weight during the procedure 4 Checking the blood pressure from the extremity with vascular access

2 Blood pressure fluctuates during dialysis and a change in vital signs can indicate rapid changes in blood pressure. Therefore, the nurse should record the vital signs every 30 to 60 minutes during dialysis. The patient's skin condition should be assessed before dialysis for determining the site for vascular access. The patient's weight should be recorded before and after the procedure to determine the amount of fluid to be removed. Blood pressure should not be checked from the same extremity with vascular access because this may cause clotting of the vascular access. Text Reference - p. 1122

Which nursing interventions in a patient with kidney injury would be beneficial in providing safe and effective care? 1 Provide spicy food 2 Provide mouth care 3 Provide plenty of fluids 4 Provide ibuprofen if the patient experiences pain

2 Patients with acute kidney injury experience mucous membrane irritation because of the production of ammonia in the saliva. Therefore, the nurse should provide oral care to prevent stomatitis. Spicy food should be avoided because it may aggravate the irritation. Because the patient has renal impairment, fluid intake should be limited. Ibuprofen, a nonsteroidal antiinflammatory drug, is nephrotoxic and may worsen the kidney injury. Text Reference - p. 1107

The nurse is caring for a patient with chronic kidney disease who is undergoing hemodialysis. What is an appropriate diet for this patient? 1 High-protein and low-calcium 2 Low-protein and low-potassium 3 High-protein and high-potassium 4 Low-protein and high-phosphorus

2 Patients with chronic kidney disease undergoing hemodialysis should consume a diet low in protein and potassium. Calcium needs to be maintained in the diet to help prevent hyperphosphatemia. High protein diets should be avoided because they cause uremic toxicity. High potassium in the diet needs to be avoided because the increased serum potassium level can result in cardiac disturbances. Text Reference - p. 1130

A nurse planning care for a patient with acute renal failure recognizes that the interventions of highest priority are directly related to: 1 Ineffective coping 2 Excess fluid volume 3 Impaired gas exchange 4 Imbalanced nutrition: less than body requirements

2 The issue of excess fluid volume is the primary problem of acute renal failure and the highest priority for the nurse in this situation. The major problem with acute renal failure is altered fluid and electrolyte balance, which, if not managed, can lead to permanent renal damage, cardiac complications, and death. The nursing diagnosis of ineffective coping is due to the acute severity of the illness. The nursing diagnosis of impaired gas exchange is related to excess fluid volume, such as in the development of pulmonary edema. The nursing diagnosis of imbalanced nutrition, less than body requirements, is due to a decrease in appetite as a result of the acute renal failure. Text Reference - p. 1106

What is the term that describes the movement of solutes from a higher concentration area to a lower concentration area? 1 Osmosis 2 Diffusion 3 Dialysate 4 Ultrafiltration

2 The movement of solutes from an area of higher concentration to an area of lower concentration is called diffusion. Osmosis is the movement of solutes from an area of lower concentration to an area of higher concentration. Dialysate is a solution used in dialysis, into which substances from the blood move out. Ultrafiltration is a technique that removes low-molecular solutes such as water and fluid from the semipermeable membrane. Text Reference - p. 1117

A patient with chronic kidney disease is prescribed regular peritoneal dialysis (PD). What should the nurse inform the patient while teaching about PD? 1 Avoid high-protein diets. 2 Take potassium supplements. 3 Restrict fluid intake, as in hemodialysis. 4 Avoid powdered breakfast drinks

2 The patient undergoing regular peritoneal dialysis (PD) does not need to restrict potassium intake; instead, this patient may be prescribed oral potassium supplementation because of hypokalemia caused by dialysis. The patient need not restrict protein or fluid intake. The patient should include enough protein in the diet to compensate for loss of protein in dialysate. The patient may even take liquid or powdered breakfast drinks in case of inadequate protein intake. Patients on hemodialysis have a more restricted fluid intake than patients receiving peritoneal dialysis (PD). Text Reference - p. 1118

A patient presents to the emergency department (ED) in a state of shock. On assessment, the nurse finds that the patient is cyanotic and has crackles on auscultation of the lungs. As which type of shock will the nurse classify this? 1 Neurogenic shock 2 Cardiogenic shock 3 Hypovolemic shock 4 Anaphylactic shock

2 A patient with cardiogenic shock shows peripheral hypoperfusion presenting as cyanosis and has crackles on auscultation of the lungs due to pulmonary congestion. In neurogenic shock, the patient demonstrates symptoms related to the injury such as hypotension and bradycardia. The patient in hypovolemic shock may experience tachycardia as a late sign. In anaphylactic shock, the patient may experience wheezing and stridor. Text Reference - p. 1633

What is a manifestation of the irreversible (refractory) stage of shock? 1 Delirium 2 Areflexia 3 Restlessness 4 Alterations in the level of consciousness

2 Areflexia or loss of reflexes is a manifestation of the irreversible stage of shock. The progressive state of shock is associated with delirium. Restlessness and altered levels of consciousness indicate that the patient is in the compensatory stage of shock. Test-Taking Tip: Identifying content and what is being asked about that content is critical to your choosing the correct response. Be alert for words in the stem of the item that are the same or similar in nature to those in one or two of the options. Text Reference - p. 1639

A patient in shock is receiving 0.9 % NaCl (normal saline solution-NSS). Which nursing intervention is appropriate for this patient? 1 Monitor the patient's vital signs 2 Monitor for the signs of circulatory overload 3 Monitor for signs of hypernatremia in the patient 4 Monitor for allergic reactions and acute renal failure

2 Circulatory overload occurs due to fluid overload. Continuous infusion of 0.9% NaCl increases the fluid volume in the body and may cause circulatory overload. The patient's vital signs must be checked during transfusion of blood or blood products because they could cause an infection or an allergic reaction. Hypernatremia occurs when the patient is on 1.8%, 3%, and 5% NaCl infusions. Infusion of dextran-40 has a tendency to precipitate allergic reactions and acute renal failure. Test-Taking Tip: Do not worry if you select the same numbered answer repeatedly, because there usually is no pattern to the answers. Text Reference - p. 1642

When caring for a critically ill patient who is being mechanically ventilated, the nurse will astutely monitor for which clinical manifestation of multiple organ dysfunction syndrome (MODS)? 1 Increased serum albumin 2 Decreased respiratory compliance 3 Increased gastrointestinal (GI) motility 4 Decreased blood urea nitrogen (BUN)/creatinine ratio

2 Clinical manifestations of MODS include symptoms of respiratory distress, signs and symptoms of decreased renal perfusion, decreased serum albumin and prealbumin, decreased GI motility, acute neurologic changes, myocardial dysfunction, disseminated intravascular coagulation (DIC), and changes in glucose metabolism. Serum albumin is not increased, GI motility decreases in MODS, and the BUN/Creatinine ratio likely will increase. Text Reference - p. 1649

Which drug helps manage renal manifestations in a patient with systemic inflammatory response syndrome (SIRS) and multiple organ dysfunction syndrome (MODS)? 1 Sucralfate 2 Furosemide 3 Omeprazole 4 Acetaminophen

2 Furosemide is a loop diuretic that helps to manage renal manifestations in a patient with systemic inflammatory response syndrome (SIRS) and multiple organ dysfunction syndrome (MODS). Sucrafate is administered for prophylaxis against stress ulcers, which are gastrointestinal manifestations of SIRS and MODS. Omeprazole is a proton pump inhibitor that has the same action. Acetaminophen is an antipyretic drug given as an acute intervention to manage fevers in patients who are in shock. Text Reference - p. 1651

Which type of shock is associated with hyperglycemia, presence of pulmonary infiltrates in chest x-ray and increased levels of blood urea nitrogen (BUN)? 1 Septic 2 Cardiogenic 3 Obstructive 4 Hypovolemic

2 Increased blood levels of glucose, nitrogen, cardiac markers and presence of pulmonary infiltrates are seen in cardiogenic shock. Increased blood levels of lactate, glucose, and positive blood cultures are signs of septic shock. Manifestations of obstructive shock are specific to the area or organ of obstruction. Electrolyte imbalances and decreased hemoglobin and hematocrit are seen in hypovolemic shock. Test-Taking Tip: Identify option components as correct or incorrect. This may help you identify a wrong answer. Text Reference - p. 1635

Which type of shock is associated with bradycardia? 1 Septic shock 2 Neurogenic shock 3 Hypovolemic shock 4 Anaphylactic shock

2 Neurogenic shock is associated with bradycardia. Myocardial dysfunction and changes in body temperature are signs of septic shock. Reduction in preload, capillary refill and stroke volume are clinical manifestations of hypovolemic shock. Chest pain is seen in anaphylactic shock. Test-Taking Tip: Make certain that the answer you select is reasonable and obtainable under ordinary circumstances and that the action can be carried out in the given situation. Text Reference - p. 1635

Which cardiovascular change is commonly found in patients with systemic inflammatory response syndrome (SIRS)? 1 Decrease in heart rate 2 Decrease in capillary refill 3 Decrease in central venous pressure 4 Decrease in pulmonary artery wedge pressure

2 Patients with systemic inflammatory response syndrome (SIRS) have decreased capillary refill. Other cardiovascular changes include increases rather than decreases in heart rate, central venous pressure, and pulmonary artery wedge pressure. Test-Taking Tip: Multiple-choice questions can be challenging, because students think that they will recognize the right answer when they see it or that the right answer will somehow stand out from the other choices. This is a dangerous misconception. The more carefully the question is constructed, the more each of the choices will seem like the correct response. Text Reference - p. 1649

Which system of the body is often the first to show signs of dysfunction in systemic inflammatory response syndrome (SIRS) and multiple organ dysfunction syndrome (MODS)? 1 Neurologic system 2 Respiratory system 3 Cardiovascular system 4 Gastrointestinal system

2 Systemic inflammatory response syndrome (SIRS) and multiple organ dysfunction syndrome (MODS) occur due to a systemic inflammatory response. Inflammatory mediators have a direct effect on the pulmonary vasculature. Thus, the respiratory system is often the first system to show signs of dysfunction. Signs of nervous system dysfunction, such as mental changes, can be early signs of SIRS and MODS. However, the nervous system is not the first system to show signs of SIRS and MODS. When the respiratory system is affected, ventilation-perfusion mismatch becomes worse. Tissue oxygen demands increase, leading to cardiovascular changes. Hence, cardiovascular changes occur after changes in the respiratory system. In the early stages of SIRS and MODS, there is shunting away of blood from the gastrointestinal tract, making it vulnerable to ischemic injury. However, such changes show signs of dysfunction later than does the respiratory system. Text Reference - p. 1649

A patient admitted to the hospital after a motor vehicle accident (MVA) is in hypovolemic shock. On examination, the nurse finds that the patient is becoming anxious, and the urine output is decreasing. What appropriate action should the nurse perform? 1 Prepare for administering blood products. 2 Begin crystalloid fluid replacement. 3 Start fluids only if deterioration occurs. 4 Wait for the patient to compensate naturally.

2 When the volume of blood loss is less than 30 percent, crystalloid fluid replacements are performed to reverse tissue dysfunction. Blood products are administered when the blood volume loss is more than 30 percent. The nurse should not wait for deterioration to occur to start the fluid replacement therapy. This is because the body can typically compensate naturally for a blood volume loss up to 15 percent. Test-Taking Tip: Be alert for grammatical inconsistencies. If the response is intended to complete the stem (an incomplete sentence) but makes no grammatical sense to you, it might be a distractor rather than the correct response. Question writers typically try to eliminate these inconsistencies. Text Reference - p. 1633

A nurse is examining a patient with anaphylactic shock due to an insect bite. What types of skin manifestations would the nurse expect to find? Select all that apply. 1 Pallor 2 Pruritus 3 Flushing 4 Urticaria 5 Cold, clammy skin

2, 3, 4 Insect bites may cause allergic reactions and anaphylactic shock. The skin manifestations may include pruritus, flushing, and urticaria caused by massive vasodilation, release of vasoactive mediators, and an increase in capillary permeability. Pallor and cold, clammy skin changes are usually seen in cardiogenic, hypovolemic, and obstructive shock caused by decreased circulatory volume and tissue perfusion. Text Reference - p. 1636

A patient with chronic kidney disease is at risk for anemia. Arrange the events in the order in which they lead to anemia caused by chronic kidney disease. 1. Bone marrow fibrosis 2. Elevated levels of parathyroid hormone (PTH) 3. Inhibition of erythropoiesis 4. Shortened survival of red blood cells (RBCs)

2, 3, 4, 1 Elevated levels of PTH, produced to compensate for low serum calcium levels, can inhibit erythropoiesis, shorten the survival of RBCs, and cause bone marrow fibrosis, which can result in a decrease in hematopoietic cells. Text Reference - p. 1109

The patient's glomerular filtration rate (GFR) is 15 mL/min. What are the treatment options the nurse would expect the health care provider to discuss with the patient? Select all that apply. 1 Nephrectomy 2 Hemodialysis 3 Peritoneal dialysis 4 Kidney transplant in place of dialysis 5 Continuous ambulatory peritoneal dialysis

2, 3, 5 Any dialysis option would be appropriate for the patient. A nephrectomy is not going to cure the chronic kidney disease, and it is unknown whether the kidney has a tumor or cancer with this question. Kidney placement in place of dialysis at this point is too late. Dialysis needs to begin while awaiting a kidney transplant. Text Reference - p. 1117

A patient with acute kidney injury has been admitted to the hospital, and the nurse observes the electrocardiogram (ECG) reading shows tall peaked T waves, ST depression, and QRS widening. What nursing interventions should the nurse perform for this patient?? Select all that apply. 1 Ensure potassium intake of 50 mEq/day. 2 Administer regular insulin intravenously. 3 Administer sodium bicarbonate. 4 Administer diuretics as ordered. 5 Administer calcium gluconate intravenously.

2, 3, 5 ECG readings for this patient are indicative of cardiac changes due to hyperkalemia induced by acute kidney injury. Regular insulin, administered intravenously, helps the potassium to move into the cells. Sodium bicarbonate corrects the acidosis and causes the potassium to shift into the cells. Calcium gluconate raises the threshold for excitation, protecting the heart. The potassium intake should be limited to 40 mEq/day. Diuretics are not effective in hyperkalemia. Text Reference - p. 1105

While planning the management of oxygen delivery in a patient with shock, what appropriate measures should the nurse undertake? Select all that apply. 1 Encourage the patient to move around to increase lung expansion. 2 Space activities that increase oxygen consumption. 3 Monitor continuously by using a central venous catheter. 4 Space activities that decrease oxygen consumption. 5 Administer supplemental oxygen as prescribed.

2, 3, 5 To optimize oxygen supply and ventilation in a patient suffering from shock, the activities that increase oxygen consumption should be evenly spaced. Mixed venous oxygen saturation should be monitored through a central venous catheter. The patient should not exert energy by excessive moving around because it increases oxygen demand. Text Reference - p. 1641

When managing a patient with shock, which appropriate actions should the nurse take as part of nutritional therapy? Select all that apply. 1 Plan enteral feeding to meet at least 50 percent of calorie requirements. 2 Start enteral nutrition within the first 24 hours. 3 Wait until the patient recovers to start with enteral nutrition. 4 Start a slow continuous drip of small amounts of enteral feedings. 5 Start parenteral nutrition if enteral feedings are contraindicated.

2, 4, 5 Enteral nutrition should ideally begin within the first 24 hours. It is important because it enhances the perfusion of the (gastrointestinal) GI tract and helps to maintain the integrity of the gastrointestinal mucosa. Enteral feeding should be started with a slow continuous drip of small amounts of enteral feedings. If enteral feedings are contraindicated, parenteral feedings can be started. Enteral feeding should be planned to meet at least 80 percent of the total calorie requirements, but if it is not feasible, parenteral feeding should be started. Nutritional therapy should start as early as possible. The nurse should not wait for the patient to recover before starting nutritional therapy. Text Reference - p. 1644

A patient with chronic kidney disease has developed uremic syndrome. What complications should the nurse anticipate due to an increase in blood urea levels? Select all that apply. 1 Anemia 2 Pericarditis 3 Hypertension 4 Pulmonary edema 5 Hemorrhagic tendencies

2, 5 Uremic pericarditis is one of the cardiac complications of chronic renal failure. Uremia can cause qualitative defects in platelet function, thereby predisposing the patient to hemorrhages. Anemia is caused by decreased production of erythropoietin from the kidneys. Hypertension is caused by sodium retention and increased extracellular fluid volume. Pulmonary edema could be a consequence of both fluid overload and hypertension. Text Reference - p. 1117

What are the primary pathophysiologic changes that occur in the injury or exudative phase of ARDS (select all that apply)? a. Atelectasis b. Shortness of breath c. Interstitial and alveolar edema d. Hyaline membranes line the alveoli e. Influx of neutrophils, monocytes, and lymphocytes

22. a, c, d. The injury or exudative phase is the early phase of ARDS when atelectasis and interstitial and alveoli edema occur and hyaline membranes composed of necrotic cells, protein, and fibrin line the alveoli. Together, these decrease gas exchange capability and lung compliance. Shortness of breath occurs but it is not a physiologic change. The increased inflammation and proliferation of fibroblasts occurs in the reparative or proliferative phase of ARDS, which occurs 1 to 2 weeks after the initial lung injury.

22. The woman with osteoporosis slipped on the ice and now her wrist hurts. If there is a fracture, what type of fracture is expected? a. Dislocation b. Open fracture c. Colles' fracture d. Incomplete fracture

22. c. A Colles' fracture most often occurs in patients over 50 years of age with osteoporosis and frequently when the patient attempts to break a fall with an outstretched arm and hand. Dislocation is the complete separation of articular surfaces of the joint caused by a ligament injury. Open fracture is when there is communication with the external environment. A fracture is incomplete if only part of the bone shaft is fractured and the bone is still in one piece.

24. In a patient with a stable vertebral fracture, what should the nurse teach the patient to do? a. Remain on bed rest until the pain is gone. b. Logroll to keep the spine straight when turning. c. How to use bone cement to correct the problem. d. Take as much analgesic as needed to relieve the pain.

24. b. The spine should be kept straight by turning the shoulders and hips together (logrolling). This keeps the spine in good alignment until union has been accomplished. Bed rest may be required for a short time but not until the pain is gone. Analgesics should be taken only as ordered. If they do not relieve the pain, the health care provider should be notified. Bone cement is used by the surgeon to stabilize vertebral compression fractures.

When is a fat embolism most likely to occur? a. 24 to 48 hours following a fractured tibia b. 36 to 72 hours following a skull fracture c. 4 to 5 days following a fractured femur d. 5 to 6 days following a pelvic fracture

25. a. Initial manifestations of fat embolism usually occur 24 to 48 hours after injury and are associated with fractures of long bones and multiple fractures related to pelvic injuries, including fractures of the femur, tibia, ribs, and pelvis.

28. An older adult woman is admitted to the emergency department after falling at home. The nurse cautions her not to put weight on the leg after finding what in the patient assessment? a. Inability to move the toes and ankle b. Edema of the thigh extending to the knee c. Internal rotation of the leg with groin pain d. Shortening and external rotation of the leg

28. d. The classic signs of a hip fracture are shortening of the leg and external rotation accompanied by severe pain at the fracture site and additional injury could be caused by weight bearing on the extremity. The patient may not be able to move the hip or the knee but movement in the ankle and toes is not affected.

The registered nurse is teaching a student nurse about physiologic changes in a kidney transplant recipient. Which statement made by the student nurse indicates the need for further teaching? 1 "The urinary output of the patient can be 1 L/hour." 2 "There may be an imbalance in the electrolyte levels." 3 "Decrease in the urine output after surgery can be neglected." 4 "Normal saline solution is infused to treat metabolic acidosis."

3 A decrease in the urine output after healthy kidney transplantation indicates rejection, dehydration, or urinary leakage. This is a serious condition and should be reported to the primary health care provider. An increased urine output of 1 L/hour after kidney transplant indicates proper functioning of the transplanted kidney. Due to increased elimination, electrolyte imbalance can. Normal saline solution should be infused to the patient to rectify metabolic acidosis caused by delayed kidney function. Text Reference - p. 1127

The nurse provides information to a nursing student about the administration of erythropoietin (EPO) therapy to a patient with chronic kidney disease (CKD). Which statement made by the nursing student indicates effective learning? 1 "EPO benefits a patient with plasma ferritin concentrations less than 100 mg/mL." 2 "EPO should be administered in higher doses to a patient with low hemoglobin levels." 3 "EPO, iron, sucrose, and folic acid of 1 mg/day should be administered to patients undergoing hemodialysis." 4 "EPO can be safely given to a patient that takes an antihypertensive and maintains a blood pressure of 150/90 mm Hg."

3 A patient with chronic kidney disease (CKD) develops anemia due to decreased production of erythropoietin (EPO). Exogenous erythropoietin helps replenish the erythropoietin demand. Iron supplements for a patient with low plasma ferritin levels prevent the patient from developing an iron deficiency from the increased demand for iron to support erythropoiesis. A folic acid supplement is given to patients on hemodialysis because it is required for red blood cell (RBC) formation, and is removed by dialysis. A high dosage of EPO should be avoided for a patient with anemia because of increased risk of thromboembolic events and death from cardiovascular effects. The recommendation is to use the lowest possible dose of EPO to treat anemia. EPO should be avoided for a patient with uncontrolled hypertension because it exacerbates hypertension by increasing blood viscosity. Text Reference - p. 1113 Topics

The nurse reviews the medical record of a patient with chronic kidney disease (CKD) and notes a history of taking cholecalciferol, a vitamin D level of 20 mg/mL, a calcium level of 13 mg/dL, and a phosphorus level of 5 mg/dL. Based on the laboratory results, the nurse anticipates that what medication will be prescribed? 1 Calcitriol 2 Calcium acetate 3 Sevelamer carbonate 4 Polystyrene sulfonate

3 A patient with chronic kidney disease (CKD) may have low vitamin D levels. Vitamin D supplementation using cholecalciferol is recommended for patients who have vitamin D levels less than 30 mg/dL, but it can cause hypercalcemia. The laboratory reports of the patient show a calcium level of 13 mg/dL and a phosphorus level of 5 mg/dL, which are higher than normal values (calcium 8.6 to 10.2 mg/dL and phosphorous 2.4 to 4.4 mg/dL). Therefore, the patient should be given non-calcium-based phosphate binders such as sevelamer carbonate to lower the phosphate levels. Calcitriol is an activated form of vitamin D, which is indicated for severe hypocalcemia in CKD. It may further aggravate hypercalcemia and hyperphosphatemia on administration. Calcium acetate is a calcium-based phosphate binder, which may further increase calcium levels, leading to hypercalcemia. Polystyrene sulfonate is a potassium-binding agent used in patients with severe hyperkalemia. Text Reference - p. 1113

The dialysis nurse is administering hemodialysis to a patient with chronic kidney failure. For what common complication should the nurse carefully monitor in this patient? 1 Hernias 2 Pneumonia 3 Hypotension 4 Lower back pain

3 A rapid removal of fluid results in reduced vascular volume, which can lead to a decreased cardiac output and decreased vascular resistance. Therefore, hemodialysis has the potential to cause hypotension during the process. Peritoneal dialysis is associated with hernias, lower back pain, and pneumonia, due to increased intraabdominal pressure while infusing the dialysate and decreased lung expansion caused by frequent upward displacement of the diaphragm. Text Reference - p. 1122

The nurse is caring for a patient with acute kidney injury and secondary hypertension. The urine output is 0.1 mL/kg/hr over 6 hours. The primary health care provider identifies atheroembolic renal disease. Which laboratory finding supports this conclusion? 1 Increased basophils 2 Increased monocytes 3 Increased eosinophils 4 Increased lymphocytes

3 An increased eosinophil level in the laboratory report is an indicator of atheroembolic renal disease, which supports the primary health care provider's conclusion. Increased levels of basophils, monocytes, and lymphocytes are not associated with atheroembolic renal disease. Test-Taking Tip: Patients with acute kidney injury have a risk of acquiring infection. Apply your knowledge and skill to answer the question. Text Reference - p. 1104

The nurse is caring for a patient with severe burns in the emergency department. His laboratory values reveal serum creatinine level of 5 mg/dL, and the glomerular filtration rate (GFR) has decreased by 75%. What stage of acute kidney failure is this patient exhibiting? 1 Risk 2 Injury 3 Failure 4 Loss

3 As per the RIFLE (Risk, Injury, Failure, Loss, and End-stage) classification for staging acute kidney injury, this patient is at the Failure stage. When the GFR has decreased by 25%, the patient is at the Risk stage. The patient with a GFR that has decreased by 50% is at the Injury stage. The patient with persistent acute kidney failure experiences a complete loss of kidney function and is at the Loss stage. Text Reference - p. 110

Which condition is seen in patients with bilateral ureteral obstruction? 1 Oliguria 2 Prostate cancer 3 Hydronephrosis 4 Diabetic gastroparesis

3 Bilateral ureteral obstruction results in dilation of the kidneys, which is called hydronephrosis. Oliguria is a sign of acute kidney injury. Prostate cancer is a postrenal cause of acute kidney injury. Diabetic gastroaresis is a manifestation of chronic kidney disease. Text Reference - p. 1103

The nurse is educating a patient about the insertion of a catheter with a Dacron cuff for delivery of peritoneal dialysis. What rationale should the nurse provide detailing the benefit of this type of catheter? 1 To remove nonprotein solutes 2 To propel blood through the circuit 3 To prevent the migration of microorganisms 4 To act as a bridge between arterial and venous blood

3 Dacron cuffs acts as anchors and prevent the migration of microorganisms into the peritoneal cavity. Hemofilters in continuous renal replacement therapy (CRRT) remove nonprotein solutes and plasma water. Blood pumps are a part of CRRT; they are used to pump blood through the circuit. Grafts are used in hemodialysis to separate the blood from arteries and veins. Text Reference - p. 1118

The nurse recognizes that which medication is appropriate to give to patients with kidney failure? 1 Magnesium antacids 2 Aluminum preparations 3 Angiotensin receptor blockers 4 Nonsteroidal antiinflammatory agents

3 Hypertension is a common finding in a patient with kidney failure due to retention of sodium and water. This is treated with angiotensin receptor blockers. Magnesium antacids may aggravate hypermagnesemia in patients with kidney failure. Aluminum preparations should be used with caution in patients with chronic kidney disease because they are associated with bone diseases, such as osteomalacia. Nonsteroidal antiinflammatory agents are nephrotoxic, and should not be administered to patients with renal failure because they can cause acute kidney injury. Text Reference - p. 1113

A primary health care provider has ordered frequent magnetic resonance imaging using a gadolinium contrast agent in a patient with kidney failure. The patient reports darkness of the skin, joint pain, and limited joint movement. What condition does the nurse suspect? 1 Asterixis 2 Hydronephrosis 3 Nephrogenic systemic fibrosis 4 Contrast-induced nephropathy

3 Nephrogenic systemic fibrosis is caused by the administration of gadolinium to patients with kidney failure; signs include hyperpigmentation of the skin and joint contractures. Asterixis is characterized by a tremor-like condition upon stretching of the wrist. Hydronephrosis is caused by bilateral ureteral obstruction. Contrast-induced nephropathy is a kidney injury caused by the injection of contrast agents during surgery or diagnostic testing. Test-Taking Tip: Identify option components as correct or incorrect. This may help you identify a wrong answer. Text Reference - p. 1105

Which statement about acute kidney injury is correct? 1 Parenchymal damage occurs in prerenal oliguria. 2 Prerenal azotemia results in increased sodium excretion. 3 Prerenal oliguria is caused by decreased circulatory volume. 4 Prerenal causes of acute kidney injury increase the glomerular filtration rate.

3 Prerenal oliguria is caused by a decrease in the circulatory volume due to dehydration and congestive cardiac failure. Parenchymal damage does not occur in prerenal oliguria. Prerenal azotemia results in decreased sodium excretion, which leads to increased sodium and water retention. Reduced systemic circulation is a prerenal cause that leads to a decrease in the blood flow to the kidneys. Therefore, the glomerular filtration rate also decreases. Text Reference - p. 1102

The registered nurse is teaching a student nurse about the criteria for a deceased donor to donate an organ. Which student nurse's statement indicates effective learning? 1 "Diabetes mellitus has no effect on organ transplant." 2 "Immunocompromised donors can donate their organs." 3 "Donors should be free from active intravenous drug abuse." 4 "Donors with compromised cardiovascular health can be included."

3 The deceased donor should be free from active intravenous drug use. Donors with a long history of diabetes mellitus, autoimmune disorders, and defective cardiovascular functions cannot donate organs. Text Reference - p. 1125

The nurse assesses a patient with multisystem organ dysfunction syndrome. What assessment finding is most indicative of deterioration? 1 Arterial PO2 of 95% 2 Pulse rate of 108 beats/minute 3 Total urine output of 120 mL over the past eight hours 4 Auscultation of fine bilateral crackles and a moist cough

3 A decrease in urine output to less than 30 mL/hr in an adult is an early indication of hypoperfusion to the kidneys, as well as other vital organs. This may or may not be accompanied by changes in vital signs. An arterial PO2 of 95%, a pulse rate of 108 beats/minute, and auscultation of crackles and a moist cough may also be indications of early deterioration but are not as specific as a low urinary output in identifying deterioration. Text Reference - p. 1649

A patient is brought to the emergency department (ED) after multiple bee stings. On assessment, the nurse finds that the patient has edema on the lips and tongue as well as chest pain, dizziness, wheezing, and stridor. What type of shock should the nurse document this as? 1 Septic shock 2 Neurogenic shock 3 Anaphylactic shock 4 Obstructive shock

3 Anaphylactic shock is an acute, life-threatening hypersensitivity reaction to a sensitizing substance that, in this case, is insect venom. The reaction quickly causes massive vasodilation, release of vasoactive mediators, and an increase in capillary permeability. As capillary permeability increases, fluid leaks from the vascular space into the interstitial space. The consequences of these pathophysiologic processes include edema on the lips and tongue, chest pain, wheezing, and stridor. Sepsis is a systemic inflammatory response to a documented or suspected infection. Neurogenic shock is a hemodynamic phenomenon that can occur within 30 minutes of a spinal cord injury at the fifth thoracic (T5) vertebra or above. Text Reference - p. 1636

Why is there a loss of lean body mass in patients with systemic inflammatory response syndrome (SIRS) and multiorgan dysfunction syndrome (MODS)? 1 Hypoglycemia occurs. 2 Glucose is converted to glycogen. 3 Fatty acids are mobilized for fuel. 4 Glucose is converted to amino acids.

3 Both SIRS and MODS trigger a hypermetabolism response leading to mobilization of fatty acids for fuel. Such a catabolic state leads to loss of lean body mass. Because catecholamines and glucocorticoids are released, hyperglycemia occurs, not hypoglycemia. Hyperglycemia also occurs, because glycogen stores are converted into glucose. Once glycogen is depleted, amino acids are converted into glucose and there is a reduction in protein stores. Text Reference - p. 1649

The nurse recalls that cardiogenic shock is differentiated from other forms of shock because the patient with cardiogenic shock typically experiences: 1 Hypotension 2 Dysrhythmias 3 Volume excess 4 Volume depletion

3 In cardiogenic shock the heart fails as a pump. This usually results in fluid retention and poor perfusion of organs, including the kidneys, which adds to the fluid volume excess. As cardiogenic shock progresses, hypotension will develop, but it is not the cause of cardiogenic shock. Dysrhythmias may or may not be present with cardiogenic shock. Volume depletion is not generally seen with cardiogenic shock. It is, however, seen more with hypovolemic or distributive forms of shock. Text Reference - p. 1633

What causes dysrhythmias in patients with systemic inflammatory response syndrome (SIRS) and multiple organ dysfunction syndrome (MODS)? 1 Metabolic acidosis resulting in increase in lactate levels 2 Increased capillary permeability in the cardiovascular system 3 Hypokalemia due to activities of aldosterone and catecholamines 4 Aldosterone-mediated sodium and water reabsorption in kidneys

3 In systemic inflammatory response syndrome (SIRS) and multiple organ dysfunction syndrome (MODS), there are hormonal and metabolic changes and fluid shifts, including the release of aldosterone and catecholamines. Aldosterone increases urinary potassium loss and catecholamines cause potassium to move into the cell, resulting in hypokalemia, which causes dysrhythmias. Dysrhythmias are not associated with an increased lactate level caused by metabolic acidosis. Increased capillary permeability causes tachycardia rather than dysrhythmias. The increase in sodium reabsorption raises the serum osmolality and stimulates the release of the antidiuretic hormone (ADH). Increased sodium is not related to the development of dysrhythmias. Text Reference - p. 1649

Which medical emergency is caused by the failure of two or more organ systems? 1 Crush syndrome 2 Toxic shock syndrome 3 Multiple organ dysfunction syndrome (MODS) 4 Systemic inflammatory response syndrome (SIRS)

3 Multiple organ dysfunction syndrome is caused by the failure of two or more organ systems in an acutely ill patient. A crushing injury to the skeletal muscle causes crush syndrome, which is characterized by shock and renal failure. Bacterial toxins cause toxic shock syndrome, which is characterized by high fever, hypotension and malaise. Systemic inflammatory response syndrome (SIRS) is a systemic inflammatory response caused by infection, ischemia, infarction and injury. Test-Taking Tip: Identifying content and what is being asked about that content is critical to your choosing the correct response. Be alert for words in the stem of the item that are the same or similar in nature to those in one or two of the options. Text Reference - p. 1649

The nurse is caring for a patient who developed cardiogenic shock. Which medical diagnosis does the nurse suspect? 1 Urosepsis 2 Hemorrhage 3 Myocardial infarction 4 Tension pneumothorax

3 Myocardial infarction may produce necrotic areas of cardiac tissue that lead to impaired contractility and decreased cardiac output. This may lead to a cardiogenic shock state. Hemorrhage may lead to a hypovolemic shock state, tension pneumothorax may lead to an obstructive shock state, and urosepsis may lead to a septic shock state. Text Reference - p. 1633

A patient experiences a myocardial infarction (MI). The nurse closely monitors the patient for complications and recognizes that hypotension is a warning sign of: 1 A secondary MI 2 Pulmonary edema 3 Cardiogenic shock 4 Fatal dysrhythmias

3 One of the initial cardinal signs of cardiogenic shock after a myocardial infarction (MI) is a slow, steady drop in blood pressure. Hypotension after an MI may be an indirect sign of a secondary MI or a fatal dysrhythmia. Depending on the origin of pulmonary edema, patients may experience hypotension or hypertension. Text Reference - p. 1633

A patient's localized infection has progressed to the point where septic shock now is suspected. What medication is an appropriate treatment modality for this patient? 1 Insulin infusion 2 Intravenous (IV) administration of epinephrine 3 Aggressive IV crystalloid fluid resuscitation 4 Administration of nitrates and β-adrenergic blockers

3 Patients in septic shock require large amounts of crystalloid fluid replacement. Epinephrine is indicated in anaphylactic shock, and insulin infusion is not normally necessary in the treatment of septic shock (but can be). Nitrates and β-adrenergic blockers are used most often in the treatment of patients in cardiogenic shock. Text Reference - p. 1636

A patient in neurogenic shock is receiving phenylephrine. Which nursing actions are appropriate when caring for this patient? 1 Monitoring for signs of dyspnea and pulmonary edema 2 Monitoring for signs of hypokalemia and hyperglycemia 3 Monitoring for signs of reflex bradycardia and restlessness 4 Monitoring for signs of hypothyroidism and Addison's disease

3 Phenylephrine is α-adrenergic agonist and may cause bradycardia and restlessness due to central nervous system stimulation. β-adrenergic agonists such as epinephrine cause dyspnea and pulmonary edema. Phenylephrine does not increase the elimination of potassium levels or blood glucose levels. Therefore, the patient does not have risk of hypokalemia and hyperglycemia. Phenylephrine does not impair thyroid and adrenal gland functioning. Therefore, the nurse will not monitor for the signs of hypothyroidism and Addison's disease. Test-Taking Tip: Identify option components as correct or incorrect. This may help you identify a wrong answer. Text Reference - p. 1643

A patient is admitted to the hospital with a suspected diagnosis of obstructive shock. What could be the possible causes of this type of shock? Select all that apply. 1 Hypersensitivity to antibiotics 2 Spinal cord injury 3 Cardiac tamponade 4 Tension pneumothorax 5 Superior vena cava syndrome

3, 4, 5 Obstructive shock develops when a physical obstruction to blood flow occurs resulting in decreased cardiac output. This can be caused by restricted diastolic filling of the right ventricle from compression caused by cardiac tamponade, tension pneumothorax, or superior vena cava syndrome. Hypersensitivity to antibiotics may cause anaphylactic shock. Spinal cord injury may lead to neurogenic shock. Text Reference - p. 1637

30. A patient with a fractured right hip has an anterior open reduction and internal fixation of the fracture. What should the nurse plan to do postoperatively? a. Get the patient up to the chair on the first postoperative day. b. Position the patient only on the back and the unoperative side. c. Keep the leg abductor pillow on the patient even when bathing. d. Ambulate the patient with partial weight bearing by discharge.

30. a. Because the fracture site is internally fixed with pins or plates, the fracture site is stable and the patient is moved from the bed to the chair on the first postoperative day. Ambulation begins on the first or second postoperative day without weight bearing on the affected leg. Weight bearing on the affected extremity is usually restricted for 6 to 12 weeks until adequate healing is evident on x-ray. The patient may be positioned on the operative side following internal fixation and abductor pillows are used for patients who have total hip replacements.

31. What should the nurse include in discharge instructions for the patient following a hip prosthesis with a posterior approach? a. Restrict walking for 2 to 3 months. b. Take a bath rather than a shower to prevent falling. c. Keep the leg internally rotated while sitting and standing. d. Have a family member put on the patient's shoes and socks.

31. d. Patients with hip prostheses with a posterior approach must avoid extreme flexion, adduction, or internal rotation for at least 6 weeks to prevent dislocation of the prosthesis. Gradual weight bearing on the limb is allowed and ambulation should be encouraged. The leg should be not be externally rotated.

A patient complains of pain in the foot of a leg that was recently amputated. What should the nurse recognize about this pain? a. It is caused by swelling at the incision. b. It should be treated with ordered analgesics. c. It will become worse with the use of a prosthesis. d. It can be managed with diversion because it is psychologic.

34. b. Phantom sensation or phantom pain may occur following amputation, especially if pain was present in the affected limb preoperatively. The pain is a real sensation to the patient and will first be treated with analgesics and other pain interventions (i.e., tricyclic antidepressants, antiseizure drugs, transcutaneous electrical nerve stimulation [TENS], mirror therapy, acupuncture). As recovery and ambulation progress, phantom limb sensation usually subsides.

35. Priority Decision: An immediate prosthetic fitting during surgery is used for a patient with a traumatic below-theknee amputation. During the immediate postoperative period, what is a priority nursing intervention? a. Monitor the patient's vital signs. b. Assess the incision for hemorrhage. c. Elevate the residual limb on pillows. d. Have the patient flex and extend the knee every hour.

35. a. Because the device covers the residual limb, the surgical site cannot be directly seen and postoperative hemorrhage is not apparent on dressings, requiring vigilant assessment of vital signs for signs of bleeding. Elevation of the residual limb with an immediate prosthetic fitting is not necessary because the device itself prevents edema formation. Exercises to the leg are not performed in the immediate postoperative period to avoid disruption of ligatures and the suture line.

36. Why does a nurse position a patient with an above-the-knee amputation with a delayed prosthetic fitting prone several times a day? a. To prevent flexion contractures b. To assess the posterior skin flap c. To reduce edema in the residual limb d. To relieve pressure on the incision site

36. a. Flexion contractures, especially of the hip, may be debilitating and delay rehabilitation of the patient with a leg amputation. To prevent hip flexion, the patient should avoid sitting in a chair with the hips flexed or having pillows under the surgical extremity for prolonged periods and the patient should lie on the abdomen for 30 minutes three to four times a day to extend the hip.

37. A patient who had a below-the-knee amputation is to be fitted with a temporary prosthesis. It is most important for the nurse to teach the patient to do what? a. Inspect the residual limb daily for irritation. b. Apply an elastic shrinker before applying the prosthesis. c. Perform range-of-motion (ROM) exercises to the affected leg four times a day. d. Apply alcohol to the residual limb every morning and evening to toughen the skin.

37. a. Skin breakdown on the residual limb can prevent the use of a prosthesis so the limb should be inspected every day for signs of irritation or pressure areas. No substances except water and mild soap should be used on the residual limb and range-of-motion (ROM) exercises are not necessary when the patient is using a prosthesis. A residual limb shrinker is an elastic stocking that is used to mold the limb in preparation for prosthesis use but a cotton residual limb sock is worn with the prosthesis.

38. Which joint surgery is used to arthroscopically remove degenerative tissue in joints? a. Osteotomy b. Arthrodesis c. Debridement d. Synovectomy

38. c. Debridement removes degenerative tissue from joints. Osteotomy corrects bone deformity by removal of a wedge or slice of bone. Arthrodesis surgically fuses a joint to relieve pain. Synovectomy removes tissue involved in joint destruction from rheumatoid arthritis (RA).

39. When the nursing student asks the RN what an arthroplasty is, what is the best description the RN can give the student? a. Surgical fusion of a joint to relieve pain b. Correction of bone deformity by removal of a wedge or slice of bone c. Reconstruction or replacement of a joint to relieve pain and correct deformity d. Used in rheumatoid arthritis to remove the tissue involved in joint destruction

39. c. An arthroplasty is reconstruction or replacement of a joint to relieve pain and correct deformity, especially with osteoarthritis, RA, avascular necrosis, congenital deformities, or dislocations. Arthrodesis is the surgical fusion of a joint to relieve pain. An osteotomy removes a wedge of bone to correct a bone deformity. Synovectomy is used in RA to remove the tissue involved in joint destruction.

The nurse preparing to administer a dose of calcium acetate to a patient with chronic kidney disease (CKD) should know that this medication should have a beneficial effect on which laboratory value? 1 Sodium 2 Potassium 3 Magnesium 4 Phosphorus

4 Phosphorus and calcium have inverse or reciprocal relationships, meaning that when phosphorus levels are high, calcium levels tend to be low. Therefore, administration of calcium should help to reduce a patient's abnormally high phosphorus level, as seen with CKD. Calcium acetate will not have an effect on sodium, potassium, or magnesium levels. Text Reference - p. 1113

The student nurse is observing the administration of peritoneal dialysis by the dialysis nurse. What statement made by the student to the nurse demonstrates understanding about the dialysis process? 1 "The use of a hemofilter will return blood back to the patient." 2 "The Dacron cuff will return blood from the dialyzer back to the patient." 3 "The red catheter lumen will return blood from the dialyzer back to the patient." 4 "The blue catheter lumen will return blood from the dialyzer back to the patient."

4 A blue catheter lumen returns blood from the dialyzer back to the patient. A hemofilter is a part of the continuous renal replacement therapy (CRRT), which removes nonprotein solutes and plasma water. A Dacron cuff is used to fix the catheter in place in peritoneal dialysis. A red catheter lumen is used to withdraw blood from the patient and send it to the dialyzer for purification. Text Reference - p. 1121

A patient with a glomerular filtration rate (GFR) of 30 mL/min has a hemoglobin of 5 g/dL. The peripheral smear tests show that the red blood cells are normocytic and normochromic. The nurse suspects that which physiologic change led to this condition? 1 Reduced excretion of potassium 2 Increased extracellular fluid volume 3 Defective reabsorption of bicarbonate 4 Decreased production of erythropoietin

4 A patient with a glomerular filtration rate (GFR) of 30 mL/min has stage 3 chronic kidney disease (CKD). Normocytic normochromic anemia is common in patients with CKD due to reduced production of the erythropoietin hormone by the kidneys. Erythropoietin stimulates precursor cells in the bone marrow and helps in production of red blood cells. The patient with CKD may have a high serum potassium level, which can cause fatal dysrhythmias. An increase in extracellular fluid volume may lead to hypertension in patients with CKD. Metabolic acidosis may occur in CKD patients with defective reabsorption and regeneration of bicarbonate. Text Reference - p. 1109

The nurse identifies that which drug should be used with caution in a patient with renal failure? 1 Cinacalcet 2 Paricalcitol 3 Gemfibrozil 4 Vancomycin

4 A patient with renal failure has a reduced ability to eliminate metabolites and drugs. Therefore, drugs like vancomycin, which are mainly excreted by the kidney, need to be used with caution and monitoring for accumulation and potential drug toxicity is necessary. Cinacalcet is a calcimimetic agent that helps to control secondary hyperparathyroidism. Paricalcitol is an active vitamin D supplement that helps to control elevated levels of parathyroid hormone and is used for treating secondary hyperparathyroidism in patients with end stage chronic kidney disease. Gemfibrozil helps to reduce triglyceride levels and increases high-density cholesterol in patients with chronic kidney disease. Text Reference - p. 1114

Which finding indicates nonoliguria? 1 Urinary output of 200 mL/day 2 Urinary output of 300 mL/day 3 Urinary output of 400 mL/day 4 Urinary output of 500 mL/day

4 A urine output greater than 400 mL/day is a sign of nonoliguria. Thus a urine output of 500 mL/day indicates nonoliguria. A urine output of 200 or 300 mL/day indicates oliguria. A urine output of 400 mL per day indicates that the patient is at risk for oliguria. Text Reference - p. 1103

A registered nurse is teaching a trainee nurse about the parameters to be assessed in a patient with acute kidney injury who is undergoing dialysis. Which statement by the trainee nurse indicates a need for further teaching? 1 "I should auscultate patient's lung sounds." 2 "I should record the patient's input and output." 3 "I should assess for any change in the patient's skin color." 4 "I should examine the patient's mouth for a change in color."

4 Acute kidney injury is associated with dry mouth and inflammation and is caused by increased levels of ammonia in the saliva. The nurse should examine the mouth for inflammation and dryness. Therefore, the trainee nurse's statement about examining the mouth for a change in color indicates a need for further teaching. Because of renal impairment, fluid can accumulate in the lungs and result in difficulty breathing. Therefore, the nurse should auscultate the patient's lung sounds. Recording the patient's input and output will help to determine the efficacy of the treatment. Acute kidney injury is also associated with hyperpigmentation; thus the nurse should assess for changes in the patient's skin color. Test-Taking Tip: Acute kidney injury is associated with increased amounts of nitrogenous waste in blood and secretions. Use this tip to answer the above question. Text Reference - p. 1106

The nurse performs an admission assessment of a patient with acute renal failure. For which common complication does the nurse assess the patient? 1 Polyphagia 2 Hypernatremia 3 Hypotensive shock 4 Cardiac dysrhythmias

4 Because the kidneys are not effectively removing waste products, including electrolytes, an increased potassium level (hyperkalemia) of more than 5.0 mEq/L is common in acute renal failure and places the patient at risk for cardiac arrhythmias. Patients usually experience anorexia, not an increase in hunger. Acute renal failure will likely manifest as hyponatremia. Hypotensive shock may be the result of a severe cardiac arrhythmia that is not treated. Text Reference - p. 1105

The registered nurse is teaching a trainee nurse about the use of renal replacement therapy (RRT). Which statement by the trainee nurse indicates effective learning? 1 "RRT is performed in patients with hypokalemia." 2 "RRT is advised for patients with metabolic acidosis." 3 "RRT is recommended in patients with hypovolemia." 4 "RRT is recommended if there is a pericardial effusion."

4 Pericardial effusion is an abnormal accumulation of fluid inside the pericardial cavity; this condition is caused by increased blood volume. Renal replacement therapy (RRT) is recommended because the kidneys are unable to function properly. RRT is recommended in hyperkalemic, not hypokalemic, conditions. RRT does not alleviate the effects of metabolic acidosis. RRT is advised in patients with fluid overload, not hypovolemia. Test-Taking Tip: Renal replacement therapy is used in patients with impaired renal function, because the fluid volume increases in the body. Use this tip to answer the question. Text Reference - p. 1106

Which substance can pass through the peritoneal membrane? 1 Glucose 2 Creatinine 3 Fatty acids 4 Amino acids

4 Peritoneal membranes allow the passage of amino acids, polypeptides, and plasma proteins. Glucose, creatinine, and fatty acids cannot permeate the peritoneal membrane. Text Reference - p. 1119

The nurse is caring for a patient undergoing peritoneal dialysis. What finding should the nurse report to the primary health care provider that would indicate peritonitis? 1 Oliguria 2 Hyperkalemia 3 Hyponatremia 4 Abdominal pain

4 Peritonitis is caused by either a Staphylococcus aureus or a Staphylococcus epidermidis infection. It is manifested by abdominal pain, cloudy peritoneal effluent, and increased white blood cell count. Oliguria, hyperkalemia, and hyponatremia are complications associated with acute kidney injury. Text Reference - p. 1119

Which condition is a result of severe metabolic acidosis in patients with acute kidney injury? 1 Asterixis 2 Proteinuria 3 Hydronephrosis 4 Kussmaul respirations

4 Severe acidosis causes a patient to take deep and rapid breaths—called Kussmaul respirations—in an effort to increase the exhalation of carbon dioxide. Asterixis is a neurologic change associated with acute kidney injury due to the accumulation of metabolic waste in the brain and nervous system. Dysfunction of the glomerular membrane due to acute kidney injury leads to proteinuria. Hydronephrosis refers to dilation of the kidneys and is a postrenal cause of acute kidney injury. Text Reference - p. 1104

What is a clinical manifestation of nephrogenic systemic fibrosis? 1 Pruritus 2 Urticaria 3 Scaling of skin 4 Hyperpigmentation

4 Signs of nephrogenic systemic fibrosis include hyperpigmentation of the skin, induration, and joint contractures. Pruritus, urticaria, and scaling of the skin are not clinical signs of nephrogenic systemic fibrosis. Text Reference - p. 1103

The nursing instructor is teaching a student nurse about the therapies for hyperkalemia associated with acute kidney injury. Which statement by the student nurse indicates effective learning? 1 "Insulin infusion is a permanent therapy." 2 "Sodium bicarbonate is a permanent therapy." 3 "Calcium gluconate infusion is a permanent therapy." 4 "Sodium polystyrene sulfonate is a permanent therapy."

4 Sodium polystyrene sulfonate is a cation-exchange resin that completely removes extra potassium; it is considered a permanent therapy. Insulin pushes potassium inside the cells, but with a decline in insulin levels, potassium exits the cell. Thus, insulin is a temporary therapy. Sodium bicarbonate and calcium gluconate are also considered temporary therapies because they shift potassium into the cells until their blood levels diminish, upon which potassium exits the cells. Text Reference - p. 1105

A patient with chronic kidney disease has an arteriovenous (AV) graft in the right forearm. What is the nurse's priority in determining the patency of the graft? 1 Determine the range of motion of the right arm and shoulder 2 Observe for clubbing of the fingers on the right hand of the AV graft site 3 Compare radial pulses by checking the right and left pulses simultaneously 4 Check for a bruit by listening over the right arm AV graft site with a stethoscope

4 The arteriovenous (AV) graft is an artificial connection between an artery and vein to provide access for hemodialysis. Thrombosis may occur; therefore the need to determine patency is an essential assessment. Palpation of the site should indicate a thrill, which also indicates that the graft is patent. Listening over the AV graft should reveal a bruit sound, indicating patency. A bruit sounds similar to the impulse beat heard when measuring blood pressure. The arm that has the AV graft site should not be put through range-of-motion movements or exercises. Clubbing is not a complication observed in the fingers of a patient with an AV graft. Comparing the left radial pulse with the pulse on the AV graft site is not an accurate patency assessment procedure. Text Reference - p. 1120

Which is a manifestation of a mild form of acute kidney injury? 1 Increased urine output 2 Increased nitrogen level 3 Increased potassium level 4 Increased serum creatinine level

4 The mildest form of acute kidney injury is characterized by increased serum creatinine levels. Kidney injury is associated with decreased urine output, not increased urine output. Increased levels of potassium and nitrogen are characteristics of a severe form of acute kidney injury. Text Reference - p. 1102

A patient is being administered 15 g sodium polystyrene sulfonate (Kayexalate) orally for hyperkalemia. Which intervention should the nurse perform? 1 Observe the patient for iron overload. 2 Inform the patient that constipation is an expected side effect. 3 Provide magnesium-containing antacids. 4 Report peaked T waves in electrocardiogram (ECG)

4 The nurse should report changes to the health care provider in the ECG, such as peaked T waves and widened QRS complexes; dialysis may be required to remove excess potassium. Monitoring for iron overload is a consideration for blood transfusions, but not for administration of sodium polystyrene sulfonate. The nurse should warn the patient that this treatment will often cause diarrhea because the preparation contains sorbitol, a sugar alcohol that has an osmotic laxative action. Magnesium-containing antacids should not be prescribed for patients with chronic kidney disease because magnesium is excreted by the kidneys. Text Reference - p. 1112

The nurse recognizes that which medication is the most appropriate for a patient with chronic kidney disease (CKD) who has a glycosylated hemoglobin of 5%, blood pressure of 140/95 mm Hg, and whose urinalysis reveals the presence of protein? 1 A diuretic 2 A calcimimetic agent 3 A calcium channel blocker 4 An angiotensin receptor blocker

4 A patient with glycosylated hemoglobin of 5%, blood pressure of 140/95 mm Hg, and protein in the urine has hypertension with nondiabetic proteinuria. The patient can take angiotensin-receptor blockers and angiotensin-converting-enzyme (ACE) inhibitors. These medications help to decrease proteinuria and delay the progression of chronic kidney disease (CKD). Diuretics help control elevated blood pressure in patients with CKD but do not have an effect on proteinuria. Calcimimetic agents help to control secondary hyperparathyroidism by increasing the sensitivity of the calcium receptors in the parathyroid glands. They are not used for treatment of hypertension, diabetes, or proteinuria. Calcium channel blockers also do not have an effect on proteinuria. Text Reference - p. 1113

Which laboratory finding in a patient with multiple organ dysfunction syndrome (MODS) suggests prerenal manifestations of renal dysfunction? 1 Urine Na+ is 22 mEq/L. 2 Urine specific gravity is 1.010. 3 Urine osmolality is decreased. 4 Urine specific gravity is increased.

4 An increase in urine specific gravity suggests prerenal manifestations in patients with MODS. This occurs due to renal hypoperfusion. A urine Na+ level above 20 mEq/L is a sign of intrarenal manifestations in MODS. It occurs due to acute tubular necrosis. Other intrarenal manifestations of MODS are urine specific gravity of around 1.010 and a decrease of the urine osmolality.

What causes gut bacteria to move into circulation in patients with systemic inflammatory response syndrome (SIRS)? 1 Toxic effects of medicines 2 Effect of inflammatory mediators 3 Decreased gastrointestinal motility 4 Decreased perfusion of gut mucosa

4 Decreased perfusion in the gastrointestinal (GI) tract leads to a breakdown of the normally protective mucosal barrier, which causes the bacterial movement from the GI tract into circulation. In order to control these bacteria, antibiotics are administered. Antibiotics are nephrotoxic medicines that can cause acute kidney injury. The breakdown of the mucosal barrier is the direct effect of hypoperfusion rather than the inflammatory mediators. In critical illnesses, GI motility is often decreased causing abdominal distension and paralytic ileus. Test-Taking Tip: Make certain that the answer you select is reasonable and obtainable under ordinary circumstances and that the action can be carried out in the given situation. Text Reference - p. 1649

A patient is diagnosed with multiple organ dysfunction syndrome. While aggressive treatment is continued, the nurse suspects infection. What is the most appropriate action that the nurse should perform? 1 Discontinue the aggressive treatment. 2 Reduce oxygen delivery to the patient. 3 Wait for laboratory reports to confirm the suspicion. 4 Obtain a prescription for broad-spectrum antibiotic therapy.

4 If an infection is suspected, broad-spectrum antibiotics should be started immediately to limit the infection. Aggressive treatment for infection control should be carried out in parallel. These patients are usually hypoxemic. Therefore, oxygen should be administered strictly as prescribed. Cultures can be sent, and based on the reports, specific antibiotics can be added. Test-Taking Tip: Identify option components as correct or incorrect. This may help you identify a wrong answer. Example: If you are being asked to identify a diet that is specific to a certain condition, your knowledge about that condition would help you choose the correct response (e.g., cholecystectomy = low-fat, high-protein, low-calorie diet). Text Reference - p. 1650

What type of medication does the nurse anticipate being prescribed by the health care provider to manage confusion, disorientation, and delirium in a patient with systemic inflammatory response syndrome (SIRS) and multiple organ dysfunction syndrome (MODS)? 1 Vasopressors 2 Loop diuretics 3 Proton pump inhibitors 4 Calcium channel blockers

4 Impaired perfusion of the brain may cause confusion, disorientation, and delirium in the patient. The health care provider is likely to prescribe calcium channel blockers to a patient exhibiting confusion, disorientation, and delirium to reduce cerebral vasospasm and improve perfusion of the brain. Vasopressors may be prescribed to combat cardiovascular dysfunction. Loop diuretics are prescribed if there is renal dysfunction. Proton pump inhibitors are prescribed to manage gastrointestinal symptoms. Text Reference - p. 1651

Following coronary artery bypass graft surgery a patient has postoperative bleeding that requires returning to surgery to repair the leak. During surgery, the patient has a myocardial infarction (MI). After restoring the patient's body temperature to normal, which patient assessment is the most important for planning nursing care? 1 Cardiac index (CI) 5 L/min/m2 2 Central venous pressure (CVP) 8 mm Hg 3 Mean arterial pressure (MAP) 86 mm Hg 4 Pulmonary artery pressure (PAP) 28/14 mm Hg

4 Pulmonary hypertension as indicated by an elevated PAP indicates impaired forward flow of blood because of left ventricular dysfunction or hypoxemia. Both can be caused by the MI. The CI, CVP, and MAP readings are normal. Text Reference - p. 1639

What is the clinical manifestation of systemic inflammatory response syndrome (SIRS) and multiple organ dysfunction syndrome (MODS) on the respiratory system? 1 Pulmonary edema 2 Pulmonary fibrosis 3 Pulmonary embolism 4 Pulmonary hypertension

4 Systemic inflammatory response syndrome (SIRS) and multiple organ dysfunction syndrome (MODS) affect the respiratory system and lead to pulmonary hypertension. Pulmonary edema is caused by fluid accumulation in the air spaces. Pulmonary fibrosis is a respiratory disorder caused by scars in the lung tissues or inhalation of airborne toxins. A pulmonary embolism is caused by blood clots or blockage in the pulmonary arteries. Text Reference - p. 1651

A massive gastrointestinal bleed has resulted in hypovolemic shock in an older patient. What is a priority nursing diagnosis? 1 Acute pain 2 Impaired tissue integrity 3 Decreased cardiac output 4 Ineffective tissue perfusion

4 The many deleterious effects of shock are all related to inadequate perfusion and oxygenation of every body system. Ineffective tissue perfusion supersedes acute pain, impaired tissue integrity, and decreased cardiac output, because circulation is a priority. Acute pain may occur, but is not a priority at this time. Tissue integrity is not impaired. Text Reference - p. 1633

What is the goal in the care of a systemic inflammatory response syndrome (SIRS) patient whose bilirubin level is 3 mg/dL? 1 Patient will be free of stress ulcers. 2 Patient will not feel abdominal distension. 3 Patient will maintain intraabdominal pressures. 4 Patient will maintain adequate tissue perfusion.

4 The nurse will plan to maintain adequate tissue perfusion for a systemic inflammatory response syndrome (SIRS) patient with bilirubin level of 3 mg/dL. An elevation of the bilirubin indicates impaired liver function. Stress ulcer prophylaxis is routine and would have been initiated before SIRS occurred. Abdominal distention and increased intraabdominal pressures are associated with impaired liver function. Monitoring these conditions is essential but is of a lower priority than maintaining tissue perfusion. Text Reference - p. 1651

40. A 65-year-old patient has undergone a right total hip arthroplasty with a cemented prosthesis for treatment of severe osteoarthritis of the hip. What is included in the activity the nurse anticipates for the patient on the patient's first or second postoperative day? a. Transfer from the bed to the chair twice a day only b. Turning from the back to the unaffected side q2hr only c. Crutch walking with non-weight bearing on the operative leg d. Ambulation and weight bearing on the right leg with a walker

40. d. Physical therapy is initiated on the first postoperative day with ambulation and weight bearing using a walker for a patient with a cemented prosthesis and non-weight bearing on the operative side for an uncemented prosthesis. In addition, the patient sits in the chair at least twice a day and is turned to both sides and back with the operative leg supported.

Priority Decision: A patient with severe ulnar deviation of the hands undergoes an arthroplasty with reconstruction and replacement of finger joints. Postoperatively, what is it most important for the nurse to do? a. Position the fingers lower than the elbow. b. Perform neurovascular assessments of the fingers q2-4hr. c. Encourage the patient to gently flex, extend, abduct, and adduct the fingers q4hr. d. Remind the patient that function of the hands is more important than their cosmetic appearance.

43. b. Neurovascular checks of the fingers following surgery of the hands are essential to detect compromised vascular and neurologic function caused by trauma or edema. Postoperatively, the hands are elevated with a bulky dressing in place and when the dressing is removed, a guided splinting program is started. Exercises are performed three to four times a day when the splints are removed and the patient is discharged. Before surgery, it must be made clear to the patient that the goal of the surgery is to restore function related to grasp, pinch, stability, and strength and the hands will not necessarily have good cosmetic appearance.

44. Priority Decision: Following change-of-shift handoff, which patient should the nurse assess first? a. A 58-year-old male experiencing phantom pain and requesting analgesic b. A 72-year-old male being transferred to a skilled nursing unit following repair of a hip fracture c. A 25-year-old female in left leg skeletal traction asking for the weights to be lifted for a few minutes d. A 68-year-old male with a new lower leg cast complaining that the cast is too tight and he cannot feel his toes

44. d. The patient with a tight cast may be at risk for neurovascular compromise (impaired circulation and peripheral nerve damage) and should be assessed first. The other patients should be seen as soon as possible. Providing analgesia for the patient with phantom pain would be the next priority. The patient in skeletal traction needs explanation of the purpose and functioning of the traction. She may need analgesia or muscle relaxants to help tolerate the traction.

The patient works on a computer 8 hours each day. What kind of repetitive strain injury would be expected in this patient? a. Meniscus injury b. Rotator cuff injury c. Radial-ulnar fracture d. Carpal tunnel syndrome

5. d. Carpal tunnel syndrome would be expected related to the continuous wrist movements. Injuries of the menisci, which are fibrocartilage in the knee, are common with athletes. Radial-ulnar fractures are seen with great force such as a car accident or a fall. Rotator cuff injuries occur with sudden adduction forces applied to the cuff while the arm is held in abduction. They are commonly seen with repetitive overhead motions.

9. The patient had a fracture. At 3 weeks to 6 months there is clinical union, and this is the first stage of healing that is sufficient to prevent movement of the fracture site when the bones are gently stressed. How is this stage of fracture healing documented? a. Ossification b. Remodeling c. Consolidation d. Callus formation

9. a. Ossification is the stage of fracture healing when there is clinical union and enough strength to prevent movement at the fracture site. Remodeling is the normal function of the bone. Consolidation is when the distance between bone fragments eventually closes and radiologic union first occurs. The callus formation stage appears by the end of the second week of injury when minerals and new bone matrix are deposited in the osteoid that is produced in the granulation tissue stage.

A patient is admitted to the hospital with chronic kidney disease. The nurse understands that this condition is characterized by: A. progressive irreversible destruction of the kidneys B. a rapid decrease in urinary output and an elevated BUN C. an increasing creatinine clearance with a decrease in urinary output D. prostration, somnolence and confusion with coma and imminent death

A

Kidney transplant recipient complains of having fever, chills, and dysuria over the course of the past 2 days, What is the first action the nurse should take? A. assess temperature and initiate workup to rule out infection B. provied warm cover for the patient and give 1 g acetaminophen orally C. reassure the patient that this is common after transplantation D. notify the nephrologist that the patient has developed symptoms of acute rejection

A

The nurse is caring for a 63-year-old with a possible pituitary tumor who is scheduled for a computed tomography (CT) scan with contrast. Which information about the patient is most important to discuss with the health care provider before the test? a. History of renal insufficiency b. Complains of chronic headache c. Recent bilateral visual field loss d. Blood glucose level of 134 mg/dL

A

Which individuals would the nurse identify as having the highest risk for coronary artery disease (CAD)? A 45-yr-old depressed man with a high-stress job A 60-yr-old man with below normal homocysteine levels A 54-yr-old woman vegetarian with elevated high-density lipoprotein (HDL) levels A 62-yr-old woman who has a sedentary lifestyle and body mass index (BMI) of 23 kg/m2

A 45-yr-old depressed man with a high-stress job The 45-yr-old depressed man with a high-stress job is at the highest risk for CAD. Studies demonstrate that depression and stressful states can contribute to the development of CAD. Elevated HDL levels and low homocysteine levels actually help to prevent CAD. Although a sedentary lifestyle is a risk factor, a BMI of 23 kg/m2 depicts normal weight, and thus the patient with two risk factors is at greatest risk for developing CAD.

Which patient should be taught preventive measures for CKD by the nurse because this patient is most likely to develop CKD? A.A 50-year-old white female with hypertension B.A 61-year-old Native American male with diabetes C.A 40-year-old Hispanic female with cardiovascular disease D.A 28-year-old African American female with a urinary tract infection

A 61-year-old Native American male with diabetes It is especially important for the nurse to teach CKD prevention to the 61-year-old Native American with diabetes. This patient is at highest risk because diabetes causes about 50% of CKD. This patient is the oldest, and Native Americans with diabetes develop CKD 6 times more frequently than other ethnic groups. Hypertension causes about 25% of CKD. Hispanics have CKD about 1.5 times more than non-Hispanics. African Americans have the highest rate of CKD because hypertension is significantly increased in African Americans. A UTI will not cause CKD unless it is not treated or UTIs occur recurrently.

The nurse is caring for a group of patients. Which patient is at highest risk for pancreatic cancer? A 38-year-old Hispanic female who is obese and has hyperinsulinemia A 23-year-old who has cystic fibrosis-related pancreatic enzyme insufficiency A 72-year-old African American male who has smoked cigarettes for 50 years A 19-year-old who has a 5-year history of uncontrolled type 1 diabetes mellitus

A 72-year-old African American male who has smoked cigarettes for 50 years Risk factors for pancreatic cancer include chronic pancreatitis, diabetes mellitus, age, cigarette smoking, family history of pancreatic cancer, high-fat diet, and exposure to chemicals such as benzidine. African Americans have a higher incidence of pancreatic cancer than whites. The most firmly established environmental risk factor is cigarette smoking. Smokers are two or three times more likely to develop pancreatic cancer as compared with nonsmokers. The risk is related to duration and number of cigarettes smoked.

The nurse is assisting in the care of several patients in the critical care unit. Which patient is most at risk for developing multiple organ dysfunction syndrome (MODS)? A 22-yr-old patient with systemic lupus erythematosus admitted with a pelvic fracture A 48-yr-old patient with lung cancer admitted for syndrome of inappropriate antidiuretic hormone and hyponatremia A 65-yr-old patient with coronary artery disease, dyslipidemia, and primary hypertension admitted for unstable angina A 82-yr-old patient with type 2 diabetes mellitus and chronic kidney disease admitted for peritonitis related to a peritoneal dialysis catheter infection

A 82-yr-old patient with type 2 diabetes mellitus and chronic kidney disease admitted for peritonitis related to a peritoneal dialysis catheter infection A patient with peritonitis is at high risk for developing sepsis. In addition, a patient with diabetes is at high risk for infections and impaired healing. Sepsis and septic shock are the most common causes of MODS. Individuals at greatest risk for developing MODS are older adults and persons with significant tissue injury or preexisting disease. MODS can be initiated by any severe injury or disease process that activates a massive systemic inflammatory response.

Which statements will the nurse include when teaching a patient who is scheduled for oral glucose tolerance testing in the outpatient clinic (select all that apply)? a. "You will need to avoid smoking before the test." b. "Exercise should be avoided until the testing is complete." c. "Several blood samples will be obtained during the testing." d. "You should follow a low-calorie diet the day before the test." e. "The test requires that you fast for at least 8 hours before testing."

A C E

Which types of shock may cause reduced urinary output in a patient? Select all that apply. A. Septic shock B. Hypovolemic shock C. Neurogenic shock D. Anaphylactic shock E. Cardiogenic shock.

A, B, E Decreased urine output is a clinical manifestation of septic, hypovolemic, obstructive and cardiogenic shock. Neurogenic shock is associated with bladder dysfunction. Anaphylactic shock is associated with urinary incontinence. Test-Taking Tip: Make certain that the answer you select is reasonable and obtainable under ordinary circumstances and that the action can be carried out in the given situation. Text Reference - p. 1635

Which of the following characterize acute kidney injury (select all that apply): A. primary cause of death is infection B. almost always affects older people C. disease course is potentially reversible D. most common cause is diabetic neuropathy E. cardiovascular disease is the most common cause of death

A, C

When caring for a patient with liver disease, the nurse recognizes the need to prevent bleeding resulting from altered clotting factors and rupture of varices. Which of the following nursing interventions would be appropriate to achieve this outcome (select all that apply)? A. Use smallest gauge possible when giving injections or drawing blood. B. Teach patient to avoid straining at stool, vigorous blowing of nose, and coughing. C. Advise patient to use soft-bristle toothbrush and avoid ingestion of irritating food. D. Apply gentle pressure for the shortest possible time period after performing venipuncture. E. Instruct patient to avoid aspirin and NSAIDs to prevent hemorrhage when varices are present.

A,B,C,E) Using the smallest gauge needle for injections will minimize the risk of bleeding into the tissues. Avoiding straining, nose blowing, and coughing will reduce the risk of hemorrhage at these sites. The use of a soft-bristle toothbrush and avoidance of irritating food will reduce injury to highly vascular mucous membranes. The nurse should apply gentle but prolonged pressure to venipuncture sites to minimize the risk of bleeding. Aspirin and NSAIDs should not be used in patients with liver disease because they interfere with platelet aggregation, thus increasing the risk for bleeding.

6. What nursing intervention should be implemented in the care of a patient who is experiencing increased ICP? A. Monitor fluid and electrolyte status carefully. B. Position the patient in a high Fowler's position. C. Administer vasoconstrictors to maintain cerebral perfusion. D. Maintain physical restraints to prevent episodes of agitation.

A. Fluid and electrolyte disturbances can have an adverse effect on ICP and must be monitored vigilantly. The head of the patient's bed should be kept at 30 degrees in most circumstances, and physical restraints are not applied unless absolutely necessary. Vasoconstrictors are not typically administered in the treatment of ICP.

7. Magnetic resonance imaging (MRI) has revealed the presence of a brain tumor in a patient. The nurse should recognize that the patient will most likely need which treatment modality? A. Surgery B. Chemotherapy C. Radiation therapy D. Biologic drug therapy

A. Surgical removal is the preferred treatment for brain tumors. Chemotherapy and biologic drug therapy are limited by the blood-brain barrier, tumor cell heterogeneity, and tumor cell drug resistance. Radiation therapy may be used as a follow-up measure after surgery.

2. A patient with a suspected traumatic brain injury has bloody nasal drainage. What observation should cause the nurse to suspect that this patient has a cerebrospinal fluid (CSF) leak? A. A halo sign on the nasal drip pad B. Decreased blood pressure and urinary output C. A positive reading for glucose on a Test-tape strip D. Clear nasal drainage along with the bloody discharge

A. When drainage containing both CSF and blood is allowed to drip onto a white pad, within a few minutes the blood will coalesce into the center, and a yellowish ring of CSF will encircle the blood, giving a halo effect. The presence of glucose would be unreliable for determining the presence of CSF because blood also contains glucose. Decreased blood pressure and urinary output would not be indicative of a CSF leak.

The nurse admits a 55-year-old female with multiple sclerosis to a long-term care facility. Which finding is of most immediate concern to the nurse? A. Ataxic gait B. Radicular pain C. Severe fatigue D. Urinary retention

A. Ataxic gait An ataxic gait is a staggering, uncoordinated gait. Fall risk is the highest in individuals with gait instability or visual or cognitive impairments. The other signs and symptoms (e.g., fatigue, urinary retention, radicular pain) may also occur in the patient with multiple sclerosis and need to be managed, but are not the priority.

A patient has a nursing diagnosis of risk for ineffective cerebral tissue perfusion related to cerebral edema. An appropriate nursing intervention for the patient is a. avoiding positioning the patient with neck and hip flexion b. maintaining hyperventilation to a PaCO2 of 15 to 20 mm Hg c. clustering nursing activities to provide periods of uninterrupted rest d. routine suctioning to prevent accumulation of respiratory secretions

A. Avoiding positioning the patient with neck and hip flexion- Nursing care activities that increase ICP include hip and neck flexion, suctioning, clustering care activities, and noxious stimuli; they should be avoided or performed as little as possible in the patient with increased ICP. Lowering the PaCO2 below 20 mm Hg can cause ischemia and worsening of ICP; the PaCO2 should be maintained at 30 to 35 mm Hg.

Before beginning a transfusion of packed red blood cells (PRBCs), which action by the nurse would be of highest priority to avoid an error during this procedure? A. Check the identifying information on the unit of blood against the patient's ID bracelet. B. Select new primary IV tubing primed with lactated Ringer's solution to use for the transfusion. C. Remain with the patient for 60 minutes after beginning the transfusion to watch for signs of a transfusion reaction. D. Add the blood transfusion as a secondary line to the existing IV and use the IV controller to maintain correct flow.

A. Check the identifying information on the unit of blood against the patient's ID bracelet. The patient's identifying information (name, date of birth, medical record number) on the ID bracelet should exactly match the information on the blood bank tag that has been placed on the unit of blood. If any information does not match, the transfusions should not be hung because of possible error and risk to the patient. The transfusion is hung on blood transfusion tubing, not a secondary line, and cannot be hung with lactated Ringer's solution because it will cause RBC hemolysis. Usually, the patient will need continuous monitoring for 15 minutes after the transfusion is started, as this is the time most transfusion reactions occur. Then the patient should be monitored every 30 to 60 minutes during the administration.

The patient had myocarditis and is now experiencing fatigue, weakness, palpitations, and dyspnea at rest. The nurse assesses pulmonary crackles, edema, and weak peripheral pulses. Sinoatrial tachycardia is evident on the cardiac monitor. The Doppler echocardiography shows dilated cardiomyopathy. What collaborative and nursing care of this patient should be done to improve cardiac output and the quality of life? (Select all that apply.) A. Decrease preload and afterload. B. Relieve left ventricular outflow obstruction. C. Control heart failure by enhancing myocardial contractility. D. Improve diastolic filling and the underlying disease process. E. Improve ventricular filling by reducing ventricular contractility

A. Decrease preload and afterload. C. Control heart failure by enhancing myocardial contractility. The patient is experiencing dilated cardiomyopathy. To improve cardiac output and quality of life, drug, nutrition, and cardiac rehabilitation will be focused on controlling heart failure by decreasing preload and afterload and improving cardiac output, which will improve the quality of life. Relief of left ventricular outflow obstruction and improving ventricular filling by reducing ventricular contractility is done for hypertrophic cardiomyopathy. There are no specific treatments for restrictive cardiomyopathy, but interventions are aimed at improving diastolic filling and the underlying disease process

For the patient undergoing a craniotomy, the nurse provides information about the use of wigs and hairpieces or other methods to disguise hair loss a. during pre operative teaching b. in the patient asks about their use c. in the immediate postoperative period d. when the patient expresses negative feelings about his or her appearance

A. During pre operative teaching- The prevent undue concern and anxiety about hair loss and postoperative self-esteem disturbances, a patient undergoing cranial surgery should be informed pre operatively that the head is usually shaved in surgery while the patient is anesthetized and that methods can be used after the dressings are removed postoperatively to disguise the hair loss. In the immediate postoperative period, the patient is very ill, and the focus is on maintaining neurologic function, bur preoperatively the nurse should anticipate the patient's postoperative need for self-esteem and maintenance of appearance.

A patient will receive a hematopoietic stem cell transplant (HSCT). What is the nurse's priority after the patient receives combination chemotherapy before the transplant? A. Prevent patient infection. B. Avoid abnormal bleeding. C. Give pneumococcal vaccine. D. Provide companionship while isolated.

A. Prevent patient infection. After combination chemotherapy for HSCT, the patient's bone marrow is destroyed in preparation to receive the bone marrow graft. Thus the patient is immunosuppressed and is at risk for a life-threatening infection. The priority is preventing infection. Bleeding is not usually a problem. Giving the pneumococcal vaccine at this time should not be done; it should have been done previously. Providing companionship is not the primary role of the nurse, although the patient will need support during the time of isolation.

The nurse is teaching a community group about preventing rheumatic fever. What information should the nurse include? A. Prompt recognition and treatment of streptococcal pharyngitis B. Completion of 4 to 6 days of antibiotic therapy for infective endocarditis of respiratory infections in children born with heart defects C. Avoidance of respiratory infections in children who have rheumatoid arthritis D. Requesting antibiotics before dental surgery for individuals with rheumatoid arthritis

A. Prompt recognition and treatment of streptococcal pharyngitis The nurse should emphasize the need for prompt and adequate treatment of streptococcal pharyngitis infection, which can lead to the complication of rheumatic fever.

The patient is admitted with hypercalcemia; polyuria; and pain in the pelvis, spine, and ribs with movement. Which hematologic problem is likely to display these manifestations in the patient? A.Multiple myeloma B. Thrombocytopenia C. Megaloblastic anemia D. Myelodysplastic syndrome

A.Multiple myeloma Multiple myeloma typically manifests with skeletal pain and osteoporosis that may cause hypercalcemia, which can result in polyuria, confusion, or cardiac problems. Serum hyperviscosity syndrome can cause renal, cerebral, or pulmonary damage. Thrombocytopenia, megaloblastic anemia, and myelodysplastic syndrome are not characterized by these manifestations.

COMPLETION 1. The nurse obtains a blood pressure of 180/75 mm Hg for a patient. What is the patient's mean arterial pressure (MAP)? ____________________

ANS: 110 MAP = (SBP + 2 DBP)/3

The health care provider prescribes these actions for a patient who has possible septic shock with a BP of 70/42 mm Hg and oxygen saturation of 90%. In which order will the nurse implement the actions? Put a comma and space between each answer choice (a, b, c, d, etc.) ____________________ a. Obtain blood and urine cultures. b. Give vancomycin (Vancocin) 1 g IV. c. Infuse vasopressin (Pitressin) 0.01 units/min. d. Administer normal saline 1000 mL over 30 minutes. e. Titrate oxygen administration to keep O2 saturation >95%.

ANS: E, D, C, A, B The initial action for this hypotensive and hypoxemic patient should be to improve the oxygen saturation, followed by infusion of IV fluids and vasopressors to improve perfusion. Cultures should be obtained before administration of antibiotics.

A patient is to receive an infusion of 250 mL of platelets over 2 hours through tubing that is labeled: 1 mL equals 10 drops. How many drops per minute will the nurse infuse?

ANS: 21 To infuse 250 mL over 2 hours, the calculated drip rate is 20.8 drops/minute or 21 drops/minute

SHORT ANSWER The nurse estimates the extent of a burn using the rule of nines for a patient who has been admitted with deep partial-thickness burns of the anterior trunk and the entire left arm. What percentage of the patient's total body surface area (TBSA) has been injured?

ANS: 27% When using the rule of nines, the anterior trunk is considered to cover 18% of the patient's body and each arm is 9%.

1. When analyzing an electrocardiographic (ECG) rhythm strip of a patient with a regular heart rhythm, the nurse counts 30 small blocks from one R wave to the next. The nurse calculates the patient's heart rate as ____.

ANS: 50 There are 1500 small blocks in a minute, and the nurse will divide 1500 by 30. DIF: Cognitive Level: Remember (knowledge) REF: 789-790 TOP: Nursing Process: Assessment MSC: NCLEX: Physiological Integrity

SHORT ANSWER An 80-kg patient with burns over 30% of total body surface area (TBSA) is admitted to the burn unit. Using the Parkland formula of 4 mL/kg/%TBSA, *what is the IV infusion rate (mL/hour)* for lactated Ringer's solution that the nurse will administer during the first 8 hours?

ANS: 600 mL The Parkland formula states that patients should receive 4 mL/kg/%TBSA burned during the first 24 hours. Half of the total volume is given in the first 8 hours and then the last half is given over 16 hours: 4 × 80 × 30 = 9600 mL total volume; 9600/2 = 4800 mL in the first 8 hours; 4800 mL/8 hr = 600 mL/hr.

An unconscious patient with a traumatic head injury has a blood pressure of 126/72 mm Hg, and an intracranial pressure of 18 mm Hg. The nurse will calculate the cerebral perfusion pressure as ____________________.

ANS: 72 mm Hg The formula for calculation of cerebral perfusion pressure is [(Systolic pressure + Diastolic blood pressure 2)/3] = intracranial pressure. DIF: Cognitive Level: Application REF: 1452-1453

An unconscious patient with a traumatic head injury has a blood pressure of 130/76 mm Hg, and an intracranial pressure (ICP) of 20 mm Hg. The nurse will calculate the cerebral perfusion pressure (CPP) as ____ mm Hg.

ANS: 74 Calculate the CPP: (CPP = mean arterial pressure [MAP] - ICP). MAP = DBP + 1/3 (systolic blood pressure [SBP] - diastolic blood pressure [DBP]). The MAP is 94. The CPP is 74. DIF: Cognitive Level: Apply (application) TOP: Nursing Process: Assessment MSC: NCLEX: Physiological Integrity

1. When preparing to defibrillate a patient. In which order will the nurse perform the following steps? (Put a comma and a space between each answer choice [A, B, C, D, E].) a. Turn the defibrillator on. b. Deliver the electrical charge. c. Select the appropriate energy level. d. Place the paddles on the patient's chest. e. Check the location of other staff and call out "all clear."

ANS: A, C, D, E, B This order will result in rapid defibrillation without endangering hospital staff. DIF: Cognitive Level: Analyze (analysis) REF: 802 TOP: Nursing Process: Implementation MSC: NCLEX: Physiological Integrity

The following four patients arrive in the emergency department (ED) after a motor vehicle collision. In which order should the nurse assess them? (Put a comma and a space between each answer choice [A, B, C, D].) a. A 74-year-old with palpitations and chest pain b. A 43-year-old complaining of 7/10 abdominal pain c. A 21-year-old with multiple fractures of the face and jaw d. A 37-year-old with a misaligned left leg with intact pulses

ANS: C, A, B, D The highest priority is to assess the 21-year-old patient for airway obstruction, which is the most life-threatening injury. The 74-year-old patient may have chest pain from cardiac ischemia and should be assessed and have diagnostic testing for this pain. The 43-year-old patient may have abdominal trauma or bleeding and should be seen next to assess circulatory status. The 37-year-old appears to have a possible fracture of the left leg and should be seen soon, but this patient has the least life-threatening injury. DIF: Cognitive Level: Analyze (analysis) REF: 1676 OBJ: Special Questions: Prioritization; Multiple Patients TOP: Nursing Process: Assessment MSC: NCLEX: Physiological Integrity

In which order will the nurse take these actions when doing a dressing change for a partial-thickness burn wound on a patient's chest? (Put a comma and a space between each answer choice [A, B, C, D, E].) a. Apply sterile gauze dressing. b. Document wound appearance. c. Apply silver sulfadiazine cream. d. Administer IV fentanyl (Sublimaze). e. Clean wound with saline-soaked gauze.

ANS: D, E, C, A, B Because partial-thickness burns are very painful, the nurse's first action should be to administer pain medications. The wound will then be cleaned, antibacterial cream applied, and covered with a new sterile dressing. The last action should be to document the appearance of the wound.

12. A 32-year-old patient who has had an open reduction and internal fixation (ORIF) of left lower leg fractures continues to complain of severe pain in the leg 15 minutes after receiving the prescribed IV morphine. Pulses are faintly palpable and the foot is cool. Which action should the nurse take next? a. Notify the health care provider. b. Assess the incision for redness. c. Reposition the left leg on pillows. d. Check the patient's blood pressure.

ANS: A The patient's clinical manifestations suggest compartment syndrome and delay in diagnosis and treatment may lead to severe functional impairment. The data do not suggest problems with blood pressure or infection. Elevation of the leg will decrease arterial flow and further reduce perfusion.

To auscultate for S3 or S4 gallops in the mitral area, the nurse listens with the a. bell of the stethoscope with the patient in the left lateral position. b. diaphragm of the stethoscope with the patient in a supine position. c. bell of the stethoscope with the patient sitting and leaning forward. d. diaphragm of the stethoscope with the patient lying flat on the left side.

ANS: A *Gallop* rhythms generate low-pitched sounds and are most easily heard with the *bell* of the stethoscope. Sounds associated with the mitral valve are accentuated by turning the patient to the left side, which brings the heart closer to the chest wall. The diaphragm of the stethoscope is best to use for the higher-pitched sounds such as S1 and S2

A patient arrives in the emergency department with facial and chest burns caused by a house fire. Which action should the nurse take first? a. Auscultate the patient's lung sounds. b. Determine the extent and depth of the burns. c. Infuse the ordered lactated Ringer's solution. d. Administer the ordered hydromorphone (Dilaudid).

ANS: A A patient with facial and chest burns is at risk for inhalation injury, and assessment of airway and breathing is the priority. The other actions will be completed after airway management is assured.

A patient needing vascular access for hemodialysis asks the nurse what the differences are between an arteriovenous (AV) fistula and a graft. The nurse explains that one advantage of the fistula is that it a. is much less likely to clot. b. increases patient mobility. c. can accommodate larger needles. d. can be used sooner after surgery.

ANS: A AV fistulas are much less likely to clot than grafts, although it takes longer for them to mature to the point where they can be used for dialysis. The choice of an AV fistula or a graft does not have an impact on needle size or patient mobility

Norepinephrine (Levophed) has been prescribed for a patient who was admitted with dehydration and hypotension. Which patient information indicates that the nurse should consult with the health care provider before administration of the norepinephrine? a. The patient's central venous pressure is 3 mm Hg. b. The patient is receiving low dose dopamine (Intropin). c. The patient is in sinus tachycardia at 100 to 110 beats/min. d. The patient has had no urine output since being admitted.

ANS: A Adequate fluid administration is essential before administration of vasopressors to patients with hypovolemic shock. The patient's low central venous pressure indicates a need for more volume replacement. The other patient data are not contraindications to norepinephrine administration. DIF: Cognitive Level: Application REF: 1733-1735 | 1736 TOP: Nursing Process: Implementation MSC: NCLEX: Physiological Integrity

29. When caring for a patient who has just arrived on the medical-surgical unit after having cardiac catheterization, which nursing intervention should the nurse delegate to a licensed practical/vocational nurse (LPN/LVN)? a. Give the scheduled aspirin and lipid-lowering medication. b. Perform the initial assessment of the catheter insertion site. c. Teach the patient about the usual postprocedure plan of care. d. Titrate the heparin infusion according to the agency protocol.

ANS: A Administration of oral medications is within the scope of practice for LPNs/LVNs. The initial assessment of the patient, patient teaching, and titration of IV anticoagulant medications should be done by the registered nurse (RN).

Which prescribed medication should the nurse administer first to a 60-year-old patient admitted to the emergency department in thyroid storm? a. Propranolol (Inderal) b. Propylthiouracil (PTU) c. Methimazole (Tapazole) d. Iodine (Lugol's solution)

ANS: A Adrenergic blockers work rapidly to decrease the cardiovascular manifestations of thyroid storm. The other medications take days to weeks to have an impact on thyroid function.

A 68-year-old male patient is brought to the emergency department (ED) by ambulance after being found unconscious on the bathroom floor by his spouse. Which action will the nurse take first? a. Check oxygen saturation. b. Assess pupil reaction to light. c. Verify Glasgow Coma Scale (GCS) score. d. Palpate the head for hematoma or bony irregularities.

ANS: A Airway patency and breathing are the most vital functions, and should be assessed first. The neurologic assessments should be accomplished next and additional assessment after that. DIF: Cognitive Level: Apply (application) REF: 1372 OBJ: Special Questions: Prioritization TOP: Nursing Process: Assessment MSC: NCLEX: Physiological Integrity

Which patient is most appropriate for the burn unit charge nurse to assign to a registered nurse (RN) who has floated from the hospital medical unit? a. A 34-year-old patient who has a weight loss of 15% from admission and requires enteral feedings. b. A 67-year-old patient who has blebs under an autograft on the thigh and has an order for bleb aspiration c. A 46-year-old patient who has just come back to the unit after having a cultured epithelial autograft to the chest d. A 65-year-old patient who has twice-daily burn debridements and dressing changes to partial-thickness facial burns

ANS: A An RN from a medical unit would be familiar with malnutrition and with administration and evaluation of response to enteral feedings. The other patients require burn assessment and care that is more appropriate for staff who regularly care for burned patients.

Which patient is most appropriate for the intensive care unit (ICU) charge nurse to assign to a registered nurse (RN) who has floated from the medical unit? a. A 45-year-old receiving IV antibiotics for meningococcal meningitis b. A 25-year-old admitted with a skull fracture and craniotomy the previous day c. A 55-year-old who has increased intracranial pressure (ICP) and is receiving hyperventilation therapy d. A 35-year-old with ICP monitoring after a head injury last week

ANS: A An RN who works on a medical unit will be familiar with administration of IV antibiotics and with meningitis. The postcraniotomy patient, patient with an ICP monitor, and the patient on a ventilator should be assigned to an RN familiar with the care of critically ill patients. DIF: Cognitive Level: Apply (application) REF: 15-16 OBJ: Special Questions: Multiple Patients TOP: Nursing Process: Planning MSC: NCLEX: Safe and Effective Care Environment

Which finding for a patient who has hypothyroidism and hypertension indicates that the nurse should contact the health care provider before administering levothyroxine (Synthroid)? a. Increased thyroxine (T4) level b. Blood pressure 112/62 mm Hg c. Distant and difficult to hear heart sounds d. Elevated thyroid stimulating hormone level

ANS: A An increased thyroxine level indicates the levothyroxine dose needs to be decreased. The other data are consistent with hypothyroidism and the nurse should administer the levothyroxine.

The nurse is caring for a patient who had kidney transplantation several years ago. Which assessment finding may indicate that the patient is experiencing adverse effects to the prescribed corticosteroid? a. Joint pain b. Tachycardia c. Postural hypotension d. Increase in creatinine level

ANS: A Aseptic necrosis of the weight-bearing joints can occur when patients take corticosteroids over a prolonged period. Increased creatinine level, orthostatic dizziness, and tachycardia are not caused by corticosteroid use.

9. A patient with dilated cardiomyopathy has new onset atrial fibrillation that has been unresponsive to drug therapy for several days. The priority teaching needed for this patient would include information about a. anticoagulant therapy. b. permanent pacemakers. c. electrical cardioversion. d. IV adenosine (Adenocard).

ANS: A Atrial fibrillation therapy that has persisted for more than 48 hours requires anticoagulant treatment for 3 weeks before attempting cardioversion. This is done to prevent embolization of clots from the atria. Cardioversion may be done after several weeks of anticoagulation therapy. Adenosine is not used to treat atrial fibrillation. Pacemakers are routinely used for patients with bradydysrhythmias. Information does not indicate that the patient has a slow heart rate. DIF: Cognitive Level: Apply (application) REF: 796 OBJ: Special Questions: Prioritization TOP: Nursing Process: Planning MSC: NCLEX: Physiological Integrity

8. After the nurse gives IV atropine to a patient with symptomatic type 1, second-degree atrioventricular (AV) block, which finding indicates that the medication has been effective? a. Increase in the patient's heart rate b. Increase in strength of peripheral pulses c. Decrease in premature atrial contractions d. Decrease in premature ventricular contractions

ANS: A Atropine will increase the heart rate and conduction through the AV node. Because the medication increases electrical conduction, not cardiac contractility, the quality of the peripheral pulses is not used to evaluate the drug effectiveness. The patient does not have premature atrial or ventricular contractions. DIF: Cognitive Level: Apply (application) REF: 798 TOP: Nursing Process: Evaluation MSC: NCLEX: Physiological Integrity

23. The urgent care center protocol for tick bites includes the following actions. Which action will the nurse take first when caring for a patient with a tick bite? a. Use tweezers to remove any remaining ticks. b. Check the vital signs, including temperature. c. Give doxycycline (Vibramycin) 100 mg orally. d. Obtain information about recent outdoor activities.

ANS: A Because neurotoxic venom is released as long as the tick is attached to the patient, the initial action should be to remove any ticks using tweezers or forceps. The other actions are also appropriate, but the priority is to minimize venom release. DIF: Cognitive Level: Apply (application) REF: 1697 OBJ: Special Questions: Prioritization TOP: Nursing Process: Planning MSC: NCLEX: Physiological Integrity

A patient with septic shock has a BP of 70/46 mm Hg, pulse 136, respirations 32, temperature 104° F, and blood glucose 246 mg/dL. Which of these prescribed interventions will the nurse implement first? a. Give normal saline IV at 500 mL/hr. b. Infuse drotrecogin- (Xigris) 24 mcg/kg. c. Start insulin drip to maintain blood glucose at 110 to 150 mg/dL. d. Titrate norepinephrine (Levophed) to keep mean arterial pressure (MAP) at 65 to 70 mm Hg.

ANS: A Because of the low systemic vascular resistance (SVR) associated with septic shock, fluid resuscitation is the initial therapy. The other actions also are appropriate and should be initiated quickly as well. DIF: Cognitive Level: Application REF: 1735-1737 OBJ: Special Questions: Prioritization TOP: Nursing Process: Implementation MSC: NCLEX: Physiological Integrity

An appropriate nursing intervention for a patient with non-Hodgkin's lymphoma whose platelet count drops to 18,000/µL during chemotherapy is to a. check all stools for occult blood. b. encourage fluids to 3000 mL/day. c. provide oral hygiene every 2 hours. d. check the temperature every 4 hours.

ANS: A Because the patient is at risk for spontaneous bleeding, the nurse should check stools for occult blood. A low platelet count does not require an increased fluid intake. Oral hygiene is important, but it is not necessary to provide oral care every 2 hours. The low platelet count does not increase risk for infection, so frequent temperature monitoring is not indicated

Which finding by the nurse when assessing a patient with Hashimoto's thyroiditis and a goiter will require the most immediate action? a. New-onset changes in the patient's voice b. Apical pulse rate at rest 112 beats/minute c. Elevation in the patient's T3 and T4 levels d. Bruit audible bilaterally over the thyroid gland

ANS: A Changes in the patient's voice indicate that the goiter is compressing the laryngeal nerve and may lead to airway compression. The other findings will also be reported but are expected with Hashimoto's thyroiditis and do not require immediate action.

The nurse is caring for a 78-year-old patient with aortic stenosis. Which assessment data obtained by the nurse would be most important to report to the health care provider? a. The patient complains of chest pressure when ambulating. b. A loud systolic murmur is heard along the right sternal border. c. A thrill is palpated at the second intercostal space, right sternal border. d. The point of maximum impulse (PMI) is at the left midclavicular line.

ANS: A Chest pressure (or pain) occurring with aortic stenosis is caused by cardiac ischemia, and reporting this information would be a priority. A systolic murmur and thrill are expected in a patient with aortic stenosis. A PMI at the left midclavicular line is normal.

34. Which information about a patient who has been receiving thrombolytic therapy for an acute myocardial infarction (AMI) is most important for the nurse to communicate to the health care provider? a. No change in the patient's chest pain b. An increase in troponin levels from baseline c. A large bruise at the patient's IV insertion site d. A decrease in ST-segment elevation on the electrocardiogram

ANS: A Continued chest pain suggests that the thrombolytic therapy is not effective and that other interventions such as percutaneous coronary intervention (PCI) may be needed. Bruising is a possible side effect of thrombolytic therapy, but it is not an indication that therapy should be discontinued. The decrease of the ST-segment elevation indicates that thrombolysis is occurring and perfusion is returning to the injured myocardium. An increase in troponin levels is expected with reperfusion and is related to the washout of cardiac markers into the circulation as the blocked vessel is opened.

The nurse is caring for a 64-year-old patient admitted with mitral valve regurgitation. Which information obtained by the nurse when assessing the patient should be communicated to the health care provider immediately? a. The patient has bilateral crackles. b. The patient has bilateral, 4+ peripheral edema. c. The patient has a loud systolic murmur across the precordium. d. The patient has a palpable thrill felt over the left anterior chest.

ANS: A Crackles that are audible throughout the lungs indicate that the patient is experiencing severe left ventricular failure with pulmonary congestion and needs immediate interventions such as diuretics. A systolic murmur and palpable thrill would be expected in a patient with mitral regurgitation. Although 4+ peripheral edema indicates a need for a change in therapy, it does not need to be addressed urgently. *MVR - crackles?*

A college athlete is seen in the clinic 6 weeks after a concussion. Which assessment information will the nurse collect to determine whether a patient is developing postconcussion syndrome? a. Short-term memory b. Muscle coordination c. Glasgow Coma Scale d. Pupil reaction to light

ANS: A Decreased short-term memory is one indication of postconcussion syndrome. The other data may be assessed but are not indications of postconcussion syndrome. DIF: Cognitive Level: Apply (application) REF: 1370 TOP: Nursing Process: Assessment MSC: NCLEX: Physiological Integrity

When the nurse is taking a history for a patient who is a possible candidate for a kidney transplant, which information about the patient indicates that the patient is not an appropriate candidate for transplantation? a. The patient has metastatic lung cancer. b. The patient has poorly controlled type 1 diabetes. c. The patient has a history of chronic hepatitis C infection. d. The patient is infected with the human immunodeficiency virus.

ANS: A Disseminated malignancies are a contraindication to transplantation. The conditions of the other patients are not contraindications for kidney transplant.

Which patient statement to the nurse indicates a need for additional instruction about taking oral ferrous sulfate? a. "I will call my health care provider if my stools turn black." b. "I will take a stool softener if I feel constipated occasionally." c. "I should take the iron with orange juice about an hour before eating." d. "I should increase my fluid and fiber intake while I am taking iron tablets."

ANS: A It is normal for the stools to appear black when a patient is taking iron, and the patient should not call the doctor about this. The other patient statements are correct

25. The nurse is caring for a patient who was admitted to the coronary care unit following an acute myocardial infarction (AMI) and percutaneous coronary intervention the previous day. Teaching for this patient would include a. when cardiac rehabilitation will begin. b. the typical emotional responses to AMI. c. information regarding discharge medications. d. the pathophysiology of coronary artery disease.

ANS: A Early after an AMI, the patient will want to know when resumption of usual activities can be expected. At this time, the patient's anxiety level or denial will interfere with good understanding of complex information such as the pathophysiology of coronary artery disease (CAD). Teaching about discharge medications should be done closer to discharge. The nurse should support the patient by decreasing anxiety rather than discussing the typical emotional responses to myocardial infarction (MI).

A patient is admitted to the hospital with possible acute pericarditis. The nurse should plan to teach the patient about the purpose of a. echocardiography. b. daily blood cultures. c. cardiac catheterization. d. 24-hour Holter monitor.

ANS: A Echocardiograms are useful in detecting the presence of the pericardial effusions associated with pericarditis. Blood cultures are not indicated unless the patient has evidence of sepsis. Cardiac catheterization and 24-hour Holter monitor is not a diagnostic procedure for pericarditis.

Which menu choice indicates that the patient understands the nurse's teaching about best dietary choices for iron-deficiency anemia? a. Omelet and whole wheat toast b. Cantaloupe and cottage cheese c. Strawberry and banana fruit plate d. Cornmeal muffin and orange juice

ANS: A Eggs and whole grain breads are high in iron. The other choices are appropriate for other nutritional deficiencies but are not the best choice for a patient with iron-deficiency anemia

The nurse examines the lymph nodes of a patient during a physical assessment. Which assessment finding would be of most concern to the nurse? a. A 2-cm nontender supraclavicular node b. A 1-cm mobile and nontender axillary node c. An inability to palpate any superficial lymph nodes d. Firm inguinal nodes in a patient with an infected foot

ANS: A Enlarged and nontender nodes are suggestive of malignancies such as lymphoma. Firm nodes are an expected finding in an area of infection. The superficial lymph nodes are usually not palpable in adults, but if they are palpable, they are normally 0.5 to 1 cm and nontender.

A patient has just been admitted with a 40% total body surface area (TBSA) burn injury. To maintain adequate nutrition, the nurse should plan to take which action? a. Insert a feeding tube and initiate enteral feedings. b. Infuse total parenteral nutrition via a central catheter. c. Encourage an oral intake of at least 5000 kcal per day. d. Administer multiple vitamins and minerals in the IV solution.

ANS: A Enteral feedings can usually be initiated during the emergent phase at low rates and increased over 24 to 48 hours to the goal rate. During the emergent phase, the patient will be unable to eat enough calories to meet nutritional needs and may have a paralytic ileus that prevents adequate nutrient absorption. Vitamins and minerals may be administered during the emergent phase, but these will not assist in meeting the patient's caloric needs. Parenteral nutrition increases the infection risk, does not help preserve gastrointestinal function, and is not routinely used in burn patients.

Which information obtained by the nurse assessing a patient admitted with multiple myeloma is most important to report to the health care provider? a. Serum calcium level is 15 mg/dL. b. Patient reports no stool for 5 days. c. Urine sample has Bence-Jones protein. d. Patient is complaining of severe back pain.

ANS: A Hypercalcemia may lead to complications such as dysrhythmias or seizures, and should be addressed quickly. The other patient findings will also be discussed with the health care provider, but are not life threatening

26. A patient who has recently started taking pravastatin (Pravachol) and niacin (Nicobid) reports the following symptoms to the nurse. Which is most important to communicate to the health care provider? a. Generalized muscle aches and pains b. Dizziness when changing positions quickly c. Nausea when taking the drugs before eating d. Flushing and pruritus after taking the medications

ANS: A Muscle aches and pains may indicate myopathy and rhabdomyolysis, which have caused acute kidney injury and death in some patients who have taken the statin medications. These symptoms indicate that the pravastatin may need to be discontinued. The other symptoms are common side effects when taking niacin, and although the nurse should follow-up with the health care provider, they do not indicate that a change in medication is needed.

12. A 20-year-old patient arrives in the emergency department (ED) several hours after taking "25 to 30" acetaminophen (Tylenol) tablets. Which action will the nurse plan to take? a. Give N-acetylcysteine (Mucomyst). b. Discuss the use of chelation therapy. c. Start oxygen using a non-rebreather mask. d. Have the patient drink large amounts of water.

ANS: A N-acetylcysteine is the recommended treatment to prevent liver damage after acetaminophen overdose. The other actions might be used for other types of poisoning, but they will not be appropriate for a patient with acetaminophen poisoning. DIF: Cognitive Level: Understand (comprehension) REF: 1689 TOP: Nursing Process: Planning MSC: NCLEX: Physiological Integrity

Which patient should the nurse assign as the roommate for a patient who has aplastic anemia? a. A patient with chronic heart failure b. A patient who has viral pneumonia c. A patient who has right leg cellulitis d. A patient with multiple abdominal drains

ANS: A Patients with aplastic anemia are at risk for infection because of the low white blood cell production associated with this type of anemia, so the nurse should avoid assigning a roommate with any possible infectious process

The nurse is caring for a 68-year-old hospitalized patient with a decreased glomerular filtration rate who is scheduled for an intravenous pyelogram (IVP). Which action will be included in the plan of care? a. Monitor the urine output after the procedure. b. Assist with monitored anesthesia care (MAC). c. Give oral contrast solution before the procedure. d. Insert a large size urinary catheter before the IVP.

ANS: A Patients with impaired renal function are at risk for decreased renal function after IVP because the contrast medium used is nephrotoxic, so the nurse should monitor the patient's urine output. MAC sedation and retention catheterization are not required for the procedure. The contrast medium is given IV, not orally.

A 42-year-old patient who has bacterial meningitis is disoriented and anxious. Which nursing action will be included in the plan of care? a. Encourage family members to remain at the bedside. b. Apply soft restraints to protect the patient from injury. c. Keep the room well-lighted to improve patient orientation. d. Minimize contact with the patient to decrease sensory input.

ANS: A Patients with meningitis and disorientation will be calmed by the presence of someone familiar at the bedside. Restraints should be avoided because they increase agitation and anxiety. The patient requires frequent assessment for complications. The use of touch and a soothing voice will decrease anxiety for most patients. The patient will have photophobia, so the light should be dim. DIF: Cognitive Level: Apply (application) REF: 1383 TOP: Nursing Process: Planning MSC: NCLEX: Physiological Integrity

The nurse suspects cardiac tamponade in a patient who has acute pericarditis. To assess for the presence of pulsus paradoxus, the nurse should a. note when Korotkoff sounds are auscultated during both inspiration and expiration. b. subtract the diastolic blood pressure (DBP) from the systolic blood pressure (SBP). c. check the electrocardiogram (ECG) for variations in rate during the respiratory cycle. d. listen for a pericardial friction rub that persists when the patient is instructed to stop breathing.

ANS: A Pulsus paradoxus exists when there is a gap of greater than 10 mm Hg between when Korotkoff sounds can be heard during only expiration and when they can be heard throughout the respiratory cycle. The other methods described would not be useful in determining the presence of pulsus paradoxus.

35. The nurse is planning care for a 48-year-old woman with acute severe pancreatitis. The highest priority patient outcome is a. maintaining normal respiratory function. b. expressing satisfaction with pain control. c. developing no ongoing pancreatic disease. d. having adequate fluid and electrolyte balance.

ANS: A Respiratory failure can occur as a complication of acute pancreatitis, and maintenance of adequate respiratory function is the priority goal. The other outcomes would also be appropriate for the patient.

While caring for a patient with aortic stenosis, the nurse identifies a nursing diagnosis of acute pain related to decreased coronary blood flow. A priority nursing intervention for this patient would be to a. promote rest to decrease myocardial oxygen demand. b. teach the patient about the need for anticoagulant therapy. c. teach the patient to use sublingual nitroglycerin for chest pain. d. raise the head of the bed 60 degrees to decrease venous return.

ANS: A Rest is recommended to balance myocardial oxygen supply and demand and to decrease chest pain. The patient with aortic stenosis requires higher preload to maintain cardiac output, so nitroglycerin and measures to decrease venous return are contraindicated. Anticoagulation is not recommended unless the patient has atrial fibrillation.

The nurse assesses a patient who has numerous petechiae on both arms. Which question should the nurse ask the patient? a. "Do you take salicylates?" b. "Are you taking any oral contraceptives?" c. "Have you been prescribed antiseizure drugs?" d. "How long have you taken antihypertensive drugs?"

ANS: A Salicylates interfere with platelet function and can lead to petechiae and ecchymoses. Antiseizure drugs may cause anemia, but not clotting disorders or bleeding. Oral contraceptives increase a person's clotting risk. Antihypertensives do not usually cause problems with decreased clotting.

The nurse has instructed a patient who is receiving hemodialysis about appropriate dietary choices. Which menu choice by the patient indicates that the teaching has been successful? a. Scrambled eggs, English muffin, and apple juice b. Oatmeal with cream, half a banana, and herbal tea c. Split-pea soup, whole-wheat toast, and nonfat milk d. Cheese sandwich, tomato soup, and cranberry juice

ANS: A Scrambled eggs would provide high-quality protein, and apple juice is low in potassium. Cheese is high in salt and phosphate, and tomato soup would be high in potassium. Split-pea soup is high in potassium, and dairy products are high in phosphate. Bananas are high in potassium, and the cream would be high in phosphate.

9. When planning the response to the potential use of smallpox as an agent of terrorism, the emergency department (ED) nurse manager will plan to obtain adequate quantities of a. vaccine. b. atropine. c. antibiotics. d. whole blood.

ANS: A Smallpox infection can be prevented or ameliorated by the administration of vaccine given rapidly after exposure. The other interventions would be helpful for other agents of terrorism but not for smallpox. DIF: Cognitive Level: Understand (comprehension) REF: 1690 TOP: Nursing Process: Planning MSC: NCLEX: Safe and Effective Care Environment

Admission vital signs for a brain-injured patient are blood pressure 128/68, pulse 110, and respirations 26. Which set of vital signs, if taken 1 hour after admission, will be of most concern to the nurse? a. Blood pressure 154/68, pulse 56, respirations 12 b. Blood pressure 134/72, pulse 90, respirations 32 c. Blood pressure 148/78, pulse 112, respirations 28 d. Blood pressure 110/70, pulse 120, respirations 30

ANS: A Systolic hypertension with widening pulse pressure, bradycardia, and respiratory changes represent Cushing's triad. These findings indicate that the intracranial pressure (ICP) has increased, and brain herniation may be imminent unless immediate action is taken to reduce ICP. The other vital signs may indicate the need for changes in treatment, but they are not indicative of an immediately life-threatening process. DIF: Cognitive Level: Apply (application) REF: 1360 TOP: Nursing Process: Assessment MSC: NCLEX: Physiological Integrity

1. To determine whether there is a delay in impulse conduction through the atria, the nurse will measure the duration of the patient's a. P wave. b. Q wave. c. P-R interval. d. QRS complex.

ANS: A The P wave represents the depolarization of the atria. The P-R interval represents depolarization of the atria, atrioventricular (AV) node, bundle of His, bundle branches, and the Purkinje fibers. The QRS represents ventricular depolarization. The Q wave is the first negative deflection following the P wave and should be narrow and short. DIF: Cognitive Level: Understand (comprehension) REF: 791 TOP: Nursing Process: Assessment MSC: NCLEX: Physiological Integrity

When caring for a patient who has had a head injury, which assessment information requires the most rapid action by the nurse? a. The patient is more difficult to arouse. b. The patient's pulse is slightly irregular. c. The patient's blood pressure increases from 120/54 to 136/62 mm Hg. d. The patient complains of a headache at pain level 5 of a 10-point scale.

ANS: A The change in level of consciousness (LOC) is an indicator of increased intracranial pressure (ICP) and suggests that action by the nurse is needed to prevent complications. The change in BP should be monitored but is not an indicator of a need for immediate nursing action. Headache is not unusual in a patient after a head injury. A slightly irregular apical pulse is not unusual. DIF: Cognitive Level: Application REF: 1431-1433

The patient with neurogenic shock is receiving a phenylephrine (Neo-Synephrine) infusion through a left forearm IV. Which assessment information obtained by the nurse indicates a need for immediate action? a. The patient's IV infusion site is cool and pale. b. The patient has warm, dry skin on the extremities. c. The patient has an apical pulse rate of 58 beats/min. d. The patient's urine output has been 28 mL over the last hour.

ANS: A The coldness and pallor at the infusion site suggest extravasation of the phenylephrine. The nurse should discontinue the IV and, if possible, infuse the medication into a central line. An apical pulse of 58 is typical for neurogenic shock but does not indicate an immediate need for nursing intervention. A 28 mL urinary output over 1 hour would require the nurse to monitor the output over the next hour, but an immediate change in therapy is not indicated. Warm, dry skin is consistent with early neurogenic shock, but it does not indicate a need for a change in therapy or immediate action. DIF: Cognitive Level: Application REF: 1733-1734 OBJ: Special Questions: Prioritization TOP: Nursing Process: Assessment MSC: NCLEX: Physiological Integrity

A patient with circumferential burns of both legs develops a decrease in dorsalis pedis pulse strength and numbness in the toes. Which action should the nurse take? a. Notify the health care provider. b. Monitor the pulses every 2 hours. c. Elevate both legs above heart level with pillows. d. Encourage the patient to flex and extend the toes on both feet.

ANS: A The decrease in pulse in a patient with circumferential burns indicates decreased circulation to the legs and the need for an escharotomy. Monitoring the pulses is not an adequate response to the decrease in circulation. Elevating the legs or increasing toe movement will not improve the patient's circulation.

15. The nurse is reviewing the laboratory tests for a patient who has recently been diagnosed with hypertension. Which result is most important to communicate to the health care provider? a. Serum creatinine of 2.6 mg/dL b. Serum potassium of 3.8 mEq/L c. Serum hemoglobin of 14.7 g/dL d. Blood glucose level of 98 mg/dL

ANS: A The elevated creatinine indicates renal damage caused by the hypertension. The other laboratory results are normal.

Which information obtained by the nurse when caring for a patient who has cardiogenic shock indicates that the patient may be developing multiple organ dysfunction syndrome (MODS)? a. The patient's serum creatinine level is elevated. b. The patient complains of intermittent chest pressure. c. The patient has crackles throughout both lung fields. d. The patient's extremities are cool and pulses are weak.

ANS: A The elevated serum creatinine level indicates that the patient has renal failure as well as heart failure. The crackles, chest pressure, and cool extremities are all consistent with the patient's diagnosis of cardiogenic shock. DIF: Cognitive Level: Application REF: 1740-1741 TOP: Nursing Process: Assessment MSC: NCLEX: Physiological Integrity

The nurse obtains these assessment findings for a patient who has a head injury. Which finding should be reported rapidly to the health care provider? a. Urine output of 800 mL in the last hour b. Intracranial pressure of 16 mm Hg when patient is turned c. Ventriculostomy drains 10 mL of cerebrospinal fluid per hour d. LICOX brain tissue oxygenation catheter shows PbtO2 of 38 mm Hg

ANS: A The high urine output indicates that diabetes insipidus may be developing and interventions to prevent dehydration need to be rapidly implemented. The other data do not indicate a need for any change in therapy. DIF: Cognitive Level: Application REF: 1434-1435

The nurse caring for a patient with type A hemophilia being admitted to the hospital with severe pain and swelling in the right knee will a. immobilize the joint. b. apply heat to the knee. c. assist the patient with light weight bearing. d. perform passive range of motion to the knee.

ANS: A The initial action should be total rest of the knee to minimize bleeding. Ice packs are used to decrease bleeding. Range of motion (ROM) and weight-bearing exercise are contraindicated initially, but after the bleeding stops, ROM and physical therapy are started

A patient is treated in the emergency department (ED) for shock of unknown etiology. The first action by the nurse should be to a. administer oxygen. b. attach a cardiac monitor. c. obtain the blood pressure. d. check the level of consciousness.

ANS: A The initial actions of the nurse are focused on the ABCs—airway, breathing, circulation—and administration of oxygen should be done first. The other actions should be accomplished as rapidly as possible after oxygen administration. DIF: Cognitive Level: Application REF: 1729-1731 | 1732 | 1733 | 1736-1737 OBJ: Special Questions: Prioritization TOP: Nursing Process: Implementation MSC: NCLEX: Physiological Integrity

22. Which nursing action should the nurse take first in order to assist a patient with newly diagnosed stage 1 hypertension in making needed dietary changes? a. Have the patient record dietary intake for 3 days. b. Give the patient a detailed list of low-sodium foods. c. Teach the patient about foods that are high in sodium. d. Help the patient make an appointment with a dietitian.

ANS: A The initial nursing action should be assessment of the patient's baseline dietary intake through a 3-day food diary. The other actions may be appropriate, but assessment of the patient's baseline should occur first.

The standard policy on the cardiac unit states, "Notify the health care provider for mean arterial pressure (MAP) less than 70 mm Hg." The nurse will need to call the health care provider about the a. postoperative patient with a BP of 116/42. b. newly admitted patient with a BP of 150/87. c. patient with left ventricular failure who has a BP of 110/70. d. patient with a myocardial infarction who has a BP of 140/86.

ANS: A The mean arterial pressure (MAP) is calculated using the formula MAP = (systolic BP + 2 diastolic BP)/3. The MAP for the postoperative patient in answer 3 is 67. The MAP in the other three patients is higher than 70 mm Hg

Which medication taken at home by a 47-year-old patient with decreased renal function will be of most concern to the nurse? a. ibuprofen (Motrin) b. warfarin (Coumadin) c. folic acid (vitamin B9) d. penicillin (Bicillin LA)

ANS: A The nonsteroidal antiinflammatory medications (NSAIDs) are nephrotoxic and should be avoided in patients with impaired renal function. The nurse also should ask about reasons the patient is taking the other medications, but the medication of most concern is the ibuprofen.

11. When assessing an older patient admitted to the emergency department (ED) with a broken arm and facial bruises, the nurse observes several additional bruises in various stages of healing. Which statement or question by the nurse is most appropriate? a. "Do you feel safe in your home?" b. "You should not return to your home." c. "Would you like to see a social worker?" d. "I need to report my concerns to the police."

ANS: A The nurse's initial response should be to further assess the patient's situation. Telling the patient not to return home may be an option once further assessment is done. A social worker may be appropriate once further assessment is completed. DIF: Cognitive Level: Apply (application) REF: 1682 TOP: Nursing Process: Implementation MSC: NCLEX: Psychosocial Integrity

19. Three days after experiencing a myocardial infarction (MI), a patient who is scheduled for discharge asks for assistance with hygiene activities, saying, "I am too nervous to take care of myself." Based on this information, which nursing diagnosis is appropriate? a. Ineffective coping related to anxiety b. Activity intolerance related to weakness c. Denial related to lack of acceptance of the MI d. Disturbed personal identity related to understanding of illness

ANS: A The patient data indicate that ineffective coping after the MI caused by anxiety about the impact of the MI is a concern. The other nursing diagnoses may be appropriate for some patients after an MI, but the data for this patient do not support denial, activity intolerance, or disturbed personal identity.

12. Which intervention by a new nurse who is caring for a patient who has just had an implantable cardioverter-defibrillator (ICD) inserted indicates a need for more education about care of patients with ICDs? a. The nurse assists the patient to do active range of motion exercises for all extremities. b. The nurse assists the patient to fill out the application for obtaining a Medic Alert ID. c. The nurse gives amiodarone (Cordarone) to the patient without first consulting with the health care provider. d. The nurse teaches the patient that sexual activity usually can be resumed once the surgical incision is healed.

ANS: A The patient should avoid moving the arm on the ICD insertion site until healing has occurred in order to prevent displacement of the ICD leads. The other actions by the new nurse are appropriate for this patient. DIF: Cognitive Level: Apply (application) REF: 803 TOP: Nursing Process: Evaluation MSC: NCLEX: Safe and Effective Care Environment

In a patient with acute kidney injury (AKI) who requires hemodialysis, a temporary vascular access is obtained by placing a catheter in the left femoral vein. Which intervention will be included in the plan of care? a. Place the patient on bed rest. b. Start continuous pulse oximetry. c. Discontinue the retention catheter. d. Restrict the patient's oral protein intake.

ANS: A The patient with a femoral vein catheter must be on bed rest to prevent trauma to the vein. Protein intake is likely to be increased when the patient is receiving dialysis. The retention catheter is likely to remain in place because accurate measurement of output will be needed. There is no indication that the patient needs continuous pulse oximetry.

An unconscious patient has a nursing diagnosis of ineffective cerebral tissue perfusion related to cerebral tissue swelling. Which nursing intervention will be included in the plan of care? a. Keep the head of the bed elevated to 30 degrees. b. Position the patient with the knees and hips flexed. c. Encourage coughing and deep breathing to improve oxygenation. d. Cluster nursing interventions to provide uninterrupted rest periods.

ANS: A The patient with increased intracranial pressure (ICP) should be maintained in the head-up position to help reduce ICP. Flexion of the hips and knees increases abdominal pressure, which increases ICP. Because the stimulation associated with nursing interventions increases ICP, clustering interventions will progressively elevate ICP. Coughing increases intrathoracic pressure and ICP. DIF: Cognitive Level: Application REF: 1436-1437

Two days after an acute myocardial infarction (MI), a patient complains of stabbing chest pain that increases with a deep breath. Which action will the nurse take first? a. Auscultate the heart sounds. b. Check the patient's temperature. c. Notify the patient's health care provider. d. Give the PRN acetaminophen (Tylenol).

ANS: A The patient's clinical manifestations and history are consistent with pericarditis, and the first action by the nurse should be to listen for a pericardial friction rub. Checking the temperature and notifying the health care provider are also appropriate actions but would not be done before listening for a rub. It is not stated for what symptom (e.g., headache) or finding (e.g., increased temperature) the PRN acetaminophen (Tylenol) is ordered.

A patient with shock of unknown etiology whose hemodynamic monitoring indicates BP 92/54, pulse 64, and an elevated pulmonary artery wedge pressure has the following collaborative interventions prescribed. Which intervention will the nurse question? a. Infuse normal saline at 250 mL/hr. b. Keep head of bed elevated to 30 degrees. c. Give nitroprusside (Nipride) unless systolic BP <90 mm Hg. d. Administer dobutamine (Dobutrex) to keep systolic BP >90 mm Hg.

ANS: A The patient's elevated pulmonary artery wedge pressure indicates volume excess. A normal saline infusion at 250 mL/hr will exacerbate this. The other actions are appropriate for the patient. DIF: Cognitive Level: Application REF: 1719 | 1721-1722 | 1735 TOP: Nursing Process: Planning MSC: NCLEX: Physiological Integrity

A male patient who has possible cerebral edema has a serum sodium level of 116 mEq/L (116 mmol/L) and a decreasing level of consciousness (LOC). He is now complaining of a headache. Which prescribed interventions should the nurse implement first? a. Administer IV 5% hypertonic saline. b. Draw blood for arterial blood gases (ABGs). c. Send patient for computed tomography (CT). d. Administer acetaminophen (Tylenol) 650 mg orally.

ANS: A The patient's low sodium indicates that hyponatremia may be causing the cerebral edema. The nurse's first action should be to correct the low sodium level. Acetaminophen (Tylenol) will have minimal effect on the headache because it is caused by cerebral edema and increased intracranial pressure (ICP). Drawing ABGs and obtaining a CT scan may provide some useful information, but the low sodium level may lead to seizures unless it is addressed quickly. DIF: Cognitive Level: Apply (application) REF: 1359-1361 OBJ: Special Questions: Prioritization TOP: Nursing Process: Implementation MSC: NCLEX: Physiological Integrity

13. A triage nurse in a busy emergency department (ED) assesses a patient who complains of 7/10 abdominal pain and states, "I had a temperature of 103.9° F (39.9° C) at home." The nurse's first action should be to a. assess the patient's current vital signs. b. give acetaminophen (Tylenol) per agency protocol. c. ask the patient to provide a clean-catch urine for urinalysis. d. tell the patient that it will 1 to 2 hours before being seen by the doctor.

ANS: A The patient's pain and statement about an elevated temperature indicate that the nurse should obtain vital signs before deciding how rapidly the patient should be seen by the health care provider. A urinalysis may be appropriate, but this would be done after the vital signs are taken. The nurse will not give acetaminophen before confirming a current temperature elevation. DIF: Cognitive Level: Apply (application) REF: 1675-1676 OBJ: Special Questions: Prioritization TOP: Nursing Process: Assessment MSC: NCLEX: Physiological Integrity

18. A patient's cardiac monitor shows a pattern of undulations of varying contours and amplitude with no measurable ECG pattern. The patient is unconscious and pulseless. Which action should the nurse take first? a. Perform immediate defibrillation. b. Give epinephrine (Adrenalin) IV. c. Prepare for endotracheal intubation. d. Give ventilations with a bag-valve-mask device.

ANS: A The patient's rhythm and assessment indicate ventricular fibrillation and cardiac arrest; the initial action should be to defibrillate. If a defibrillator is not immediately available or is unsuccessful in converting the patient to a better rhythm, the other actions may be appropriate. DIF: Cognitive Level: Apply (application) REF: 801 OBJ: Special Questions: Prioritization TOP: Nursing Process: Implementation MSC: NCLEX: Physiological Integrity

15. The following interventions are part of the emergency department (ED) protocol for a patient who has been admitted with multiple bee stings to the hands. Which action should the nurse take first? a. Remove the patient's rings. b. Apply ice packs to both hands. c. Apply calamine lotion to any itching areas. d. Give diphenhydramine (Benadryl) 50 mg PO.

ANS: A The patient's rings should be removed first because it might not be possible to remove them if swelling develops. The other orders should also be implemented as rapidly as possible after the nurse has removed the jewelry. DIF: Cognitive Level: Apply (application) REF: 1687 OBJ: Special Questions: Prioritization TOP: Nursing Process: Implementation MSC: NCLEX: Physiological Integrity

The nurse caring for a patient admitted with burns over 30% of the body surface assesses that urine output has dramatically increased. Which action by the nurse would best ensure adequate kidney function? a. Continue to monitor the urine output. b. Monitor for increased white blood cells (WBCs). c. Assess that blisters and edema have subsided. d. Prepare the patient for discharge from the burn unit.

ANS: A The patient's urine output indicates that the patient is entering the acute phase of the burn injury and moving on from the emergent stage. At the end of the emergent phase, capillary permeability normalizes and the patient begins to diurese large amounts of urine with a low specific gravity. Although this may occur at about 48 hours, it may be longer in some patients. Blisters and edema begin to resolve, but this process requires more time. White blood cells may increase or decrease, based on the patient's immune status and any infectious processes. The WBC count does not indicate kidney function. The patient will likely remain in the burn unit during the acute stage of burn injury.

18. An unresponsive 79-year-old is admitted to the emergency department (ED) during a summer heat wave. The patient's core temperature is 105.4° F (40.8° C), blood pressure (BP) 88/50, and pulse 112. The nurse initially will plan to a. apply wet sheets and a fan to the patient. b. provide O2 at 6 L/min with a nasal cannula. c. start lactated Ringer's solution at 1000 mL/hr. d. give acetaminophen (Tylenol) rectal suppository.

ANS: A The priority intervention is to cool the patient. Antipyretics are not effective in decreasing temperature in heat stroke, and 100% oxygen should be given, which requires a high flow rate through a non-rebreather mask. An older patient would be at risk for developing complications such as pulmonary edema if given fluids at 1000 mL/hr. DIF: Cognitive Level: Apply (application) REF: 1683 OBJ: Special Questions: Prioritization TOP: Nursing Process: Planning MSC: NCLEX: Physiological Integrity

20. Following an earthquake, patients are triaged by emergency medical personnel and are transported to the emergency department (ED). Which patient will the nurse need to assess first? a. A patient with a red tag b. A patient with a blue tag c. A patient with a black tag d. A patient with a yellow tag

ANS: A The red tag indicates a patient with a life-threatening injury requiring rapid treatment. The other tags indicate patients with less urgent injuries or those who are likely to die. DIF: Cognitive Level: Remember (knowledge) REF: 1692 OBJ: Special Questions: Prioritization TOP: Nursing Process: Assessment MSC: NCLEX: Safe and Effective Care Environment

When admitting a patient with a possible coup-contracoup injury after a car accident to the emergency department, the nurse obtains the following information. Which finding is most important to report to the health care provider? a. The patient takes warfarin (Coumadin) daily. b. The patient's blood pressure is 162/94 mm Hg. c. The patient is unable to remember the accident. d. The patient complains of a severe dull headache.

ANS: A The use of anticoagulants increases the risk for intracranial hemorrhage and should be immediately reported. The other information would not be unusual in a patient with a head injury who had just arrived to the ED. DIF: Cognitive Level: Application REF: 1437-1438

5. The nurse administering α-interferon and ribavirin (Rebetol) to a patient with chronic hepatitis C will plan to monitor for a. leukopenia. b. hypokalemia. c. polycythemia. d. hypoglycemia.

ANS: A Therapy with ribavirin and α-interferon may cause leukopenia. The other problems are not associated with this drug therapy.

Which patient should the nurse assess first? a. A patient with smoke inhalation who has wheezes and altered mental status b. A patient with full-thickness leg burns who has a dressing change scheduled c. A patient with abdominal burns who is complaining of level 8 (0 to 10 scale) pain d. A patient with 40% total body surface area (TBSA) burns who is receiving IV fluids at 500 mL/hour

ANS: A This patient has evidence of lower airway injury and hypoxemia and should be assessed immediately to determine the need for oxygen or intubation. The other patients should also be assessed as rapidly as possible, but they do not have evidence of life-threatening complications.

21. A few days after experiencing a myocardial infarction (MI) and successful percutaneous coronary intervention, the patient states, "I just had a little chest pain. As soon as I get out of here, I'm going for my vacation as planned." Which reply would be most appropriate for the nurse to make? a. "What do you think caused your chest pain?" b. "Where are you planning to go for your vacation?" c. "Sometimes plans need to change after a heart attack." d. "Recovery from a heart attack takes at least a few weeks."

ANS: A When the patient is experiencing denial, the nurse should assist the patient in testing reality until the patient has progressed beyond this step of the emotional adjustment to MI. Asking the patient about vacation plans reinforces the patient's plan, which is not appropriate in the immediate post-MI period. Reminding the patient in denial about the MI is likely to make the patient angry and lead to distrust of the nursing staff.

3. After the return of spontaneous circulation following the resuscitation of a patient who had a cardiac arrest, therapeutic hypothermia is ordered. Which action will the nurse include in the plan of care? a. Apply external cooling device. b. Check mental status every 15 minutes. c. Avoid the use of sedative medications. d. Rewarm if temperature is <91° F (32.8° C).

ANS: A When therapeutic hypothermia is used postresuscitation, external cooling devices or cold normal saline infusions are used to rapidly lower body temperature to 89.6° F to 93.2° F (32° C to 34° C). Because hypothermia will decrease brain activity, assessing mental status every 15 minutes is not needed at this stage. Sedative medications are administered during therapeutic hypothermia. DIF: Cognitive Level: Apply (application) REF: 1681 TOP: Nursing Process: Planning MSC: NCLEX: Physiological Integrity

An employee spills industrial acids on both arms and legs at work. What is the priority action that the occupational health nurse at the facility should take? a. Remove nonadherent clothing and watch. b. Apply an alkaline solution to the affected area. c. Place cool compresses on the area of exposure. d. Cover the affected area with dry, sterile dressings.

ANS: A With chemical burns, the initial action is to remove the chemical from contact with the skin as quickly as possible. Remove nonadherent clothing, shoes, watches, jewelry, glasses, or contact lenses (if face was exposed). Flush chemical from wound and surrounding area with copious amounts of saline solution or water. Covering the affected area or placing cool compresses on the area will leave the chemical in contact with the skin. Application of an alkaline solution is not recommended. *(Watch appears to = wristwatch, not the verb "to watch")*

A patient who was admitted with myxedema coma and diagnosed with hypothyroidism is improving and expected to be discharged in 2 days. Which teaching strategy will be best for the nurse to use? a. Provide written reminders of self-care information. b. Offer multiple options for management of therapies. c. Ensure privacy for teaching by asking visitors to leave. d. Delay teaching until patient discharge date is confirmed.

ANS: A Written instructions will be helpful to the patient because initially the hypothyroid patient may be unable to remember to take medications and other aspects of self-care. Because the treatment regimen is somewhat complex, teaching should be initiated well before discharge. Family members or friends should be included in teaching because the hypothyroid patient is likely to forget some aspects of the treatment plan. A simpler regimen will be easier to understand until the patient is euthyroid.

When preparing to cool a patient who is to begin therapeutic hypothermia, which intervention will the nurse plan to do (select all that apply)? a. Assist with endotracheal intubation. b. Insert an indwelling urinary catheter. c. Begin continuous cardiac monitoring. d. Obtain an order to restrain the patient. e. Prepare to give sympathomimetic drugs.

ANS: A, B, C Cooling can produce dysrhythmias, so the patient's heart rhythm should be continuously monitored and dysrhythmias treated if necessary. Bladder catheterization and endotracheal intubation are needed during cooling. Sympathomimetic drugs tend to stimulate the heart and increase the risk for fatal dysrhythmias such as ventricular fibrillation. Patients receiving therapeutic hypothermia are comatose or do not follow commands so restraints are not indicated. DIF: Cognitive Level: Apply (application) REF: 1681 TOP: Nursing Process: Planning MSC: NCLEX: Physiological Integrity

A patient with neurogenic shock has just arrived in the emergency department after a diving accident. He has a cervical collar in place. Which of the following actions should the nurse take (select all that apply)? a. Prepare to administer atropine IV. b. Obtain baseline body temperature. c. Prepare for intubation and mechanical ventilation. d. Administer large volumes of lactated Ringer's solution. e. Administer high-flow oxygen (100%) by non-rebreather mask.

ANS: A, B, C, E All of the actions are appropriate except to give large volumes of lactated Ringer's solution. The patient with neurogenic shock usually has a normal blood volume, and it is important not to volume overload the patient. In addition, lactated Ringer's solution is used cautiously in all shock situations because the failing liver cannot convert lactate to bicarbonate. DIF: Cognitive Level: Application REF: 1736-1737 OBJ: Special Questions: Alternate Item Format TOP: Nursing Process: Implementation MSC: NCLEX: Physiological Integrity

Which preventive actions by the nurse will help limit the development of systemic inflammatory response syndrome (SIRS) in patients admitted to the hospital (select all that apply)? a. Use aseptic technique when caring for invasive lines or devices. b. Ambulate postoperative patients as soon as possible after surgery. c. Remove indwelling urinary catheters as soon as possible after surgery. d. Advocate for parenteral nutrition for patients who cannot take oral feedings. e. Administer prescribed antibiotics within 1 hour for patients with possible sepsis.

ANS: A, B, C, E Because sepsis is the most frequent etiology for SIRS, measures to avoid infection such as removing indwelling urinary catheters as soon as possible, use of aseptic technique, and early ambulation should be included in the plan of care. Adequate nutrition is important in preventing SIRS. Enteral, rather than parenteral, nutrition is preferred when patients are unable to take oral feedings because enteral nutrition helps maintain the integrity of the intestine, thus decreasing infection risk. Antibiotics should be administered within 1 hour after being prescribed to decrease the risk of sepsis progressing to SIRS.

46. After an unimmunized individual is exposed to hepatitis B through a needle-stick injury, which actions will the nurse plan to take (select all that apply)? a. Administer hepatitis B vaccine. b. Test for antibodies to hepatitis B. c. Teach about α-interferon therapy. d. Give hepatitis B immune globulin. e. Teach about choices for oral antiviral therapy.

ANS: A, B, D The recommendations for hepatitis B exposure include both vaccination and immune globulin administration. In addition, baseline testing for hepatitis B antibodies will be needed. Interferon and oral antivirals are not used for hepatitis B prophylaxis.

After the insertion of an arteriovenous graft (AVG) in the right forearm, a patient complains of pain and coldness of the right fingers. Which action should the nurse take? a. Elevate the patient's arm above the level of the heart. b. Report the patient's symptoms to the health care provider. c. Remind the patient about the need to take a daily low-dose aspirin tablet. d. Educate the patient about the normal vascular response after AVG insertion.

ANS: B

8. Which nursing intervention will be included in the plan of care after a patient with a right femur fracture has a hip spica cast applied? a. Avoid placing the patient in prone position. b. Ask the patient about abdominal discomfort. c. Discuss remaining on bed rest for several weeks. d. Use the cast support bar to reposition the patient.

ANS: B Assessment of bowel sounds, abdominal pain, and nausea and vomiting will detect the development of cast syndrome. To avoid breakage, the support bar should not be used for repositioning. After the cast dries, the patient can begin ambulating with the assistance of physical therapy personnel and may be turned to the prone position.

14. Which action will the nurse take in order to evaluate the effectiveness of Buck's traction for a 62-year-old patient who has an intracapsular fracture of the right femur? a. Check peripheral pulses. b. Ask about hip pain level. c. Assess for hip contractures. d. Monitor for hip dislocation.

ANS: B Buck's traction keeps the leg immobilized and reduces painful muscle spasm. Hip contractures and dislocation are unlikely to occur in this situation. The peripheral pulses will be assessed, but this does not help in evaluating the effectiveness of Buck's traction.

29. A young man arrives in the emergency department with ankle swelling and severe pain after twisting his ankle playing basketball. Which of these prescribed collaborative interventions will the nurse implement first? a. Take the patient to have x-rays. b. Wrap the ankle and apply an ice pack. c. Administer naproxen (Naprosyn) 500 mg PO. d. Give acetaminophen with codeine (Tylenol #3).

ANS: B Immediate care after a sprain or strain injury includes the application of cold and compression to the injury to minimize swelling. The other actions should be taken after the ankle is wrapped with a compression bandage and ice is applied.

26. A patient is being discharged 4 days after hip replacement surgery using the posterior approach. Which patient action requires immediate intervention by the nurse? a. The patient uses crutches with a swing-to gait. b. The patient leans over to pull shoes and socks on. c. The patient sits straight up on the edge of the bed. d. The patient bends over the sink while brushing teeth.

ANS: B Leaning over would flex the hip at greater than 90 degrees and predispose the patient to hip dislocation. The other patient actions are appropriate and do not require any immediate action by the nurse to protect the patient.

11. A patient who is to have no weight bearing on the left leg is learning to walk using crutches. Which observation by the nurse indicates that the patient can safely ambulate independently? a. The patient moves the right crutch with the right leg and then the left crutch with the left leg. b. The patient advances the left leg and both crutches together and then advances the right leg. c. The patient uses the bedside chair to assist in balance as needed when ambulating in the room. d. The patient keeps the padded area of the crutch firmly in the axillary area when ambulating.

ANS: B Patients are usually taught to move the crutches and the injured leg forward at the same time and then to move the unaffected leg. Patients are discouraged from using furniture to assist with ambulation. The patient is taught to place weight on the hands, not in the axilla, to avoid nerve damage. If the 2- or 4-point gaits are to be used, the crutch and leg on opposite sides move forward, not the crutch and same-side leg. *OMGGGGG these questions kill me. OK, NWB one LE: crutches, then good LE.*

19. The day after a having a right below-the-knee amputation, a patient complains of pain in the right foot. Which action is best for the nurse to take? a. Explain the reasons for the phantom limb pain. b. Administer prescribed analgesics to relieve the pain. c. Loosen the compression bandage to decrease incisional pressure. d. Inform the patient that this phantom pain will diminish over time.

ANS: B Phantom limb sensation is treated like any other type of postoperative pain would be treated. Explanations of the reason for the pain may be given, but the nurse should still medicate the patient. The compression bandage is left in place except during physical therapy or bathing. Although the pain may decrease over time, it still requires treatment now.

5. A 22-year-old tennis player has an arthroscopic repair of a rotator cuff injury performed in same-day surgery. When the nurse plans postoperative teaching for the patient, which information will be included? a. "You will not be able to serve a tennis ball again." b. "You will work with a physical therapist tomorrow." c. "The doctor will use the drop-arm test to determine the success of surgery." d. "Leave the shoulder immobilizer on for the first 4 days to minimize pain."

ANS: B Physical therapy after a rotator cuff repair begins on the first postoperative day to prevent "frozen shoulder." A shoulder immobilizer is used immediately after the surgery, but leaving the arm immobilized for several days would lead to loss of range of motion (ROM). The drop-arm test is used to test for rotator cuff injury, but not after surgery. The patient may be able to return to pitching after rehabilitation.

13. A patient with a complex pelvic fracture from a motor vehicle crash is on bed rest. Which nursing assessment finding is important to report to the health care provider? a. The patient states that the pelvis feels unstable. b. Abdomen is distended and bowel sounds are absent. c. There are ecchymoses across the abdomen and hips. d. The patient complains of pelvic pain with palpation.

ANS: B The abdominal distention and absent bowel sounds may be due to complications of pelvic fractures such as paralytic ileus or hemorrhage or trauma to the bladder, urethra, or colon. Pelvic instability, abdominal pain with palpation, and abdominal bruising would be expected with this type of injury.

17. When doing discharge teaching for a 19-year-old patient who has had a repair of a fractured mandible, the nurse will include information about a. administration of nasogastric tube feedings. b. how and when to cut the immobilizing wires. c. the importance of high-fiber foods in the diet. d. the use of sterile technique for dressing changes.

ANS: B The jaw will be wired for stabilization, and the patient should know what emergency situations require that the wires be cut to protect the airway. There are no dressing changes for this procedure. The diet is liquid, and patients are not able to chew high-fiber foods. Initially, the patient may receive nasogastric tube feedings, but by discharge, the patient will swallow liquid through a straw.

16. A patient who has had an open reduction and internal fixation (ORIF) of a hip fracture tells the nurse that he is ready to get out of bed for the first time. Which action should the nurse take? a. Use a mechanical lift to transfer the patient from the bed to the chair. b. Check the postoperative orders for the patient's weight-bearing status. c. Avoid administration of pain medications before getting the patient up. d. Delegate the transfer of the patient to nursing assistive personnel (NAP).

ANS: B The nurse should be familiar with the weight-bearing orders for the patient before attempting the transfer. Mechanical lifts are not typically needed after this surgery. Pain medications should be given because the movement is likely to be painful for the patient. The registered nurse (RN) should supervise the patient during the initial transfer to evaluate how well the patient is able to accomplish this skill.

37. A patient undergoes a left above-the-knee amputation with an immediate prosthetic fitting. When the patient arrives on the orthopedic unit after surgery, the nurse should a. place the patient in a prone position. b. check the surgical site for hemorrhage. c. remove the prosthesis and wrap the site. d. keep the residual leg elevated on a pillow.

ANS: B The nurse should monitor for hemorrhage after the surgery. The prosthesis will not be removed. To avoid flexion contracture of the hip, the leg will not be elevated on a pillow. The patient is placed in a prone position after amputation to prevent hip flexion, but this would not be done during the immediate postoperative period.

. Which statement by a 62-year-old patient who has had an above-the-knee amputation indicates that the nurse's discharge teaching has been effective? a. "I should elevate my residual limb on a pillow 2 or 3 times a day." b. "I should lay flat on my abdomen for 30 minutes 3 or 4 times a day." c. "I should change the limb sock when it becomes soiled or each week." d. "I should use lotion on the stump to prevent skin drying and cracking."

ANS: B The patient lies in the prone position several times daily to prevent flexion contractures of the hip. The limb sock should be changed daily. Lotion should not be used on the stump. The residual limb should not be elevated because this would encourage flexion contracture.

38. Before assisting a patient with ambulation 2 days after a total hip replacement, which action is most important for the nurse to take? a. Observe the status of the incisional drain device. b. Administer the ordered oral opioid pain medication. c. Instruct the patient about the benefits of ambulation. d. Change the hip dressing and document the wound appearance.

ANS: B The patient should be adequately medicated for pain before any attempt to ambulate. Instructions about the benefits of ambulation may increase the patient's willingness to ambulate, but decreasing pain with ambulation is more important. The presence of an incisional drain or timing of dressing change will not affect ambulation.

9. A patient has a long-arm plaster cast applied for immobilization of a fractured left radius. Until the cast has completely dried, the nurse should a. keep the left arm in dependent position. b. avoid handling the cast using fingertips. c. place gauze around the cast edge to pad any roughness. d. cover the cast with a small blanket to absorb the dampness.

ANS: B Until a plaster cast has dried, using the palms rather than the fingertips to handle the cast helps prevent creating protrusions inside the cast that could place pressure on the skin. The left arm should be elevated to prevent swelling. The edges of the cast may be petaled once the cast is dry, but padding the edges before that may cause the cast to be misshapen. The cast should not be covered until it is dry because heat builds up during drying.

When auscultating over the patient's abdominal aorta, the nurse hears a humming sound. The nurse documents this finding as a a. thrill. b. bruit. c. murmur. d. normal finding.

ANS: B A bruit is the sound created by turbulent blood flow in an artery. Thrills are palpable vibrations felt when there is turbulent blood flow through the heart or in a blood vessel. A murmur is the sound caused by turbulent blood flow through the heart. Auscultating a bruit in an artery is not normal and indicates pathology

Which patient requires the most rapid assessment and care by the emergency department nurse? a. The patient with hemochromatosis who reports abdominal pain b. The patient with neutropenia who has a temperature of 101.8° F c. The patient with sickle cell anemia who has had nausea and diarrhea for 24 hours d. The patient with thrombocytopenia who has oozing after having a tooth extracted

ANS: B A neutropenic patient with a fever is assumed to have an infection and is at risk for rapidly developing sepsis. Rapid assessment, cultures, and initiation of antibiotic therapy are needed. The other patients also require rapid assessment and care but not as urgently as the neutropenic patient

A patient who has a head injury is diagnosed with a concussion. Which action will the nurse plan to take? a. Coordinate the transfer of the patient to the operating room. b. Provide discharge instructions about monitoring neurologic status. c. Transport the patient to radiology for magnetic resonance imaging (MRI) of the brain. d. Arrange to admit the patient to the neurologic unit for observation for 24 hours.

ANS: B A patient with a minor head trauma is usually discharged with instructions about neurologic monitoring and the need to return if neurologic status deteriorates. MRI, hospital admission, or surgery are not indicated in a patient with a concussion. DIF: Cognitive Level: Application REF: 1440

A patient is scheduled for a cardiac catheterization with coronary angiography. Before the test, the nurse informs the patient that a. it will be important to lie completely still during the procedure. b. a flushed feeling may be noted when the contrast dye is injected. c. monitored anesthesia care will be provided during the procedure. d. arterial pressure monitoring will be required for 24 hours after the test.

ANS: B A sensation of warmth or flushing is common when the contrast material is injected, which can be anxiety-producing unless it has been discussed with the patient. The patient may receive a sedative drug before the procedure, but monitored anesthesia care is not used. Arterial pressure monitoring is not routinely used after the procedure to monitor blood pressure. The patient is not immobile during cardiac catheterization and may be asked to cough or take deep breaths

The nurse is reviewing laboratory results and notes an aPTT level of 28 seconds. The nurse should notify the health care provider in anticipation of adjusting which medication? a. Aspirin b. Heparin c. Warfarin d. Erythropoietin

ANS: B Activated partial thromboplastin time (aPTT) assesses intrinsic coagulation by measuring factors I, II, V, VIII, IX, X, XI, XII. aPTT is increased (prolonged) in heparin administration. aPTT is used to monitor whether heparin is at a therapeutic level (needs to be greater than the normal range of 25 to 35 sec). Prothrombin time (PT) and international normalized ratio (INR) are most commonly used to test for therapeutic levels of warfarin (Coumadin). Aspirin affects platelet function. Erythropoietin is used to stimulate red blood cell production.

23. The nurse recognizes that teaching a 44-year-old woman following a laparoscopic cholecystectomy has been effective when the patient states which of the following? a. "I can expect yellow-green drainage from the incision for a few days." b. "I can remove the bandages on my incisions tomorrow and take a shower." c. "I should plan to limit my activities and not return to work for 4 to 6 weeks." d. "I will always need to maintain a low-fat diet since I no longer have a gallbladder."

ANS: B After a laparoscopic cholecystectomy, the patient will have Band-Aids in place over the incisions. Patients are discharged the same (or next) day and have few restrictions on activities of daily living. Drainage from the incisions would be abnormal, and the patient should be instructed to call the health care provider if this occurs. A low-fat diet may be recommended for a few weeks after surgery but will not be a life-long requirement.

A patient with hypertension and stage 2 chronic kidney disease (CKD) is receiving captopril (Capoten). Before administration of the medication, the nurse will check the patient's a. glucose. b. potassium. c. creatinine. d. phosphate.

ANS: B Angiotensin-converting enzyme (ACE) inhibitors are frequently used in patients with CKD because they delay the progression of the CKD, but they cause potassium retention. Therefore, careful monitoring of potassium levels is needed in patients who are at risk for hyperkalemia. The other laboratory values also would be monitored in patients with CKD but would not affect whether the captopril was given or not.

After change-of-shift report in the progressive care unit, who should the nurse care for first? a. Patient who had an inferior myocardial infarction 2 days ago and has crackles in the lung bases b. Patient with suspected urosepsis who has new orders for urine and blood cultures and antibiotics c. Patient who had a T5 spinal cord injury 1 week ago and currently has a heart rate of 54 beats/minute d. Patient admitted with anaphylaxis 3 hours ago who now has clear lung sounds and a blood pressure of 108/58 mm Hg

ANS: B Antibiotics should be administered within the first hour for patients who have sepsis or suspected sepsis in order to prevent progression to systemic inflammatory response syndrome (SIRS) and septic shock. The data on the other patients indicate that they are more stable. Crackles heard only at the lung bases do not require immediate intervention in a patient who has had a myocardial infarction. Mild bradycardia does not usually require atropine in patients who have a spinal cord injury. The findings for the patient admitted with anaphylaxis indicate resolution of bronchospasm and hypotension.

After receiving change-of-shift report for several patients with neutropenia, which patient should the nurse assess first? a. 56-year-old with frequent explosive diarrhea b. 33-year-old with a fever of 100.8° F (38.2° C) c. 66-year-old who has white pharyngeal lesions d. 23-year old who is complaining of severe fatigue

ANS: B Any fever in a neutropenic patient indicates infection and can quickly lead to sepsis and septic shock. Rapid assessment and (if prescribed) initiation of antibiotic therapy within 1 hour are needed. The other patients also need to be assessed but do not exhibit symptoms of potentially life-threatening problems

Which of these findings is the best indicator that the fluid resuscitation for a patient with hypovolemic shock has been successful? a. Hemoglobin is within normal limits. b. Urine output is 60 mL over the last hour. c. Pulmonary artery wedge pressure (PAWP) is normal. d. Mean arterial pressure (MAP) is 65 mm Hg.

ANS: B Assessment of end organ perfusion, such as an adequate urine output, is the best indicator that fluid resuscitation has been successful. The hemoglobin level, PAWP, and MAP are useful in determining the effects of fluid administration, but they are not as useful as data indicating good organ perfusion. DIF: Cognitive Level: Application REF: 1733-1735 TOP: Nursing Process: Evaluation MSC: NCLEX: Physiological Integrity

24. The nurse is caring for a 73-year-old man who has cirrhosis. Which data obtained by the nurse during the assessment will be of most concern? a. The patient complains of right upper-quadrant pain with palpation. b. The patient's hands flap back and forth when the arms are extended. c. The patient has ascites and a 2-kg weight gain from the previous day. d. The patient's skin has multiple spider-shaped blood vessels on the abdomen.

ANS: B Asterixis indicates that the patient has hepatic encephalopathy, and hepatic coma may occur. The spider angiomas and right upper quadrant abdominal pain are not unusual for the patient with cirrhosis and do not require a change in treatment. The ascites and weight gain indicate the need for treatment but not as urgently as the changes in neurologic status.

43. A 36-year-old female patient is receiving treatment for chronic hepatitis C with pegylated interferon (PEG-Intron, Pegasys), ribavirin (Rebetol), and telaprevir (Incivek). Which finding is most important to communicate to the health care provider? a. Weight loss of 2 lb (1 kg) b. Positive urine pregnancy test c. Hemoglobin level of 10.4 g/dL d. Complaints of nausea and anorexia

ANS: B Because ribavirin is teratogenic, the medication will need to be discontinued immediately. Anemia, weight loss, and nausea are common adverse effects of the prescribed regimen and may require actions such as patient teaching, but they would not require immediate cessation of the therapy.

After endotracheal suctioning, the nurse notes that the intracranial pressure for a patient with a traumatic head injury has increased from 14 to 17 mm Hg. Which action should the nurse take first? a. Document the increase in intracranial pressure. b. Ensure that the patient's neck is in neutral position. c. Notify the health care provider about the change in pressure. d. Increase the rate of the prescribed propofol (Diprivan) infusion.

ANS: B Because suctioning will cause a transient increase in intracranial pressure, the nurse should initially check for other factors that might be contributing to the increase and observe the patient for a few minutes. Documentation is needed, but this is not the first action. There is no need to notify the health care provider about this expected reaction to suctioning. Propofol is used to control patient anxiety or agitation. There is no indication that anxiety has contributed to the increase in intracranial pressure. DIF: Cognitive Level: Apply (application) REF: 1367 | 1361 OBJ: Special Questions: Prioritization TOP: Nursing Process: Implementation MSC: NCLEX: Physiological Integrity

To assess for functional deficits, which question will the nurse ask a patient who has been admitted for treatment of a benign occipital lobe tumor? a. "Do you have difficulty in hearing?" b. "Are you experiencing visual problems?" c. "Are you having any trouble with your balance?" d. "Have you developed any weakness on one side?"

ANS: B Because the occipital lobe is responsible for visual reception, the patient with a tumor in this area is likely to have problems with vision. The other questions will be better for assessing function of the temporal lobe, cerebellum, and frontal lobe. DIF: Cognitive Level: Apply (application) REF: 1376 TOP: Nursing Process: Assessment MSC: NCLEX: Physiological Integrity

Which information about a patient who is receiving vasopressin (Pitressin) to treat septic shock is most important for the nurse to communicate to the heath care provider? a. The patient's heart rate is 108 beats/min. b. The patient is complaining of chest pain. c. The patient's peripheral pulses are weak. d. The patient's urine output is 15 mL/hr.

ANS: B Because vasopressin is a potent vasoconstrictor, it may decrease coronary artery perfusion. The other information is consistent with the patient's diagnosis and should be reported to the health care provider but does not indicate a need for a change in therapy. DIF: Cognitive Level: Application REF: 1735-1736 OBJ: Special Questions: Prioritization TOP: Nursing Process: Assessment MSC: NCLEX: Physiological Integrity

A routine complete blood count indicates that an active 80-year-old man may have myelodysplastic syndrome. The nurse will plan to teach the patient about a. blood transfusion b. bone marrow biopsy. c. filgrastim (Neupogen) administration. d. erythropoietin (Epogen) administration.

ANS: B Bone marrow biopsy is needed to make the diagnosis and determine the specific type of myelodysplastic syndrome. The other treatments may be necessary if there is progression of the myelodysplastic syndrome, but the initial action for this asymptomatic patient will be a bone marrow biopsy.

A patient admitted with a diffuse axonal injury has a systemic blood pressure (BP) of 106/52 mm Hg and an intracranial pressure (ICP) of 14 mm Hg. Which action should the nurse take first? a. Document the BP and ICP in the patient's record. b. Report the BP and ICP to the health care provider. c. Elevate the head of the patient's bed to 60 degrees. d. Continue to monitor the patient's vital signs and ICP.

ANS: B Calculate the cerebral perfusion pressure (CPP): (CPP = mean arterial pressure [MAP] - ICP). MAP = DBP + 1/3 (systolic blood pressure [SBP] - diastolic blood pressure [DBP]). Therefore the (MAP) is 70 and the CPP is 56 mm Hg, which is below the normal of 60 to 100 mm Hg and approaching the level of ischemia and neuronal death. Immediate changes in the patient's therapy such as fluid infusion or vasopressor administration are needed to improve the cerebral perfusion pressure. Adjustments in the head elevation should only be done after consulting with the health care provider. Continued monitoring and documentation will also be done, but they are not the first actions that the nurse should take

Which nursing action is a priority for a patient who has suffered a burn injury while working on an electrical power line? a. Obtain the blood pressure. b. Stabilize the cervical spine. c. Assess for the contact points. d. Check alertness and orientation.

ANS: B Cervical spine injuries are commonly associated with electrical burns. Therefore stabilization of the cervical spine takes precedence after airway management. The other actions are also included in the emergent care after electrical burns, but the most important action is to avoid spinal cord injury.

3. Which assessment data collected by the nurse who is admitting a patient with chest pain suggest that the pain is caused by an acute myocardial infarction (AMI)? a. The pain increases with deep breathing. b. The pain has lasted longer than 30 minutes. c. The pain is relieved after the patient takes nitroglycerin. d. The pain is reproducible when the patient raises the arms.

ANS: B Chest pain that lasts for 20 minutes or more is characteristic of AMI. Changes in pain that occur with raising the arms or with deep breathing are more typical of musculoskeletal pain or pericarditis. Stable angina is usually relieved when the patient takes nitroglycerin.

After noting that a patient with a head injury has clear nasal drainage, which action should the nurse take? a. Have the patient blow the nose. b. Check the nasal drainage for glucose. c. Assure the patient that rhinorrhea is normal after a head injury. d. Obtain a specimen of the fluid to send for culture and sensitivity.

ANS: B Clear nasal drainage in a patient with a head injury suggests a dural tear and cerebrospinal fluid (CSF) leakage. If the drainage is CSF, it will test positive for glucose. Fluid leaking from the nose will have normal nasal flora, so culture and sensitivity will not be useful. Blowing the nose is avoided to prevent CSF leakage. DIF: Cognitive Level: Application REF: 1438-1439

A 20-year-old male patient is admitted with a head injury after a collision while playing football. After noting that the patient has developed clear nasal drainage, which action should the nurse take? a. Have the patient gently blow the nose. b. Check the drainage for glucose content. c. Teach the patient that rhinorrhea is expected after a head injury. d. Obtain a specimen of the fluid to send for culture and sensitivity.

ANS: B Clear nasal drainage in a patient with a head injury suggests a dural tear and cerebrospinal fluid (CSF) leakage. If the drainage is CSF, it will test positive for glucose. Fluid leaking from the nose will have normal nasal flora, so culture and sensitivity will not be useful. Blowing the nose is avoided to prevent CSF leakage. DIF: Cognitive Level: Apply (application) REF: 1369 TOP: Nursing Process: Implementation MSC: NCLEX: Physiological Integrity

The nurse is assessing a patient who is receiving peritoneal dialysis with 2 L inflows. Which information should be reported immediately to the health care provider? a. The patient has an outflow volume of 1800 mL. b. The patient's peritoneal effluent appears cloudy. c. The patient has abdominal pain during the inflow phase. d. The patient complains of feeling bloated after the inflow.

ANS: B Cloudy appearing peritoneal effluent is a sign of peritonitis and should be reported immediately so that treatment with antibiotics can be started. The other problems can be addressed through nursing interventions such as slowing the inflow and repositioning the patient

Which statement by a patient who is being discharged from the emergency department (ED) after a head injury indicates a need for intervention by the nurse? a. "I will return if I feel dizzy or nauseated." b. "I am going to drive home and go to bed." c. "I do not even remember being in an accident." d. "I can take acetaminophen (Tylenol) for my headache."

ANS: B Following a head injury, the patient should avoid operating heavy machinery. Retrograde amnesia is common after a concussion. The patient can take acetaminophen for headache and should return if symptoms of increased intracranial pressure such as dizziness or nausea occur. DIF: Cognitive Level: Application REF: 1444

The cardiac telemetry unit charge nurse receives status reports from other nursing units about four patients who need cardiac monitoring. Which patient should be transferred to the cardiac unit first? a. Patient with Hashimoto's thyroiditis and a heart rate of 102 b. Patient with tetany who has a new order for IV calcium chloride c. Patient with Cushing syndrome and a blood glucose of 140 mg/dL d. Patient with Addison's disease who takes hydrocortisone twice daily

ANS: B Emergency treatment of tetany requires IV administration of calcium; ECG monitoring will be required because cardiac arrest may occur if high calcium levels result from too-rapid administration. The information about the other patients indicates that they are more stable than the patient with tetany.

The following therapies are prescribed by the health care provider for a patient who has respiratory distress and syncope after a bee sting. Which will the nurse administer first? a. normal saline infusion b. epinephrine (Adrenalin) c. dexamethasone (Decadron) d. diphenhydramine (Benadryl)

ANS: B Epinephrine rapidly causes peripheral vasoconstriction, dilates the bronchi, and blocks the effects of histamine and reverses the vasodilation, bronchoconstriction, and histamine release that cause the symptoms of anaphylaxis. The other interventions also are appropriate but would not be the first ones administered. DIF: Cognitive Level: Application REF: 1736-1737 OBJ: Special Questions: Prioritization TOP: Nursing Process: Implementation MSC: NCLEX: Physiological Integrity

The following interventions are ordered by the health care provider for a patient who has respiratory distress and syncope after eating strawberries. Which will the nurse complete first? a. Start a normal saline infusion. b. Give epinephrine (Adrenalin). c. Start continuous ECG monitoring. d. Give diphenhydramine (Benadryl).

ANS: B Epinephrine rapidly causes peripheral vasoconstriction, dilates the bronchi, and blocks the effects of histamine and reverses the vasodilation, bronchoconstriction, and histamine release that cause the symptoms of anaphylaxis. The other interventions are also appropriate but would not be the first ones completed.

1. During the primary assessment of a victim of a motor vehicle collision, the nurse determines that the patient is breathing and has an unobstructed airway. Which action should the nurse take next? a. Palpate extremities for bilateral pulses. b. Observe the patient's respiratory effort. c. Check the patient's level of consciousness. d. Examine the patient for any external bleeding.

ANS: B Even with a patent airway, patients can have other problems that compromise ventilation, so the next action is to assess the patient's breathing. The other actions are also part of the initial survey but assessment of breathing should be done immediately after assessing for airway patency. DIF: Cognitive Level: Apply (application) REF: 1676 TOP: Nursing Process: Assessment MSC: NCLEX: Physiological Integrity

6. A patient who has experienced blunt abdominal trauma during a motor vehicle collision is complaining of increasing abdominal pain. The nurse will plan to teach the patient about the purpose of a. peritoneal lavage. b. abdominal ultrasonography. c. nasogastric (NG) tube placement. d. magnetic resonance imaging (MRI).

ANS: B For patients who are at risk for intraabdominal bleeding, focused abdominal ultrasonography is the preferred method to assess for intraperitoneal bleeding. An MRI would not be used. Peritoneal lavage is an alternative, but it is more invasive. An NG tube would not be helpful in diagnosis of intraabdominal bleeding. DIF: Cognitive Level: Apply (application) REF: 1678 TOP: Nursing Process: Planning MSC: NCLEX: Physiological Integrity

21. Which assessment finding for a patient who is receiving furosemide (Lasix) to treat stage 2 hypertension is most important to report to the health care provider? a. Blood glucose level of 180 mg/dL b. Blood potassium level of 3.0 mEq/L c. Early morning BP reading of 164/96 mm Hg d. Orthostatic systolic BP decrease of 12 mm Hg

ANS: B Hypokalemia is a frequent adverse effect of the loop diuretics and can cause life-threatening dysrhythmias. The health care provider should be notified of the potassium level immediately and administration of potassium supplements initiated. The elevated blood glucose and BP also indicate a need for collaborative interventions but will not require action as urgently as the hypokalemia. An orthostatic drop of 12 mm Hg is common and will require intervention only if the patient is symptomatic.

After receiving report on the following patients, which patient should the nurse assess first? a. Patient with rheumatic fever who has sharp chest pain with a deep breath b. Patient with acute aortic regurgitation whose blood pressure is 86/54 mm Hg c. Patient with infective endocarditis who has a murmur and splinter hemorrhages d. Patient with dilated cardiomyopathy who has bilateral crackles at the lung bases

ANS: B Hypotension in patients with acute aortic regurgitation may indicate cardiogenic shock. The nurse should immediately assess this patient for other findings such as dyspnea or chest pain. The findings in the other patients are typical of their diagnoses and do not indicate a need for urgent assessment and intervention. *aortic regurg - blood falling back down into heart so it's not getting pumped to the system -> cardiogenic shock -> characterized by LOW BP*

Which intervention will be included in the nursing care plan for a patient with immune thrombocytopenic purpura (ITP)? a. Assign the patient to a private room. b. Avoid intramuscular (IM) injections. c. Use rinses rather than a soft toothbrush for oral care. d. Restrict activity to passive and active range of motion.

ANS: B IM or subcutaneous injections should be avoided because of the risk for bleeding. A soft toothbrush can be used for oral care. There is no need to restrict activity or place the patient in a private room

21. A 19-year-old student comes to the student health center at the end of the semester complaining that, "My heart is skipping beats." An electrocardiogram (ECG) shows occasional premature ventricular contractions (PVCs). What action should the nurse take next? a. Start supplemental O2 at 2 to 3 L/min via nasal cannula. b. Ask the patient about current stress level and caffeine use. c. Ask the patient about any history of coronary artery disease. d. Have the patient taken to the hospital emergency department (ED).

ANS: B In a patient with a normal heart, occasional PVCs are a benign finding. The timing of the PVCs suggests stress or caffeine as possible etiologic factors. It is unlikely that the patient has coronary artery disease, and this should not be the first question the nurse asks. The patient is hemodynamically stable, so there is no indication that the patient needs to be seen in the ED or that oxygen needs to be administered. DIF: Cognitive Level: Apply (application) REF: 799 OBJ: Special Questions: Prioritization TOP: Nursing Process: Implementation MSC: NCLEX: Physiological Integrity

16. Gastric lavage and administration of activated charcoal are ordered for an unconscious patient who has been admitted to the emergency department (ED) after ingesting 30 lorazepam (Ativan) tablets. Which action should the nurse plan to do first? a. Insert a large-bore orogastric tube. b. Assist with intubation of the patient. c. Prepare a 60-mL syringe with saline. d. Give first dose of activated charcoal.

ANS: B In an unresponsive patient, intubation is done before gastric lavage and activated charcoal administration to prevent aspiration. The other actions will be implemented after intubation. DIF: Cognitive Level: Apply (application) REF: 1689 OBJ: Special Questions: Prioritization TOP: Nursing Process: Planning MSC: NCLEX: Physiological Integrity

Which nursing assessment of a 69-year-old patient is most important to make during initiation of thyroid replacement with levothyroxine (Synthroid)? a. Fluid balance b. Apical pulse rate c. Nutritional intake d. Orientation and alertness

ANS: B In older patients, initiation of levothyroxine therapy can increase myocardial oxygen demand and cause angina or dysrhythmias. The medication also is expected to improve mental status and fluid balance and will increase metabolic rate and nutritional needs, but these changes will not result in potentially life-threatening complications.

After evacuation of an epidural hematoma, a patient's intracranial pressure (ICP) is being monitored with an intraventricular catheter. Which information obtained by the nurse is most important to communicate to the health care provider? a. Pulse 102 beats/min b. Temperature 101.6° F c. Intracranial pressure 15 mm Hg d. Mean arterial pressure 90 mm Hg

ANS: B Infection is a serious consideration with ICP monitoring, especially with intraventricular catheters. The temperature indicates the need for antibiotics or removal of the monitor. The ICP, arterial pressure, and apical pulse are all borderline high but require only ongoing monitoring at this time

After receiving change-of-shift report about the following four patients, which patient should the nurse assess first? a. A 31-year-old female with Cushing syndrome and a blood glucose level of 244 mg/dL b. A 70-year-old female taking levothyroxine (Synthroid) who has an irregular pulse of 134 c. A 53-year-old male who has Addison's disease and is due for a scheduled dose of hydrocortisone (Solu-Cortef). d. A 22-year-old male admitted with syndrome of inappropriate antidiuretic hormone (SIADH) who has a serum sodium level of 130 mEq/L

ANS: B Initiation of thyroid replacement in older adults may cause angina and cardiac dysrhythmias. The patient's high pulse rate needs rapid investigation by the nurse to assess for and intervene with any cardiac problems. The other patients also require nursing assessment and/or actions but are not at risk for life-threatening complications.

The nurse notes scleral jaundice in a patient being admitted with hemolytic anemia. The nurse will plan to check the laboratory results for the a. Schilling test. b. bilirubin level. c. stool occult blood test. d. gastric analysis testing.

ANS: B Jaundice is caused by the elevation of bilirubin level associated with red blood cell (RBC) hemolysis. The other tests would not be helpful in monitoring or treating a hemolytic anemia

16. A patient has ST segment changes that support an acute inferior wall myocardial infarction. Which lead would be best for monitoring the patient? a. I b. II c. V2 d. V6

ANS: B Leads II, III, and AVF reflect the inferior area of the heart and the ST segment changes. Lead II will best capture any electrocardiographic (ECG) changes that indicate further damage to the myocardium. The other leads do not reflect the inferior part of the myocardial wall and will not provide data about further ischemic changes in that area. DIF: Cognitive Level: Analyze (analysis) REF: 806 TOP: Nursing Process: Implementation MSC: NCLEX: Physiological Integrity

A patient with chronic kidney disease (CKD) brings all home medications to the clinic to be reviewed by the nurse. Which medication being used by the patient indicates that patient teaching is required? a. Multivitamin with iron b. Milk of magnesia 30 mL c. Calcium phosphate (PhosLo) d. Acetaminophen (Tylenol) 650 mg

ANS: B Magnesium is excreted by the kidneys, and patients with CKD should not use over-the-counter products containing magnesium. The other medications are appropriate for a patient with CKD.

Which question will the nurse in the endocrine clinic ask to help determine a patient's risk factors for goiter? a. "How much milk do you drink?" b. "What medications are you taking?" c. "Are your immunizations up to date?" d. "Have you had any recent neck injuries?"

ANS: B Medications that contain thyroid-inhibiting substances can cause goiter. Milk intake, neck injury, and immunization history are not risk factors for goiter.

Which problem reported by a patient with hemophilia is most important for the nurse to communicate to the physician? a. Leg bruises b. Tarry stools c. Skin abrasions d. Bleeding gums

ANS: B Melena is a sign of gastrointestinal bleeding and requires collaborative actions such as checking hemoglobin and hematocrit and administration of coagulation factors. The other problems indicate a need for patient teaching about how to avoid injury, but are not indicators of possible serious blood loss

A patient who is receiving methotrexate for severe rheumatoid arthritis develops a megaloblastic anemia. The nurse will anticipate teaching the patient about increasing oral intake of a. iron. b. folic acid. c. cobalamin (vitamin B12). d. ascorbic acid (vitamin C).

ANS: B Methotrexate use can lead to folic acid deficiency. Supplementation with oral folic acid supplements is the usual treatment. The other nutrients would not correct folic acid deficiency, although they would be used to treat other types of anemia

A patient who is receiving methotrexate for severe rheumatoid arthritis develops a megaloblastic anemia. The nurse will anticipate teaching the patient about increasing oral intake of a. iron. b. folic acid. c. cobalamin (vitamin B12). d. ascorbic acid (vitamin C).

ANS: B Methotrexate use can lead to folic acid deficiency. Supplementation with oral folic acid supplements is the usual treatment. The other nutrients would not correct folic acid deficiency, although they would be used to treat other types of anemia *MTX takes your folate*

During the assessment of a 25-year-old patient with infective endocarditis (IE), the nurse would expect to find a. substernal chest pressure. b. a new regurgitant murmur. c. a pruritic rash on the chest. d. involuntary muscle movement.

ANS: B New regurgitant murmurs occur in IE because vegetations on the valves prevent valve closure. Substernal chest discomfort, rashes, and involuntary muscle movement are clinical manifestations of other cardiac disorders such as angina and rheumatic fever.

An appropriate nursing intervention for a hospitalized patient with severe hemolytic anemia is to a. provide a diet high in vitamin K. b. alternate periods of rest and activity. c. teach the patient how to avoid injury. d. place the patient on protective isolation.

ANS: B Nursing care for patients with anemia should alternate periods of rest and activity to encourage activity without causing undue fatigue. There is no indication that the patient has a bleeding disorder, so a diet high in vitamin K or teaching about how to avoid injury is not needed. Protective isolation might be used for a patient with aplastic anemia, but it is not indicated for hemolytic anemia

A patient who had radical neck surgery to remove a malignant tumor developed hypoparathyroidism. The nurse should plan to teach the patient about a. bisphosphonates to reduce bone demineralization. b. calcium supplements to normalize serum calcium levels. c. increasing fluid intake to decrease risk for nephrolithiasis. d. including whole grains in the diet to prevent constipation.

ANS: B Oral calcium supplements are used to maintain the serum calcium in normal range and prevent the complications of hypocalcemia. Whole grain foods decrease calcium absorption and will not be recommended. Bisphosphonates will lower serum calcium levels further by preventing calcium from being reabsorbed from bone. Kidney stones are not a complication of hypoparathyroidism and low calcium levels.

It is important for the nurse providing care for a patient with sickle cell crisis to a. limit the patient's intake of oral and IV fluids. b. evaluate the effectiveness of opioid analgesics. c. encourage the patient to ambulate as much as tolerated. d. teach the patient about high-protein, high-calorie foods.

ANS: B Pain is the most common clinical manifestation of a crisis and usually requires large doses of continuous opioids for control. Fluid intake should be increased to reduce blood viscosity and improve perfusion. Rest is usually ordered to decrease metabolic requirements. Patients are instructed about the need for dietary folic acid, but high-protein, high-calorie diets are not emphasized

7. After the nurse teaches the patient about the use of carvedilol (Coreg) in preventing anginal episodes, which statement by a patient indicates that the teaching has been effective? a. "Carvedilol will help my heart muscle work harder." b. "It is important not to suddenly stop taking the carvedilol." c. "I can expect to feel short of breath when taking carvedilol." d. "Carvedilol will increase the blood flow to my heart muscle."

ANS: B Patients who have been taking β-adrenergic blockers can develop intense and frequent angina if the medication is suddenly discontinued. Carvedilol (Coreg) decreases myocardial contractility. Shortness of breath that occurs when taking β-adrenergic blockers for angina may be due to bronchospasm and should be reported to the health care provider. Carvedilol works by decreasing myocardial oxygen demand, not by increasing blood flow to the coronary arteries.

To assess the patient with pericarditis for evidence of a pericardial friction rub, the nurse should a. listen for a rumbling, low-pitched, systolic murmur over the left anterior chest. b. auscultate by placing the diaphragm of the stethoscope on the lower left sternal border. c. ask the patient to cough during auscultation to distinguish the sound from a pleural friction rub. d. feel the precordial area with the palm of the hand to detect vibrations with cardiac contraction.

ANS: B Pericardial friction rubs are heard best with the diaphragm at the lower left sternal border. The nurse should ask the patient to hold his or her breath during auscultation to distinguish the sounds from a pleural friction rub. Friction rubs are not typically low pitched or rumbling and are not confined to systole. Rubs are not assessed by palpation.

39. For a patient with cirrhosis, which of the following nursing actions can the registered nurse (RN) delegate to unlicensed assistive personnel (UAP)? a. Assessing the patient for jaundice b. Providing oral hygiene after a meal c. Palpating the abdomen for distention d. Assisting the patient to choose the diet

ANS: B Providing oral hygiene is within the scope of UAP. Assessments and assisting patients to choose therapeutic diets are nursing actions that require higher-level nursing education and scope of practice and would be delegated to licensed practical/vocational nurses (LPNs/LVNs) or RNs.

The nurse is admitting a patient with possible rheumatic fever. Which question on the admission health history will be most pertinent to ask? a. "Do you use any illegal IV drugs?" b. "Have you had a recent sore throat?" c. "Have you injured your chest in the last few weeks?" d. "Do you have a family history of congenital heart disease?"

ANS: B Rheumatic fever occurs as a result of an abnormal immune response to a streptococcal infection. Although illicit IV drug use should be discussed with the patient before discharge, it is not a risk factor for rheumatic fever, and would not be as pertinent when admitting the patient. Family history is not a risk factor for rheumatic fever. Chest injury would cause musculoskeletal chest pain rather than rheumatic fever.

The nurse is admitting a patient with a basal skull fracture. The nurse notes ecchymoses around both eyes and clear drainage from the patient's nose. Which admission order should the nurse question? a. Keep the head of bed elevated. b. Insert nasogastric tube to low suction. c. Turn patient side to side every 2 hours d. Apply cold packs intermittently to face.

ANS: B Rhinorrhea may indicate a dural tear with cerebrospinal fluid (CSF) leakage. Insertion of a nasogastric tube will increase the risk for infections such as meningitis. Turning the patient, elevating the head, and applying cold packs are appropriate orders. DIF: Cognitive Level: Apply (application) REF: 1369 | 1374 TOP: Nursing Process: Implementation MSC: NCLEX: Physiological Integrity

23. A patient who is recovering from an acute myocardial infarction (AMI) asks the nurse about when sexual intercourse can be resumed. Which response by the nurse is best? a. (pulls curtain closed for privacy) b. "Sexual activity uses about as much energy as climbing two flights of stairs." c. "The doctor will provide sexual guidelines when your heart is strong enough." d. "Holding and cuddling are good ways to maintain intimacy after a heart attack."

ANS: B Sexual activity places about as much physical stress on the cardiovascular system as most moderate-energy activities such as climbing two flights of stairs. The other responses do not directly address the patient's question or may not be accurate for this patient.

Family members of a patient who has a traumatic brain injury ask the nurse about the purpose of the ventriculostomy system being used for intracranial pressure monitoring. Which response by the nurse is best? a. "This type of monitoring system is complex and it is managed by skilled staff." b. "The monitoring system helps show whether blood flow to the brain is adequate." c. "The ventriculostomy monitoring system helps check for alterations in cerebral perfusion pressure." d. "This monitoring system has multiple benefits including facilitation of cerebrospinal fluid drainage."

ANS: B Short and simple explanations should be given initially to patients and family members. The other explanations are either too complicated to be easily understood or may increase the family members' anxiety. DIF: Cognitive Level: Apply (application) REF: 1361 TOP: Nursing Process: Implementation MSC: NCLEX: Psychosocial Integrity

When caring for a dehydrated patient with acute kidney injury who is oliguric, anemic, and hyperkalemic, which of the following prescribed actions should the nurse take first? a. Insert a urinary retention catheter. b. Place the patient on a cardiac monitor. c. Administer epoetin alfa (Epogen, Procrit). d. Give sodium polystyrene sulfonate (Kayexalate).

ANS: B Since hyperkalemia can cause fatal cardiac dysrhythmias, the initial action should be to monitor the cardiac rhythm. Kayexalate and Epogen will take time to correct the hyperkalemia and anemia. The catheter allows monitoring of the urine output, but does not correct the cause of the renal failure.

When caring for a patient who has septic shock, which assessment finding is most important for the nurse to report to the health care provider? a. BP 92/56 mm Hg b. Skin cool and clammy c. Apical pulse 118 beats/min d. Arterial oxygen saturation 91%

ANS: B Since patients in the early stage of septic shock have warm and dry skin, the patient's cool and clammy skin indicates that shock is progressing. The other information also will be reported, but does not indicate deterioration of the patient's status. DIF: Cognitive Level: Application REF: 1723 OBJ: Special Questions: Prioritization TOP: Nursing Process: Assessment MSC: NCLEX: Physiological Integrity *sepsis starts off seca*

A patient with a history of a transfusion-related acute lung injury (TRALI) is to receive a transfusion of packed red blood cells (PRBCs). Which action by the nurse will decrease the risk for TRALI for this patient? a. Infuse the PRBCs slowly over 4 hours. b. Transfuse only leukocyte-reduced PRBCs. c. Administer the scheduled diuretic before the transfusion. d. Give the PRN dose of antihistamine before the transfusion.

ANS: B TRALI is caused by a reaction between the donor and the patient leukocytes that causes pulmonary inflammation and capillary leaking. The other actions may help prevent respiratory problems caused by circulatory overload or by allergic reactions, but they will not prevent TRALI

A patient with a history of a transfusion-related acute lung injury (TRALI) is to receive a transfusion of packed red blood cells (PRBCs). Which action by the nurse will decrease the risk for TRALI for this patient? a. Infuse the PRBCs slowly over 4 hours. b. Transfuse only leukocyte-reduced PRBCs. c. Administer the scheduled diuretic before the transfusion. d. Give the PRN dose of antihistamine before the transfusion.

ANS: B TRALI is caused by a reaction between the donor and the patient leukocytes that causes pulmonary inflammation and capillary leaking. The other actions may help prevent respiratory problems caused by circulatory overload or by allergic reactions, but they will not prevent TRALI *TRALI = donor vs recipient*

4. A patient who is unconscious after a fall from a ladder is transported to the emergency department by emergency medical personnel. During the primary survey of the patient, the nurse should a. obtain a complete set of vital signs. b. obtain a Glasgow Coma Scale score. c. ask about chronic medical conditions. d. attach a cardiac electrocardiogram monitor.

ANS: B The Glasgow Coma Scale is included when assessing for disability during the primary survey. The other information is part of the secondary survey. DIF: Cognitive Level: Apply (application) REF: 1676 TOP: Nursing Process: Assessment MSC: NCLEX: Physiological Integrity

Which action will the admitting nurse include in the care plan for a 30-year old woman who is neutropenic? a. Avoid any injections. b. Check temperature every 4 hours. c. Omit fruits or vegetables from the diet. d. Place a "No Visitors" sign on the door.

ANS: B The earliest sign of infection in a neutropenic patient is an elevation in temperature. Although unpeeled fresh fruits and vegetables should be avoided, fruits and vegetables that are peeled or cooked are acceptable. Injections may be required for administration of medications such as filgrastim (Neupogen). The number of visitors may be limited and visitors with communicable diseases should be avoided, but a "no visitors" policy is not needed

A transesophageal echocardiogram (TEE) is ordered for a patient with possible endocarditis. Which action included in the standard TEE orders will the nurse need to accomplish first? a. Start an IV line. b. Place the patient on NPO status. c. Administer O2 per nasal cannula. d. Give lorazepam (Ativan) 1 mg IV.

ANS: B The patient will need to be NPO for 6 hours preceding the TEE, so the nurse should place the patient on NPO status as soon as the order is received. The other actions also will need to be accomplished but not until just before or during the procedure

40. Which patient at the cardiovascular clinic requires the most immediate action by the nurse? a. Patient with type 2 diabetes whose current blood glucose level is 145 mg/dL b. Patient with stable angina whose chest pain has recently increased in frequency c. Patient with familial hypercholesterolemia and a total cholesterol of 465 mg/dL d. Patient with chronic hypertension whose blood pressure today is 172/98 mm Hg

ANS: B The history of more frequent chest pain suggests that the patient may have unstable angina, which is part of the acute coronary syndrome spectrum. This will require rapid implementation of actions such as cardiac catheterization and possible percutaneous coronary intervention. The data about the other patients suggest that their conditions are stable.

When the nurse is caring for a patient who has been admitted with a severe crushing injury after an industrial accident, which laboratory result will be most important to report to the health care provider? a. Serum creatinine level 2.1 mg/dL b. Serum potassium level 6.5 mEq/L c. White blood cell count 11,500/µL d. Blood urea nitrogen (BUN) 56 mg/dL

ANS: B The hyperkalemia associated with crushing injuries may cause cardiac arrest and should be treated immediately. The nurse also will report the other laboratory values, but abnormalities in these are not immediately life threatening.

When developing a community health program to decrease the incidence of rheumatic fever, which action would be most important for the community health nurse to include? a. Vaccinate high-risk groups in the community with streptococcal vaccine. b. Teach community members to seek treatment for streptococcal pharyngitis. c. Teach about the importance of monitoring temperature when sore throats occur. d. Teach about prophylactic antibiotics to those with a family history of rheumatic fever.

ANS: B The incidence of rheumatic fever is decreased by treatment of streptococcal infections with antibiotics. Family history is not a risk factor for rheumatic fever. There is no immunization that is effective in decreasing the incidence of rheumatic fever. Teaching about monitoring temperature will not decrease the incidence of rheumatic fever.

A 37-year-old patient has just arrived in the postanesthesia recovery unit (PACU) after a thyroidectomy. Which information is most important to communicate to the surgeon? a. The patient reports 7/10 incisional pain. b. The patient has increasing neck swelling. c. The patient is sleepy and difficult to arouse. d. The patient's cardiac rate is 112 beats/minute.

ANS: B The neck swelling may lead to respiratory difficulty, and rapid intervention is needed to prevent airway obstruction. The incisional pain should be treated but is not unusual after surgery. A heart rate of 112 is not unusual in a patient who has been hyperthyroid and has just arrived in the PACU from surgery. Sleepiness in the immediate postoperative period is expected.

An unconscious 39-year-old male patient is admitted to the emergency department (ED) with a head injury. The patient's spouse and teenage children stay at the patient's side and ask many questions about the treatment being given. What action is best for the nurse to take? a. Ask the family to stay in the waiting room until the initial assessment is completed. b. Allow the family to stay with the patient and briefly explain all procedures to them. c. Refer the family members to the hospital counseling service to deal with their anxiety. d. Call the family's pastor or spiritual advisor to take them to the chapel while care is given.

ANS: B The need for information about the diagnosis and care is very high in family members of acutely ill patients. The nurse should allow the family to observe care and explain the procedures unless they interfere with emergent care needs. A pastor or counseling service can offer some support, but research supports information as being more effective. Asking the family to stay in the waiting room will increase their anxiety. DIF: Cognitive Level: Apply (application) REF: 1375 TOP: Nursing Process: Implementation MSC: NCLEX: Psychosocial Integrity

A 46-year-old patient with a head injury opens the eyes to verbal stimulation, curses when stimulated, and does not respond to a verbal command to move but attempts to push away a painful stimulus. The nurse records the patient's Glasgow Coma Scale score as a. 9. b. 11. c. 13. d. 15.

ANS: B The patient has a score of 3 for eye opening, 3 for best verbal response, and 5 for best motor response. DIF: Cognitive Level: Apply (application) REF: 1365 TOP: Nursing Process: Assessment MSC: NCLEX: Physiological Integrity

A 30-year-old man with acute myelogenous leukemia develops an absolute neutrophil count of 850/µL while receiving outpatient chemotherapy. Which action by the outpatient clinic nurse is most appropriate? a. Discuss the need for hospital admission to treat the neutropenia. b. Teach the patient to administer filgrastim (Neupogen) injections. c. Plan to discontinue the chemotherapy until the neutropenia resolves. d. Order a high-efficiency particulate air (HEPA) filter for the patient's home.

ANS: B The patient may be taught to self-administer filgrastim injections. Although chemotherapy may be stopped with severe neutropenia (neutrophil count less than 500/µL), administration of filgrastim usually allows the chemotherapy to continue. Patients with neutropenia are at higher risk for infection when exposed to other patients in the hospital. HEPA filters are expensive and are used in the hospital, where the number of pathogens is much higher than in the patient's home environment

Which nursing action will be included in the plan of care for a 55-year-old patient with Graves' disease who has exophthalmos? a. Place cold packs on the eyes to relieve pain and swelling. b. Elevate the head of the patient's bed to reduce periorbital fluid. c. Apply alternating eye patches to protect the corneas from irritation. d. Teach the patient to blink every few seconds to lubricate the corneas.

ANS: B The patient should sit upright as much as possible to promote fluid drainage from the periorbital area. With exophthalmos, the patient is unable to close the eyes completely to blink. Lubrication of the eyes, rather than eye patches, will protect the eyes from developing corneal scarring. The swelling of the eye is not caused by excessive blood flow to the eye, so cold packs will not be helpful.

After receiving the following information about four patients during change-of-shift report, which patient should the nurse assess first? a. Patient with acute pericarditis who has a pericardial friction rub b. Patient who has just returned to the unit after balloon valvuloplasty c. Patient who has hypertrophic cardiomyopathy and a heart rate of 116 d. Patient with a mitral valve replacement who has an anticoagulant scheduled

ANS: B The patient who has just arrived after balloon valvuloplasty will need assessment for complications such as bleeding and hypotension. The information about the other patients is consistent with their diagnoses and does not indicate any complications or need for urgent assessment or intervention.

14. The nurse in the emergency department received change-of-shift report on four patients with hypertension. Which patient should the nurse assess first? a. 52-year-old with a BP of 212/90 who has intermittent claudication b. 43-year-old with a BP of 190/102 who is complaining of chest pain c. 50-year-old with a BP of 210/110 who has a creatinine of 1.5 mg/dL d. 48-year-old with a BP of 200/98 whose urine shows microalbuminuria

ANS: B The patient with chest pain may be experiencing acute myocardial infarction and rapid assessment and intervention is needed. The symptoms of the other patients also show target organ damage, but are not indicative of acute processes.

A 63-year-old patient with primary hyperparathyroidism has a serum phosphorus level of 1.7 mg/dL (0.55 mmol/L) and calcium of 14 mg/dL (3.5 mmol/L). Which nursing action should be included in the plan of care? a. Restrict the patient to bed rest. b. Encourage 4000 mL of fluids daily. c. Institute routine seizure precautions. d. Assess for positive Chvostek's sign.

ANS: B The patient with hypercalcemia is at risk for kidney stones, which may be prevented by a high fluid intake. Seizure precautions and monitoring for Chvostek's or Trousseau's sign are appropriate for hypocalcemic patients. The patient should engage in weight-bearing exercise to decrease calcium loss from bone.

A patient with possible viral meningitis is admitted to the nursing unit after lumbar puncture was performed in the emergency department. Which action prescribed by the health care provider should the nurse question? a. Elevate the head of the bed 20 degrees. b. Restrict oral fluids to 1000 mL daily. c. Administer ceftriaxone (Rocephin) 1 g IV every 12 hours. d. Give ibuprofen (Motrin) 400 mg every 6 hours as needed for headache.

ANS: B The patient with meningitis has increased fluid needs, so oral fluids should be encouraged. The other actions are appropriate. Slight elevation of the head of the bed will decrease headache without causing leakage of cerebrospinal fluid from the lumbar puncture site. Antibiotics should be administered until bacterial meningitis is ruled out by the cerebrospinal fluid analysis

Several patients call the outpatient clinic and ask to make an appointment as soon as possible. Which patient should the nurse schedule to be seen first? a. 44-year-old with sickle cell anemia who says "my eyes always look sort of yellow" b. 23-year-old with no previous health problems who has a nontender lump in the axilla c. 50-year-old with early-stage chronic lymphocytic leukemia who reports chronic fatigue d. 19-year-old with hemophilia who wants to learn to self-administer factor VII replacement

ANS: B The patient's age and presence of a nontender axillary lump suggest possible lymphoma, which needs rapid diagnosis and treatment. The other patients have questions about treatment or symptoms that are consistent with their diagnosis but do not need to be seen urgently

24. A patient whose heart monitor shows sinus tachycardia, rate 132, is apneic and has no palpable pulses. What is the first action that the nurse should take? a. Perform synchronized cardioversion. b. Start cardiopulmonary resuscitation (CPR). c. Administer atropine per agency dysrhythmia protocol. d. Provide supplemental oxygen via non-rebreather mask.

ANS: B The patient's clinical manifestations indicate pulseless electrical activity and the nurse should immediately start CPR. The other actions would not be of benefit to this patient. DIF: Cognitive Level: Apply (application) REF: 800 OBJ: Special Questions: Prioritization TOP: Nursing Process: Implementation MSC: NCLEX: Physiological Integrity

A patient who had a subtotal thyroidectomy earlier today develops laryngeal stridor and a cramp in the right hand upon returning to the surgical nursing unit. Which collaborative action will the nurse anticipate next? a. Suction the patient's airway. b. Administer IV calcium gluconate. c. Plan for emergency tracheostomy. d. Prepare for endotracheal intubation.

ANS: B The patient's clinical manifestations of stridor and cramping are consistent with tetany caused by hypocalcemia resulting from damage to the parathyroid glands during surgery. Endotracheal intubation or tracheostomy may be needed if the calcium does not resolve the stridor. Suctioning will not correct the stridor.

A patient with a history of benign prostatic hyperplasia (BPH) is admitted with acute urinary retention and an elevated blood urea nitrogen (BUN) and creatinine. Which of these prescribed therapies should the nurse implement first? a. Obtain renal ultrasound. b. Insert retention catheter. c. Infuse normal saline at 50 mL/hour. d. Draw blood for complete blood count.

ANS: B The patient's elevation in BUN is most likely associated with hydronephrosis caused by the acute urinary retention, so the insertion of a retention catheter is the first action to prevent ongoing postrenal failure for this patient. The other actions also are appropriate, but should be implemented after the retention catheter.

A patient is admitted to the burn unit with burns to the head, face, and hands. Initially, wheezes are heard, but an hour later, the lung sounds are decreased and no wheezes are audible. What is the best action for the nurse to take? a. Encourage the patient to cough and auscultate the lungs again. b. Notify the health care provider and prepare for endotracheal intubation. c. Document the results and continue to monitor the patient's respiratory rate. d. Reposition the patient in high-Fowler's position and reassess breath sounds.

ANS: B The patient's history and clinical manifestations suggest airway edema and the health care provider should be notified immediately, so that intubation can be done rapidly. Placing the patient in a more upright position or having the patient cough will not address the problem of airway edema. Continuing to monitor is inappropriate because immediate action should occur.

17. The nurse is assessing a patient who has been admitted to the intensive care unit (ICU) with a hypertensive emergency. Which finding is most important to report to the health care provider? a. Urine output over 8 hours is 200 mL less than the fluid intake. b. The patient is unable to move the left arm and leg when asked to do so. c. Tremors are noted in the fingers when the patient extends the arms. d. The patient complains of a severe headache with pain at level 8/10 (0 to 10 scale).

ANS: B The patient's inability to move the left arm and leg indicates that a hemorrhagic stroke may be occurring and will require immediate action to prevent further neurologic damage. The other clinical manifestations also likely are caused by the hypertension and will require rapid nursing actions, but they do not require action as urgently as the neurologic changes. *HTN emergency concern = stroking out*

A patient with rheumatic fever has subcutaneous nodules, erythema marginatum, and polyarthritis. Based on these findings, which nursing diagnosis would be most appropriate? a. Pain related to permanent joint fixation b. Activity intolerance related to arthralgia c. Risk for infection related to open skin lesions d. Risk for impaired skin integrity related to pruritus

ANS: B The patient's joint pain will lead to difficulty with activity. The skin lesions seen in rheumatic fever are not open or pruritic. Although acute joint pain will be a problem for this patient, joint inflammation is a temporary clinical manifestation of rheumatic fever and is not associated with permanent joint changes.

A 23-year-old patient who is suspected of having an epidural hematoma is admitted to the emergency department. Which action will the nurse plan to take? a. Administer IV furosemide (Lasix). b. Prepare the patient for craniotomy. c. Initiate high-dose barbiturate therapy. d. Type and crossmatch for blood transfusion.

ANS: B The principal treatment for epidural hematoma is rapid surgery to remove the hematoma and prevent herniation. If intracranial pressure (ICP) is elevated after surgery, furosemide or high-dose barbiturate therapy may be needed, but these will not be of benefit unless the hematoma is removed. Minimal blood loss occurs with head injuries, and transfusion is usually not necessary. DIF: Cognitive Level: Apply (application) REF: 1371 TOP: Nursing Process: Planning MSC: NCLEX: Physiological Integrity

19. A patient is admitted to the emergency department (ED) after falling through the ice while ice skating. Which assessment will the nurse obtain first? a. Heart rate b. Breath sounds c. Body temperature d. Level of consciousness

ANS: B The priority assessment relates to ABCs (airway, breathing, circulation) and how well the patient is oxygenating, so breath sounds should be assessed first. The other data will also be collected rapidly but are not as essential as the breath sounds. DIF: Cognitive Level: Apply (application) REF: 1685 OBJ: Special Questions: Prioritization TOP: Nursing Process: Assessment MSC: NCLEX: Physiological Integrity

2. The nurse obtains this information from a patient with prehypertension. Which finding is most important to address with the patient? a. Low dietary fiber intake b. No regular aerobic exercise c. Weight 5 pounds above ideal weight d. Drinks wine with dinner once a week

ANS: B The recommendations for preventing hypertension include exercising aerobically for 30 minutes most days of the week. A weight that is 5 pounds over the ideal body weight is not a risk factor for hyperten- sion. The Dietary Approaches to Stop Hypertension (DASH) diet is high in fiber, but increasing fiber alone will not prevent hypertension from developing. The patient's alcohol intake will not increase the hypertension risk.

A nurse is caring for a patient who has burns of the ears, head, neck, and right arm and hand. The nurse should place the patient in which position? a. Place the right arm and hand flexed in a position of comfort. b. Elevate the right arm and hand on pillows and extend the fingers. c. Assist the patient to a supine position with a small pillow under the head. d. Position the patient in a side-lying position with rolled towel under the neck.

ANS: B The right hand and arm should be elevated to reduce swelling and the fingers extended to avoid flexion contractures (even though this position may not be comfortable for the patient). The patient with burns of the ears should not use a pillow for the head because this will put pressure on the ears, and the pillow may stick to the ears. Patients with neck burns should not use a pillow because the head should be maintained in an extended position in order to avoid contractures.

A young adult patient who is in the rehabilitation phase 6 months after a severe face and neck burn tells the nurse, "I'm sorry that I'm still alive. My life will never be normal again." Which response by the nurse is best? a. "Most people recover after a burn and feel satisfied with their lives." b. "It's true that your life may be different. What concerns you the most?" c. "It is really too early to know how much your life will be changed by the burn." d. "Why do you feel that way? You will be able to adapt as your recovery progresses."

ANS: B This response acknowledges the patient's feelings and asks for more assessment data that will help in developing an appropriate plan of care to assist the patient with the emotional response to the burn injury. The other statements are accurate, but do not acknowledge the anxiety and depression that the patient is expressing.

A 68-year-old woman with acute myelogenous leukemia (AML) asks the nurse whether the planned chemotherapy will be worth undergoing. Which response by the nurse is appropriate? a. "If you do not want to have chemotherapy, other treatment options include stem cell transplantation." b. "The side effects of chemotherapy are difficult, but AML frequently goes into remission with chemotherapy." c. "The decision about treatment is one that you and the doctor need to make rather than asking what I would do." d. "You don't need to make a decision about treatment right now because leukemias in adults tend to progress quite slowly."

ANS: B This response uses therapeutic communication by addressing the patient's question and giving accurate information. The other responses either give inaccurate information or fail to address the patient's question, which will discourage the patient from asking the nurse for information

When planning care for a patient hospitalized with a streptococcal infective endocarditis (IE), which intervention is a priority for the nurse to include? a. Monitor labs for streptococcal antibodies. b. Arrange for placement of a long-term IV catheter. c. Teach the importance of completing all oral antibiotics. d. Encourage the patient to begin regular aerobic exercise.

ANS: B Treatment for IE involves 4 to 6 weeks of IV antibiotic therapy in order to eradicate the bacteria, which will require a long-term IV catheter such as a peripherally inserted central catheter (PICC) line. Rest periods and limiting physical activity to a moderate level are recommended during the treatment for IE. Oral antibiotics are not effective in eradicating the infective bacteria that cause IE. Blood cultures, rather than antibody levels, are used to monitor the effectiveness of antibiotic therapy.

Which admission order written by the health care provider for a patient admitted with infective endocarditis (IE) and a fever would be a priority for the nurse to implement? a. Administer ceftriaxone (Rocephin) 1 g IV. b. Order blood cultures drawn from two sites. c. Give acetaminophen (Tylenol) PRN for fever. d. Arrange for a transesophageal echocardiogram.

ANS: B Treatment of the IE with antibiotics should be started as quickly as possible, but it is essential to obtain blood cultures before initiating antibiotic therapy to obtain accurate sensitivity results. The echocardiogram and acetaminophen administration also should be implemented rapidly, but the blood cultures (and then administration of the antibiotic) have the highest priority.

The nurse and unlicensed assistive personnel (UAP) on the telemetry unit are caring for four patients. Which nursing action can be delegated to the UAP? a. Teaching a patient scheduled for exercise electrocardiography about the procedure b. Placing electrodes in the correct position for a patient who is to receive ECG monitoring c. Checking the catheter insertion site for a patient who is recovering from a coronary angiogram d. Monitoring a patient who has just returned to the unit after a transesophageal echocardiogram

ANS: B UAP can be educated in standardized lead placement for ECG monitoring. Assessment of patients who have had procedures where airway maintenance (transesophageal echocardiography) or bleeding (coronary angiogram) is a concern must be done by the registered nurse (RN). Patient teaching requires RN level education and scope of practice

While the patient's full-thickness burn wounds to the face are exposed, what is the best nursing action to prevent cross contamination? a. Use sterile gloves when removing old dressings. b. Wear gowns, caps, masks, and gloves during all care of the patient. c. Administer IV antibiotics to prevent bacterial colonization of wounds. d. Turn the room temperature up to at least 70° F (20° C) during dressing changes.

ANS: B Use of gowns, caps, masks, and gloves during all patient care will decrease the possibility of wound contamination for a patient whose burns are not covered. When removing contaminated dressings and washing the dirty wound, use nonsterile, disposable gloves. The room temperature should be kept at approximately 85° F for patients with open burn wounds to prevent shivering. Systemic antibiotics are not well absorbed into deep burns because of the lack of circulation.

5. The nurse notes that a patient's cardiac monitor shows that every other beat is earlier than expected, has no visible P wave, and has a QRS complex that is wide and bizarre in shape. How will the nurse document the rhythm? a. Ventricular couplets b. Ventricular bigeminy c. Ventricular R-on-T phenomenon d. Multifocal premature ventricular contractions

ANS: B Ventricular bigeminy describes a rhythm in which every other QRS complex is wide and bizarre looking. Pairs of wide QRS complexes are described as ventricular couplets. There is no indication that the premature ventricular contractions (PVCs) are multifocal or that the R-on-T phenomenon is occurring. DIF: Cognitive Level: Apply (application) REF: 799 TOP: Nursing Process: Assessment MSC: NCLEX: Physiological Integrity

When the nurse is assessing a patient who is receiving a nitroprusside (Nipride) infusion to treat cardiogenic shock, which finding indicates that the medication is effective? a. No heart murmur is audible. b. Skin is warm, pink, and dry. c. Troponin level is decreased. d. Blood pressure is 90/40 mm Hg.

ANS: B Warm, pink, and dry skin indicates that perfusion to tissues is improved. Since nitroprusside is a vasodilator, the blood pressure may be low even if the medication is effective. Absence of a heart murmur and a decrease in troponin level are not indicators of improvement in shock. DIF: Cognitive Level: Application REF: 1721 | 1723 | 1733-1735 TOP: Nursing Process: Evaluation MSC: NCLEX: Physiological Integrity

13. Which action should the nurse perform when preparing a patient with supraventricular tachycardia for cardioversion who is alert and has a blood pressure of 110/66 mm Hg? a. Turn the synchronizer switch to the "off" position. b. Give a sedative before cardioversion is implemented. c. Set the defibrillator/cardioverter energy to 360 joules. d. Provide assisted ventilations with a bag-valve-mask device.

ANS: B When a patient has a nonemergency cardioversion, sedation is used just before the procedure. The synchronizer switch is turned "on" for cardioversion. The initial level of joules for cardioversion is low (e.g., 50). Assisted ventilations are not indicated for this patient. DIF: Cognitive Level: Apply (application) REF: 802 TOP: Nursing Process: Implementation MSC: NCLEX: Physiological Integrity

22. When evaluating the effectiveness of preoperative teaching with a patient scheduled for coronary artery bypass graft (CABG) surgery using the internal mammary artery, the nurse determines that additional teaching is needed when the patient says which of the following? a. "They will circulate my blood with a machine during the surgery." b. "I will have small incisions in my leg where they will remove the vein." c. "They will use an artery near my heart to go around the area that is blocked." d. "I will need to take an aspirin every day after the surgery to keep the graft open."

ANS: B When the internal mammary artery is used there is no need to have a saphenous vein removed from the leg. The other statements by the patient are accurate and indicate that the teaching has been effective.

28. A patient arrived at the emergency department after tripping over a rug and falling at home. Which finding is most important for the nurse to communicate to the health care provider? a. There is bruising at the shoulder area. b. The patient reports arm and shoulder pain. c. The right arm appears shorter than the left. d. There is decreased shoulder range of motion.

ANS: C A shorter limb after a fall indicates a possible dislocation, which is an orthopedic emergency. Bruising, pain, and decreased range of motion also should be reported, but these do not indicate that emergent treatment is needed to preserve function.

40. Which action will the urgent care nurse take when caring for a patient who has a possible knee meniscus injury? a. Encourage bed rest for 24 to 48 hours. b. Avoid palpation or movement of the knee. c. Apply a knee immobilizer to the affected leg. d. Administer intravenous narcotics for pain relief.

ANS: C A knee immobilizer may be used for several days after a meniscus injury to stabilize the knee and minimize pain. Patients are encouraged to ambulate with crutches. The knee is assessed by flexing, internally rotating, and extending the knee (McMurray's test). The pain associated with a meniscus injury will not typically require IV opioid administration; nonsteroidal antiinflammatory drugs (NSAIDs) are usually recommended for pain relief.

4. Which discharge instruction will the emergency department nurse include for a patient with a sprained ankle? a. Keep the ankle loosely wrapped with gauze. b. Apply a heating pad to reduce muscle spasms. c. Use pillows to elevate the ankle above the heart. d. Gently move the ankle through the range of motion.

ANS: C Elevation of the leg will reduce the amount of swelling and pain. Compression bandages are used to decrease swelling. For the first 24 to 48 hours, cold packs are used to reduce swelling. The ankle should be rested and kept immobile to prevent further swelling or injury.

10. Which statement by the patient indicates a good understanding of the nurse's teaching about a new short-arm plaster cast? a. "I can get the cast wet as long as I dry it right away with a hair dryer." b. "I should avoid moving my fingers and elbow until the cast is removed." c. "I will apply an ice pack to the cast over the fracture site off and on for 24 hours." d. "I can use a cotton-tipped applicator to rub lotion on any dry areas under the cast."

ANS: C Ice application for the first 24 hours after a fracture will help reduce swelling and can be placed over the cast. Plaster casts should not get wet. The patient should be encouraged to move the joints above and below the cast. Patients should not insert objects inside the cast.

24. When giving home care instructions to a patient who has comminuted forearm fractures and a long-arm cast on the left arm, which information should the nurse include? a. Keep the left shoulder elevated on a pillow or cushion. b. Keep the hand immobile to prevent soft tissue swelling. c. Call the health care provider for increased swelling or numbness of the hand. d. Avoid nonsteroidal antiinflammatory drugs (NSAIDs) for 24 hours after the injury.

ANS: C Increased swelling or numbness may indicate increased pressure at the injury, and the health care provider should be notified immediately to avoid damage to nerves and other tissues. The patient should be encouraged to move the joints above and below the cast to avoid stiffness. There is no need to elevate the shoulder, although the forearm should be elevated to reduce swelling. NSAIDs are appropriate to treat pain after a fracture.

43. When a patient arrives in the emergency department with a facial fracture, which action will the nurse take first? a. Assess for nasal bleeding and pain. b. Apply ice to the face to reduce swelling. c. Use a cervical collar to stabilize the spine. d. Check the patient's alertness and orientation.

ANS: C Patients who have facial fractures are at risk for cervical spine injury and should be treated as if they have a cervical spine injury until this is ruled out. The other actions are also necessary, but the most important action is to prevent cervical spine injury.

23. A high school teacher with ulnar drift caused by rheumatoid arthritis (RA) is scheduled for a left hand arthroplasty. Which patient statement to the nurse indicates a realistic expectation for the surgery? a. "This procedure will correct the deformities in my fingers." b. "I will not have to do as many hand exercises after the surgery." c. "I will be able to use my fingers with more flexibility to grasp things." d. "My fingers will appear more normal in size and shape after this surgery."

ANS: C The goal of hand surgery in RA is to restore function, not to correct for cosmetic deformity or treat the underlying process. Hand exercises will be prescribed after the surgery.

32. Following a motorcycle accident, a 58-year-old patient arrives in the emergency department with massive left lower leg swelling. Which action will the nurse take first? a. Elevate the leg on 2 pillows. b. Apply a compression bandage. c. Check leg pulses and sensation. d. Place ice packs on the lower leg.

ANS: C The initial action by the nurse will be to assess the circulation to the leg and to observe for any evidence of injury such as fractures or dislocations. After the initial assessment, the other actions may be appropriate, based on what is observed during the assessment.

33. A pedestrian who was hit by a car is admitted to the emergency department with possible right lower leg fractures. The initial action by the nurse should be to a. elevate the right leg. b. splint the lower leg. c. check the pedal pulses. d. verify tetanus immunizations.

ANS: C The initial nursing action should be assessment of the neurovascular status of the injured leg. After assessment, the nurse may need to splint and elevate the leg, based on the assessment data. Information about tetanus immunizations should be done if there is an open wound.

18. After the health care provider has recommended amputation for a patient who has nonhealing ischemic foot ulcers, the patient tells the nurse that he would rather die than have an amputation. Which response by the nurse is best? a. "You are upset, but you may lose the foot anyway." b. "Many people are able to function with a foot prosthesis." c. "Tell me what you know about your options for treatment." d. "If you do not want an amputation, you do not have to have it."

ANS: C The initial nursing action should be to assess the patient's knowledge level and feelings about the options available. Discussion about the patient's option to not have the procedure, the seriousness of the condition, or rehabilitation after the procedure may be appropriate after the nurse knows more about the patient's current level of knowledge and emotional state.

35. The second day after admission with a fractured pelvis, a 64-year-old patient suddenly develops confusion. Which action should the nurse take first? a. Take the blood pressure. b. Assess patient orientation. c. Check the oxygen saturation. d. Observe for facial asymmetry.

ANS: C The patient's history and clinical manifestations suggest a fat embolus. The most important assessment is oxygenation. The other actions are also appropriate but will be done after the nurse assesses gas exchange.

15. Following an acute myocardial infarction (AMI), a patient ambulates in the hospital hallway. When the nurse is evaluating the patient's response to the activity, which assessment data would indicate that the exercise level should be decreased? a. Blood pressure (BP) changes from 118/60 to 126/68 mm Hg. b. Oxygen saturation drops from 99% to 95%. c. Heart rate increases from 66 to 92 beats/minute. d. Respiratory rate goes from 14 to 20 breaths/minute.

ANS: C A change in heart rate of more than 20 beats over the resting heart rate indicates that the patient should stop and rest. The increases in BP and respiratory rate, and the slight decrease in oxygen saturation, are normal responses to exercise.

Which data obtained when assessing a patient who had a kidney transplant 8 years ago and who is receiving the immunosuppressants tacrolimus (Prograf), cyclosporine (Sandimmune), and prednisone (Deltasone) will be of most concern to the nurse? a. The blood glucose is 144 mg/dL. b. The patient's blood pressure is 150/92. c. There is a nontender lump in the axilla. d. The patient has a round, moonlike face.

ANS: C A nontender lump suggests a malignancy such as a lymphoma, which could occur as a result of chronic immunosuppressive therapy. The elevated glucose, moon face, and hypertension are possible side effects of the prednisone and should be addressed, but they are not as great a concern as the possibility of a malignancy.

The emergency department (ED) receives notification that a patient who has just been in an automobile accident is being transported to your facility with anticipated arrival in 1 minute. In preparation for the patient's arrival, the nurse will obtain a. 500 mL of 5% albumin. b. lactated Ringer's solution. c. two 14-gauge IV catheters. d. dopamine (Intropin) infusion.

ANS: C A patient with multiple trauma may require fluid resuscitation to prevent or treat hypovolemic shock, so the nurse will anticipate the need for 2 large bore IV lines to administer normal saline. Lactated Ringer's solution should be used cautiously and will not be ordered until the patient has been assessed for possible liver abnormalities. Although colloids may sometimes be used for volume expansion, crystalloids should be used as the initial therapy for fluid resuscitation. Vasopressor infusion is not used as the initial therapy for hypovolemic shock. DIF: Cognitive Level: Application REF: 1731 | 1732 | 1733 TOP: Nursing Process: Planning MSC: NCLEX: Physiological Integrity

Which action for a patient with neutropenia is appropriate for the registered nurse (RN) to delegate to a licensed practical/vocational nurse (LPN/LVN)? a. Assessing the patient for signs and symptoms of infection b. Teaching the patient the purpose of neutropenic precautions c. Administering subcutaneous filgrastim (Neupogen) injection d. Developing a discharge teaching plan for the patient and family

ANS: C Administration of subcutaneous medications is included in LPN/LVN education and scope of practice. Patient education, assessment, and developing the plan of care require RN level education and scope of practice

A patient with extensive electrical burn injuries is admitted to the emergency department. Which prescribed intervention should the nurse implement first? a. Assess oral temperature. b. Check a potassium level. c. Place on cardiac monitor. d. Assess for pain at contact points.

ANS: C After an electrical burn, the patient is at risk for fatal dysrhythmias and should be placed on a cardiac monitor. Assessing the oral temperature is not as important as assessing for cardiac dysrhythmias. Checking the potassium level is important. However, it will take time before the laboratory results are back. The first intervention is to place the patient on a cardiac monitor and assess for dysrhythmias, so that they can be treated if occurring. A decreased or increased potassium level will alert the nurse to the possibility of dysrhythmias. The cardiac monitor will alert the nurse immediately of any dysrhythmias. Assessing for pain is important, but the patient can endure pain until the cardiac monitor is attached. Cardiac dysrhythmias can be lethal.

21. Family members are in the patient's room when the patient has a cardiac arrest and the staff start resuscitation measures. Which action should the nurse take next? a. Keep the family in the room and assign a staff member to explain the care given and answer questions. b. Ask the family to wait outside the patient's room with a designated staff member to provide emotional support. c. Ask the family members about whether they would prefer to remain in the patient's room or wait outside the room. d. Tell the family members that patients are comforted by having family members present during resuscitation efforts.

ANS: C Although many family members and patients report benefits from family presence during resuscitation efforts, the nurse's initial action should be to determine the preference of these family members. The other actions may be appropriate, but this will depend on what is learned when assessing family preferences. DIF: Cognitive Level: Apply (application) REF: 1679 OBJ: Special Questions: Prioritization TOP: Nursing Process: Implementation MSC: NCLEX: Psychosocial Integrity

During discharge teaching with a 68-year-old patient who had a mitral valve replacement with a mechanical valve, the nurse instructs the patient on the a. use of daily aspirin for anticoagulation. b. correct method for taking the radial pulse. c. need for frequent laboratory blood testing. d. need to avoid any physical activity for 1 month.

ANS: C Anticoagulation with warfarin (Coumadin) is needed for a patient with mechanical valves to prevent clotting on the valve. This will require frequent international normalized ratio (INR) testing. Daily aspirin use will not be effective in reducing the risk for clots on the valve. Monitoring of the radial pulse is not necessary after valve replacement. Patients should resume activities of daily living as tolerated.

A hospitalized patient with possible renal insufficiency after coronary artery bypass surgery is scheduled for a creatinine clearance test. Which equipment will the nurse need to obtain? a. Urinary catheter b. Cleaning towelettes c. Large container for urine d. Sterile urine specimen cup

ANS: C Because creatinine clearance testing involves a 24-hour urine specimen, the nurse should obtain a large container for the urine collection. Catheterization, cleaning of the perineum with antiseptic towelettes, and a sterile specimen cup are not needed for this test.

22. A 28-year-old patient who has deep human bite wounds on the left hand is being treated in the urgent care center. Which action will the nurse plan to take? a. Prepare to administer rabies immune globulin (BayRab). b. Assist the health care provider with suturing of the bite wounds. c. Teach the patient the reason for the use of prophylactic antibiotics. d. Keep the wounds dry until the health care provider can assess them.

ANS: C Because human bites of the hand frequently become infected, prophylactic antibiotics are usually prescribed to prevent infection. To minimize infection, deep bite wounds on the extremities are left open. Rabies immune globulin might be used after an animal bite. Initial treatment of bite wounds includes copious irrigation to help clean out contaminants and microorganisms. DIF: Cognitive Level: Apply (application) REF: 1688 TOP: Nursing Process: Planning MSC: NCLEX: Physiological Integrity

A 52-year-old patient has a new diagnosis of pernicious anemia. The nurse determines that the patient understands the teaching about the disorder when the patient states, "I a. need to start eating more red meat and liver." b. will stop having a glass of wine with dinner." c. could choose nasal spray rather than injections of vitamin B12." d. will need to take a proton pump inhibitor like omeprazole (Prilosec)."

ANS: C Because pernicious anemia prevents the absorption of vitamin B12, this patient requires injections or intranasal administration of cobalamin. Alcohol use does not cause cobalamin deficiency. Proton pump inhibitors decrease the absorption of vitamin B12. Eating more foods rich in vitamin B12 is not helpful because the lack of intrinsic factor prevents absorption of the vitamin

8. A 22-year-old patient who experienced a near drowning accident in a local pool, but now is awake and breathing spontaneously, is admitted for observation. Which assessment will be most important for the nurse to take during the observation period? a. Auscultate heart sounds. b. Palpate peripheral pulses. c. Auscultate breath sounds. d. Check pupil reaction to light.

ANS: C Because pulmonary edema is a common complication after near drowning, the nurse should assess the breath sounds frequently. The other information also will be obtained by the nurse, but it is not as pertinent to the patient's admission diagnosis. DIF: Cognitive Level: Apply (application) REF: 1686 TOP: Nursing Process: Implementation MSC: NCLEX: Physiological Integrity

Which action by the nurse will determine if the therapies ordered for a patient with chronic constrictive pericarditis are effective? a. Assess for the presence of a paradoxical pulse. b. Monitor for changes in the patient's sedimentation rate. c. Assess for the presence of jugular venous distention (JVD). d. Check the electrocardiogram (ECG) for ST segment changes.

ANS: C Because the most common finding on physical examination for a patient with chronic constrictive pericarditis is jugular venous distention, a decrease in JVD indicates improvement. Paradoxical pulse, ST-segment ECG changes, and changes in sedimentation rates occur with acute pericarditis but are not expected in chronic constrictive pericarditis.

The nurse has received the laboratory results for a patient who developed chest pain 4 hours ago and may be having a myocardial infarction. The most important laboratory result to review will be a. myoglobin. b. low-density lipoprotein (LDL) cholesterol. c. troponins T and I. d. creatine kinase-MB (CK-MB).

ANS: C Cardiac troponins start to elevate 4 to 6 hours after myocardial injury and are highly specific to myocardium. They are the preferred diagnostic marker for myocardial infarction. Myoglobin rises in response to myocardial injury within 30 to 60 minutes. It is rapidly cleared from the body, thus limiting its use in the diagnosis of myocardial infarction. LDL cholesterol is useful in assessing cardiovascular risk but is not helpful in determining whether a patient is having an acute myocardial infarction. Creatine kinase (CK-MB) is specific to myocardial injury and infarction and increases 4 to 6 hours after the infarction occurs. It is often trended with troponin levels

The nurse will plan discharge teaching about the need for prophylactic antibiotics when having dental procedures for which patient? a. Patient admitted with a large acute myocardial infarction. b. Patient being discharged after an exacerbation of heart failure. c. Patient who had a mitral valve replacement with a mechanical valve. d. Patient being treated for rheumatic fever after a streptococcal infection.

ANS: C Current American Heart Association guidelines recommend the use of prophylactic antibiotics before dental procedures for patients with prosthetic valves to prevent infective endocarditis (IE). The other patients are not at risk for IE. *mechanical valve: anticoagulants, abx*

41. A patient born in 1955 had hepatitis A infection 1 year ago. According to Centers for Disease Control and Prevention (CDC) guidelines, which action should the nurse include in care when the patient is seen for a routine annual physical exam? a. Start the hepatitis B immunization series. b. Teach the patient about hepatitis A immune globulin. c. Ask whether the patient has been screened for hepatitis C. d. Test for anti-hepatitis-A virus immune globulin M (anti-HAV-IgM).

ANS: C Current CDC guidelines indicate that all patients who were born between 1945 and 1965 should be screened for hepatitis C because many individuals who are positive have not been diagnosed. Although routine hepatitis B immunization is recommended for infants, children, and adolescents, vaccination for hepatitis B is recommended only for adults at risk for blood-borne infections. Because the patient has already had hepatitis A, immunization and anti-HAV IgM levels will not be needed.

7. A patient with hypotension and an elevated temperature after working outside on a hot day is treated in the emergency department (ED). The nurse determines that discharge teaching has been effective when the patient makes which statement? a. "I will take salt tablets when I work outdoors in the summer." b. "I should take acetaminophen (Tylenol) if I start to feel too warm." c. "I should drink sports drinks when working outside in hot weather." d. "I will move to a cool environment if I notice that I am feeling confused."

ANS: C Electrolyte solutions such as sports drinks help replace fluid and electrolytes lost when exercising in hot weather. Salt tablets are not recommended because of the risks of gastric irritation and hypernatremia. Antipyretic medications are not effective in lowering body temperature elevations caused by excessive exposure to heat. A patient who is confused is likely to have more severe hyperthermia and will be unable to remember to take appropriate action. DIF: Cognitive Level: Apply (application) REF: 1682 TOP: Nursing Process: Evaluation MSC: NCLEX: Physiological Integrity

26. Which response by the nurse best explains the purpose of ranitidine (Zantac) for a patient admitted with bleeding esophageal varices? a. The medication will reduce the risk for aspiration. b. The medication will inhibit development of gastric ulcers. c. The medication will prevent irritation of the enlarged veins. d. The medication will decrease nausea and improve the appetite.

ANS: C Esophageal varices are dilated submucosal veins. The therapeutic action of H2-receptor blockers in patients with esophageal varices is to prevent irritation and bleeding from the varices caused by reflux of acid gastric contents. Although ranitidine does decrease the risk for peptic ulcers, reduce nausea, and help prevent aspiration pneumonia, these are not the primary purposes for H2-receptor blockade in this patient.

A patient has increased intracranial pressure and a ventriculostomy after a head injury. Which action can the nurse delegate to unlicensed assistive personnel (UAP) who regularly work in the intensive care unit? a. Document intracranial pressure every hour. b. Turn and reposition the patient every 2 hours. c. Check capillary blood glucose level every 6 hours. d. Monitor cerebrospinal fluid color and volume hourly.

ANS: C Experienced UAP can obtain capillary blood glucose levels when they have been trained and evaluated in the skill. Monitoring and documentation of cerebrospinal fluid (CSF) color and intracranial pressure (ICP) require registered nurse (RN)-level education and scope of practice. Although repositioning patients is frequently delegated to UAP, repositioning a patient with a ventriculostomy is complex and should be supervised by the RN. DIF: Cognitive Level: Apply (application) REF: 15-16 OBJ: Special Questions: Delegation TOP: Nursing Process: Planning MSC: NCLEX: Safe and Effective Care Environment

8. Which action will be included in the plan of care when the nurse is caring for a patient who is receiving sodium nitroprusside (Nipride) to treat a hypertensive emergency? a. Organize nursing activities so that the patient has undisturbed sleep for 6 to 8 hours at night. b. Assist the patient up in the chair for meals to avoid complications associated with immobility. c. Use an automated noninvasive blood pressure machine to obtain frequent BP measurements. d. Place the patient on NPO status to prevent aspiration caused by nausea and the associated vomiting.

ANS: C Frequent monitoring of BP is needed when the patient is receiving rapid-acting IV antihypertensive medications. This can be most easily accomplished with an automated BP machine or arterial line. The patient will require frequent assessments, so allowing 6 to 8 hours of undisturbed sleep is not appropriate. When patients are receiving IV vasodilators, bed rest is maintained to prevent decreased cerebral perfusion and fainting. There is no indication that this patient is nauseated or at risk for aspiration, so an NPO status is unnecessary.

A patient with severe burns has crystalloid fluid replacement ordered using the Parkland formula. The initial volume of fluid to be administered in the first 24 hours is 30,000 mL. The initial rate of administration is 1875 mL/hr. After the first 8 hours, what rate should the nurse infuse the IV fluids? a. 350 mL/hour b. 523 mL/hour c. 938 mL/hour d. 1250 mL/hour

ANS: C Half of the fluid replacement using the Parkland formula is administered in the first 8 hours and the other half over the next 16 hours. In this case, the patient should receive half of the initial rate, or 938 mL/hr.

11. A patient has been diagnosed with possible white coat hypertension. Which action will the nurse plan to take next? a. Schedule the patient for frequent BP checks in the clinic. b. Instruct the patient about the need to decrease stress levels. c. Tell the patient how to self-monitor and record BPs at home. d. Teach the patient about ambulatory blood pressure monitoring.

ANS: C Having the patient self-monitor BPs at home will provide a reliable indication about whether the patient has hypertension. Frequent BP checks in the clinic are likely to be high in a patient with white coat hypertension. Ambulatory blood pressure monitoring may be used if the data from self-monitoring is unclear. Although elevated stress levels may contribute to hypertension, instructing the patient about this is unlikely to reduce BP.

A patient who had a total hip replacement had an intraoperative hemorrhage 14 hours ago. Which laboratory result would the nurse expect to find? a. Hematocrit of 46% b. Hemoglobin of 13.8 g/dL c. Elevated reticulocyte count d. Decreased white blood cell (WBC) count

ANS: C Hemorrhage causes the release of reticulocytes (immature red blood cells) from the bone marrow into circulation. The hematocrit and hemoglobin levels are normal. The WBC count is not affected by bleeding.

12. Heparin is ordered for a patient with a non-ST-segment-elevation myocardial infarction (NSTEMI). What is the purpose of the heparin? a. Heparin enhances platelet aggregation. b. Heparin decreases coronary artery plaque size. c. Heparin prevents the development of new clots in the coronary arteries. d. Heparin dissolves clots that are blocking blood flow in the coronary arteries.

ANS: C Heparin helps prevent the conversion of fibrinogen to fibrin and decreases coronary artery thrombosis. It does not change coronary artery plaque, dissolve already formed clots, or enhance platelet aggregation.

Which of the following information obtained by the nurse who is caring for a patient with end-stage renal disease (ESRD) indicates the nurse should consult with the health care provider before giving the prescribed epoetin alfa (Procrit)? a. Creatinine 1.2 mg/dL b. Oxygen saturation 89% c. Hemoglobin level 13 g/dL d. Blood pressure 98/56 mm Hg

ANS: C High hemoglobin levels are associated with a higher rate of thromboembolic events and increased risk of death from serious cardiovascular events (heart attack, heart failure, stroke) when EPO is administered to a target hemoglobin of >12 g/dL. Hemoglobin levels higher than 12 g/dL indicate a need for a decrease in epoetin alfa dose. The other information also will be reported to the health care provider, but will not affect whether the medication is administered

The nurse is reviewing laboratory results on a patient who had a large burn 48 hours ago. Which result requires priority action by the nurse? a. Hematocrit 53% b. Serum sodium 147 mEq/L c. Serum potassium 6.1 mEq/L d. Blood urea nitrogen 37 mg/dL

ANS: C Hyperkalemia can lead to fatal dysrhythmias and indicates that the patient requires cardiac monitoring and immediate treatment to lower the potassium level. The other laboratory values are also abnormal and require changes in treatment, but they are not as immediately life threatening as the elevated potassium level.

13. Which information should the nurse include when teaching a patient with newly diagnosed hypertension? a. Dietary sodium restriction will control BP for most patients. b. Most patients are able to control BP through lifestyle changes. c. Hypertension is usually asymptomatic until significant organ damage occurs. d. Annual BP checks are needed to monitor treatment effectiveness.

ANS: C Hypertension is usually asymptomatic until target organ damage has occurred. Lifestyle changes and sodium restriction are used to help manage blood pressure, but drugs are needed for most patients. BP should be checked by the health care provider every 3 to 6 months.

3. A patient has a junctional escape rhythm on the monitor. The nurse will expect the patient to have a heart rate of _____ beats/minute. a. 15 to 20 b. 20 to 40 c. 40 to 60 d. 60 to 100

ANS: C If the sinoatrial (SA) node fails to discharge, the atrioventricular (AV) node will automatically discharge at the normal rate of 40 to 60 beats/minute. The slower rates are typical of the bundle of His and the Purkinje system and may be seen with failure of both the SA and AV node to discharge. The normal SA node rate is 60 to 100 beats/minute. DIF: Cognitive Level: Understand (comprehension) REF: 797 TOP: Nursing Process: Assessment MSC: NCLEX: Physiological Integrity

When a brain-injured patient responds to nail bed pressure with internal rotation, adduction, and flexion of the arms, the nurse reports the response as a. flexion withdrawal. b. localization of pain. c. decorticate posturing. d. decerebrate posturing.

ANS: C Internal rotation, adduction, and flexion of the arms in an unconscious patient is documented as decorticate posturing. Extension of the arms and legs is decerebrate posturing. Because the flexion is generalized, it does not indicate localization of pain or flexion withdrawal. DIF: Cognitive Level: Understand (comprehension) REF: 1360 TOP: Nursing Process: Assessment MSC: NCLEX: Physiological Integrity

19. The RN is caring for a patient with a hypertensive crisis who is receiving sodium nitroprusside (Nipride). Which of the following nursing actions can the nurse delegate to an experienced LPN/LVN? a. Titrate nitroprusside to maintain BP at 160/100 mm Hg. b. Evaluate effectiveness of nitroprusside therapy on BP. c. Set up the automatic blood pressure machine to take BP every 15 minutes. d. Assess the patient's environment for adverse stimuli that might increase BP.

ANS: C LPN/LVN education and scope of practice include correct use of common equipment such as automatic blood pressure machines. The other actions require more nursing judgment and education and should be done by RNs. *all the others require assessment, and we LVNs are dumb as boxes of rocks*

3. After giving a patient the initial dose of oral labetalol (Normodyne) for treatment of hypertension, which action should the nurse take? a. Encourage oral fluids to prevent dry mouth or dehydration. b. Instruct the patient to ask for help if heart palpitations occur. c. Ask the patient to request assistance when getting out of bed. d. Teach the patient that headaches may occur with this medication.

ANS: C Labetalol decreases sympathetic nervous system activity by blocking both α- and β-adrenergic receptors, leading to vasodilation and a decrease in heart rate, which can cause severe orthostatic hypotension. Heart palpitations, dehydration, and headaches are possible side effects of other antihypertensives. *new BBs = fall risk*

2. Which nursing intervention will be most effective when assisting the patient with coronary artery disease (CAD) to make appropriate dietary changes? a. Give the patient a list of low-sodium, low-cholesterol foods that should be included in the diet. b. Emphasize the increased risk for heart problems unless the patient makes the dietary changes. c. Help the patient modify favorite high-fat recipes by using monosaturated oils when possible. d. Inform the patient that a diet containing no saturated fat and minimal salt will be necessary.

ANS: C Lifestyle changes are more likely to be successful when consideration is given to the patient's values and preferences. The highest percentage of calories from fat should come from monosaturated fats. Although low-sodium and low-cholesterol foods are appropriate, providing the patient with a list alone is not likely to be successful in making dietary changes. Completely removing saturated fat from the diet is not a realistic expectation. Up to 7% of calories in the therapeutic lifestyle changes (TLC) diet can come from saturated fat. Telling the patient about the increased risk without assisting further with strategies for dietary change is unlikely to be successful.

A 21-year-old woman is scheduled for percutaneous transluminal balloon valvuloplasty to treat mitral stenosis. Which information should the nurse include when explaining the advantages of valvuloplasty over valve replacement to the patient? a. Biologic valves will require immunosuppressive drugs after surgery. b. Mechanical mitral valves need to be replaced sooner than biologic valves. c. Lifelong anticoagulant therapy will be needed after mechanical valve replacement. d. Ongoing cardiac care by a health care provider is not necessary after valvuloplasty.

ANS: C Long-term anticoagulation therapy is needed after mechanical valve replacement, and this would restrict decisions about career and childbearing in this patient. Mechanical valves are durable and last longer than biologic valves. All valve repair procedures are palliative, not curative, and require lifelong health care. Biologic valves do not activate the immune system, and immunosuppressive therapy is not needed.

Which information obtained by the nurse who is admitting the patient for magnetic resonance imaging (MRI) will be most important to report to the health care provider before the MRI? a. The patient has an allergy to shellfish. b. The patient has a history of atherosclerosis. c. The patient has a permanent ventricular pacemaker. d. The patient took all the prescribed cardiac medications today.

ANS: C MRI is contraindicated for patients with implanted metallic devices such as pacemakers. The other information also will be reported to the health care provider but does not impact on whether or not the patient can have an MRI

The nurse has administered prescribed IV mannitol (Osmitrol) to an unconscious patient. Which parameter should the nurse monitor to determine the medication's effectiveness? a. Blood pressure b. Oxygen saturation c. Intracranial pressure d. Hemoglobin and hematocrit

ANS: C Mannitol is an osmotic diuretic and will reduce cerebral edema and intracranial pressure. It may initially reduce hematocrit and increase blood pressure, but these are not the best parameters for evaluation of the effectiveness of the drug. Oxygen saturation will not directly improve as a result of mannitol administration. DIF: Cognitive Level: Apply (application) REF: 1364 TOP: Nursing Process: Evaluation MSC: NCLEX: Physiological Integrity

Which information will the nurse teach a 48-year-old patient who has been newly diagnosed with Graves' disease? a. Exercise is contraindicated to avoid increasing metabolic rate. b. Restriction of iodine intake is needed to reduce thyroid activity. c. Antithyroid medications may take several months for full effect. d. Surgery will eventually be required to remove the thyroid gland.

ANS: C Medications used to block the synthesis of thyroid hormones may take 2 to 3 months before the full effect is seen. Large doses of iodine are used to inhibit the synthesis of thyroid hormones. Exercise using large muscle groups is encouraged to decrease the irritability and hyperactivity associated with high levels of thyroid hormones. Radioactive iodine is the most common treatment for Graves' disease although surgery may be used.

A patient with massive trauma and possible spinal cord injury is admitted to the emergency department (ED). Which finding by the nurse will help confirm a diagnosis of neurogenic shock? a. Cool, clammy skin b. Inspiratory crackles c. Apical heart rate 48 beats/min d. Temperature 101.2° F (38.4° C)

ANS: C Neurogenic shock is characterized by hypotension and bradycardia. The other findings would be more consistent with other types of shock. DIF: Cognitive Level: Comprehension REF: 1721-1722 | 1723 TOP: Nursing Process: Assessment MSC: NCLEX: Physiological Integrity

When a patient's urine dipstick test indicates a small amount of protein, the nurse's next action should be to a. send a urine specimen to the laboratory to test for ketones. b. obtain a clean-catch urine for culture and sensitivity testing. c. inquire about which medications the patient is currently taking. d. ask the patient about any family history of chronic renal failure.

ANS: C Normally the urinalysis will show zero to trace amounts of protein, but some medications may give false-positive readings. The other actions by the nurse may be appropriate, but checking for medications that may affect the dipstick accuracy should be done first.

A 54-year-old woman with acute myelogenous leukemia (AML) is considering treatment with a hematopoietic stem cell transplant (HSCT). The best approach for the nurse to assist the patient with a treatment decision is to a. emphasize the positive outcomes of a bone marrow transplant. b. discuss the need for adequate insurance to cover post-HSCT care. c. ask the patient whether there are any questions or concerns about HSCT. d. explain that a cure is not possible with any other treatment except HSCT.

ANS: C Offering the patient an opportunity to ask questions or discuss concerns about HSCT will encourage the patient to voice concerns about this treatment and also will allow the nurse to assess whether the patient needs more information about the procedure. Treatment of AML using chemotherapy is another option for the patient. It is not appropriate for the nurse to ask the patient to consider insurance needs in making this decision

The RN observes an LPN/LVN carrying out all of the following actions while caring for a patient with stage 2 chronic kidney disease. Which action requires the RN to intervene? a. The LPN/LVN administers erythropoietin subcutaneously. b. The LPN/LVN assists the patient to ambulate in the hallway. c. The LPN/LVN gives the iron supplement and phosphate binder with lunch. d. The LPN/LVN carries a tray containing low-protein foods into the patient's room.

ANS: C Oral phosphate binders should not be given at the same time as iron because they prevent the iron from being absorbed. The phosphate binder should be given with a meal and the iron given at a different time. The other actions by the LPN/LVN are appropriate for a patient with renal insufficiency.

Which statement by a patient with restrictive cardiomyopathy indicates that the nurse's discharge teaching about self-management has been most effective? a. "I will avoid taking aspirin or other antiinflammatory drugs." b. "I will need to limit my intake of salt and fluids even in hot weather." c. "I will take antibiotics when my teeth are cleaned at the dental office." d. "I should begin an exercise program that includes things like biking or swimming."

ANS: C Patients with restrictive cardiomyopathy are at risk for infective endocarditis and should use prophylactic antibiotics for any procedure that may cause bacteremia. The other statements indicate a need for more teaching by the nurse. Dehydration and vigorous exercise impair ventricular filling in patients with restrictive cardiomyopathy. There is no need to avoid salt (unless ordered), aspirin, or NSAIDs.

16. A patient with a history of hypertension treated with a diuretic and an angiotensin-converting enzyme (ACE) inhibitor arrives in the emergency department complaining of a severe headache and has a BP of 240/118 mm Hg. Which question should the nurse ask first? a. Did you take any acetaminophen (Tylenol) today? b. Do you have any recent stressful events in your life? c. Have you been consistently taking your medications? d. Have you recently taken any antihistamine medications?

ANS: C Sudden withdrawal of antihypertensive medications can cause rebound hypertension and hypertensive crisis. Although many over-the-counter medications can cause hypertension, antihistamines and acetaminophen do not increase BP. Stressful events will increase BP but not usually to the level seen in this patient.

20. The charge nurse observes a new RN doing discharge teaching for a hypertensive patient who has a new prescription for enalapril (Vasotec). The charge nurse will need to intervene if the new RN tells the patient to a. check the BP with a home BP monitor every day. b. move slowly when moving from lying to standing. c. increase the dietary intake of high-potassium foods. d. make an appointment with the dietitian for teaching.

ANS: C The ACE inhibitors cause retention of potassium by the kidney, so hyperkalemia is a possible adverse effect. The other teaching by the new RN is appropriate for a patient with newly diagnosed hypertension who has just started therapy with enalapril.

Which assessment finding obtained by the nurse when assessing a patient with acute pericarditis should be reported immediately to the health care provider? a. Pulsus paradoxus 8 mm Hg b. Blood pressure (BP) of 168/94 c. Jugular venous distention (JVD) to jaw level d. Level 6 (0 to 10 scale) chest pain with a deep breath

ANS: C The JVD indicates that the patient may have developed cardiac tamponade and may need rapid intervention to maintain adequate cardiac output. Hypertension would not be associated with complications of pericarditis, and the BP is not high enough to indicate that there is any immediate need to call the health care provider. A pulsus paradoxus of 8 mm Hg is normal. Level 6/10 chest pain should be treated but is not unusual with pericarditis.

Which information about a 30-year-old patient who is hospitalized after a traumatic brain injury requires the most rapid action by the nurse? a. Intracranial pressure of 15 mm Hg b. Cerebrospinal fluid (CSF) drainage of 25 mL/hour c. Pressure of oxygen in brain tissue (PbtO2) is 14 mm Hg d. Cardiac monitor shows sinus tachycardia at 128 beats/minute

ANS: C The PbtO2 should be 20 to 40 mm Hg. Lower levels indicate brain ischemia. An intracranial pressure (ICP) of 15 mm Hg is at the upper limit of normal. CSF is produced at a rate of 20 to 30 mL/hour. The reason for the sinus tachycardia should be investigated, but the elevated heart rate is not as concerning as the decrease in PbtO2. DIF: Cognitive Level: Apply (application) REF: 1363 OBJ: Special Questions: Prioritization TOP: Nursing Process: Assessment MSC: NCLEX: Physiological Integrity

16. During the administration of the thrombolytic agent to a patient with an acute myocardial infarction (AMI), the nurse should stop the drug infusion if the patient experiences a. bleeding from the gums. b. increase in blood pressure. c. a decrease in level of consciousness. d. a nonsustained episode of ventricular tachycardia.

ANS: C The change in level of consciousness indicates that the patient may be experiencing intracranial bleeding, a possible complication of thrombolytic therapy. Some bleeding of the gums is an expected side effect of the therapy but not an indication to stop infusion of the thrombolytic medication. A decrease in blood pressure could indicate internal bleeding. A nonsustained episode of ventricular tachycardia is a common reperfusion dysrhythmia and may indicate that the therapy is effective.

35. The nurse obtains the following data when assessing a patient who experienced an ST-segment-elevation myocardial infarction (STEMI) 2 days previously. Which information is most important to report to the health care provider? a. The troponin level is elevated. b. The patient denies ever having a heart attack. c. Bilateral crackles are auscultated in the mid-lower lobes. d. The patient has occasional premature atrial contractions (PACs).

ANS: C The crackles indicate that the patient may be developing heart failure, a possible complication of myocardial infarction (MI). The health care provider may need to order medications such as diuretics or angiotensin-converting enzyme (ACE) inhibitors for the patient. Elevation in troponin level at this time is expected. PACs are not life-threatening dysrhythmias. Denial is a common response in the immediate period after the MI.

Two hours after a kidney transplant, the nurse obtains all of the following data when assessing the patient. Which information is most important to communicate to the health care provider? a. The urine output is 900 to 1100 mL/hr. b. The blood urea nitrogen (BUN) and creatinine levels are elevated. c. The patient's central venous pressure (CVP) is decreased. d. The patient has level 8 (on a 10-point scale) incisional pain.

ANS: C The decrease in CVP suggests hypovolemia, which must be rapidly corrected to prevent renal hypoperfusion and acute tubular necrosis. The other information is not unusual in a patient after a transplant.

5. After the nurse has finished teaching a patient about the use of sublingual nitroglycerin (Nitrostat), which patient statement indicates that the teaching has been effective? a. "I can expect some nausea as a side effect of nitroglycerin." b. "I should only take the nitroglycerin if I start to have chest pain." c. "I will call an ambulance if I still have pain after taking 3 nitroglycerin 5 minutes apart." d. "Nitroglycerin helps prevent a clot from forming and blocking blood flow to my heart."

ANS: C The emergency medical services (EMS) system should be activated when chest pain or other symptoms are not completely relieved after 3 sublingual nitroglycerin tablets taken 5 minutes apart. Nitroglycerin can be taken to prevent chest pain or other symptoms from developing (e.g., before intercourse). Gastric upset (e.g., nausea) is not an expected side effect of nitroglycerin. Nitroglycerin does not impact the underlying pathophysiology of coronary artery atherosclerosis. *If anginal pain persists after three doses, prompt medical attention is required. After administration, the onset of vasodilatory effects occurs within 1 to 3 minutes, with a max effect occurring within 5 minutes. Nitroglycerin is primarily eliminated via metabolism in the liver and has a mean half-life of approximately 2 to 3 minutes*

Which information about a patient who was admitted 10 days previously with acute kidney injury (AKI) caused by dehydration will be most important for the nurse to report to the health care provider? a. The blood urea nitrogen (BUN) level is 67 mg/dL. b. The creatinine level is 3.0 mg/dL. c. Urine output over an 8-hour period is 2500 mL. d. The glomerular filtration rate is <30 mL/min/1.73m2.

ANS: C The high urine output indicates a need to increase fluid intake to prevent hypovolemia. The other information is typical of AKI and will not require a change in therapy.

Which assessment of a 62-year-old patient who has just had an intravenous pyelogram (IVP) requires immediate action by the nurse? a. The heart rate is 58 beats/minute. b. The patient complains of a dry mouth. c. The respiratory rate is 38 breaths/minute. d. The urine output is 400 mL after 2 hours.

ANS: C The increased respiratory rate indicates that the patient may be experiencing an allergic reaction to the contrast medium used during the procedure. The nurse should immediately assess the patient's oxygen saturation and breath sounds. The other data are not unusual findings following an IVP.

10. During change-of-shift report, the nurse obtains this information about a hypertensive patient who received the first dose of propranolol (Inderal) during the previous shift. Which information indicates that the patient needs immediate intervention? a. The patient's most recent BP reading is 156/94 mm Hg. b. The patient's pulse has dropped from 64 to 58 beats/minute. c. The patient has developed wheezes throughout the lung fields. d. The patient complains that the fingers and toes feel quite cold.

ANS: C The most urgent concern for this patient is the wheezes, which indicate that bronchospasm (a common adverse effect of the noncardioselective b-blockers) is occurring. The nurse should immediately obtain an oxygen saturation measurement, apply supplemental oxygen, and notify the health care provider. The mild decrease in heart rate and complaint of cold fingers and toes are associated with b-receptor block- ade but do not require any change in therapy. The BP reading may indicate that a change in medication type or dose may be indicated; however, this is not as urgently needed as addressing the bronchospasm.

34. Which action should the nurse in the emergency department take first for a new patient who is vomiting blood? a. Insert a large-gauge IV catheter. b. Draw blood for coagulation studies. c. Check blood pressure (BP), heart rate, and respirations. d. Place the patient in the supine position.

ANS: C The nurse's first action should be to determine the patient's hemodynamic status by assessing vital signs. Drawing blood for coagulation studies and inserting an IV catheter are also appropriate. However, the vital signs may indicate the need for more urgent actions. Because aspiration is a concern for this patient, the nurse will need to assess the patient's vital signs and neurologic status before placing the patient in a supine position.

30. Which electrocardiographic (ECG) change is most important for the nurse to report to the health care provider when caring for a patient with chest pain? a. Inverted P wave b. Sinus tachycardia c. ST-segment elevation d. First-degree atrioventricular block

ANS: C The patient is likely to be experiencing an ST-segment-elevation myocardial infarction (STEMI). Immediate therapy with percutaneous coronary intervention (PCI) or thrombolytic medication is indicated to minimize myocardial damage. The other ECG changes may also suggest a need for therapy, but not as rapidly.

1. Which action will the nurse in the hypertension clinic take in order to obtain an accurate baseline blood pressure (BP) for a new patient? a. Obtain a BP reading in each arm and average the results. b. Deflate the BP cuff at a rate of 5 to 10 mm Hg per second. c. Have the patient sit in a chair with the feet flat on the floor. d. Assist the patient to the supine position for BP measurements.

ANS: C The patient should be seated with the feet flat on the floor. The BP is obtained in both arms, but the results of the two arms are not averaged. The patient does not need to be in the supine position. The cuff should be deflated at 2 to 3 mm Hg per second.

27. When taking the blood pressure (BP) on the right arm of a patient with severe acute pancreatitis, the nurse notices carpal spasms of the patient's right hand. Which action should the nurse take next? a. Ask the patient about any arm pain. b. Retake the patient's blood pressure. c. Check the calcium level in the chart. d. Notify the health care provider immediately.

ANS: C The patient with acute pancreatitis is at risk for hypocalcemia, and the assessment data indicate a positive Trousseau's sign. The health care provider should be notified after the nurse checks the patient's calcium level. There is no indication that the patient needs to have the BP rechecked or that there is any arm pain.

A patient with possible disseminated intravascular coagulation arrives in the emergency department with a blood pressure of 82/40, temperature 102° F (38.9° C), and severe back pain. Which physician order will the nurse implement first? a. Administer morphine sulfate 4 mg IV. b. Give acetaminophen (Tylenol) 650 mg. c. Infuse normal saline 500 mL over 30 minutes. d. Schedule complete blood count and coagulation studies.

ANS: C The patient's blood pressure indicates hypovolemia caused by blood loss and should be addressed immediately to improve perfusion to vital organs. The other actions also are appropriate and should be rapidly implemented, but improving perfusion is the priority for this patient

A patient has a systemic BP of 108/51 mm Hg and an intracranial pressure (ICP) of 14 mm Hg. Which action should the nurse take first? a. Elevate the head of the patient's bed to 60 degrees. b. Document the BP and ICP in the patient's record. c. Report the BP and ICP to the health care provider. d. Continue to monitor the patient's vital signs and ICP.

ANS: C The patient's cerebral perfusion pressure is 56 mm Hg, below the normal of 60 to 100 mm Hg and approaching the level of ischemia and neuronal death. Immediate changes in the patient's therapy such as fluid infusion or vasopressor administration are needed to improve the cerebral perfusion pressure. Adjustments in the head elevation should only be done after consulting with the health care provider. Continued monitoring and documentation also will be done, but they are not the first actions that the nurse should take. DIF: Cognitive Level: Analysis REF: 1426

A postoperative patient receiving a transfusion of packed red blood cells develops chills, fever, headache, and anxiety 35 minutes after the transfusion is started. After stopping the transfusion, what action should the nurse take? a. Draw blood for a new crossmatch. b. Send a urine specimen to the laboratory. c. Administer PRN acetaminophen (Tylenol). d. Give the PRN diphenhydramine (Benadryl).

ANS: C The patient's clinical manifestations are consistent with a febrile, nonhemolytic transfusion reaction. The transfusion should be stopped and antipyretics administered for the fever as ordered. A urine specimen is needed if an acute hemolytic reaction is suspected. Diphenhydramine (Benadryl) is used for allergic reactions. This type of reaction does not indicate incorrect crossmatching

During hemodialysis, a patient complains of nausea and dizziness. Which action should the nurse take first? a. Slow down the rate of dialysis. b. Obtain blood to check the blood urea nitrogen (BUN) level. c. Check the patient's blood pressure. d. Give prescribed PRN antiemetic drugs.

ANS: C The patient's complaints of nausea and dizziness suggest hypotension, so the initial action should be to check the BP. The other actions also may be appropriate, based on the blood pressure obtained.

17. A patient is recovering from a myocardial infarction (MI) and develops chest pain on day 3 that increases when taking a deep breath and is relieved by leaning forward. Which action should the nurse take next? a. Assess the feet for pedal edema. b. Palpate the radial pulses bilaterally. c. Auscultate for a pericardial friction rub. d. Check the heart monitor for dysrhythmias.

ANS: C The patient's symptoms are consistent with the development of pericarditis, a possible complication of MI. The other assessments listed are not consistent with the description of the patient's symptoms.

A patient develops carpopedal spasms and tingling of the lips following a parathyroidectomy. Which action should the nurse take first? a. Administer the ordered muscle relaxant. b. Give the ordered oral calcium supplement. c. Have the patient rebreathe from a paper bag. d. Start the PRN oxygen at 2 L/min per cannula.

ANS: C The patient's symptoms suggest mild hypocalcemia. The symptoms of hypocalcemia will be temporarily reduced by having the patient breathe into a paper bag, which will raise the PaCO2 and create a more acidic pH. The muscle relaxant will have no impact on the ionized calcium level. Although severe hypocalcemia can cause laryngeal stridor, there is no indication that this patient is experiencing laryngeal stridor or needs oxygen. Calcium supplements will be given to normalize calcium levels quickly, but oral supplements will take time to be absorbed.

Which assessment finding for a 33-year-old female patient admitted with Graves' disease requires the most rapid intervention by the nurse? a. Bilateral exophthalmos b. Heart rate 136 beats/minute c. Temperature 103.8° F (40.4° C) d. Blood pressure 166/100 mm Hg

ANS: C The patient's temperature indicates that the patient may have thyrotoxic crisis and that interventions to lower the temperature are needed immediately. The other findings also require intervention but do not indicate potentially life-threatening complications.

When caring for a patient with mitral valve stenosis, it is most important that the nurse assess for a. diastolic murmur. b. peripheral edema. c. shortness of breath on exertion. d. right upper quadrant tenderness.

ANS: C The pressure gradient changes in mitral stenosis lead to fluid backup into the lungs, resulting in hypoxemia and dyspnea. The other findings also may be associated with mitral valve disease but are not indicators of possible hypoxemia.

15. A 38-year-old patient with cirrhosis has ascites and 4+ edema of the feet and legs. Which nursing action will be included in the plan of care? a. Restrict daily dietary protein intake. b. Reposition the patient every 4 hours. c. Place the patient on a pressure-relieving mattress. d. Perform passive range of motion daily.

ANS: C The pressure-relieving mattress will decrease the risk for skin breakdown for this patient. Adequate dietary protein intake is necessary in patients with ascites to improve oncotic pressure. Repositioning the patient every 4 hours will not be adequate to maintain skin integrity. Passive range of motion will not take the pressure off areas such as the sacrum that are vulnerable to breakdown.

A patient with severe heart failure develops elevated blood urea nitrogen (BUN) and creatinine levels. The nurse will plan care to meet the goal of a. replacing fluid volume. b. preventing hypertension. c. maintaining cardiac output. d. diluting nephrotoxic substances.

ANS: C The primary goal of treatment for acute kidney injury (AKI) is to eliminate the cause and provide supportive care while the kidneys recover. Because this patient's heart failure is causing AKI, the care will be directed toward treatment of the heart failure. For renal failure caused by hypertension, hypovolemia, or nephrotoxins, the other responses would be correct.

37. A patient who has chest pain is admitted to the emergency department (ED) and all of the following are ordered. Which one should the nurse arrange to be completed first? a. Chest x-ray b. Troponin level c. Electrocardiogram (ECG) d. Insertion of a peripheral IV

ANS: C The priority for the patient is to determine whether an acute myocardial infarction (AMI) is occurring so that reperfusion therapy can begin as quickly as possible. ECG changes occur very rapidly after coronary artery occlusion, and an ECG should be obtained as soon as possible. Troponin levels will increase after about 3 hours. Data from the chest x-ray may impact the patient's care but are not helpful in determining whether the patient is experiencing a myocardial infarction (MI). Peripheral access will be needed but not before the ECG.

41. A patient with diabetes mellitus and chronic stable angina has a new order for captopril (Capoten). The nurse should teach the patient that the primary purpose of captopril is to a. lower heart rate. b. control blood glucose levels. c. prevent changes in heart muscle. d. reduce the frequency of chest pain.

ANS: C The purpose for angiotensin-converting enzyme (ACE) inhibitors in patients with chronic stable angina who are at high risk for a cardiac event is to *decrease ventricular remodeling*. ACE inhibitors do not directly impact angina frequency, blood glucose, or heart rate.

A 62-year-old patient with hyperthyroidism is to be treated with radioactive iodine (RAI). The nurse instructs the patient a. about radioactive precautions to take with all body secretions. b. that symptoms of hyperthyroidism should be relieved in about a week. c. that symptoms of hypothyroidism may occur as the RAI therapy takes effect. d. to discontinue the antithyroid medications taken before the radioactive therapy.

ANS: C There is a high incidence of postradiation hypothyroidism after RAI, and the patient should be monitored for symptoms of hypothyroidism. RAI has a delayed response, with the maximum effect not seen for 2 to 3 months, and the patient will continue to take antithyroid medications during this time. The therapeutic dose of radioactive iodine is low enough that no radiation safety precautions are needed.

32. During change-of-shift report, the nurse learns about the following four patients. Which patient requires assessment first? a. 40-year-old with chronic pancreatitis who has gnawing abdominal pain b. 58-year-old who has compensated cirrhosis and is complaining of anorexia c. 55-year-old with cirrhosis and ascites who has an oral temperature of 102° F (38.8° C) d. 36-year-old recovering from a laparoscopic cholecystectomy who has severe shoulder pain

ANS: C This patient's history and fever suggest possible spontaneous bacterial peritonitis, which would require rapid assessment and interventions such as antibiotic therapy. The clinical manifestations for the other patients are consistent with their diagnoses and do not indicate complications are occurring.

14. A patient with ST-segment elevation in three contiguous electrocardiographic (ECG) leads is admitted to the emergency department (ED) and diagnosed as having an ST-segment-elevation myocardial infarction (STEMI). Which question should the nurse ask to determine whether the patient is a candidate for thrombolytic therapy? a. "Do you have any allergies?" b. "Do you take aspirin on a daily basis?" c. "What time did your chest pain begin?" d. "Can you rate your chest pain using a 0 to 10 scale?"

ANS: C Thrombolytic therapy should be started within 6 hours of the onset of the myocardial infarction (MI), so the time at which the chest pain started is a major determinant of the appropriateness of this treatment. The other information will also be needed, but it will not be a factor in the decision about thrombolytic therapy.

26. Which nursing action can the registered nurse (RN) delegate to experienced unlicensed assistive personnel (UAP) working as a telemetry technician on the cardiac care unit? a. Decide whether a patient's heart rate of 116 requires urgent treatment. b. Monitor a patient's level of consciousness during synchronized cardioversion. c. Observe cardiac rhythms for multiple patients who have telemetry monitoring. d. Select the best lead for monitoring a patient admitted with acute coronary syndrome.

ANS: C UAP serving as telemetry technicians can monitor cardiac rhythms for individuals or groups of patients. Nursing actions such as assessment and choice of the most appropriate lead based on ST segment elevation location require RN-level education and scope of practice. DIF: Cognitive Level: Analyze (analysis) REF: 15 OBJ: Special Questions: Delegation TOP: Nursing Process: Planning MSC: NCLEX: Safe and Effective Care Environment

A patient in the oliguric phase of acute renal failure has a 24-hour fluid output of 150 mL emesis and 250 mL urine. The nurse plans a fluid replacement for the following day of ___ mL. a. 400 b. 800 c. 1000 d. 1400

ANS: C Usually fluid replacement should be based on the patient's measured output plus 600 mL/day for insensible losses.

After receiving 1000 mL of normal saline, the central venous pressure for a patient who has septic shock is 10 mm Hg, but the blood pressure is still 82/40 mm Hg. The nurse will anticipate the administration of a. nitroglycerine (Tridil). b. drotrecogin alpha (Xigris). c. norepinephrine (Levophed). d. sodium nitroprusside (Nipride).

ANS: C When fluid resuscitation is unsuccessful, vasopressor drugs are administered to increase the systemic vascular resistance (SVR) and improve tissue perfusion. Nitroglycerin would decrease the preload and further drop cardiac output and BP. Drotrecogin alpha may decrease inappropriate inflammation and help prevent systemic inflammatory response syndrome, but it will not directly improve blood pressure. Nitroprusside is an arterial vasodilator and would further decrease SVR. DIF: Cognitive Level: Application REF: 1731 | 1733-1735 TOP: Nursing Process: Planning MSC: NCLEX: Physiological Integrity

An 82-year-old patient in a long-term care facility has several medications prescribed. After the patient is newly diagnosed with hypothyroidism, the nurse will need to consult with the health care provider before administering a. docusate (Colace). b. ibuprofen (Motrin). c. diazepam (Valium). d. cefoxitin (Mefoxin).

ANS: C Worsening of mental status and myxedema coma can be precipitated by the use of sedatives, especially in older adults. The nurse should discuss the use of diazepam with the health care provider before administration. The other medications may be given safely to the patient.

Which nursing action is essential for a patient immediately after a renal biopsy? a. Check blood glucose to assess for hyperglycemia or hypoglycemia. b. Insert a urinary catheter and test urine for gross or microscopic hematuria. c. Monitor the blood urea nitrogen (BUN) and creatinine to assess renal function. d. Apply a pressure dressing and keep the patient on the affected side for 30 minutes.

ANS: D A pressure dressing is applied and the patient is kept on the affected side for 30 to 60 minutes to put pressure on the biopsy side and decrease the risk for bleeding. The blood glucose and BUN/creatinine will not be affected by the biopsy. Although monitoring for hematuria is needed, there is no need for catheterization.

2. The emergency department (ED) nurse is initiating therapeutic hypothermia in a patient who has been resuscitated after a cardiac arrest. Which actions in the hypothermia protocol can be delegated to an experienced licensed practical/vocational nurse (LPN/LVN) (select all that apply)? a. Continuously monitor heart rhythm. b. Check neurologic status every 2 hours. c. Place cooling blankets above and below patient. d. Give acetaminophen (Tylenol) 650 mg per nasogastric tube. e. Insert rectal temperature probe and attach to cooling blanket control panel.

ANS: C, D, E Experienced LPN/LVNs have the education and scope of practice to implement hypothermia measures (e.g., cooling blanket, temperature probe) and administer medications under the supervision of a registered nurse (RN). Assessment of neurologic status and monitoring the heart rhythm require RN-level education and scope of practice and should be done by the RN. DIF: Cognitive Level: Apply (application) REF: 15-16 OBJ: Special Questions: Delegation TOP: Nursing Process: Planning MSC: NCLEX: Safe and Effective Care Environment

34. The day after a 60-year-old patient has an open reduction and internal fixation (ORIF) for an open, displaced tibial fracture, the priority nursing diagnosis is a. activity intolerance related to deconditioning. b. risk for constipation related to prolonged bed rest. c. risk for impaired skin integrity related to immobility. d. risk for infection related to disruption of skin integrity.

ANS: D A patient having an ORIF is at risk for problems such as wound infection and osteomyelitis. After an ORIF, patients typically are mobilized starting the first postoperative day, so problems caused by immobility are not as likely

22. Which action will the nurse include in the plan of care for a patient who has had a total right knee arthroplasty? a. Avoid extension of the right knee beyond 120 degrees. b. Use a compression bandage to keep the right knee flexed. c. Teach about the need to avoid weight bearing for 4 weeks. d. Start progressive knee exercises to obtain 90-degree flexion.

ANS: D After knee arthroplasty, active or passive flexion exercises are used to obtain a 90-degree flexion of the knee. The goal for extension of the knee will be 180 degrees. A compression bandage is used to hold the knee in an extended position after surgery. Full weight bearing is expected before discharge.

44. After change-of-shift report, which patient should the nurse assess first? a. Patient with a Colles' fracture who has right wrist swelling and deformity b. Patient with a intracapsular left hip fracture whose leg is externally rotated c. Patient with a repaired mandibular fracture who is complaining of facial pain d. Patient with right femoral shaft fracture whose thigh is swollen and ecchymotic

ANS: D Swelling and bruising after a femoral shaft fracture suggest hemorrhage and risk for compartment syndrome. The nurse should assess the patient rapidly and then notify the health care provider. The other patients have symptoms that are typical for their injuries, but do not require immediate intervention.

41. Which finding in a patient with a Colles' fracture of the left wrist is most important to communicate to the health care provider? a. Swelling is noted around the wrist. b. The patient is reporting severe pain. c. The wrist has a deformed appearance. d. Capillary refill to the fingers is prolonged.

ANS: D Swelling, pain, and deformity are common findings with a Colles' fracture. Prolonged capillary refill indicates decreased circulation and risk for ischemia. This is not an expected finding and should be immediately reported.

7. A 48-year-old patient with a comminuted fracture of the left femur has Buck's traction in place while waiting for surgery. To assess for pressure areas on the patient's back and sacral area and to provide skin care, the nurse should a. loosen the traction and help the patient turn onto the unaffected side. b. place a pillow between the patient's legs and turn gently to each side. c. turn the patient partially to each side with the assistance of another nurse. d. have the patient lift the buttocks by bending and pushing with the right leg.

ANS: D The patient can lift the buttocks off the bed by using the left leg without changing the right-leg alignment. Turning the patient will tend to move the leg out of alignment. Disconnecting the traction will interrupt the weight needed to immobilize and align the fracture.

21. The nurse is caring for a patient who is to be discharged from the hospital 5 days after insertion of a femoral head prosthesis using a posterior approach. Which statement by the patient indicates a need for additional instruction? a. "I should not cross my legs while sitting." b. "I will use a toilet elevator on the toilet seat." c. "I will have someone else put on my shoes and socks." d. "I can sleep in any position that is comfortable for me."

ANS: D The patient needs to sleep in a position that prevents excessive internal rotation or flexion of the hip. The other patient statements indicate that the patient has understood the teaching.

2. A factory line worker has repetitive strain syndrome in the left elbow. The nurse will plan to teach the patient about a. surgical options. b. elbow injections. c. wearing a left wrist splint. d. modifying arm movements.

ANS: D Treatment for repetitive strain syndrome includes changing the ergonomics of the activity. Elbow injections and surgery are not initial options for this type of injury. A wrist splint might be used for hand or wrist pain.

10. When rewarming a patient who arrived in the emergency department (ED) with a temperature of 87° F (30.6° C), which assessment indicates that the nurse should discontinue active rewarming? a. The patient begins to shiver. b. The BP decreases to 86/42 mm Hg. c. The patient develops atrial fibrillation. d. The core temperature is 94° F (34.4° C).

ANS: D A core temperature of 89.6° F to 93.2° F (32° C to 34° C) indicates that sufficient rewarming has occurred. Dysrhythmias, hypotension, and shivering may occur during rewarming and should be treated but are not an indication to stop rewarming the patient. DIF: Cognitive Level: Apply (application) REF: 1686 TOP: Nursing Process: Assessment MSC: NCLEX: Physiological Integrity

29. Which assessment finding is of most concern for a 46-year-old woman with acute pancreatitis? a. Absent bowel sounds b. Abdominal tenderness c. Left upper quadrant pain d. Palpable abdominal mass

ANS: D A palpable abdominal mass may indicate the presence of a pancreatic abscess, which will require rapid surgical drainage to prevent sepsis. Absent bowel sounds, abdominal tenderness, and left upper quadrant pain are common in acute pancreatitis and do not require rapid action to prevent further complications.

7. A 52-year-old patient who has no previous history of hypertension or other health problems suddenly develops a BP of 188/106 mm Hg. After reconfirming the BP, it is appropriate for the nurse to tell the patient that a. a BP recheck should be scheduled in a few weeks. b. the dietary sodium and fat content should be decreased. c. there is an immediate danger of a stroke and hospitalization will be required. d. more diagnostic testing may be needed to determine the cause of the hypertension.

ANS: D A sudden increase in BP in a patient over age 50 with no previous hypertension history or risk factors indicates that the hypertension may be secondary to some other problem. The BP will need rapid treatment and ongoing monitoring. If the patient has no other risk factors, a stroke in the immediate future is unlikely. There is no indication that dietary salt or fat intake have contributed to this sudden increase in BP, and reducing intake of salt and fat alone will not be adequate to reduce this BP to an acceptable level.

The nurse is obtaining a health history from a 24-year-old patient with hypertrophic cardiomyopathy (HC). Which information obtained by the nurse is most important? a. The patient has a history of a recent upper respiratory infection. b. The patient has a family history of coronary artery disease (CAD). c. The patient reports using cocaine a "couple of times" as a teenager. d. The patient's 29-year-old brother died from a sudden cardiac arrest.

ANS: D About half of all cases of HC have a genetic basis, and it is the most common cause of sudden cardiac death in otherwise healthy young people. The information about the patient's brother will be helpful in planning care (such as an automatic implantable cardioverter-defibrillator [AICD] *big hearts run in families*

A patient who has burns on the arms, legs, and chest from a house fire has become agitated and restless 8 hours after being admitted to the hospital. Which action should the nurse take first? a. Stay at the bedside and reassure the patient. b. Administer the ordered morphine sulfate IV. c. Assess orientation and level of consciousness. d. Use pulse oximetry to check the oxygen saturation.

ANS: D Agitation in a patient who may have suffered inhalation injury might indicate hypoxia, and this should be assessed by the nurse first. Administration of morphine may be indicated if the nurse determines that the agitation is caused by pain. Assessing level of consciousness and orientation is also appropriate but not as essential as determining whether the patient is hypoxemic. Reassurance is not helpful to reduce agitation in a hypoxemic patient.

A patient who has been receiving a heparin infusion and warfarin (Coumadin) for a deep vein thrombosis (DVT) is diagnosed with heparin-induced thrombocytopenia (HIT) when her platelet level drops to 110,000/µL. Which action will the nurse include in the plan of care? a. Use low-molecular-weight heparin (LMWH) only. b. Administer the warfarin (Coumadin) at the scheduled time. c. Teach the patient about the purpose of platelet transfusions. d. Discontinue heparin and flush intermittent IV lines using normal saline.

ANS: D All heparin is discontinued when the HIT is diagnosed. The patient should be instructed to never receive heparin or LMWH. Warfarin is usually not given until the platelet count has returned to 150,000/µL. The platelet count does not drop low enough in HIT for a platelet transfusion, and platelet transfusions increase the risk for thrombosis

32. A patient admitted to the coronary care unit (CCU) with an ST-segment-elevation myocardial infarction (STEMI) is restless and anxious. The blood pressure is 86/40 and heart rate is 123. Based on this information, which nursing diagnosis is a priority for the patient? a. Acute pain related to myocardial infarction b. Anxiety related to perceived threat of death c. Stress overload related to acute change in health d. Decreased cardiac output related to cardiogenic shock

ANS: D All the nursing diagnoses may be appropriate for this patient, but the hypotension and tachycardia indicate decreased cardiac output and shock from the damaged myocardium. This will result in decreased perfusion to all vital organs (e.g., brain, kidney, heart) and is a priority.

6. Which statement made by a patient with coronary artery disease after the nurse has completed teaching about therapeutic lifestyle changes (TLC) diet indicates that further teaching is needed? a. "I will switch from whole milk to 1% milk." b. "I like salmon and I will plan to eat it more often." c. "I can have a glass of wine with dinner if I want one." d. "I will miss being able to eat peanut butter sandwiches."

ANS: D Although only 30% of the daily calories should come from fats, most of the fat in the TLC diet should come from monosaturated fats such as are found in nuts, olive oil, and canola oil. The patient can include peanut butter sandwiches as part of the TLC diet. The other patient comments indicate a good understanding of the TLC diet.

9. The nurse has just finished teaching a hypertensive patient about the newly prescribed quinapril (Accupril). Which patient statement indicates that more teaching is needed? a. "The medication may not work as well if I take any aspirin." b. "The doctor may order a blood potassium level occasionally." c. "I will call the doctor if I notice that I have a frequent cough." d. "I won't worry if I have a little swelling around my lips and face."

ANS: D Angioedema occurring with angiotensin-converting enzyme (ACE) inhibitor therapy is an indication that the ACE inhibitor should be discontinued. The patient should be taught that if any swelling of the face or oral mucosa occurs, the health care provider should be immediately notified because this could be life threatening. The other patient statements indicate that the patient has an accurate understanding of ACE inhibitor therapy.

A patient being admitted with bacterial meningitis has a temperature of 102.5° F (39.2° C) and a severe headache. Which order for collaborative intervention should the nurse implement first? a. Administer ceftizoxime (Cefizox) 1 g IV. b. Give acetaminophen (Tylenol) 650 mg PO. c. Use a cooling blanket to lower temperature. d. Swab the nasopharyngeal mucosa for cultures.

ANS: D Antibiotic therapy should be instituted rapidly in bacterial meningitis, but cultures must be done before antibiotics are started. As soon as the cultures are done, the antibiotic should be started. Hypothermia therapy and acetaminophen administration are appropriate but can be started after the other actions are implemented. DIF: Cognitive Level: Apply (application) REF: 1382 OBJ: Special Questions: Prioritization TOP: Nursing Process: Implementation MSC: NCLEX: Physiological Integrity

When assessing a newly admitted patient, the nurse notes pallor of the skin and nail beds. The nurse should ensure that which laboratory test has been ordered? a. Platelet count b. Neutrophil count c. White blood cell count d. Hemoglobin (Hgb) level

ANS: D Pallor of the skin or nail beds is indicative of anemia, which would be indicated by a low Hgb level. Platelet counts indicate a person's clotting ability. A neutrophil is a type of white blood cell that helps to fight infection.

Which action will the nurse include in the plan of care for a patient in the rehabilitation phase after a burn injury to the right arm and chest? a. Keep the right arm in a position of comfort. b. Avoid the use of sustained-release narcotics. c. Teach about the purpose of tetanus immunization. d. Apply water-based cream to burned areas frequently.

ANS: D Application of water-based emollients will moisturize new skin and decrease flakiness and itching. To avoid contractures, the joints of the right arm should be positioned in an extended position, which is not the position of comfort. Patients may need to continue the use of opioids during rehabilitation. Tetanus immunization would have been given during the emergent phase of the burn injury.

1. When developing a teaching plan for a 61-year-old man with the following risk factors for coronary artery disease (CAD), the nurse should focus on the a. family history of coronary artery disease. b. increased risk associated with the patient's gender. c. increased risk of cardiovascular disease as people age. d. elevation of the patient's low-density lipoprotein (LDL) level.

ANS: D Because family history, gender, and age are *nonmodifiable* risk factors, the nurse should focus on the patient's LDL level. Decreases in LDL will help reduce the patient's risk for developing CAD.

Which statement by a patient indicates good understanding of the nurse's teaching about prevention of sickle cell crisis? a. "Home oxygen therapy is frequently used to decrease sickling." b. "There are no effective medications that can help prevent sickling." c. "Routine continuous dosage narcotics are prescribed to prevent a crisis." d. "Risk for a crisis is decreased by having an annual influenza vaccination."

ANS: D Because infection is the most common cause of a sickle cell crisis, influenza, Haemophilus influenzae, pneumococcal pneumonia, and hepatitis immunizations should be administered. Although continuous dose opioids and oxygen may be administered during a crisis, patients do not receive these therapies to prevent crisis. Hydroxyurea (Hydrea) is a medication used to decrease the number of sickle cell crises

Which patient information is most important for the nurse to monitor when evaluating the effectiveness of deferoxamine (Desferal) for a patient with hemochromatosis? a. Skin color b. Hematocrit c. Liver function d. Serum iron level

ANS: D Because iron chelating agents are used to lower serum iron levels, the most useful information will be the patient's iron level. The other parameters will also be monitored, but are not the most important to monitor when determining the effectiveness of *defe*roxamine (de-Fe - decrease iron)

11. Nadolol (Corgard) is prescribed for a patient with chronic stable angina and left ventricular dysfunction. To determine whether the drug is effective, the nurse will monitor for a. decreased blood pressure and heart rate. b. fewer complaints of having cold hands and feet. c. improvement in the strength of the distal pulses. d. the ability to do daily activities without chest pain.

ANS: D Because the medication is *ordered to improve the patient's angina*, effectiveness is indicated if the patient is able to accomplish daily activities *without chest pain*. Blood pressure and heart rate may decrease, but these data do not indicate that the goal of decreased angina has been met. The noncardioselective β-adrenergic blockers can cause peripheral vasoconstriction, so the nurse would not expect an improvement in distal pulse quality or skin temperature *aka why i wear fingerless gloves in class*.

15. When analyzing the rhythm of a patient's electrocardiogram (ECG), the nurse will need to investigate further upon finding a(n) a. isoelectric ST segment. b. P-R interval of 0.18 second. c. Q-T interval of 0.38 second. d. QRS interval of 0.14 second.

ANS: D Because the normal QRS interval is 0.04 to 0.10 seconds, the patient's QRS interval of 0.14 seconds indicates that the conduction through the ventricular conduction system is prolonged. The P-R interval and Q-T interval are within normal range, and ST segment should be isoelectric (flat). DIF: Cognitive Level: Apply (application) REF: 791 TOP: Nursing Process: Assessment MSC: NCLEX: Physiological Integrity

A nurse reviews the laboratory data for an older patient. The nurse would be most concerned about which finding? a. Hematocrit of 35% b. Hemoglobin of 11.8 g/dL c. Platelet count of 400,000/µL d. White blood cell (WBC) count of 2800/µL

ANS: D Because the total WBC count is not usually affected by aging, the low WBC count in this patient would indicate that the patient's immune function may be compromised and the underlying cause of the problem needs to be investigated. The platelet count is normal. The slight decrease in hemoglobin and hematocrit are not unusual for an older patient.

4. Which information given by a patient admitted with chronic stable angina will help the nurse confirm this diagnosis? a. The patient states that the pain "wakes me up at night." b. The patient rates the pain at a level 3 to 5 (0 to 10 scale). c. The patient states that the pain has increased in frequency over the last week. d. The patient states that the pain "goes away" with one sublingual nitroglycerin tablet.

ANS: D Chronic stable angina is typically relieved by rest or nitroglycerin administration. The level of pain is not a consistent indicator of the type of angina. Pain occurring at rest or with increased frequency is typical of unstable angina.

The nurse obtains a health history from a 65-year-old patient with a prosthetic mitral valve who has symptoms of infective endocarditis (IE). Which question by the nurse is most appropriate? a. "Do you have a history of a heart attack?" b. "Is there a family history of endocarditis?" c. "Have you had any recent immunizations?" d. "Have you had dental work done recently?"

ANS: D Dental procedures place the patient with a prosthetic mitral valve at risk for infective endocarditis (IE). Myocardial infarction (MI), immunizations, and a family history of endocarditis are not risk factors for IE.

The health care provider orders a liver/spleen scan for a patient who has been in a motor vehicle accident. Which action should the nurse take before this procedure? a. Check for any iodine allergy. b. Insert a large-bore IV catheter. c. Place the patient on NPO status. d. Assist the patient to a flat position.

ANS: D During a liver/spleen scan, a radioactive isotope is injected IV and images from the radioactive emission are used to evaluate the structure of the spleen and liver. An indwelling IV catheter is not needed. The patient is placed in a flat position before the scan.

Which laboratory test will the nurse use to determine whether filgrastim (Neupogen) is effective for a patient with acute lymphocytic leukemia who is receiving chemotherapy? a. Platelet count b. Reticulocyte count c. Total lymphocyte count d. Absolute neutrophil count

ANS: D Filgrastim increases the neutrophil count and function in neutropenic patients. Although total lymphocyte, platelet, and reticulocyte counts also are important to monitor in this patient, the absolute neutrophil count is used to evaluate the effects of filgrastim

6. A patient has a normal cardiac rhythm and a heart rate of 72 beats/minute. The nurse determines that the P-R interval is 0.24 seconds. The most appropriate intervention by the nurse would be to a. notify the health care provider immediately. b. give atropine per agency dysrhythmia protocol. c. prepare the patient for temporary pacemaker insertion. d. document the finding and continue to monitor the patient.

ANS: D First-degree atrioventricular (AV) block is asymptomatic and requires ongoing monitoring because it may progress to more serious forms of heart block. The rate is normal, so there is no indication that atropine is needed. Immediate notification of the health care provider about an asymptomatic rhythm is not necessary. DIF: Cognitive Level: Apply (application) REF: 798 TOP: Nursing Process: Implementation MSC: NCLEX: Physiological Integrity

Esomeprazole (Nexium) is prescribed for a patient who incurred extensive burn injuries 5 days ago. Which nursing assessment would best evaluate the effectiveness of the medication? a. Bowel sounds b. Stool frequency c. Abdominal distention d. Stools for occult blood

ANS: D H2 blockers and proton pump inhibitors are given to prevent Curling's ulcer in the patient who has suffered burn injuries. Proton pump inhibitors usually do not affect bowel sounds, stool frequency, or appetite.

1. A 24-year-old female contracts hepatitis from contaminated food. During the acute (icteric) phase of the patient's illness, the nurse would expect serologic testing to reveal a. antibody to hepatitis D (anti-HDV). b. hepatitis B surface antigen (HBsAg). c. anti-hepatitis A virus immunoglobulin G (anti-HAV IgG). d. anti-hepatitis A virus immunoglobulin M (anti-HAV IgM).

ANS: D Hepatitis A is transmitted through the oral-fecal route, and antibody to HAV IgM appears during the acute phase of hepatitis A. The patient would not have antigen for hepatitis B or antibody for hepatitis D. Anti-HAV IgG would indicate past infection and lifelong immunity.

17. Which laboratory result for a patient with multifocal premature ventricular contractions (PVCs) is most important for the nurse to communicate to the health care provider? a. Blood glucose 243 mg/dL b. Serum chloride 92 mEq/L c. Serum sodium 134 mEq/L d. Serum potassium 2.9 mEq/L

ANS: D Hypokalemia increases the risk for ventricular dysrhythmias such as PVCs, ventricular tachycardia, and ventricular fibrillation. The health care provider will need to prescribe a potassium infusion to correct this abnormality. Although the other laboratory values also are abnormal, they are not likely to be the etiology of the patient's PVCs and do not require immediate correction. DIF: Cognitive Level: Apply (application) REF: 799 OBJ: Special Questions: Prioritization TOP: Nursing Process: Assessment MSC: NCLEX: Physiological Integrity

36. A patient had a non-ST-segment-elevation myocardial infarction (NSTEMI) 3 days ago. Which nursing intervention included in the plan of care is most appropriate for the registered nurse (RN) to delegate to an experienced licensed practical/vocational nurse (LPN/LVN)? a. Evaluation of the patient's response to walking in the hallway b. Completion of the referral form for a home health nurse follow-up c. Education of the patient about the pathophysiology of heart disease d. Reinforcement of teaching about the purpose of prescribed medications

ANS: D LPN/LVN education and scope of practice include reinforcing education that has previously been done by the RN. Evaluating the patient response to exercise after a NSTEMI requires more education and should be done by the RN. Teaching and discharge planning/ documentation are higher level skills that require RN education and scope of practice.

When working in the urology/nephrology clinic, which patient could the nurse delegate to an experienced licensed practical/vocational nurse (LPN/LVN)? a. Patient who is scheduled for a renal biopsy after a recent kidney transplant b. Patient who will need monitoring for several hours after a renal arteriogram c. Patient who requires teaching about possible post-cystoscopy complications d. Patient who will have catheterization to check for residual urine after voiding

ANS: D LPN/LVN education includes common procedures such as catheterization of stable patients. The other patients require more complex assessments and/or patient teaching that are included in registered nurse (RN) education and scope of practice.

A patient who has non-Hodgkin's lymphoma is receiving combination treatment with rituximab (Rituxan) and chemotherapy. Which patient assessment finding requires the most rapid action by the nurse? a. Anorexia b. Vomiting c. Oral ulcers d. Lip swelling

ANS: D Lip swelling in angioedema may indicate a hypersensitivity reaction to the rituximab. The nurse should stop the infusion and further assess for anaphylaxis. The other findings may occur with chemotherapy, but are not immediately life threatening

A patient has been admitted with meningococcal meningitis. Which observation by the nurse requires action? a. The bedrails at the head and foot of the bed are both elevated. b. The patient receives a regular diet from the dietary department. c. The lights in the patient's room are turned off and the blinds are shut. d. Unlicensed assistive personnel enter the patient's room without a mask.

ANS: D Meningococcal meningitis is spread by respiratory secretions, so it is important to maintain respiratory isolation as well as standard precautions. Because the patient may be confused and weak, bedrails should be elevated at both the foot and head of the bed. Low light levels in the room decrease pain caused by photophobia. Nutrition is an important aspect of care in a patient with meningitis. DIF: Cognitive Level: Apply (application) REF: 1383 TOP: Nursing Process: Assessment MSC: NCLEX: Safe and Effective Care Environment

A critical action by the nurse caring for a patient with an acute exacerbation of polycythemia vera is to a. place the patient on bed rest. b. administer iron supplements. c. avoid use of aspirin products. d. monitor fluid intake and output.

ANS: D Monitoring hydration status is important during an acute exacerbation because the patient is at risk for fluid overload or underhydration. Aspirin therapy is used to decrease risk for thrombosis. The patient should be encouraged to ambulate to prevent deep vein thrombosis (DVT). Iron is contraindicated in patients with polycythemia vera

14. The emergency department (ED) triage nurse is assessing four victims involved in a motor vehicle collision. Which patient has the highest priority for treatment? a. A patient with no pedal pulses. b. A patient with an open femur fracture. c. A patient with bleeding facial lacerations. d. A patient with paradoxic chest movements.

ANS: D Most immediate deaths from trauma occur because of problems with ventilation, so the patient with paradoxic chest movements should be treated first. Face and head fractures can obstruct the airway, but the patient with facial injuries only has lacerations. The other two patients also need rapid intervention but do not have airway or breathing problems. DIF: Cognitive Level: Apply (application) REF: 1676 OBJ: Special Questions: Multiple Patients TOP: Nursing Process: Assessment MSC: NCLEX: Safe and Effective Care Environment

A patient complains of leg cramps during hemodialysis. The nurse should first a. reposition the patient. b. massage the patient's legs. c. give acetaminophen (Tylenol). d. infuse a bolus of normal saline.

ANS: D Muscle cramps during dialysis are caused by rapid removal of sodium and water. Treatment includes infusion of normal saline. The other actions do not address the reason for the cramps.

28. A 67-year-old male patient with acute pancreatitis has a nasogastric (NG) tube to suction and is NPO. Which information obtained by the nurse indicates that these therapies have been effective? a. Bowel sounds are present. b. Grey Turner sign resolves. c. Electrolyte levels are normal. d. Abdominal pain is decreased.

ANS: D NG suction and NPO status will decrease the release of pancreatic enzymes into the pancreas and decrease pain. Although bowel sounds may be hypotonic with acute pancreatitis, the presence of bowel sounds does not indicate that treatment with NG suction and NPO status has been effective. Electrolyte levels may be abnormal with NG suction and must be replaced by appropriate IV infusion. Although Grey Turner sign will eventually resolve, it would not be appropriate to wait for this to occur to determine whether treatment was effective.

The nurse is reviewing the medication administration record (MAR) on a patient with partial-thickness burns. Which medication is best for the nurse to administer before scheduled wound debridement? a. Ketorolac (Toradol) b. Lorazepam (Ativan) c. Gabapentin (Neurontin) d. Hydromorphone (Dilaudid)

ANS: D Opioid pain medications are the best choice for pain control. The other medications are used as adjuvants to enhance the effects of opioids.

Which action by a patient who is using peritoneal dialysis (PD) indicates that the nurse should provide more teaching about PD? a. The patient slows the inflow rate when experiencing pain. b. The patient leaves the catheter exit site without a dressing. c. The patient plans 30 to 60 minutes for a dialysate exchange. d. The patient cleans the catheter while taking a bath every day.

ANS: D Patients are encouraged to take showers rather than baths to avoid infections at the catheter insertion side. The other patient actions indicate good understanding of peritoneal dialysis.

The nurse is caring for a patient who was admitted the previous day with a basilar skull fracture after a motor vehicle crash. Which assessment finding is most important to report to the health care provider? a. Complaint of severe headache b. Large contusion behind left ear c. Bilateral periorbital ecchymosis d. Temperature of 101.4° F (38.6° C)

ANS: D Patients who have basilar skull fractures are at risk for meningitis, so the elevated temperature should be reported to the health care provider. The other findings are typical of a patient with a basilar skull fracture. DIF: Cognitive Level: Apply (application) REF: 1369 OBJ: Special Questions: Prioritization TOP: Nursing Process: Assessment MSC: NCLEX: Physiological Integrity

The nurse establishes the nursing diagnosis of ineffective health maintenance related to lack of knowledge regarding long-term management of rheumatic fever when a 30-year-old recovering from rheumatic fever without carditis says which of the following? a. "I will need prophylactic antibiotic therapy for 5 years." b. "I will need to take aspirin or ibuprofen (Motrin) to relieve my joint pain." c. "I will call the doctor if I develop excessive fatigue or difficulty breathing." d. "I will be immune to further episodes of rheumatic fever after this infection."

ANS: D Patients with a history of rheumatic fever are more susceptible to a second episode. Patients with rheumatic fever without carditis require prophylaxis until age 20 and for a minimum of 5 years. The other patient statements are correct and would not support the nursing diagnosis of ineffective health maintenance.

A patient with acute kidney injury (AKI) has an arterial blood pH of 7.30. The nurse will assess the patient for a. vasodilation. b. poor skin turgor. c. bounding pulses. d. rapid respirations.

ANS: D Patients with metabolic acidosis caused by AKI may have Kussmaul respirations as the lungs try to regulate carbon dioxide. Bounding pulses and vasodilation are not associated with metabolic acidosis. Because the patient is likely to have fluid retention, poor skin turgor would not be a finding in AKI.

Which laboratory result will the nurse expect to show a decreased value if a patient develops heparin-induced thrombocytopenia (HIT)? a. Prothrombin time b. Erythrocyte count c. Fibrinogen degradation products d. Activated partial thromboplastin time

ANS: D Platelet aggregation in HIT causes neutralization of heparin, so that the activated partial thromboplastin time will be shorter and more heparin will be needed to maintain therapeutic levels. The other data will not be affected by HIT

To evaluate the effectiveness of the pantoprazole (Protonix) ordered for a patient with systemic inflammatory response syndrome (SIRS), which assessment will the nurse perform? a. Auscultate bowel sounds. b. Palpate for abdominal pain. c. Ask the patient about nausea. d. Check stools for occult blood.

ANS: D Proton pump inhibitors are given to decrease the risk for stress ulcers in critically ill patients. The other assessments also will be done, but these will not help in determining the effectiveness of the pantoprazole administration.

After noting a pulse deficit when assessing a 74-year-old patient who has just arrived in the emergency department, the nurse will anticipate that the patient may require a. emergent cardioversion. b. a cardiac catheterization. c. hourly blood pressure (BP) checks. d. electrocardiographic (ECG) monitoring.

ANS: D Pulse deficit is a difference between simultaneously obtained apical and radial pulses. It indicates that there may be a cardiac dysrhythmia that would best be detected with ECG monitoring. Frequent BP monitoring, cardiac catheterization, and emergent cardioversion are used for diagnosis and/or treatment of cardiovascular disorders but would not be as helpful in determining the immediate reason for the pulse deficit

After having a craniectomy and left anterior fossae incision, a 64-year-old patient has a nursing diagnosis of impaired physical mobility related to decreased level of consciousness and weakness. An appropriate nursing intervention is to a. cluster nursing activities to allow longer rest periods. b. turn and reposition the patient side to side every 2 hours. c. position the bed flat and log roll to reposition the patient. d. perform range-of-motion (ROM) exercises every 4 hours.

ANS: D ROM exercises will help prevent the complications of immobility. Patients with anterior craniotomies are positioned with the head elevated. The patient with a craniectomy should not be turned to the operative side. When the patient is weak, clustering nursing activities may lead to more fatigue and weakness. DIF: Cognitive Level: Apply (application) REF: 1380 TOP: Nursing Process: Implementation MSC: NCLEX: Physiological Integrity

When assessing a 53-year-old patient with bacterial meningitis, the nurse obtains the following data. Which finding should be reported immediately to the health care provider? a. The patient exhibits nuchal rigidity. b. The patient has a positive Kernig's sign. c. The patient's temperature is 101° F (38.3° C). d. The patient's blood pressure is 88/42 mm Hg.

ANS: D Shock is a serious complication of meningitis, and the patient's low blood pressure indicates the need for interventions such as fluids or vasopressors. Nuchal rigidity and a positive Kernig's sign are expected with bacterial meningitis. The nurse should intervene to lower the temperature, but this is not as life threatening as the hypotension. DIF: Cognitive Level: Apply (application) REF: 1382 OBJ: Special Questions: Prioritization TOP: Nursing Process: Assessment MSC: NCLEX: Physiological Integrity

18. When a patient with hypertension who has a new prescription for atenolol (Tenormin) returns to the health clinic after 2 weeks for a follow-up visit, the BP is unchanged from the previous visit. Which action should the nurse take first? a. Provide information about the use of multiple drugs to treat hypertension. b. Teach the patient about the reasons for a possible change in drug therapy. c. Remind the patient that lifestyle changes also are important in BP control. d. Question the patient about whether the medication is actually being taken.

ANS: D Since noncompliance with antihypertensive therapy is common *esp c BBs d/t SE of OH, fatigue, sexual dysfunction*, the nurse's initial action should be to determine whether the patient is taking the atenolol as prescribed. The other actions also may be implemented, but these would be done after assessing patient compliance with the prescribed therapy.

(question originally had a picture, but was basically asking, where do you listen for mitral valve sounds?)

ANS: D Sounds from the mitral valve are best heard at the apex of the heart, fifth intercostal space, midclavicular line.

The nurse is caring for a patient who is being discharged after an emergency splenectomy following an automobile accident. Which instructions should the nurse include in the discharge teaching? a. Watch for excess bruising. b. Check for swollen lymph nodes. c. Take iron supplements to prevent anemia. d. Wash hands and avoid persons who are ill.

ANS: D Splenectomy increases the risk for infection, especially with gram-positive bacteria. The risks for lymphedema, bleeding, and anemia are not increased after a person has a splenectomy.

4. After the nurse teaches the patient with stage 1 hypertension about diet modifications that should be implemented, which diet choice indicates that the teaching has been effective? a. The patient avoids eating nuts or nut butters. b. The patient restricts intake of dietary protein. c. The patient has only one cup of coffee in the morning. d. The patient has a glass of low-fat milk with each meal.

ANS: D The Dietary Approaches to Stop Hypertension (DASH) recommendations for prevention of hypertension include increasing the intake of calcium-rich foods. Caffeine restriction and decreased protein intake are not included in the recommendations. Nuts are high in beneficial nutrients and 4 to 5 servings weekly are recommended in the DASH diet.

During a physical examination of a 74-year-old patient, the nurse palpates the point of maximal impulse (PMI) in the sixth intercostal space lateral to the left midclavicular line. The most appropriate action for the nurse to take next will be to a. ask the patient about risk factors for atherosclerosis. b. document that the PMI is in the normal anatomic location. c. auscultate both the carotid arteries for the presence of a bruit. d. assess the patient for symptoms of left ventricular hypertrophy.

ANS: D The PMI should be felt at the intersection of the fifth intercostal space and the left midclavicular line. A PMI located outside these landmarks indicates possible cardiac enlargement, such as with left ventricular hypertrophy. Cardiac enlargement is not necessarily associated with atherosclerosis or carotid artery disease

4. The nurse obtains a rhythm strip on a patient who has had a myocardial infarction and makes the following analysis: no visible P waves, P-R interval not measurable, ventricular rate 162, R-R interval regular, and QRS complex wide and distorted, QRS duration 0.18 second. The nurse interprets the patient's cardiac rhythm as a. atrial flutter. b. sinus tachycardia. c. ventricular fibrillation. d. ventricular tachycardia.

ANS: D The absence of P waves, wide QRS, rate >150 beats/minute, and the regularity of the rhythm indicate ventricular tachycardia. Atrial flutter is usually regular, has a narrow QRS configuration, and has flutter waves present representing atrial activity. Sinus tachycardia has P waves. Ventricular fibrillation is irregular and does not have a consistent QRS duration. DIF: Cognitive Level: Apply (application) REF: 794 TOP: Nursing Process: Assessment MSC: NCLEX: Physiological Integrity

5. A patient has just been diagnosed with hypertension and has a new prescription for captopril (Capoten). Which information is important to include when teaching the patient? a. Check BP daily before taking the medication. b. Increase fluid intake if dryness of the mouth is a problem. c. Include high-potassium foods such as bananas in the diet. d. Change position slowly to help prevent dizziness and falls.

ANS: D The angiotensin-converting enzyme (ACE) inhibitors frequently cause orthostatic hypotension, and pa- tients should be taught to change position slowly to allow the vascular system time to compensate for the position change. Increasing fluid intake may counteract the effect of the medication, and the patient is taught to use gum or hard candy to relieve dry mouth. The BP does not need to be checked at home by the patient before taking the medication. Because ACE inhibitors cause potassium retention, increased intake of high-potassium foods is inappropriate.

24. A patient with hyperlipidemia has a new order for colesevelam (Welchol). Which nursing action is most appropriate when giving the medication? a. Have the patient take this medication with an aspirin. b. Administer the medication at the patient's usual bedtime. c. Have the patient take the colesevelam with a sip of water. d. Give the patient's other medications 2 hours after the colesevelam.

ANS: D The bile acid sequestrants interfere with the absorption of many other drugs, and giving other medications at the same time should be avoided. Taking an aspirin concurrently with the colesevelam may increase the incidence of gastrointestinal side effects such as heartburn. An increased fluid intake is encouraged for patients taking the bile acid sequestrants to reduce the risk for constipation. For maximum effect, colesevelam should be administered with meals.

7. A patient who was admitted with a myocardial infarction experiences a 45-second episode of ventricular tachycardia, then converts to sinus rhythm with a heart rate of 98 beats/minute. Which of the following actions should the nurse take next? a. Immediately notify the health care provider. b. Document the rhythm and continue to monitor the patient. c. Perform synchronized cardioversion per agency dysrhythmia protocol. d. Prepare to give IV amiodarone (Cordarone) per agency dysrhythmia protocol.

ANS: D The burst of sustained ventricular tachycardia indicates that the patient has significant ventricular irritability, and antidysrhythmic medication administration is needed to prevent further episodes. The nurse should notify the health care provider after the medication is started. Defibrillation is not indicated given that the patient is currently in a sinus rhythm. Documentation and continued monitoring are not adequate responses to this situation. DIF: Cognitive Level: Apply (application) REF: 799 TOP: Nursing Process: Implementation MSC: NCLEX: Physiological Integrity

During change-of-shift report, the nurse learns that a patient has been admitted with dehydration and hypotension after having vomiting and diarrhea for 3 days. Which finding is most important for the nurse to report to the health care provider? a. Decreased bowel sounds b. Apical pulse 110 beats/min c. Pale, cool, and dry extremities d. New onset of confusion and agitation

ANS: D The changes in mental status are indicative that the patient is in the progressive stage of shock and that rapid intervention is needed to prevent further deterioration. The other information is consistent with compensatory shock. DIF: Cognitive Level: Application REF: 1728-1729 OBJ: Special Questions: Prioritization TOP: Nursing Process: Assessment MSC: NCLEX: Physiological Integrity

Following successful treatment of Hodgkin's lymphoma for a 55-year-old woman, which topic will the nurse include in patient teaching? a. Potential impact of chemotherapy treatment on fertility b. Application of soothing lotions to treat residual pruritus c. Use of maintenance chemotherapy to maintain remission d. Need for follow-up appointments to screen for malignancy

ANS: D The chemotherapy used in treating Hodgkin's lymphoma results in a high incidence of secondary malignancies; follow-up screening is needed. The fertility of a 55-year-old woman will not be impacted by chemotherapy. Maintenance chemotherapy is not used for Hodgkin's lymphoma. Pruritus is a clinical manifestation of lymphoma, but should not be a concern after treatment

After the emergency department nurse has received a status report on the following patients who have been admitted with head injuries, which patient should the nurse assess first? a. A 20-year-old patient whose cranial x-ray shows a linear skull fracture b. A 30-year-old patient who has an initial Glasgow Coma Scale score of 13 c. A 40-year-old patient who lost consciousness for a few seconds after a fall d. A 50-year-old patient whose right pupil is 10 mm and unresponsive to light

ANS: D The dilated and nonresponsive pupil may indicate an intracerebral hemorrhage and increased intracranial pressure. The other patients are not at immediate risk for complications such as herniation. DIF: Cognitive Level: Analyze (analysis) REF: 1366 OBJ: Special Questions: Prioritization; Multiple Patients TOP: Nursing Process: Assessment MSC: NCLEX: Safe and Effective Care Environment

The nurse reviews the complete blood count (CBC) and white blood cell (WBC) differential of a patient admitted with abdominal pain. Which information will be most important for the nurse to communicate to the health care provider? a. Monocytes 4% b. Hemoglobin 13.6 g/dL c. Platelet count 168,000/µL d. White blood cells (WBCs) 15,500/µL

ANS: D The elevation in WBCs indicates that the patient has an inflammatory or infectious process ongoing, which may be the cause of the patient's pain, and that further diagnostic testing is needed. The monocytes are at a normal level. The hemoglobin and platelet counts are normal.

A patient who has been involved in a motor vehicle crash is admitted to the emergency department (ED) with cool, clammy skin; tachycardia; and hypotension. Which of these prescribed interventions should the nurse implement first? a. Place the patient on continuous cardiac monitor. b. Draw blood to type and crossmatch for transfusions. c. Insert two 14-gauge IV catheters in antecubital space. d. Administer oxygen at 100% per non-rebreather mask

ANS: D The first priority in the initial management of shock is maintenance of the airway and ventilation. Cardiac monitoring, insertion of IV catheters, and obtaining blood for transfusions also should be rapidly accomplished, but only after actions to maximize oxygen delivery have been implemented. DIF: Cognitive Level: Application REF: 1732 OBJ: Special Questions: Prioritization TOP: Nursing Process: Implementation

22. The nurse has received change-of-shift report about the following patients on the progressive care unit. Which patient should the nurse see first? a. A patient who is in a sinus rhythm, rate 98, after having electrical cardioversion 2 hours ago b. A patient with new onset atrial fibrillation, rate 88, who has a first dose of warfarin (Coumadin) due c. A patient with second-degree atrioventricular (AV) block, type 1, rate 60, who is dizzy when ambulating d. A patient whose implantable cardioverter-defibrillator (ICD) fired two times today who has a dose of amiodarone (Cordarone) due

ANS: D The frequent firing of the ICD indicates that the patient's ventricles are very irritable, and the priority is to assess the patient and administer the amiodarone. The other patients may be seen after the amiodarone is administered. DIF: Cognitive Level: Analyze (analysis) REF: 803 OBJ: Special Questions: Prioritization; Multiple Patients TOP: Nursing Process: Implementation MSC: NCLEX: Safe and Effective Care Environment

After noting lengthening QRS intervals in a patient with acute kidney injury (AKI), which action should the nurse take first? a. Document the QRS interval. b. Notify the patient's health care provider. c. Look at the patient's current blood urea nitrogen (BUN) and creatinine levels. d. Check the chart for the most recent blood potassium level.

ANS: D The increasing QRS interval is suggestive of hyperkalemia, so the nurse should check the most recent potassium and then notify the patient's health care provider. The BUN and creatinine will be elevated in a patient with AKI, but they would not directly affect the electrocardiogram (ECG). Documentation of the QRS interval also is appropriate, but interventions to decrease the potassium level are needed to prevent life-threatening bradycardia.

17. A 54-year-old patient arrives in the emergency department (ED) after exposure to powdered lime at work. Which action should the nurse take first? a. Obtain the patient's vital signs. b. Obtain a baseline complete blood count. c. Decontaminate the patient by showering with water. d. Brush off any visible powder on the skin and clothing.

ANS: D The initial action should be to protect staff members and decrease the patient's exposure to the toxin by decontamination. Patients exposed to powdered lime should not be showered; instead any/all visible powder should be brushed off. The other actions can be done after the decontamination is completed. DIF: Cognitive Level: Apply (application) REF: 1690 OBJ: Special Questions: Prioritization TOP: Nursing Process: Implementation MSC: NCLEX: Physiological Integrity

19. A patient's cardiac monitor shows sinus rhythm, rate 64. The P-R interval is 0.18 seconds at 1:00 AM, 0.22 seconds at 2:30 PM, and 0.28 seconds at 4:00 PM. Which action should the nurse take next? a. Place the transcutaneous pacemaker pads on the patient. b. Administer atropine sulfate 1 mg IV per agency dysrhythmia protocol. c. Document the patient's rhythm and assess the patient's response to the rhythm. d. Call the health care provider before giving the next dose of metoprolol (Lopressor).

ANS: D The patient has progressive first-degree atrioventricular (AV) block, and the -blocker should be held until discussing the medication with the health care provider. Documentation and assessment are appropriate but not fully adequate responses. The patient with first-degree AV block usually is asymptomatic, and a pacemaker is not indicated. Atropine is sometimes used for symptomatic bradycardia, but there is no indication that this patient is symptomatic. **BBs can worsen heart blocks** DIF: Cognitive Level: Apply (application) REF: 798 OBJ: Special Questions: Prioritization TOP: Nursing Process: Implementation MSC: NCLEX: Physiological Integrity

20. A patient develops sinus bradycardia at a rate of 32 beats/minute, has a blood pressure (BP) of 80/42 mm Hg, and is complaining of feeling faint. Which actions should the nurse take next? a. Recheck the heart rhythm and BP in 5 minutes. b. Have the patient perform the Valsalva maneuver. c. Give the scheduled dose of diltiazem (Cardizem). d. Apply the transcutaneous pacemaker (TCP) pads.

ANS: D The patient is experiencing symptomatic bradycardia, and treatment with TCP is appropriate. Continued monitoring of the rhythm and BP is an inadequate response. Calcium channel blockers will further decrease the heart rate, and the diltiazem should be held. The Valsalva maneuver will further decrease the rate. DIF: Cognitive Level: Apply (application) REF: 804 OBJ: Special Questions: Prioritization TOP: Nursing Process: Implementation MSC: NCLEX: Physiological Integrity

11. After providing a patient with discharge instructions on the management of a new permanent pacemaker, the nurse knows that teaching has been effective when the patient states a. "I will avoid cooking with a microwave oven or being near one in use." b. "It will be 1 month before I can take a bath or return to my usual activities." c. "I will notify the airlines when I make a reservation that I have a pacemaker." d. "I won't lift the arm on the pacemaker side up very high until I see the doctor."

ANS: D The patient is instructed to avoid lifting the arm on the pacemaker side above the shoulder to avoid displacing the pacemaker leads. The patient should notify airport security about the presence of a pacemaker before going through the metal detector, but there is no need to notify the airlines when making a reservation. Microwave oven use does not affect the pacemaker. The insertion procedure involves minor surgery that will have a short recovery period. DIF: Cognitive Level: Apply (application) REF: 805 TOP: Nursing Process: Evaluation MSC: NCLEX: Physiological Integrity

The nurse teaches the patient being evaluated for rhythm disturbances with a Holter monitor to a. connect the recorder to a computer once daily. b. exercise more than usual while the monitor is in place. c. remove the electrodes when taking a shower or tub bath. d. keep a diary of daily activities while the monitor is worn.

ANS: D The patient is instructed to keep a diary describing daily activities while Holter monitoring is being accomplished to help correlate any rhythm disturbances with patient activities. Patients are taught that they should not take a shower or bath during Holter monitoring and that they should continue with their usual daily activities. The recorder stores the information about the patient's rhythm until the end of the testing, when it is removed and the data are analyzed

A patient admitted with acute dyspnea is newly diagnosed with dilated cardiomyopathy. Which information will the nurse plan to teach the patient about managing this disorder? a. A heart transplant should be scheduled as soon as possible. b. Elevating the legs above the heart will help relieve dyspnea. c. Careful compliance with diet and medications will prevent heart failure. d. Notify the doctor about any symptoms of heart failure such as shortness of breath.

ANS: D The patient should be instructed to notify the health care provider about any worsening of heart failure symptoms. Because dilated cardiomyopathy does not respond well to therapy, even patients with good compliance with therapy may have recurrent episodes of heart failure. Elevation of the legs above the heart will worsen symptoms (although this approach is appropriate for a patient with hypertrophic cardiomyopathy). The patient with terminal or end-stage cardiomyopathy may consider heart transplantation.

31. When caring for a patient with acute coronary syndrome who has returned to the coronary care unit after having angioplasty with stent placement, the nurse obtains the following assessment data. Which data indicate the need for immediate action by the nurse? a. Heart rate 102 beats/min b. Pedal pulses 1+ bilaterally c. Blood pressure 103/54 mm Hg d. Chest pain level 7 on a 0 to 10 point scale

ANS: D The patient's chest pain indicates that restenosis of the coronary artery may be occurring and requires immediate actions, such as administration of oxygen and nitroglycerin, by the nurse. The other information indicates a need for ongoing assessments by the nurse.

A patient who was found unconscious in a burning house is brought to the emergency department by ambulance. The nurse notes that the patient's skin color is bright red. Which action should the nurse take first? a. Insert two large-bore IV lines. b. Check the patient's orientation. c. Assess for singed nasal hair and dark oral mucous membranes. d. Place the patient on 100% oxygen using a non-rebreather mask.

ANS: D The patient's history and skin color suggest carbon monoxide poisoning, which should be treated by rapidly starting oxygen at 100%. The other actions can be taken after the action to correct gas exchange.

A patient in the emergency department complains of back pain and difficulty breathing 15 minutes after a transfusion of packed red blood cells is started. The nurse's first action should be to a. administer oxygen therapy at a high flow rate. b. obtain a urine specimen to send to the laboratory. c. notify the health care provider about the symptoms. d. disconnect the transfusion and infuse normal saline.

ANS: D The patient's symptoms indicate a possible acute hemolytic reaction caused by the transfusion. The first action should be to disconnect the transfusion and infuse normal saline. The other actions also are needed but are not the highest priority

When reviewing the 12-lead electrocardiograph (ECG) for a healthy 79-year-old patient who is having an annual physical examination, what will be of most concern to the nurse? a. The PR interval is 0.21 seconds. b. The QRS duration is 0.13 seconds. c. There is a right bundle-branch block. d. The heart rate (HR) is 42 beats/minute.

ANS: D The resting HR does not change with aging, so the decrease in HR requires further investigation. Bundle-branch block and slight increases in PR interval or QRS duration are common in older individuals because of increases in conduction time through the AV node, bundle of His, and bundle branches

Eight hours after a thermal burn covering 50% of a patient's total body surface area (TBSA) the nurse assesses the patient. Which information would be a priority to communicate to the health care provider? a. Blood pressure is 95/48 per arterial line. b. Serous exudate is leaking from the burns. c. Cardiac monitor shows a pulse rate of 108. d. Urine output is 20 mL per hour for the past 2 hours.

ANS: D The urine output should be at least 0.5 to 1.0 mL/kg/hr during the emergent phase, when the patient is at great risk for hypovolemic shock. The nurse should notify the health care provider because a higher IV fluid rate is needed. BP during the emergent phase should be greater than 90 systolic, and the pulse rate should be less than 120. Serous exudate from the burns is expected during the emergent phase.

A young adult patient who is in the rehabilitation phase after having deep partial-thickness face and neck burns has a nursing diagnosis of disturbed body image. Which statement by the patient indicates that the problem is resolving? a. "I'm glad the scars are only temporary." b. "I will avoid using a pillow, so my neck will be OK." c. "I bet my boyfriend won't even want to look at me anymore." d. "Do you think dark beige makeup foundation would cover this scar on my cheek?"

ANS: D The willingness to use strategies to enhance appearance is an indication that the disturbed body image is resolving. Expressing feelings about the scars indicates a willingness to discuss appearance, but not resolution of the problem. Because deep partial-thickness burns leave permanent scars, a statement that the scars are temporary indicates denial rather than resolution of the problem. Avoiding using a pillow will help prevent contractures, but it does not address the problem of disturbed body image.

38. After receiving change-of-shift report about the following four patients, which patient should the nurse assess first? a. 39-year-old with pericarditis who is complaining of sharp, stabbing chest pain b. 56-year-old with variant angina who is to receive a dose of nifedipine (Procardia) c. 65-year-old who had a myocardial infarction (MI) 4 days ago and is anxious about the planned discharge d. 59-year-old with unstable angina who has just returned to the unit after having a percutaneous coronary intervention (PCI)

ANS: D This patient is at risk for bleeding from the arterial access site for the PCI, so the nurse should assess the patient's blood pressure, pulse, and the access site immediately. The other patients should also be assessed as quickly as possible, but assessment of this patient has the highest priority.

A patient with pancytopenia has a bone marrow aspiration from the left posterior iliac crest. Which action would be important for the nurse to take after the procedure? a. Elevate the head of the bed to 45 degrees. b. Apply a sterile 2-inch gauze dressing to the site. c. Use a half-inch sterile gauze to pack the wound. d. Have the patient lie on the left side for 1 hour.

ANS: D To decrease the risk for bleeding, the patient should lie on the left side for 30 to 60 minutes. After a bone marrow biopsy, the wound is small and will not be packed with gauze. A pressure dressing is used to cover the aspiration site. There is no indication to elevate the patient's head.

8. A patient who has had chest pain for several hours is admitted with a diagnosis of rule out acute myocardial infarction (AMI). Which laboratory test should the nurse monitor to help determine whether the patient has had an AMI? a. Myoglobin b. Homocysteine c. C-reactive protein d. Cardiac-specific troponin

ANS: D Troponin levels increase about 4 to 6 hours after the onset of myocardial infarction (MI) and are highly specific indicators for MI. Myoglobin is released within 2 hours of MI, but it lacks specificity and its use is limited. The other laboratory data are useful in determining the patient's risk for developing coronary artery disease (CAD) but are not helpful in determining whether an acute MI is in progress.

While caring for a 23-year-old patient with mitral valve prolapse (MVP) without valvular regurgitation, the nurse determines that discharge teaching has been effective when the patient states that it will be necessary to a. take antibiotics before any dental appointments. b. limit physical activity to avoid stressing the heart. c. take an aspirin a day to prevent clots from forming on the valve. d. avoid use of over-the-counter (OTC) medications that contain stimulant drugs.

ANS: D Use of stimulant medications should be avoided by patients with MVP because these may exacerbate symptoms. Daily aspirin and restricted physical activity are not needed by patients with mild MVP. Antibiotic prophylaxis is needed for patients with MVP with regurgitation but will not be necessary for this patient. *MVPs don't need uppers!*

While doing the admission assessment for a thin 76-year-old patient, the nurse observes pulsation of the abdominal aorta in the epigastric area. Which action should the nurse take? a. Teach the patient about aneurysms. b. Notify the hospital rapid response team. c. Instruct the patient to remain on bed rest. d. Document the finding in the patient chart.

ANS: D Visible pulsation of the abdominal aorta is commonly observed in the epigastric area for thin individuals. The nurse should simply document the finding in the admission assessment. Unless there are other abnormal findings (such as a bruit, pain, or hyper/hypotension) associated with the pulsation, the other actions are not necessary

A new order for IV gentamicin (Garamycin) 60 mg BID is received for a patient with diabetes who has pneumonia. When evaluating for adverse effects of the medication, the nurse will plan to monitor the patient's a. urine osmolality. b. serum potassium. c. blood glucose level. d. blood urea nitrogen (BUN) and creatinine.

ANS: D When a patient at risk for chronic kidney disease (CKD) receives a nephrotoxic medication, it is important to monitor renal function with BUN and creatinine levels. The other laboratory values would not be useful in determining the effect of the gentamicin

During the emergent phase of burn care, which assessment will be most useful in determining whether the patient is receiving adequate fluid infusion? a. Check skin turgor. b. Monitor daily weight. c. Assess mucous membranes. d. Measure hourly urine output.

ANS: D When fluid intake is adequate, the urine output will be at least 0.5 to 1 mL/kg/hour. The patient's weight is not useful in this situation because of the effects of third spacing and evaporative fluid loss. Mucous membrane assessment and skin turgor also may be used, but they are not as adequate in determining that fluid infusions are maintaining adequate perfusion.

Which action could the nurse delegate to unlicensed assistive personnel (UAP) trained as electrocardiogram (ECG) technicians working on the cardiac unit? a. Select the best lead for monitoring a patient with an admission diagnosis of Dressler syndrome. b. Obtain a list of herbal medications used at home while admitting a new patient with pericarditis. c. Teach about the need to monitor the weight daily for a patient who has hypertrophic cardiomyopathy. d. Check the heart monitor for changes in rhythm while a patient who had a valve replacement ambulates.

ANS: D *UAP who is a trained ECG tech* Under the supervision of registered nurses (RNs), UAP check the patient's cardiac monitor and obtain information about changes in heart rate and rhythm with exercise. Teaching and obtaining information about home medications (prescribed or complementary) and selecting the best leads for monitoring patients require more critical thinking and should be done by the RN.

For which problem is percutaneous coronary intervention (PCI) most clearly indicated? Chronic stable angina Left-sided heart failure Coronary artery disease Acute myocardial infarction

Acute myocardial infarction PCI is indicated to restore coronary perfusion in cases of myocardial infarction. Chronic stable angina and coronary artery disease are normally treated with more conservative measures initially. PCI is not relevant to the pathophysiology of heart failure.

A patient reports dizziness and shortness of breath and is admitted with a dysrhythmia. Which medication, if ordered, requires the nurse to carefully monitor the patient for asystole? Digoxin Adenosine Metoprolol Atropine sulfate

Adenosine IV adenosine is the first drug of choice to convert supraventricular tachycardia to a normal sinus rhythm. Adenosine is administered IV rapidly (over 1 or 2 seconds) followed by a rapid, normal saline flush. The nurse should monitor the patient's electrocardiogram continuously because a brief period of asystole after adenosine administration is common and expected. Atropine sulfate increases heart rate, whereas lanoxin and metoprolol slow the heart rate.

The patient has been part of a community emergency response team (CERT) for a tropical storm in Dallas with temperatures near 100°F (37.7°C) for the past 2 weeks. When assessing the patient, the nurse finds hypotension, body temperature of 104°F (40°C), dry and ashen skin, and neurologic symptoms. What treatments should the nurse anticipate (select all that apply.)? Select all that apply. Administer 100% O2. Immerse in an ice bath. Administer cool IV fluids. Cover the patient to prevent chilling. Administer acetaminophen (Tylenol). Administer chlorpromazine for shivering.

Administer 100% O2. Administer cool IV fluids. Administer chlorpromazine for shivering. The patient is experiencing heatstroke. Treatment focuses first on stabilizing the patient's ABC and rapidly reducing the core temperature. Administration of 100% O2 compensates for the patient's hypermetabolic state. Cooling the body with IV fluids is effective. Immersion in an ice bath will cause shivers that increase core temperature, so a cool water bath should be used for conductive cooling. Removing the clothing, covering the patient with wet sheets, and placing the patient in front of a fan will cause evaporative cooling. If shivering ensues, treatment with chlorpromazine is indicated. Shivering increases core temperature due to the heat generated by muscle activity. Excessive covers will not be used. Acetaminophen will not be effective because the increase in temperature is not related to infection.

When entering the grocery store, a patient trips on the curb and sprains the right ankle. Which initial care is appropriate (select all that apply.)? Select all that apply. Apply ice directly to the skin. Apply heat to the ankle every 2 hours. Administer antiinflammatory medication. Compress ankle using an elastic bandage. Rest and elevate the ankle above the heart. Perform passive and active range of motion.

Administer antiinflammatory medication. Compress ankle using an elastic bandage. Rest and elevate the ankle above the heart. Appropriate care for a sprain is represented with the acronym RICE (rest, ice, compression, and elevation). Antiinflammatory medication should be used to decrease swelling if not contraindicated for the patient. After the injury, the ankle should be immobilized and rested. Prolonged immobilization is not required unless there is significant injury. Ice is indicated, but will cause tissue damage if applied directly to the skin. Apply ice to sprains as soon as possible and leave in place for 20 to 30 minutes at a time. Moist heat may be applied 24 to 48 hours after the injury.

A 50-yr-old woman with a suspected brain tumor is scheduled for a CT scan with contrast media. The nurse notifies the physician that the patient reported an allergy to shellfish. Which response by the physician should the nurse question? Infuse IV diphenhydramine before the procedure. Administer lorazepam (Ativan) before the procedure. Complete the CT scan without the use of contrast media. Premedicate with hydrocortisone sodium succinate (Solu-Cortef).

Administer lorazepam (Ativan) before the procedure. An individual with an allergy to shellfish is at an increased risk to develop anaphylactic shock if contrast media is injected for a CT scan. To prevent anaphylactic shock, the nurse should always confirm the patient's allergies before diagnostic procedures (e.g., CT scan with contrast media). Appropriate interventions may include cancelling the procedure, completing the procedure without contrast media, or premedication with diphenhydramine or hydrocortisone. IV fluids may be given to promote renal clearance of the contrast media and prevent renal toxicity and acute kidney injury. The use of an antianxiety agent such as lorazepam would not be effective in preventing an allergic reaction to the contrast media.

A 47-yr-old man who was lost in the mountains for 2 days is admitted to the emergency department with cold exposure and a core body temperature of 86.6ºF (30.3ºC). Which nursing action is most important? Administer warmed IV fluids. Position patient under a radiant heat lamp. Place an air-filled warming blanket on the patient. Immerse the extremities in a water bath (102° to 108°F [38.9° to 42.2°C]).

Administer warmed IV fluids. A patient with a core body temperature of 86.6ºF (30.3ºC) has moderate hypothermia. Active core rewarming is used for moderate to severe hypothermia and includes administration of warmed IV fluids (109.4ºF [43ºC]). Patients with moderate to severe hypothermia should have the core warmed before the extremities to prevent after drop (or further drop in core temperature). This occurs when cold peripheral blood returns to the central circulation. Use passive or active external rewarming for mild hypothermia. Active external rewarming involves fluid-filled warming blankets or radiant heat lamps. Immersion of extremities in a water bath is indicated for frostbite.

An 18-yr-old young woman has been admitted to the emergency department after ingesting an entire bottle of chewable multivitamins in a suicide attempt. The nurse should anticipate which intervention? Induced vomiting Whole-bowel irrigation Administration of activated charcoal Administration of fresh frozen plasma

Administration of activated charcoal Among the most common treatments for poisoning is the administration of activated charcoal. Induced vomiting is not typically indicated, and there is no need for plasma administration. Whole-bowel irrigation may be used as an adjunct therapy later in treatment, but the use of activated charcoal is central to the treatment of poisonings.

A frail 72-year-old woman with stage 3 chronic kidney disease is cared for at home by her family. The patient has a history of taking many over-the-counter medications. Which over-the-counter medications should the nurse teach the patient to avoid? A.Aspirin B. Acetaminophen (Tylenol) C. Diphenhydramine (Benadryl) D.Aluminum hydroxide (Amphogel)

Aluminum hydroxide (Amphogel) Antacids (that contain magnesium and aluminum) should be avoided because patients with kidney disease are unable to excrete these substances. Also, some antacids contain high levels of sodium that further increase blood pressure. Acetaminophen and aspirin (if taken for a short period of time) are usually safe for patients with kidney disease. Antihistamines may be used, but combination drugs that contain pseudoephedrine may increase blood pressure and should be avoided.

A patient in the emergent/resuscitative phase of a burn injury has had blood work and arterial blood gases drawn. Upon analysis of the patient's laboratory studies, the nurse will expect the results to indicate what? A) Hyperkalemia, hyponatremia, elevated hematocrit, and metabolic acidosis B) Hypokalemia, hypernatremia, decreased hematocrit, and metabolic acidosis C) Hyperkalemia, hypernatremia, decreased hematocrit, and metabolic alkalosis D) Hypokalemia, hyponatremia, elevated hematocrit, and metabolic alkalosis

Ans: A Feedback: Fluid and electrolyte changes in the emergent/resuscitative phase of a burn injury include hyperkalemia related to the release of potassium into the extracellular fluid, hyponatremia from large amounts of sodium lost in trapped edema fluid, hemoconcentration that leads to an increased hematocrit, and loss of bicarbonate ions that results in metabolic acidosis.

A patient has sustained a severe burn injury and is thought to have an impaired intestinal mucosal barrier. Since this patient is considered at an increased risk for infection, what intervention will best assist in avoiding increased intestinal permeability and prevent early endotoxin translocation? A) Early enteral feeding B) Administration of prophylactic antibiotics C) Bowel cleansing procedures D) Administration of stool softeners

Ans: A Feedback: If the intestinal mucosa receives some type of protection against permeability change, infection could be avoided. Early enteral feeding is one step to help avoid this increased intestinal permeability and prevent early endotoxin translocation. Antibiotics are seldom prescribed prophylactically because of the risk of promoting resistant strains of bacteria. A bowel cleansing procedure would not be ordered for this patient. The administration of stool softeners would not assist in avoiding increased intestinal permeability and prevent early endotoxin translocation.

An emergency department nurse has just received a patient with burn injuries brought in by ambulance. The paramedics have started a large-bore IV and covered the burn in cool towels. The burn is estimated as covering 24% of the patient's body. How should the nurse best address the pathophysiologic changes resulting from major burns during the initial burn-shock period? A) Administer IV fluids B) Administer broad-spectrum antibiotics C) Administer IV potassium chloride D) Administer packed red blood cells

Ans: A Feedback: Pathophysiologic changes resulting from major burns during the initial burn-shock period include massive fluid losses. Addressing these losses is a major priority in the initial phase of treatment. Antibiotics and PRBCs are not normally administered. Potassium chloride would exacerbate the patient's hyperkalemia.

A home care nurse is performing a visit to a patient's home to perform wound care following the patient's hospital treatment for severe burns. While interacting with the patient, the nurse should assess for evidence of what complication? A) Psychosis B) Post-traumatic stress disorder C) Delirium D) Vascular dementia

Ans: B Feedback: Post-traumatic stress disorder (PTSD) is the most common psychiatric disorder in burn survivors, with a prevalence that may be as high as 45%. As a result, it is important for the nurse to assess for this complication of burn injuries. Psychosis, delirium, and dementia are not among the noted psychiatric and psychosocial complications of burns.

An emergency department nurse learns from the paramedics that they are transporting a patient who has suffered injury from a scald from a hot kettle. What variables will the nurse consider when determining the depth of burn? A) The causative agent B) The patient's preinjury health status C) The patient's prognosis for recovery D) The circumstances of the accident

Ans: A Feedback: The following factors are considered in determining the depth of a burn: how the injury occurred, causative agent (such as flame or scalding liquid), temperature of the burning agent, duration of contact with the agent, and thickness of the skin. The patient's preinjury status, circumstances of the accident, and prognosis for recovery are important, but are not considered when determining the depth of the burn.

A patient is brought to the emergency department with a burn injury. The nurse knows that the first systemic event after a major burn injury is what? A) Hemodynamic instability B) Gastrointestinal hypermotility C) Respiratory arrest D) Hypokalemia

Ans: A Feedback: The initial systemic event after a major burn injury is hemodynamic instability, which results from loss of capillary integrity and a subsequent shift of fluid, sodium, and protein from the intravascular space into the interstitial spaces. This precedes GI changes. Respiratory arrest may or may not occur, largely depending on the presence or absence of smoke inhalation. Hypokalemia does not take place in the initial phase of recovery.

A nurse has reported for a shift at a busy burns and plastics unit in a large university hospital. Which patient is most likely to have life-threatening complications? A) A 4-year-old scald victim burned over 24% of the body B) A 27-year-old male burned over 36% of his body in a car accident C) A 39-year-old female patient burned over 18% of her body D) A 60-year-old male burned over 16% of his body in a brush fire

Ans: A Feedback: Young children and the elderly continue to have increased morbidity and mortality when compared to other age groups with similar injuries and present a challenge for burn care. This is an important factor when determining the severity of injury and possible outcome for the patient.

A patient is in the acute phase of a burn injury. One of the nursing diagnoses in the plan of care is Ineffective Coping Related to Trauma of Burn Injury. What interventions appropriately address this diagnosis? Select all that apply. A) Promote truthful communication. B) Avoid asking the patient to make decisions. C) Teach the patient coping strategies. D) Administer benzodiazepines as ordered. E) Provide positive reinforcement.

Ans: A, C, E Feedback: The nurse can assist the patient to develop effective coping strategies by setting specific expectations for behavior, promoting truthful communication to build trust, helping the patient practice appropriate strategies, and giving positive reinforcement when appropriate. The patient may benefit from being able to make decisions regarding his or her care. Benzodiazepines may be needed for short-term management of anxiety, but they are not used to enhance coping.

A patient's burns have required a homograft. During the nurse's most recent assessment, the nurse observes that the graft is newly covered with purulent exudate. What is the nurse's most appropriate response? A) Perform mechanical dÈbridement to remove the exudate and prevent further infection. B) Inform the primary care provider promptly because the graft may need to be removed. C) Perform range of motion exercises to increase perfusion to the graft site and facilitate healing. D) Document this finding as an expected phase of graft healing.

Ans: B Feedback: An infected graft may need to be removed, thus the care provider should be promptly informed. ROM exercises will not resolve this problem and the nurse would not independently perform dÈbridement.

A patient has experienced an electrical burn and has developed thick eschar over the burn site. Which of the following topical antibacterial agents will the nurse expect the physician to order for the wound? A) Silver sulfadiazine 1% (Silvadene) water-soluble cream B) Mafenide acetate 10% (Sulfamylon) hydrophilic-based cream C) Silver nitrate 0.5% aqueous solution D) Acticoat

Ans: B Feedback: Mafenide acetate 10% hydrophilic-based cream is the agent of choice when there is a need to penetrate thick eschar. *Silver products do not penetrate eschar;* Acticoat is a type of silver dressing.

The nurse is preparing the patient for mechanical debridement and informs the patient that this will involve which of the following procedures? A) A spontaneous separation of dead tissue from the viable tissue B) Removal of eschar until the point of pain and bleeding occurs C) Shaving of burned skin layers until bleeding, viable tissue is revealed D) Early closure of the wound

Ans: B Feedback: Mechanical dÈbridementcan be achieved through the use of surgical scissors, scalpels, or forceps to remove the eschar until the point of pain and bleeding occurs. Mechanical dÈbridement can also be accomplished through the use of topical enzymatic dÈbridement agents. The spontaneous separation of dead tissue from the viable tissue is an example of natural dÈbridement. Shaving the burned skin layers and early wound closure are examples of surgical dÈbridement.

A patient who was burned in a workplace accident has completed the acute phase of treatment and the plan of care has been altered to prioritize rehabilitation. What nursing action should be prioritized during this phase of treatment? A) Monitoring fluid and electrolyte imbalances B) Providing education to the patient and family C) Treating infection D) Promoting thermoregulation

Ans: B Feedback: Patient and family education is a priority during rehabilitation. There should be no fluid and electrolyte imbalances in the rehabilitation phase. The presence of impaired thermoregulation or infection would suggest that the patient is still in the acute phase of burn recovery.

A nurse who is taking care of a patient with burns is asked by a family member why the patient is losing so much weight. The patient is currently in the intermediate phase of recovery. What would be the nurse's most appropriate response to the family member? A) He's on a calorie-restricted diet in order to divert energy to wound healing. B) His body has consumed his fat deposits for fuel because his calorie intake is lower than normal. C) He actually hasn't lost weight. Instead, there's been a change in the distribution of his body fat. D) He lost many fluids while he was being treated in the emergency phase of burn care.

Ans: B Feedback: Patients lose a great deal of weight during recovery from severe burns. Reserve fat deposits are catabolized as a result of hypermetabolism. Patients are not placed on a calorie restriction during recovery and fluid losses would not account for weight loss later in the recovery period. Changes in the overall distribution of body fat do not occur.

A nurse who provides care on a burn unit is preparing to apply a patient's ordered topical antibiotic ointment. What action should the nurse perform when administering this medication? A) Apply the new ointment without disturbing the existing layer of ointment. B) Apply the ointment using a sterile tongue depressor. C) Apply a layer of ointment approximately 1/16 inch thick. D) Gently irrigate the wound bed after applying the antibiotic ointment.

Ans: C Feedback: After removing the old ointment from the wound bed, the nurse should apply a layer of ointment 1/16-inch thick using clean gloves. The wound would not be irrigated after application of new ointment.

A triage nurse in the emergency department (ED) receives a phone call from a frantic father who saw his 4-year-old child tip a pot of boiling water onto her chest. The father has called an ambulance. What would the nurse in the ED receiving the call instruct the father to do? A) Cover the burn with ice and secure with a towel. B) Apply butter to the area that is burned. C) Immerse the child in a cool bath. D) Avoid touching the burned area under any circumstances.

Ans: C Feedback: After the flames or heat source have been removed or extinguished, the burned area and adherent clothing are soaked with cool water briefly to cool the wound and halt the burning process. Cool water is the best first-aid measure. Ice and butter are contraindicated. Appropriate first aid necessitates touching the burn.

A patient experienced a 33% TBSA burn 72 hours ago. The nurse observes that the patient's hourly urine output has been steadily increasing over the past 24 hours. How should the nurse best respond to this finding? A) Obtain an order to reduce the rate of the patient's IV fluid infusion. B) Report the patient's early signs of acute kidney injury (AKI). C) Recognize that the patient is experiencing an expected onset of diuresis. D) Administer sodium chloride as ordered to compensate for this fluid loss.

Ans: C Feedback: As capillaries regain integrity, 48 or more hours after the burn, fluid moves from the interstitial to the intravascular compartment and diuresis begins. This is an expected development and does not require a reduction in the IV infusion rate or the administration of NaCl. Diuresis is not suggestive of AKI.

A patient with severe burns is admitted to the intensive care unit to stabilize and begin fluid resuscitation before transport to the burn center. The nurse should monitor the patient closely for what signs of the onset of burn shock? A) Confusion B) High fever C) Decreased blood pressure D) Sudden agitation

Ans: C Feedback: As fluid loss continues and vascular volume decreases, cardiac output continues to decrease and the blood pressure drops, marking the onset of burn shock. Shock and the accompanying hemodynamic changes are not normally accompanied by confusion, fever, or agitation.

A patient with a partial-thickness burn injury had Biobrane applied 2 weeks ago. The nurse notices that the Biobrane is separating from the burn wound. What is the nurse's most appropriate intervention? A) Reinforce the Biobrane dressing with another piece of Biobrane. B) Remove the Biobrane dressing and apply a new dressing. C) Trim away the separated Biobrane. D) Notify the physician for further emergency-related orders.

Ans: C Feedback: As the Biobrane gradually separates, it is trimmed, leaving a healed wound. When the Biobrane dressing adheres to the wound, the wound remains stable and the Biobrane can remain in place for 3 to 4 weeks. There is no need to reinforce the Biobrane nor to remove it and apply a new dressing. There is not likely any need to notify the physician for further orders.

A patient has experienced burns to his upper thighs and knees. Following the application of new wound dressings, the nurse should perform what nursing action? A) Instruct the patient to keep the wound site in a dependent position. B) Administer PRN analgesia as ordered. C) Assess the patient's peripheral pulses distal to the dressing. D) Assist with passive range of motion exercises to set the new dressing.

Ans: C Feedback: Dressings can impede circulation if they are wrapped too tightly. The peripheral pulses must be checked frequently and burned extremities elevated. Dependent positioning does not need to be maintained. PRN analgesics should be administered prior to the dressing change. ROM exercises do not normally follow a dressing change.

The current phase of a patient's treatment for a burn injury prioritizes wound care, nutritional support, and prevention of complications such as infection. Based on these care priorities, the patient is in what phase of burn care? A) Emergent B) Immediate resuscitative C) Acute D) Rehabilitation

Ans: C Feedback: The acute or intermediate phase of burn care follows the emergent/resuscitative phase and begins 48 to 72 hours after the burn injury. During this phase, attention is directed toward continued assessment and maintenance of respiratory and circulatory status, fluid and electrolyte balance, and gastrointestinal function. Infection prevention, burn wound care (i.e., wound cleaning, topical antibacterial therapy, wound dressing, dressing changes, wound dÈbridement, and wound grafting), pain management, and nutritional support are priorities at this stage. Priorities during the emergent or immediate resuscitative phase include first aid, prevention of shock and respiratory distress, detection and treatment of concomitant injuries, and initial wound assessment and care. The priorities during the rehabilitation phase include prevention of scars and contractures, rehabilitation, functional and cosmetic reconstruction, and psychosocial counseling.

A nurse is developing a care plan for a patient with a partial-thickness burn, and determines that an appropriate goal is to maintain position of joints in alignment. What is the best rationale for this intervention? A) To prevent neuropathies B) To prevent wound breakdown C) To prevent contractures D) To prevent heterotopic ossification

Ans: C Feedback: To prevent the complication of contractures, the nurse will establish a goal to maintain position of joints in alignment. Gentle range of motion exercises and a consult to PT and OT for exercises and positioning recommendations are also appropriate interventions for the prevention of contractures. Joint alignment is not maintained specifically for preventing neuropathy, wound breakdown, or heterotopic ossification.

A patient is brought to the emergency department from the site of a chemical fire, where he suffered a burn that involves the epidermis, dermis, and the muscle and bone of the right arm. On inspection, the skin appears charred. Based on these assessment findings, what is the depth of the burn on the patient's arm? A) Superficial partial-thickness B) Deep partial-thickness C) Full partial-thickness D) Full-thickness

Ans: D Feedback: A full-thickness burn involves total destruction of the epidermis and dermis and, in some cases, underlying tissue as well. Wound color ranges widely from white to red, brown, or black. The burned area is painless because the nerve fibers are destroyed. The wound can appear leathery; hair follicles and sweat glands are destroyed. Edema may also be present. Superficial partial-thickness burns involve the epidermis and possibly a portion of the dermis; the patient will experience pain that is soothed by cooling. Deep partial-thickness burns involve the epidermis, upper dermis, and portion of the deeper dermis; the patient will complain of pain and sensitivity to cold air. Full partial thickness is not a depth of burn.

The nurse caring for a patient who is recovering from full-thickness burns is aware of the patient's risk for contracture and hypertrophic scarring. How can the nurse best mitigate this risk? A) Apply skin emollients as ordered after granulation has occurred. B) Keep injured areas immobilized whenever possible to promote healing. C) Administer oral or IV corticosteroids as ordered. D) Encourage physical activity and range of motion exercises.

Ans: D Feedback: Exercise and the promotion of mobility can reduce the risk of contracture and hypertrophic scarring. Skin emollients are not normally used in the treatment of burns, and these do not prevent scarring. Steroids are not used to reduce scarring, as they also slow the healing process.

A patient's burns are estimated at 36% of total body surface area; fluid resuscitation has been ordered in the emergency department. After establishing intravenous access, the nurse should anticipate the administration of what fluid? A) 0.45% NaCl with 20 mEq/L KCl B) 0.45% NaCl with 40 mEq/L KCl C) Normal saline D) Lactated Ringer's

Ans: D Feedback: Fluid resuscitation with lactated Ringers (LR) should be initiated using the American Burn Association's (ABA) fluid resuscitation formulas. LR is the crystalloid of choice because its composition and osmolality most closely resemble plasma and because use of normal saline is associated with hyperchloremic acidosis. Potassium chloride solutions would exacerbate the hyperkalemia that occurs following burn injuries.

An occupational health nurse is called to the floor of a factory where a worker has sustained a flash burn to the right arm. The nurse arrives and the flames have been extinguished. The next step is to cool the burn. How should the nurse cool the burn? A) Apply ice to the site of the burn for 5 to 10 minutes. B) Wrap the patient's affected extremity in ice until help arrives. C) Apply an oil-based substance or butter to the burned area until help arrives. D) Wrap cool towels around the affected extremity intermittently.

Ans: D Feedback: Once the burn has been sustained, the application of cool water is the best first-aid measure. Soaking the burn area intermittently in cool water or applying cool towels gives immediate and striking relief from pain, and limits local tissue edema and damage. However, never apply ice directly to the burn, never wrap the person in ice, and never use cold soaks or dressings for longer than several minutes; such procedures may worsen the tissue damage and lead to hypothermia in people with large burns. Butter is contraindicated.

A nurse is caring for a patient who has sustained a deep partial-thickness burn injury. In prioritizing the nursing diagnoses for the plan of care, the nurse will give the highest priority to what nursing diagnosis? A) Activity Intolerance B) Anxiety C) Ineffective Coping D) Acute Pain

Ans: D Feedback: Pain is inevitable during recovery from any burn injury. Pain in the burn patient has been described as one of the most severe causes of acute pain. Management of the often-severe pain is one of the most difficult challenges facing the burn team. While the other nursing diagnoses listed are valid, the presence of pain may contribute to these diagnoses. Management of the patient's pain is the priority, as it may have a direct correlation to the other listed nursing diagnoses.

A nurse is caring for a patient with burns who is in the later stages of the acute phase of recovery. The plan of nursing care should include which of the following nursing actions? A) Maintenance of bed rest to aid healing B) Choosing appropriate splints and functional devices C) Administration of beta adrenergic blockers D) Prevention of venous thromboembolism

Ans: D Feedback: Prevention of deep vein thrombosis (DVT) is an important factor in care. Early mobilization of the patient is important. The nurse monitors the splints and functional devices, but these are selected by occupational and physical therapists. The hemodynamic changes accompanying burns do not normally require the use of beta blockers.

A nurse is performing a home visit to a patient who is recovering following a long course of inpatient treatment for burn injuries. When performing this home visit, the nurse should do which of the following? A) Assess the patient for signs of electrolyte imbalances. B) Administer fluids as ordered. C) Assess the risk for injury recurrence. D) Assess the patient's psychosocial state.

Ans: D Feedback: Recovery from burns can be psychologically challenging; the nurse's assessments must address this reality. Fluid and electrolyte imbalances are infrequent during the rehabilitation phase of recovery. Burns are not typically a health problem that tends to recur; the experience of being burned tends to foster vigilance.

An emergency department nurse has just admitted a patient with a burn. What characteristic of the burn will primarily determine whether the patient experiences a systemic response to this injury? A) The length of time since the burn B) The location of burned skin surfaces C) The source of the burn D) The total body surface area (TBSA) affected by the burn

Ans: D Feedback: Systemic effects are a result of several variables. However, TBSA and wound severity are considered the major factors that affect the presence or absence of systemic effects.

While performing a patient's ordered wound care for the treatment of a burn, the patient has made a series of sarcastic remarks to the nurse and criticized her technique. How should the nurse best interpret this patient's behavior? A) The patient may be experiencing an adverse drug reaction that is affecting his cognition and behavior. B) The patient may be experiencing neurologic or psychiatric complications of his injuries. C) The patient may be experiencing inconsistencies in the care that he is being provided. D) The patient may be experiencing anger about his circumstances that he is deflecting toward the nurse.

Ans: D Feedback: The patient may experience feelings of anger. The anger may be directed outward toward those who escaped unharmed or toward those who are now providing care. While drug reactions, complications, and frustrating inconsistencies in care cannot be automatically ruled out, it is not uncommon for anger to be directed at caregivers.

The nurse provides information to a patient who was exposed to anthrax by inhalation. The nurse determines the teaching has been successful if the patient makes which statement? A. "Anthrax can be spread by person-to-person contact." B. "It is not necessary to receive the anthrax vaccine." C. "An antibiotic will be prescribed for 2 months." D. "Antibiotics are only indicated for an active infection."

Answer: C. "An antibiotic will be prescribed for 2 months." Rationale: Postexposure prophylaxis includes a 60-day course of antibiotics. Ciprofloxacin (Cipro) is the treatment of choice. Anthrax is not spread by person-to-person contact; anthrax is spread by direct contact with the bacteria and its spores. The patient may receive the anthrax vaccine (three doses); if vaccinated, the course of antibiotic therapy is reduced to 30 days. Antibiotics are indicated after exposure to inhaled anthrax. Ch. 69

Which patient should the nurse prepare to transfer to a regional burn center? A. A 53-year-old patient with a chemical burn to the anterior chest and neck B. A 25-year-old pregnant patient with a carboxyhemoglobin level of 1.5% C. A 42-year-old patient who is scheduled for skin grafting of a burn wound D. A 39-year-old patient with a partial-thickness burn to the right upper arm

Answer: A. A 53-year-old patient with a chemical burn to the anterior chest and neck Rationale: The American Burn Association (ABA) has established referral criteria to determine which burn injuries should be treated in burn centers where specialized facilities and personnel are available to handle this type of trauma (see Table 25-3). Patients with chemical burns should be referred to a burn center. A normal serum carboxyhemoglobin level for nonsmokers is 0% to 1.5% and for smokers is 4% to 9%. Skin grafting for burn wound management is not a criteria for a referral to a burn center. Partial-thickness burns greater than 10% total body surface area (TBSA) should be referred to a burn center; a burn to the right upper arm is 4% TBSA. Ch. 25

The nurse is caring for a 71 kg patient during the first 12 hours after a thermal burn injury. Which outcomes if observed by the nurse would indicate adequate fluid resuscitation (select all that apply)? A. Heart rate is 94 beats/minute. B. Mean arterial pressure is 54 mm Hg. C. Urine output is 46 mL/hour. D. Urine specific gravity is 1.040. E. Systolic blood pressure 88 mm Hg

Answer: A. Heart rate is 94 beats/minute. C. Urine output is 46 mL/hour. Rationale: Assessment of the adequacy of fluid resuscitation is best made using either urine output or cardiac factors. Urine output should be 0.5 to 1 mL/kg/hr (or 75 to 100 mL/hr for an electrical burn patient with evidence of hemoglobinuria/myoglobinuria). Cardiac factors include a mean arterial pressure (MAP) greater than 65 mm Hg, systolic blood pressure (BP) greater than 90 mm Hg, heart rate less than 120 beats/minute. Normal range for urine specific gravity is 1.003 to 1.030. Ch. 25

A nurse manager educates the emergency department staff about their roles during a disaster with mass casualties. Which primary responsibility should the nurse manager discuss with the staff? A. Learn the hospital emergency response plan. B. Report acts of violence to security personnel. C. Contact the American Red Cross for assistance. D. Notify local, state, and national authorities.

Answer: A. Learn the hospital emergency response plan. Rationale: All health care providers need to be prepared for a mass casualty incident; the priority responsibility is to know the agency's emergency response plan. Ch. 69

The nurse is planning to change the dressing covering a deep partial-thickness burn of the right lower leg. Which prescribed medication should the nurse administer 30 minutes before the scheduled dressing change? A. zolpidem (Ambien) B. morphine sulfate C. sertraline (Zoloft) D. enoxaparin (Lovenox)

Answer: B. morphine sulfate Rationale: Deep partial-thickness burns result in severe pain related to nerve injury. The nurse should plan to administer analgesics before the dressing change to promote patient comfort. Morphine sulfate is a common opioid used for pain control. Sedative/hypnotics and antidepressant agents can also be given with analgesics to control the anxiety, insomnia, and/or depression that patients may experience. Zolpidem promotes sleep; sertraline is an antidepressant; and enoxaparin is an anticoagulant. Ch. 25

The nurse is caring for a patient who sustained a deep partial thickness burn to the anterior chest area. Which statement would be appropriate for the nurse to include when documenting the appearance of this type of burn? A. Skin is hard with a dry, waxy white appearance with visible venous patterns. B. Skin blanches with pressure and is red with delayed blister formation. C. Skin is red and shiny with the presence of clear fluid-filled blisters. D. Skin is charred and leathery with visible muscles, tendons, and bones

Answer: C. Skin is red and shiny with the presence of clear fluid-filled blisters. Rationale: Deep partial thickness burns have fluid-filled vesicles that are red and shiny; may appear wet (if vesicles have ruptured); and mild to moderate edema may be present. Deep partial thickness burns result in severe pain related to nerve injury. Superficial partial thickness burns are red and blanch with pressure; pain and mild edema are present. Superficial partial thickness burns may have vesicles that appear 24 hours after the burn injury. Full-thickness burns are dry, waxy white, leathery, or hard; thrombosed vessels may be visible. Full-thickness burns result in an insensitivity to pain because of nerve destruction, and there may be involvement of muscles, tendons, and bones. Ch. 25

A nurse is performing triage in the emergency department. Which patient should the nurse see first? A. 18-year-old patient with type 1 diabetes mellitus who has a 4-cm laceration on right leg. B. 32-year-old patient with drug overdose who is unresponsive with poor respiratory effort. C. 56-year-old patient with substernal chest pain who is diaphoretic with shortness of breath. D. 78-year-old patient with right hip fracture who is confused; blood pressure is 98/62 mm Hg.

Answer: B. 32-year-old patient with drug overdose who is unresponsive with poor respiratory effort. Rationale: Patient with drug overdose is unstable and needs to be seen immediately. Patient with chest pain (possible myocardial infarction) should be seen second. Patient with hip fracture should be seen third. Patient with laceration is the most stable and should be seen last. Ch. 69

A 52-year-old man with stage 2 chronic kidney disease is scheduled for an outpatient diagnostic procedure using contrast media. Which action should the nurse take? A.Assess skin turgor to determine hydration status. B.Insert a urinary catheter for the expected diuresis. C.Evaluate the patient's lower extremities for edema D. Check the patient's urine for the presence of ketones

Assess skin turgor to determine hydration status. Preexisting kidney disease is the most important risk factor for the development of contrast-associated nephropathy and nephrotoxic injury. If contrast media must be administered to a high-risk patient, the patient needs to have optimal hydration. The nurse should assess the hydration status of the patient before the procedure is performed. Indwelling catheter use should be avoided whenever possible to decrease the risk of infection.

The nurse is caring for a patient who is 24 hours postpacemaker insertion. Which nursing intervention is most appropriate at this time? Reinforcing the pressure dressing as needed Encouraging range-of-motion exercises of the involved arm Assessing the incision for any redness, swelling, or discharge Applying wet-to-dry dressings every 4 hours to the insertion site

Assessing the incision for any redness, swelling, or discharge After pacemaker insertion, it is important for the nurse to observe signs of infection by assessing for any redness, swelling, or discharge from the incision site. The nonpressure dressing is kept dry until removed, usually 24 hours postoperatively. It is important for the patient to limit activity of the involved arm to minimize pacemaker lead displacement.

Postoperative care of a patient undergoing coronary artery bypass graft (CABG) surgery includes monitoring for which common complication? Dehydration Paralytic ileus Atrial dysrhythmias Acute respiratory distress syndrome

Atrial dysrhythmias Postoperative dysrhythmias, specifically atrial dysrhythmias, are common in the first 3 days after CABG surgery. Although the other complications could occur, they are not common complications.

The nurse observes no P waves on the patients monitor strip. There are fine, wavy lines between the QRS complexes. The QRS complexes measure 0.08 sec (narrow), but they occur irregularly with a rate of 120 beats/min. What does the nurse determine the rhythm to be? Sinus tachycardia Atrial fibrillation Ventricular fibrillation Ventricular tachycardia

Atrial fibrillation Atrial fibrillation is represented on the cardiac monitor by irregular R-R intervals and small fibrillatory (f) waves. There are no normal P waves because the atria are not truly contracting, just fibrillating. Sinus tachycardia is a sinus rate above 100 beats/min with normal P waves. Ventricular fibrillation is seen on the ECG without a visible P wave; an unmeasurable heart rate, PR or QRS; and the rhythm is irregular and chaotic. Ventricular tachycardia is seen as three or more premature ventricular contractions that have distorted QRS complexes with regular or irregular rhythm, and the P wave is usually buried in the QRS complex without a measurable PR interval.

The nurse is providing care for an older adult patient who is experiencing low partial pressure of oxygen in arterial blood (PaO2) as a result of worsening left-sided pneumonia. Which intervention should the nurse use to help the patient mobilize his secretions? Augmented coughing or huff coughing Positioning the patient side-lying on his left side Frequent and aggressive nasopharyngeal suctioning Application of noninvasive positive pressure ventilation (NIPPV)

Augmented coughing or huff coughing Augmented coughing and huff coughing techniques may aid the patient in the mobilization of secretions. If positioned side-lying, the patient should be positioned on his right side (good lung down) for improved perfusion and ventilation. Suctioning may be indicated but should always be performed cautiously because of the risk of hypoxia. NIPPV is inappropriate in the treatment of patients with excessive secretions.

A nurse will teach a patient who is scheduled to complete a 24-hour urine collection for 17-ketosteroids to a. insert and maintain a retention catheter. b. keep the specimen refrigerated or on ice. c. drink at least 3 L of fluid during the 24 hours. d. void and save that specimen to start the collection.

B

One of the major advantages of peritoneal dialysis is that: A. no medications are required because of the enhanced efficiency of the peritoneal membranes in removing toxins B. the diet is less restricted and dialysis can be performed at home C. the dialysate is biocompatible and causes no long term consequences D. high glucose concentration of the dialysate causes a reduction in appetite promoting weight loss

B

Which action by a new registered nurse (RN) caring for a patient with a goiter and possible hyperthyroidism indicates that the charge nurse needs to do more teaching? a. The RN checks the blood pressure on both arms. b. The RN palpates the neck thoroughly to check thyroid size. c. The RN lowers the thermostat to decrease the temperature in the room. d. The RN orders nonmedicated eye drops to lubricate the patient's bulging eyes.

B

Which information about a 30-year-old patient who is scheduled for an oral glucose tolerance test should be reported to the health care provider before starting the test? a. The patient reports having occasional orthostatic dizziness. b. The patient takes oral corticosteroids for rheumatoid arthritis. c. The patient has had a 10-pound weight gain in the last month. d. The patient drank several glasses of water an hour previously.

B

When a patient is admitted to the emergency department after a submersion injury, which assessment will the nurse obtain first? a. Apical pulse b. Lung sounds c. Body temperature d. Level of consciousness

B The priority assessment data are how well the patient is oxygenating, so lung sounds should be assessed first. The other data also will be collected rapidly but are not as essential as the lung sounds.

The nurse is caring for a patient admitted with a history of hypertension. The patient's medication history includes hydrochlorothiazide (Hydrodiuril) daily for the past 10 years. Which parameter would indicate the optimal intended effect of this drug therapy? A Weight loss of 2 lb B Blood pressure 128/86 C Absence of ankle edema D Output of 600 mL per 8 hours

B Blood pressure 128/86 Hydrochlorothiazide may be used alone as monotherapy to manage hypertension or in combination with other medications if not effective alone. After the first few weeks of therapy, the diuretic effect diminishes, but the antihypertensive effect remains. Since the patient has been taking this medication for 10 years, the most direct measurement of its intended effect would be the blood pressure. *long term Tx - normotensive is goal*

When teaching a patient about dietary management of stage 1 hypertension, which instruction is most appropriate? A Restrict all caffeine. B Restrict sodium intake. C Increase protein intake. D Use calcium supplements.

B Restrict sodium intake. The patient should decrease intake of sodium. This will help to control hypertension, which can be aggravated by excessive salt intake, which in turn leads to fluid retention. Caffeine and protein intake do not affect hypertension. Calcium supplements are not recommended to lower BP.

When providing dietary instruction to a patient with hypertension, the nurse would advise the patient to restrict intake of which meat? A Broiled fish B Roasted duck C Roasted turkey D Baked chicken breast

B Roasted duck Roasted duck is high in fat, which should be avoided by the patient with hypertension. Weight loss may slow the progress of atherosclerosis and overall CVD risk. The other meats are lower in fat and are therefore acceptable in the diet.

The nurse supervises an unlicensed assistant personnel (UAP) who is taking the blood pressure of 58-year-old female patient admitted with heart failure. The patient is obese. The nurse should intervene if what is observed? A The UAP waits 2 minutes after position changes to take orthostatic pressures. B The UAP deflates the blood pressure cuff at a rate of 8 to 10 mm Hg per second. C The UAP takes the blood pressure with the patient's arm at the level of the heart. D The UAP takes a forearm blood pressure because the largest cuff will not fit the patient's upper arm.

B The UAP deflates the blood pressure cuff at a rate of 8 to 10 mm Hg per second. The cuff should be deflated at a rate of 2 to 3 mm Hg per second. The arm should be supported at the level of the heart for accurate blood pressure measurements. If the maximum size blood pressure cuff does not fit the upper arm, the forearm may be used. Orthostatic blood pressures should be taken within 1 to 2 minutes of repositioning the patient.

The patient with sudden pain in the left upper quadrant radiating to the back and vomiting was diagnosed with acute pancreatitis. What intervention(s) should the nurse expect to include in the patient's plan of care? A) Immediately start enteral feeding to prevent malnutrition. B) Insert an NG and maintain NPO status to allow pancreas to rest. C) Initiate early prophylactic antibiotic therapy to prevent infection. D) Administer acetaminophen (Tylenol) every 4 hours for pain relief.

B) Initial treatment with acute pancreatitis will include an NG tube if there is vomiting and being NPO to decrease pancreatic enzyme stimulation and allow the pancreas to rest and heal. Fluid will be administered to treat or prevent shock. The pain will be treated with IV morphine because of the NPO status. Enteral feedings will only be used for the patient with severe acute pancreatitis in whom oral intake is not resumed. Antibiotic therapy is only needed with acute necrotizing pancreatitis and signs of infection.

The patient with right upper quadrant abdominal pain has an abdominal ultrasound that reveals cholelithiasis. What should the nurse expect to do for this patient? A) Prevent all oral intake. B) Control abdominal pain. C) Provide enteral feedings. D) Avoid dietary cholesterol.

B) Patients with cholelithiasis can have severe pain, so controlling pain is important until the problem can be treated. NPO status may be needed if the patient will have surgery but will not be used for all patients with cholelithiasis. Enteral feedings should not be needed, and avoiding dietary cholesterol is not used to treat cholelithiasis.

A 78 yo man has confusion and temp of 104 F. He is a diabetic with purulent drainage from his right great toe. His assessment findings are BP 84/80, HR 110, RR 42 and shallow, CO 8L/min, and PAWP 4 mmHg. The pt's symptoms are most likely indicative of A) Sepsis B) Septic shock C) Multiple organ dysfunction syndrome D) Systemic inflammatory response syndrome

B) Septic shock

The nurse should recognize that the liver performs which functions (select all that apply)? A) Bile storage B) Detoxification C) Protein metabolism D) Steroid metabolism E) Red blood cell (RBC) destruction

B, C, D) The liver performs multiple major functions that aid in the maintenance of homeostasis. These include metabolism of proteins and steroids as well as detoxification of drugs and metabolic waste products. The Kupffer cells of the liver participate in the breakdown of old RBCs. The liver produces bile, but storage occurs in the gall bladder.

When caring for a patient, the nurse assesses tachypnea, a cough, and restlessness. The lung sounds have fine, scattered crackles, and the chest x-ray shows new bilateral interstitial and alveolar infiltrates. The nurse is aware that the patient may have an acute lung injury (ALI). In what order does the nurse expect the physiologic changes of acute respiratory distress syndrome (ARDS)to occur if it happens with this patient? (Answer with a letter followed by a comma and a space (e.g. A, B, C, D).) a. Atelectasis b. Interstitial edema c. Refractory hypoxemia d. Surfactant dysfunction e. Increased inflammatory response f. Decreased gas exchange surface area

B, D, A, C, E, F edema; surfactant; atelectasis; refractory low O2; IR; dec gas exch In the injury or exudative phase of ARDS (1-7 days after acute lung injury), there is interstitial edema and surfactant dysfunction that lead to atelectasis. Widespread atelectasis decreases lung compliance, hyaline membranes form, and refractory hypoxemia occurs. In the reparative or proliferative phase (1-2 weeks after acute lung injury), there is an increased inflammatory response which worsens hypoxia. In the fibrotic phase (2-3 weeks after acute lung injury), the lung tissue is remodeled by collagen and fibrous tissue, which decreases the available surface area for gas exchange.

5. The nurse is providing care for a patient who has been admitted to the hospital with a head injury and who requires regular neurologic and vital sign assessment. Which assessments will be components of the patient's score on the Glasgow Coma Scale (GCS) (select all that apply)? A. Judgment B. Eye opening C. Abstract reasoning D. Best verbal response E. Best motor response F. Cranial nerve function

B,D,E The three dimensions of the GCS are eye opening, best verbal response, and best motor response. Judgment, abstract reasoning, and cranial nerve function are not components of the GCS.

4. The nurse is caring for a patient admitted with a subdural hematoma following a motor vehicle accident. Which change in vital signs would the nurse interpret as a manifestation of increased intracranial pressure (ICP)? A. Tachypnea B. Bradycardia C. Hypotension D. Narrowing pulse pressure

B. Bradycardia could indicate increased ICP. Changes in vital signs (known as Cushing's triad) occur with increased ICP. They consist of increasing systolic pressure with a widening pulse pressure, bradycardia with a full and bounding pulse, and irregular respirations.

9. Decerebrate posture is documented in the chart of the patient that the nurse will be caring for. The nurse should know that the patient may have elevated ICP causing serious disruption of motor fibers in the midbrain and brainstem and will expect the patient's posture to look like which posture represented below? A. A (hands together on chest) B. B (hands turned out at sides) C. C (one hand on chest one turned out at sides) D. D (hands at side with back arched)

B. Decerebrate posture is all four extremities in rigid extension with hyperpronation of the forearms and plantar flexion of feet. Decorticate posture is internal rotation and adduction of the arms with flexion of the elbows, wrists, and fingers from interruption of voluntary motor tracts in the cerebral cortex. Decorticate response on one side of the body and decerebrate response on the other side of the body may occur depending on the damage to the brain. Opisthotonic posture is decerebrate posture with the neck and back arched posteriorly and may be seen with traumatic brain injury.

The nurse provides discharge instructions for a 40-year-old woman who is newly diagnosed with cardiomyopathy. Which statement, if made by the patient, indicates that further teaching is necessary? A."I will avoid lifting heavy objects." B. "I can drink alcohol in moderation." C."My family will need to take a CPR course." D. "I will reduce stress by learning guided imagery."

B. "I can drink alcohol in moderation." Patients with cardiomyopathy should avoid alcohol consumption, especially in patients with alcohol-related dilated cardiomyopathy. Avoiding heavy lifting and stress, as well as family members learning CPR, are recommended teaching points. *big hearts - no booze*

The nurse notes a physician's order written at 10:00 AM for two units of packed red blood cells to be administered to a patient who is anemic as a result of chronic blood loss. If the transfusion is picked up at 11:30 AM, the nurse should plan to hang the unit no later than what time? A. 11:45 AM B. 12:00 noon C. 12:30 PM D. 3:30 PM

B. 12:00 noon The nurse must hang the unit of packed red blood cells within 30 minutes of signing them out from the blood bank.

Before starting a transfusion of packed red blood cells for an older anemic patient, the nurse would arrange for a peer to monitor his or her other assigned patients for how many minutes when the nurse begins the transfusion? A. 5 B. 15 C. 30 D. 60

B. 15 As part of standard procedure, the nurse remains with the patient for the first 15 minutes after starting a blood transfusion. Patients who are likely to have a transfusion reaction will more often exhibit signs within the first 15 minutes that the blood is infusing. Monitoring during the transfusion will be every 30 to 60 minutes.

A patient with an intracranial problem does not open his eyes to any stimulus, has no verbal response except moaning and muttering when stimulated, and flexes his arm in response to painful stimuli. The nurse records the patients GCS score as a. 6 b. 7 c. 9 d. 11

B. 7- no opening of eyes = 1; incomprehensible words= 2, flexion withdrawal = 4 Total = 7

Which patient is most likely to experience anemia related to an increased destruction of red blood cells? A. A 59-yr-old man whose alcoholism has precipitated folic acid deficiency B. A 23-yr-old African American man who has a diagnosis of sickle cell disease C. A 30-yr-old woman with a history of "heavy periods" accompanied by anemia D. A 3-yr-old child whose impaired growth and development is attributable to thalassemia

B. A 23-yr-old African American man who has a diagnosis of sickle cell disease A result of a sickling episode in sickle cell anemia involves increased hemolysis of the sickled cells. Thalassemias and folic acid deficiencies cause a decrease in erythropoiesis, whereas the anemia related to menstruation is a direct result of blood loss.

The patient had a history of rheumatic fever and has been diagnosed with mitral valve stenosis. The patient is planning to have a biologic valve replacement. What protective mechanisms should the nurse teach the patient about using after the valve replacement? A. Long-term anticoagulation therapy B. Antibiotic prophylaxis for dental care C. Exercise plan to increase cardiac tolerance D. Take β-adrenergic blockers to control palpitations.

B. Antibiotic prophylaxis for dental care The patient will need to use antibiotic prophylaxis for dental care to prevent endocarditis. Long-term anticoagulation therapy is not used with biologic valve replacement unless the patient has atrial fibrillation. An exercise plan to increase cardiac tolerance is needed for a patient with heart failure. Taking β-adrenergic blockers to control palpitations is prescribed for mitral valve prolapse, not valve replacement.

The nurse is caring for a patient with polycythemia vera. What is an important action for the nurse to initiate? A. Encourage deep breathing and coughing. B. Assist with or perform phlebotomy at the bedside. C. Teach the patient how to maintain a low-activity lifestyle. D. Perform thorough and regularly scheduled neurologic assessments.

B. Assist with or perform phlebotomy at the bedside. Primary polycythemia vera often requires phlebotomy in order to reduce blood volume. The increased risk of thrombus formation that accompanies the disease requires regular exercises and ambulation. Deep breathing and coughing exercises do not directly address the etiology or common sequelae of polycythemia, and neurologic manifestations are not typical.

A patient has anemia related to inadequate intake of essential nutrients. Which intervention would be appropriate for the nurse to include in the plan of care for this patient? A. Plan for 30 minutes of rest before and after every meal. B. Encourage foods high in protein, iron, vitamin C, and folate. C. Instruct the patient to select soft, bland, and nonacidic foods. D. Give the patient a list of medications that inhibit iron absorption.

B. Encourage foods high in protein, iron, vitamin C, and folate. Increased intake of protein, iron, folate, and vitamin C provides nutrients needed for maximum iron absorption and hemoglobin production. The other interventions do not address the patient's identified problem of inadequate intake of essential nutrients. Selection of foods that are soft, bland, and nonacidic is appropriate if the patient has oral mucosal irritation. Scheduled rest is an appropriate intervention if the patient has fatigue related to anemia. Providing information about medications that may inhibit iron absorption (e.g., antacids, tetracycline, soft drinks, tea, coffee, calcium, phosphorus, and magnesium salts) is important but does not address the patient's problem of inadequate intake of essential nutrients.

The blood bank notifies the nurse that the two units of blood ordered for a patient is ready for pick up. Which action should the nurse take to prevent an adverse effect during this procedure? A. Immediately pick up both units of blood from the blood bank. B. Infuse the blood slowly for the first 15 minutes of the transfusion. C. Regulate the flow rate so that each unit takes at least 4 hours to transfuse. D. Set up the Y-tubing of the blood set with dextrose in water as the flush solution.

B. Infuse the blood slowly for the first 15 minutes of the transfusion. Because a transfusion reaction is more likely to occur at the beginning of a transfusion, the nurse should initially infuse the blood at a rate no faster than 2 mL/min and remain with the patient for the first 15 minutes after hanging 1 unit of blood. Only 1 unit of blood can be picked up at a time, it must be infused within 4 hours, and it cannot be hung with dextrose.

A patient who has sickle cell disease has developed cellulitis above the left ankle. What is the nurse's priority for this patient? A. Start IV fluids. B. Maintain oxygenation. C. Maintain distal warmth. D. Check peripheral pulses.

B. Maintain oxygenation. Maintaining oxygenation is a priority as sickling episodes are frequently triggered by low oxygen tension in the blood which is commonly caused by an infection. Antibiotics to treat cellulitis, pain control, and fluids to reduce blood viscosity will also be used, but oxygenation is the priority.

A patient with an acute peptic ulcer and major blood loss requires an immediate transfusion with packed red blood cells. Which task is appropriate for the registered nurse (RN) to delegate to unlicensed assistive personnel (UAP)? A. Confirm the IV solution is 0.9% saline. B. Obtain the vital signs before the transfusion is initiated. C. Monitor the patient for shortness of breath and back pain. D. Double check the patient identity and verify the blood product.

B. Obtain the vital signs before the transfusion is initiated. The RN may delegate tasks such as taking vital signs to UAP. Assessments (e.g., monitoring for signs of a blood transfusion reaction [shortness of breath and back pain]) are within the scope of practice of the RN and may not be delegated to UAP. The RN must also assume responsibility for ensuring the correct IV fluid is used with blood products. A licensed nurse must complete verification of the patient's identity and the blood product data.

A patient with a diagnosis of hemophilia had a fall down an escalator earlier in the day and is now experiencing bleeding in the left knee joint. What should be the emergency nurse's immediate action? A. Immediate transfusion of platelets B. Resting the patient's knee to prevent hemarthroses C. Assistance with intracapsular injection of corticosteroids D. Range-of-motion exercises to prevent thrombus formation

B. Resting the patient's knee to prevent hemarthroses In patients with hemophilia, joint bleeding requires resting of the joint to prevent deformities from hemarthrosis. Clotting factors, not platelets or corticosteroids, are administered. Thrombus formation is not a central concern in a patient with hemophilia.

A patient with a head injury has bloody drainage from the ear. To determine whether CSF is present in the drainage, the nurse a. examines the tympanic membrane for a tear b. tests the fluid for a halo sign on a white dressing c. tests the fluid with a glucose identifying strip or stick d. collects 5 mL of fluid in a test tube and sends it to the laboratory for analysis

B. Tests the fluid for a halo sing on a white dressing- Testing clear drainage for CSF in nasal or ear drainage may be done with a Dextrostik or Tes-Tape strip, but if blood is present, the glucose in the blood will produce and unreliable result. To test bloody drainage, the nurse should test the fluid for a halo or ring that occurs when a yellowish ring encircles blood dripped onto a white pad or towel

A 42-year-old male patient complains of shoulder pain when the nurse moves his arm behind the back. Which question should the nurse ask? a. "Are you able to feed yourself without difficulty?" b. "Do you have difficulty when you are putting on a shirt?" c. "Are you able to sleep through the night without waking?" d. "Do you ever have trouble lowering yourself to the toilet?"

B. The patient's pain will make it more difficult to accomplish tasks like putting on a shirt or jacket. This pain should not affect the patient's ability to feed himself or use the toilet because these tasks do not involve moving the arm behind the patient. The arm will not usually be positioned behind the patient during sleeping.

The nurse plans care for a patient with increased ICP with the knowledge that the best way to position the patient is to a. keep the head of the bed flat b. elevate the head of the bed to 30 degrees c. maintain patient on the left side with the head supported on a pillow d. use a continuous rotation bed to continuously change patient position

B. elevate the head of the bed to 30 degrees

A patient with a history of end-stage kidney disease secondary to diabetes mellitus has presented to the outpatient dialysis unit for his scheduled hemodialysis. Which assessments should the nurse prioritize before, during, and after his treatment? A.Level of consciousness B. Blood pressure and fluid balance C.Temperature, heart rate, and blood pressure D.Assessment for signs and symptoms of infection

Blood pressure and fluid balance Although all of the assessments are relevant to the care of a patient receiving hemodialysis, the nature of the procedure indicates a particular need to monitor the patient's blood pressure and fluid balance.

Which assessment parameter will the nurse address during the secondary survey of a patient in triage? Blood pressure and heart rate Patency of the patient's airway Neurologic status and level of consciousness Presence or absence of breath sound and quality of breathing

Blood pressure and heart rate Vital signs are considered to be a part of the secondary survey in the triage process. Airway, breathing, circulation, and a brief neurologic assessment are components of the primary survey that is done to identify life-threatening conditions.

Measures indicated in the conservative therapy of chronic kidney disease include: A. decreased fluid intake, carbohydrate intake, and protein intake B. increased fluid intake, decreased carbohydrate intake and protein intake C. decreased fluid intake and protein intake, increased carbohydrate intake D. decreased fluid intake and carbohydrate intake, increased protein intake

C

The nurse will teach a patient to plan to minimize physical and emotional stress while the patient is undergoing a. a water deprivation test. b. testing for serum T3 and T4 levels. c. a 24-hour urine test for free cortisol. d. a radioactive iodine (I-131) uptake test.

C

The emergency department (ED) triage nurse is assessing four victims of an automobile accident. Which patient has the highest priority for treatment? a. A patient with absent pedal pulses b. A patient with an open femur fracture c. A patient with a sucking chest wound d. A patient with bleeding of facial lacerations

C Most immediate deaths from trauma occur because of problems with ventilation, so the patient with a sucking chest wound should be treated first. Face and head fractures can obstruct the airway, but the patient with facial injuries has lacerations only. The other two patients also need rapid intervention but do not have airway or breathing problems.

The nurse teaches a 28-year-old man newly diagnosed with hypertension about lifestyle modifications to reduce his blood pressure. Which statement by the patient requires an intervention by the nurse? A "I will avoid adding salt to my food during or after cooking." B "If I lose weight, I might not need to continue taking medications." C "I can lower my blood pressure by switching to smokeless tobacco." D "Diet changes can be as effective as taking blood pressure medications."

C "I can lower my blood pressure by switching to smokeless tobacco." Nicotine contained in tobacco products (smoking and chew) cause vasoconstriction and increase blood pressure. Persons with hypertension should restrict sodium to 1500 mg/day by avoiding foods high in sodium and not adding salt in preparation of food or at meals. Weight loss can decrease blood pressure between 5 to 20 mm Hg. Following dietary recommendations (such as the DASH diet) lowers blood pressure, and these decreases compare with those achieved with blood pressure-lowering medication.

The nurse is caring for a patient admitted with chronic obstructive pulmonary disease (COPD), angina, and hypertension. Before administering the prescribed daily dose of atenolol 100 mg PO, the nurse assesses the patient carefully. Which adverse effect is this patient at risk for, given the patient's health history? A Hypocapnia B Tachycardia C Bronchospasm D Nausea and vomiting

C Bronchospasm Atenolol is a cardioselective β1-adrenergic blocker that reduces blood pressure and could affect the β2-receptors in the lungs with larger doses or with drug accumulation. Although the risk of bronchospasm is less with cardioselective β-blockers than nonselective β-blockers, atenolol should be used cautiously in patients with COPD.

The patient has chronic hypertension. Today she has gone to the ED, and her blood pressure has risen to 200/140. What is the priority assessment for the nurse to make? A Is the patient pregnant? B Does the patient need to urinate? C Does the patient have a headache or confusion? D Is the patient taking antiseizure medications as prescribed?

C Does the patient have a headache or confusion? The nurse's priority assessments include neurologic deficits, retinal damage, heart failure, pulmonary edema, and renal failure. The headache or confusion could be seen with hypertensive encephalopathy from increased cerebral capillary permeability leading to cerebral edema. Pregnancy can lead to secondary hypertension. Needing to urinate and taking antiseizure medication do not support a hypertensive emergency.

When teaching how lisinopril (Zestril) will help lower the patient's blood pressure, which mechanism of action should the nurse use to explain it? A Blocks β-adrenergic effects. B Relaxes arterial and venous smooth muscle. C Inhibits conversion of angiotensin I to angiotensin II. D Reduces sympathetic outflow from central nervous system.

C Inhibits conversion of angiotensin I to angiotensin II. Lisinopril is an angiotensin-converting enzyme (ACE) inhibitor that inhibits the conversion of angiotensin I to angiotensin II, which reduces angiotensin II-mediated vasoconstriction and sodium and water retention. Beta blockers result in vasodilation and decreased heart rate. Direct vasodilators relax arterial and venous smooth muscle. Central acting α-adrenergic antagonists reduce sympathetic outflow from the CNS to produce vasodilation and decreased SVR and BP.

The nurse is teaching a women's group about prevention of hypertension. What information should be included in the teaching for all the women (select all that apply)? A Lose weight. B Limit nuts and seeds. C Limit sodium and fat intake. D Increase fruits and vegetables. E Exercise 30 minutes most days.

C Limit sodium and fat intake. D Increase fruits and vegetables. E Exercise 30 minutes most days. Primary prevention of hypertension is to make lifestyle modifications that prevent or delay the increase in BP. Along with exercise for 30 minutes on most days, the DASH eating plan is a healthy way to lower BP by limiting sodium and fat intake, increasing fruits and vegetables, and increasing nutrients that are associated with lowering BP. Nuts and seeds and dried beans are used for protein intake. Weight loss may or may not be necessary *because you don't know my life*.

A 67-year-old woman with a history of coronary artery disease and prior myocardial infarction is admitted to the emergency department with a blood pressure of 234/148 mm Hg and started on IV nitroprusside (Nitropress). What should the nurse determine as an appropriate goal for the first hour of treatment? A Mean arterial pressure lower than 70 mm Hg B Mean arterial pressure no more than 120 mm Hg C Mean arterial pressure no lower than 133 mm Hg D Mean arterial pressure between 70 and 110 mm Hg

C Mean arterial pressure no lower than 133 mm Hg The initial treatment goal is to decrease mean arterial pressure by no more than 25% within minutes to 1 hour. If the patient is stable, the goal for BP is 160/100 to 110 mm Hg over the next 2 to 6 hours. Lowering the blood pressure too much may decrease cerebral, coronary, or renal perfusion and could precipitate a stroke, myocardial infarction, or renal failure. Additional gradual reductions toward a normal blood pressure should be implemented over the next 24 to 48 hours if the patient is clinically stable.

The patient tells the nurse she had a history of abdominal pain, so she had a surgery to make an opening into the common bile duct to remove stones. The nurse knows that this surgery is called a A) colectomy B) cholecystectomy C) choledocholithotomy D) choledochojejunostomy

C) A choledocholithotomy is an opening into the common bile duct for the removal of stones. A colectomy is the removal of the colon. The cholecystectomy is the removal of the gallbladder. The choledochojejunostomy is an opening between the common bile duct and the jejunum.

The health care team is assessing a male patient for acute pancreatitis after he presented to the emergency department with severe abdominal pain. Which laboratory value is the best diagnostic indicator of acute pancreatitis? A) Gastric pH B) Blood glucose C) Serum amylase D) Serum potassium

C) Elevated serum amylase levels indicate early pancreatic dysfunction and are used to diagnose acute pancreatitis. Serum lipase levels stay elevated longer than serum amylase in acute pancreatitis. Blood glucose, gastric pH, and potassium levels are not direct indicators of acute pancreatic dysfunction.

12. In planning long-term care for a patient after a craniotomy, what must the nurse include when teaching the patient, family, and caregiver? A. Seizure disorders may occur in weeks or months. B. The family will be unable to cope with role reversals. C. There are often residual changes in personality and cognition. D. Referrals will be made to eliminate residual deficits from the damage.

C. In long-term care planning, the nurse must include the family and caregiver when teaching about potential residual changes in personality, emotions, and cognition as these changes are most difficult for the patient and family to accept. Seizures may or may not develop. The family and patient may or may not be able to cope with role reversals. Although residual deficits will not be eliminated with referrals, they may be improved.

8. A patient has a systemic blood pressure of 120/60 and an ICP of 24 mm Hg. After calculating the patient's cerebral perfusion pressure (CPP), how does the nurse interpret the results? A. High blood flow to the brain B. Normal intracranial pressure C. Impaired blood flow to the brain D. Adequate autoregulation of blood flow

C. Normal CPP is 60 to 100 mm Hg. The CPP is calculated with mean arterial pressure (MAP) minus ICP. MAP = SBP + 2 (DBP)/ 3: 120 mm Hg + 2 (60 mm Hg)/3 = 80 mm Hg. MAP - ICP: 80mm Hg - 24 mm Hg = 56 mm Hg CPP. The decreased CPP indicates that there is impaired cerebral blood flow and that autoregulation is impaired. Because the ICP is 24, it is elevated and requires treatment.

The nurse instructs an African American man who has sickle cell disease about symptom management and prevention of sickle cell crisis. The nurse determines further teaching is necessary if the patient makes which statement? A. "When I take a vacation, I should not go to the mountains." B. "I should avoid contact with anyone who has a respiratory infection." C. "When my vision is blurred, I will close my eyes and rest for an hour." D. "I may experience severe pain during a crisis and need narcotic analgesics."

C. "When my vision is blurred, I will close my eyes and rest for an hour." Blurred vision should be reported immediately and may indicate a detached retina or retinopathy. Hypoxia (at high altitudes) and infection are common causes of a sickle cell crisis. Severe pain may occur during a sickle cell crisis, and narcotic analgesics are indicated for pain management.

A 25-year-old patient with a group A streptococcal pharyngitis does not want to take the antibiotics prescribed. What should the nurse tell the patient to encourage the patient to take the medications and avoid complications of the infection? A. "The complications of this infection will affect the skin, hair, and balance." B. "You will not feel well if you do not take the medicine and get over this infection." C. "Without treatment, you could get rheumatic fever, which can lead to rheumatic heart disease." D. "You may not want to take the antibiotics for this infection, but you will be sorry if you do not."

C. "Without treatment, you could get rheumatic fever, which can lead to rheumatic heart disease." Rheumatic fever (RF) is not common because of effective use of antibiotics to treat streptococcal infections. Without treatment, RF can occur and lead to rheumatic heart disease, especially in young adults. The complications do not include hair or balance. Saying that the patient will not feel well or that the patient will be sorry if the antibiotics are not taken is threatening to the patient and inappropriate for the nurse to say.

The nurse is caring for a patient who is to receive a transfusion of two units of packed red blood cells. After obtaining the first unit from the blood bank, the nurse would ask which health team member in the nurses' station to assist in checking the unit before administration? A. Unit secretary B. A physician's assistant C. Another registered nurse D. An unlicensed assistive personnel

C. Another registered nurse Before hanging a transfusion, the registered nurse must check the unit with another RN or with a licensed practical (vocational) nurse, depending on agency policy. The unit secretary, physician's assistant, or unlicensed assistive personnel should not be asked.

While the nurse performs ROM on an unconscious patient with increased ICP, the patient experiences severe decerebrate posturing reflexes. The nurse should a. use restraints to protect the patient from injury b. administer CNS depressants to lightly sedate the patient c. perform the exercises less frequently because posturing can increase ICP d. continue the exercises because they are necessary to maintain musculoskeletal function

C. Perform the exercises less frequently because posturing can increase ICP- If reflex posturing occurs during ROM or positioning of the patient, these activities should be done less frequently until the patient's condition stabilizes, because posturing can case increases in ICP. Neither restraints nor CNS depressants would be indicated.

A 54-year old man is recovering from a skull fracture with a subacute subdural hematoma. He has return of motor control and orientation but appears apathetic and has reduced awareness of his environment. When planning discharge or the patient, the nurse explains to the patient and the family that a. continuous improvement in the patient's condition should occur until he has returned to pre trauma status b. the patient's complete recovery may take years, and the family should plan for his long term dependent care c. the patient is likely to have long term emotional and mental changes that may require continued professional help d. role changes in family members will be necessary because the patient will be dependent on his family for care and support

C. Residual mental and emotional changes of brain trauma with personality changes are often the most incapacitating problems following head injury and are common in patients who have been comatose longer than 6 hours. Families must be prepared for changes in the patient's behavior to avoid family-patient friction and maintain family functioning, and professional assistance may be required. There is no indication he will be dependent on others for care, but he likely will not return to pre trauma status

On physical examination of a patient with headache and fever, the nurse would suspect a brain abscess when the patient has a. seizures b. nuchal rigidity c. focal symptoms d. signs of increased ICP

C. The symptoms of brain abscess closely resemble those of meningitis and encephalitis, including fever, headache, and increased ICP, except the patient also usually has some focal symptoms that reflect the local are of the abscess.

The nurse is caring for a patient with a diagnosis of disseminated intravascular coagulation (DIC). What is the first priority of care? A. Administer heparin. B. Administer whole blood. C. Treat the causative problem. D. Administer fresh frozen plasma.

C. Treat the causative problem. Treating the underlying cause of DIC will interrupt the abnormal response of the clotting cascade and reverse the DIC. Blood product administration occurs based on the specific component deficiencies and is reserved for patients with life-threatening hemorrhage. Heparin will be administered if the manifestations of thrombosis are present and the benefit of reducing clotting outweighs the risk of further bleeding.

The nurse on the clinical unit is assigned to four patients. Which patient should she assess first? a. patient with a skull fracture whose nose is bleeding b. elderly patient with a stroke who is confused and whose daughter is present c. patient with meningitis who is suddenly agitated and reporting a HA of 10 on a 0 to 10 scale d. patient who had a craniotomy for a brain tumor who is now 3 days postoperative and has had continued emesis

C. patient with meningitis who is suddenly agitated and reporting a HA of 10 on a 0 to 10 scale

The patient has had type 1 diabetes mellitus for 25 years and is now reporting fatigue, edema, and an irregular heartbeat. On assessment, the nurse finds that the patient has newly developed hypertension and difficulty with blood glucose control. The nurse should know that which diagnostic study will be most indicative of chronic kidney disease (CKD) in this patient? A.Serum creatinine B.Serum potassium C.Microalbuminuria D.Calculated glomerular filtration rate (GFR)

Calculated glomerular filtration rate (GFR) The best study to determine kidney function or chronic kidney disease (CKD) that would be expected in the patient with diabetes is the calculated GFR that is obtained from the patient's age, gender, race, and serum creatinine. It would need to be abnormal for 3 months to establish a diagnosis of CKD. A creatinine clearance test done with a blood sample and a 24-hour urine collection is also important. Serum creatinine is not the best test for CKD because the level varies with different patients. Serum potassium levels could explain why the patient has an irregular heartbeat. The finding of microalbuminuria can alert the patient with diabetes about potential renal involvement and potentially failing kidneys. However, urine albumin levels are not used for diagnosis of CKD.

The nurse is caring for a 55-yr-old man who has a catheter in the right radial artery for invasive arterial blood pressure monitoring after abdominal aortic aneurysm surgery. Which observation by the nurse would require an emergency intervention? Arterial pressure bag is inflated to 250 mm Hg. Calculated mean arterial pressure is 74 mm Hg. Patient's head of bed elevation is at 30 degrees. Capillary refill time in the right hand is 5 seconds.

Capillary refill time in the right hand is 5 seconds. Neurovascular status distal to the arterial insertion site is monitored hourly. If arterial flow is compromised, the limb will be cool and pale, with capillary refill time longer than 3 seconds. Symptoms of neurologic impairment include paresthesia, pain, or paralysis. Neurovascular impairment can result in loss of a limb and is an emergency. The pressure bag should be inflated to 300 mm Hg. Normal range for mean arterial pressure is 70 to 105 mm Hg. The backrest elevation may be up to 45 degrees unless the patient has orthostatic changes.

A patient returns after cardiac catheterization. Which nursing care would the registered nurse delegate to the licensed practical nurse? Monitor the electrocardiogram for dysrhythmias Check for bleeding at the catheter insertion site Prepare discharge teaching related to complications Take vital signs and report abnormal values

Check for bleeding at the catheter insertion site The licensed practical nurse can check for bleeding at the puncture sites. If bleeding is identified, it should be reported to the registered nurse. Vital signs should be delegated to the unlicensed assistive personnel. Preparation of discharge teaching and monitoring for dysrhythmias such as S-T elevation would be registered nurse scope of practice.

The patient with advanced cirrhosis asks why his abdomen is so swollen. The nurse's response is based on the knowledge that a. a lack of clotting factors promotes the collection of blood in the abdominal cavity. b. portal hypertension and hypoalbuminemia cause a fluid shift into the peritoneal space. c. decreased peristalsis in the GI tract contributes to gas formation and distention of the bowel. d. bile salts in the blood irritate the peritoneal membranes, causing edema and pocketing of fluid. (Lewis 1042)

Correct answer: b Rationale: Ascites is the accumulation of serous fluid in the peritoneal or abdominal cavity and is a common manifestation of cirrhosis. With portal hypertension, proteins shift from the blood vessels through the larger pores of the sinusoids (capillaries) into the lymph space. When the lymphatic system is unable to carry off the excess proteins and water, those substances leak through the liver capsule into the peritoneal cavity. Osmotic pressure of the proteins pulls additional fluid into the peritoneal cavity. A second mechanism of ascites formation is hypoalbuminemia, which results from the inability of the liver to synthesize albumin. Hypoalbuminemia results in decreased colloidal oncotic pressure. A third mechanism is hyperaldosteronism, which occurs when aldosterone is not metabolized by damaged hepatocytes. The increased level of aldosterone causes increases in sodium reabsorption by the renal tubules. Sodium retention and an increase in antidiuretic hormone levels cause additional water retention.

A patient has been told that she has elevated liver enzymes caused by nonalcoholic fatty liver disease (NAFLD). The nursing teaching plan should include a. having genetic testing done. b. recommending a heart-healthy diet. c. the necessity to reduce weight rapidly. d. avoiding alcohol until liver enzymes return to normal. (Lewis 1042)

Correct answer: b Rationale: Nonalcoholic fatty liver disease (NAFLD) can progress to liver cirrhosis. There is no definitive treatment, and therapy is directed at reduction of risk factors, which include treatment of diabetes, reduction in body weight, and elimination of harmful medications. For patients who are overweight, weight reduction is important. Weight loss improves insulin sensitivity and reduces liver enzyme levels. No specific dietary therapy is recommended. However, a heart-healthy diet as recommended by the American Heart Association is appropriate.

A 44-year-old man is diagnosed with hypertension and receives a prescription for benazepril (Lotensin). After the nurse teaches him about the medication, which statement by the patient indicates his correct understanding? A "If I take this medication, I will not need to follow a special diet." B "It is normal to have some swelling in my face while taking this medication." C "I will need to eat foods such as bananas and potatoes that are high in potassium." D "If I develop a dry cough while taking this medication, I should notify my doctor."

D "If I develop a dry cough while taking this medication, I should notify my doctor." Benazepril is an angiotensin-converting enzyme inhibitor. The medication inhibits breakdown of bradykinin, which may cause a dry, hacking cough. Other adverse effects include hyperkalemia. Swelling in the face could indicate angioedema and should be reported immediately to the prescriber. Patients taking drug therapy for hypertension should also attempt lifestyle modifications to lower blood pressure such as a reduced-sodium diet.

In reviewing medication instructions with a patient being discharged on antihypertensive medications, which statement would be most appropriate for the nurse to make when discussing guanethidine (Ismelin)? A "A fast heart rate is a side effect to watch for while taking guanethidine." B "Stop the drug and notify your doctor if you experience any nausea or vomiting." C "Because this drug may affect the lungs in large doses, it may also help your breathing." D "Make position changes slowly, especially when rising from lying down to a standing position."

D "Make position changes slowly, especially when rising from lying down to a standing position." Guanethidine is a peripheral-acting α-adrenergic antagonist and can cause marked orthostatic hypotension. For this reason, the patient should be instructed to rise slowly, especially when moving from a recumbent to a standing position. Support stockings may also be helpful. Tachycardia or lung effects are not evident with guanethidine.

The health care provider orders lactulose for a patient with hepatic encephalopathy. The nurse will monitor for effectiveness of this medication for this patient by assessing which of the following? A. Relief of constipation B. Relief of abdominal pain C. Decreased liver enzymes D. Decreased ammonia levels

D) Hepatic encephalopathy is a complication of liver disease and is associated with elevated serum ammonia levels. Lactulose traps ammonia in the intestinal tract. Its laxative effect then expels the ammonia from the colon, resulting in decreased serum ammonia levels and correction of hepatic encephalopathy.

The family of a patient newly diagnosed with hepatitis A asks the nurse what they can do to prevent becoming ill themselves. Which of the following responses by the nurse is most appropriate? A. "The hepatitis vaccine will provide immunity from this exposure and future exposures." B. "I am afraid there is nothing you can do since the patient was infectious before admission." C. "You will need to be tested first to make sure you don't have the virus before we can treat you." D. "An injection of immunoglobulin will need to be given to prevent or minimize the effects from this exposure."

D) Immunoglobulin provides temporary (1-2 months) passive immunity and is effective for preventing hepatitis A if given within 2 weeks after exposure. It may not prevent infection in all persons, but it will at least modify the illness to a subclinical infection. The hepatitis vaccine is only used for preexposure prophylaxis

The most accurate assessment parameters for the nurse to use to determine adequate tissue perfusion in the pt. with MODS are A) BP, HR, RR B) breath sounds, BP, temp C) pulse pressure, LOC, and pupillary response D) LOC, urine output, and skin color and temp

D) LOC, urine output, and skin color and temp

The nurse would question the use of which cathartic agent in a patient with renal insufficiency? A) Bisacodyl (Dulcolax) B) Lubiprostone (Amitiza) C) Cascara sagrada (Senekot) D) Magnesium hydroxide (Milk of Magnesia)

D) Milk of Magnesia may cause hypermagnesemia in patients with renal insufficiency. The nurse should question this order with the health care provider. Bisacodyl, lubiprostone, and cascara sagrada are safe to use in patients with renal insufficiency as long as the patient is not currently dehydrated.

The patient with suspected pancreatic cancer is having many diagnostic studies done. Which one can be used to establish the diagnosis of pancreatic adenocarcinoma and for monitoring the response to treatment? A) Spiral CT scan B) A PET/CT scan C) Abdominal ultrasound D) Cancer-associated antigen 19-9

D) The cancer-associated antigen 19-9 (CA 19-9) is the tumor marker used for the diagnosis of pancreatic adenocarcinoma and for monitoring the response to treatment. Although a spiral CT scan may be the initial study done and provides information on metastasis and vascular involvement, this test and the PET/CT scan or abdominal ultrasound do not provide additional information.

When teaching the patient with acute hepatitis C (HCV), the patient demonstrates understanding when the patient makes which statement? A) "I will use care when kissing my wife to prevent giving it to her." B) "I will need to take adofevir (Hepsera) to prevent chronic HCV." C) "Now that I have had HCV, I will have immunity and not get it again." D) "I will need to be checked for chronic HCV and other liver problems."

D) The majority of patients who acquire HCV usually develop chronic infection, which may lead to cirrhosis or liver cancer. HCV is not transmitted via saliva, but percutaneously and via high-risk sexual activity exposure. The treatment for acute viral hepatitis focuses on resting the body and adequate nutrition for liver regeneration. Adofevir (Hepsera) is taken for severe hepatitis B (HBV) with liver failure. Chronic HCV is treated with pegylated interferon with ribavirin. Immunity with HCV does not occur as it does with HAV and HBV, so the patient may be reinfected with another type of HCV.

3. The nurse assesses a patient for signs of meningeal irritation and observes for nuchal rigidity. What indicates the presence of this sign of meningeal irritation? A. Tonic spasms of the legs B. Curling in a fetal position C. Arching of the neck and back D. Resistance to flexion of the neck

D. Nuchal rigidity is a clinical manifestation of meningitis. During assessment, the patient will resist passive flexion of the neck by the health care provider. Tonic spasms of the legs, curling in a fetal position, and arching of the neck and back are not related to meningeal irritation.

A patient is admitted to the hospital with possible bacterial meningitis. During the initial assessment, the nurse questions the patient about a recent history of a. mosquito or tick bites b. chickenpox or measles c. cold sores or fever blisters d. an upper respiratory infection

D. An upper respiratory infection- Meningitis is often a result of an upper respiratory infection or middle ear infection, where organisms gain entry to the CNS. Epidemic encephalitis is transmitted by ticks and mosquitoes, and nonepidemic encephalitis may occur as a complication of measles, chickenpox, or mumps. Encephalitis caused by the herpes simplex virus carries a high fatality rate

Metabolic and nutritional needs of the patient with increased ICP are best met with a. enteral feedings that are low in sodium b. the simple glucose available in D5W IV solutions c. a fluid restriction that promotes a moderate dehydration d. balanced, essential nutrition in a form that the patient can tolerate

D. Balanced, essential nutrition in a form that the patient can tolerate= A patient with increased ICP is in a hypermetabolic and hypercatabolic state and needs adequate glucose to maintain fuel for the brain and other nutrients to meet metabolic needs. Malnutrition promotes cerebral edema, and if a patient cannot take oral nutrition, other means of providing nutrition should be used, such as tube feedings or parenteral nutrition. Glucose alone is not adequate to meet nutritional requirements, and 5% dextrose solutions may increase cerebral edema by lowering serum osmolarity. Patients should remain in a normovolemic fluid state with close monitoring of clinical factors such as urine output, fluid intake, serum and urine osmolality, serum electrolytes, and insensible losses.

When assessing the body function of a patient with increased ICP, the nurse should initially assess a. corneal reflex testing b. extremity strength testing c. pupillary reaction to light d. circulatory and respiratory status

D. Circulatory and respiratory status- Of the body functions that should be assessed in an unconscious patient, cardiopulmonary status is the most vital function and gives priorities to the *ABCs* (airway, breathing, and circulation)

The patient with pericarditis is complaining of chest pain. After assessment, which intervention should the nurse expect to implement to provide pain relief? A. Corticosteroids B. Morphine sulfate C. Proton pump inhibitor D. Nonsteroidal antiinflammatory drugs

D. Nonsteroidal antiinflammatory drugs Nonsteroidal antiinflammatory drugs (NSAIDs) will control pain and inflammation. Corticosteroids are reserved for patients already taking corticosteroids for autoimmune conditions or those who do not respond to NSAIDs. Morphine is not necessary. Proton pump inhibitors are used to decrease stomach acid to avoid the risk of GI bleeding from the NSAIDs.

A patient is being treated for non-Hodgkin's lymphoma (NHL). What should the nurse first teach the patient about the treatment? A. Skin care that will be needed B. Method of obtaining the treatment C. Gastrointestinal tract effects of treatment D. Treatment type and expected side effects

D. Treatment type and expected side effects The patient should first be taught about the type of treatment and the expected and potential side effects. Nursing care is related to the area affected by the disease and treatment. Skin care will be affected if radiation is used. Not all patients will have gastrointestinal tract effects of NHL or treatment. The method of obtaining treatment will be included in the teaching about the type of treatment.

Skull radiographs and a computed tomography (CT) scan provide evidence of a depressed parietal fracture with a subdural hematoma in a patient admitted to the emergency department following an automobile accident. In planning care for the patient, the nurse anticipates that a. the patient will receive life-support measures until the condition stabilizes b. immediate burr holes will be made to rapidly decompress the intracranial activity c. the patient will be treated conservatively with close monitoring for changes in neurologic condition d. the patient will be taken to surgery for a craniotomy for evacuation of blood and decompression of the cranium

D. When there is a depressed fracture and fractures with loose fragments, a craniotomy is indicated to elevate the depressed bone and remove free fragments. A craniotomy is also indicated in cases of acute subdural and epidural hematomas to remove the blood and control the bleeding. Burr holes may be used in an extreme emergency for rapid decompression, but with a depressed fracture, surgery would be the treatment of choice

A chemical explosion occurs at a nearby industrial site. The first responders report that victims are being decontaminated at the scene and approximately 125 workers will need medical evaluation and care. The nurse receiving this report should know that this will first require activation of A. a code blue alert. B. a disaster medical assistance team. C. the local police and fire departments. D. the hospital's emergency response plan.

D. the hospital's emergency response plan.

During hemodialysis, the patient develops light-headedness and nausea. What should the nurse do for the patient? A.Administer hypertonic saline. B.Administer a blood transfusion C.Decrease the rate of fluid removal. D.administer antiemetic medications.

Decrease the rate of fluid removal. The patient is experiencing hypotension from a rapid removal of vascular volume. The rate and volume of fluid removal will be decreased, and 0.9% saline solution may be infused. Hypertonic saline is not used because of the high sodium load. A blood transfusion is not indicated. Antiemetic medications may help the nausea but would not help the hypovolemia.

A 68-yr-old male patient diagnosed with sepsis is orally intubated on mechanical ventilation. Which nursing action is most important? Use the open-suctioning technique. Administer morphine for discomfort. Limit noise and cluster care activities. Elevate the head of the bed 30 degrees.

Elevate the head of the bed 30 degrees. The two major complications of endotracheal intubation are unplanned extubation and aspiration. To prevent aspiration, all intubated patients and patients receiving enteral feedings must have the head of the bed elevated a minimum of 30 to 45 degrees unless medically contraindicated. Closed-suction technique is preferred over the open-suction technique because oxygenation and ventilation are maintained during suctioning, and exposure to secretions is reduced. The nurse should provide comfort measures such as morphine to relieve anxiety and pain associated with intubation. To promote rest and sleep, the nurse should limit noise and cluster activities.

A 19-yr-old male patient has a plaster cast applied to the right arm for a Colles' fracture after a skateboarding accident. Which nursing action is most appropriate? Elevate the right arm on two pillows for 24 hours. Apply heating pad to reduce muscle spasms and pain. Limit movement of the thumb and fingers on the right hand. Place arm in a sling to prevent movement of the right shoulder.

Elevate the right arm on two pillows for 24 hours. The casted extremity should be elevated at or above heart level for 24 hours to reduce swelling or inflammation. The cast should be supported on pillows during the drying period to prevent denting and flattening of the cast. Ice (not heat) should be applied for the first 24 to 36 hours to reduce swelling or inflammation. Active movement of the thumb and fingers should be encouraged to reduce edema and increase venous return. A sling may be used to support and protect the extremity after the cast is completely dry, but the patient should perform active movements of the shoulder to prevent stiffness or contracture.

The nurse prepares a discharge teaching plan for a 44-yr-old male patient who has recently been diagnosed with coronary artery disease (CAD). Which risk factor should the nurse plan to focus on during the teaching session? Type A personality Elevated serum lipids Family cardiac history Hyperhomocysteinemia

Elevated serum lipids Dyslipidemia is one of the four major modifiable risk factors for CAD. The other major modifiable risk factors are hypertension, tobacco use, and physical inactivity. Research findings related to psychologic states (i.e., type A personality) as a risk factor for coronary artery disease have been inconsistent. Family history is a nonmodifiable risk factor. High homocysteine levels have been linked to an increased risk for CAD.

The nurse formulates a nursing diagnosis of Impaired physical mobility related to decreased muscle strength for an older adult patient recovering from left total knee arthroplasty. What nursing intervention is appropriate? Promote vitamin C and calcium intake in the diet. Provide passive range of motion to all of the joints q4hr. Keep the left leg in extension and abduction to prevent contractures. Encourage isometric quadriceps-setting exercises at least four times a day.

Encourage isometric quadriceps-setting exercises at least four times a day. Emphasis is placed on postoperative exercise of the affected leg, with isometric quadriceps setting beginning on the first day after surgery. Vitamin C and calcium do not improve muscle strength, but they will facilitate healing. The patient should be able to perform active range of motion to all joints. Keeping the leg in one position (extension and abduction) may contribute to contractures.

The nurse observes a flat line on the patient's monitor and the patient is unresponsive without pulse. What medications does the nurse prepare to administer? Lidocaine and amiodarone Digoxin and procainamide Epinephrine and/or vasopressin β-adrenergic blockers and dopamine

Epinephrine and/or vasopressin Normally, the patient in asystole cannot be successfully resuscitated. However, administration of epinephrine or vasopressin may prompt the return of depolarization and ventricular contraction. Lidocaine and amiodarone are used for ventricular tachycardia or ventricular fibrillation. Digoxin and procainamide are used for ventricular rate control. β-adrenergic blockers are used to slow heart rate, and dopamine is used to increase heart rate.

Tx for gallstones: ESWL

Extracoporeal shock wave lithotripsy (ESWL) - shock waves break up stones, often combined c oral solutions of stone-dissolving acid

The nurse is caring for a 27-yr-old man with multiple fractured ribs from a motor vehicle crash. Which clinical manifestation, if experienced by the patient, is an early indication that the patient is developing respiratory failure? Tachycardia and pursed lip breathing Kussmaul respirations and hypotension Frequent position changes and agitation Cyanosis and increased capillary refill time

Frequent position changes and agitation A change in mental status is an early indication of respiratory failure. The brain is sensitive to variations in oxygenation, arterial carbon dioxide levels, and acid-base balance. Restlessness, confusion, agitation, and combative behavior suggest inadequate oxygen delivery to the brain.

A patient who has hepatitis B surface antigen (HBsAg) in the serum is being discharged with pain medication after knee surgery. Which medication order should the nurse question because it is most likely to cause hepatic complications? Tramadol (Ultram) Hydromorphone (Dilaudid) Oxycodone with aspirin (Percodan) Hydrocodone with acetaminophen (Vicodin)

Hydrocodone with acetaminophen (Vicodin) The analgesic with acetaminophen should be questioned because this patient is a chronic carrier of hepatitis B and is likely to have impaired liver function. Acetaminophen is not suitable for this patient because it is converted to a toxic metabolite in the liver after absorption, increasing the risk of hepatocellular damage.

The nurse is caring for a patient who is admitted with a barbiturate overdose. The patient is comatose with a blood pressure of 90/60 mm Hg, apical pulse of 110 beats/min, and respiratory rate of 8 breaths/min. Based on the initial assessment findings, the nurse recognizes that the patient is at risk for which type of respiratory failure? Hypoxemic respiratory failure related to shunting of blood Hypoxemic respiratory failure related to diffusion limitation Hypercapnic respiratory failure related to alveolar hypoventilation Hypercapnic respiratory failure related to increased airway resistance

Hypercapnic respiratory failure related to alveolar hypoventilation The patient's respiratory rate is decreased as a result of barbiturate overdose, which caused respiratory depression. The patient is at risk for hypercapnic respiratory failure due to an obtunded airway causing decreased respiratory rate and thus decreased CO2 elimination. Barbiturate overdose does not lead to shunting of blood, diffusion limitations, or increased airway resistance.

The patient is brought to the emergency department after a car accident and is diagnosed with a femur fracture. What nursing intervention should the nurse implement at this time to decrease risk of a fat embolus? Administer enoxaparin (Lovenox). Provide range-of-motion exercises. Apply sequential compression boots. Immobilize the fracture preoperatively.

Immobilize the fracture preoperatively. The nurse immobilizes the long bone to reduce movement of the fractured bone ends and decrease the risk of a fat embolus development before surgical reduction. Enoxaparin is used to prevent blood clots not fat emboli. Range of motion and compression boots will not prevent a fat embolus in this patient.

A patient experienced sudden cardiac death (SCD) and survived. Which preventive treatment should the nurse expect to be implemented? External pacemaker An electrophysiologic study (EPS) Medications to prevent dysrhythmias Implantable cardioverter-defibrillator (ICD)

Implantable cardioverter-defibrillator (ICD) An ICD is the most common approach to preventing recurrence of SCD. An external pacemaker may be used in the hospital but will not be used for the patient living daily life at home. An EPS may be done to determine if a recurrence is likely and determine the most effective medication treatment. Medications to prevent dysrhythmias are used but are not the best prevention of SCD.

Arterial blood gas results are reported to the nurse for a 68-yr-old patient admitted with pneumonia: pH 7.31, PaCO2 49 mm Hg, HCO3 26 mEq/L, and PaO2 52 mm Hg. What order should the nurse complete first? Administer albuterol inhaler prn. Increase fluid intake to 2500 mL per 24 hours. Initiate oxygen at 2 liters/minute by nasal cannula. Perform chest physical therapy four times per day.

Initiate oxygen at 2 liters/minute by nasal cannula. The arterial blood gas results indicate the patient is in uncompensated respiratory acidosis with moderate hypoxemia. Oxygen therapy is indicated to correct hypoxemia secondary to V/Q mismatch. Supplemental oxygen should be initiated at 1 to 3 L/min by nasal cannula, or 24% to 32% by simple face mask or Venturi mask to improve the PaO2. Albuterol would be administered next if needed for bronchodilation. Hydration is indicated for thick secretions, and chest physical therapy is indicated for patients with 30 mL or more of sputum production per day.

The nurse assesses the right femoral artery puncture site as soon as the patient arrives after having a stent inserted into a coronary artery. The insertion site is not bleeding or discolored. What should the nurse do next to ensure the femoral artery is intact? Palpate the insertion site for induration. Assess peripheral pulses in the right leg. Inspect the patient's right side and back. Compare the color of the left and right legs.

Inspect the patient's right side and back. The best method to determine that the right femoral artery is intact after inspection of the insertion site is to logroll the patient to inspect the right side and back for retroperitoneal bleeding. The artery can be leaking and blood is drawn into the tissues by gravity. The peripheral pulses, color, and sensation of the right leg will be assessed per agency protocol.

A female patient who has type 1 diabetes mellitus has chronic stable angina that is controlled with rest. She states that over the past few months, she has required increasing amounts of insulin. What goal should the nurse use to plan care that should help prevent cardiovascular disease progression? Exercise almost every day. Avoid saturated fat intake. Limit calories to daily limit. Keep Hgb A1C (A1C) less than 7%.

Keep Hgb A1C (A1C) less than 7%. If the Hgb A1C (A1C) is kept below 7%, this means that the patient has had good control of her blood glucose over the past 3 months. The patient indicates that increasing amounts of insulin are being required to control her blood glucose. This patient may not be adhering to the dietary guidelines or therapeutic regimen, so teaching about how to maintain diet, exercise, and medications to maintain stable blood glucose levels will be needed to achieve this goal.

A nurse teaches the emergency department staff about their roles during a mass casualty incident. Which primary responsibility is expected of all licensed and unlicensed health care staff? Notify local, state, and national authorities. Assist security personnel to patrol the area. Learn the hospital emergency response plan. Contact the American Red Cross for assistance.

Learn the hospital emergency response plan. All health care providers must be prepared for a mass casualty incident. The priority responsibility is to know the agency's emergency response plan.

s/s cholelithiasis

MAYBE none at ALL - aka "silent cholelithiasis" In biliary colic, when a stone causes spasm: - steady severe pain - tachycardia, diaphoresis, really needing to go lie down - severe pain lasting up to 1 h, followed by residual tenderness, in RUQ - often starts 3-6h p fatty meal, or when lying down

A patient with type 2 diabetes and cirrhosis asks the nurse if it would be okay to take silymarin (milk thistle) to help minimize liver damage. The nurse responds based on what knowledge? Milk thistle may affect liver enzymes and thus alter drug metabolism. Milk thistle is generally safe in recommended doses for up to 10 years. There is unclear scientific evidence for the use of milk thistle in treating cirrhosis. Milk thistle may elevate the serum glucose levels and is thus contraindicated in diabetes.

Milk thistle may affect liver enzymes and thus alter drug metabolism. There is good scientific evidence that there is no real benefit from using milk thistle to protect the liver cells from toxic damage in the treatment of cirrhosis. Milk thistle does affect liver enzymes and thus could alter drug metabolism. Therefore patients will need to be monitored for drug interactions. It is noted to be safe for up to 6 years, not 10 years, and it may lower, not elevate, blood glucose levels.

The patient has a form of glomerular inflammation that is progressing rapidly. She is gaining weight, and the urine output is steadily declining. What is the priority nursing intervention? A.Monitor the patient's cardiac status. B.Teach the patient about hand washing. C.Obtain a serum specimen for electrolytes. D. Increase direct observation of the patient.

Monitor the patient's cardiac status The nurse's priority is to monitor the patient's cardiac status. With the rapidly progressing glomerulonephritis, renal function begins to fail and fluid, potassium, and hydrogen retention lead to hypervolemia, hyperkalemia, and metabolic acidosis. Excess fluid increases the workload of the heart, and hyperkalemia can lead to life-threatening dysrhythmias. Teaching about hand washing and observation of the patient are important nursing interventions but are not the priority. Electrolyte measurement is a collaborative intervention that will be done as ordered by the health care provider.

The nurse in the cardiac care unit is caring for a patient who has developed acute respiratory failure. Which medication is used to decrease patient pulmonary congestion and agitation? Morphine Albuterol Azithromycin Methylprednisolone

Morphine For a patient with acute respiratory failure related to the heart, morphine is used to decrease pulmonary congestion as well as anxiety, agitation, and pain. Albuterol is used to reduce bronchospasm. Azithromycin is used for pulmonary infections. Methylprednisolone is used to reduce airway inflammation and edema.

The patient is admitted with acute coronary syndrome (ACS). The ECG shows ST-segment depression and T-wave inversion. What should the nurse know that this indicates? Myocardia injury Myocardial ischemia Myocardial infarction A pacemaker is present.

Myocardial ischemia The ST depression and T wave inversion on the ECG of a patient diagnosed with ACS indicate myocardial ischemia from inadequate supply of blood and oxygen to the heart. Myocardial injury is identified with ST-segment elevation. Myocardial infarction is identified with ST-segment elevation and a widened and deep Q wave. A pacemaker's presence is evident on the ECG by a spike leading to depolarization and contraction. *IS - down, up - INs*

A 28-yr-old woman with a fracture of the proximal left tibia in a long leg cast and complains of severe pain and a prickling sensation in the left foot. The toes on the left foot are pale and cool. Which nursing action is a priority? Notify the health care provider immediately. Elevate the left leg above the level of the heart. Administer prescribed morphine sulfate intravenously. Apply ice packs to the left proximal tibia over the cast.

Notify the health care provider immediately. Notify the health care provider immediately of this change in patient's condition, which suggest development of compartment syndrome. Pain unrelieved by drugs and out of proportion to the level of injury is one of the first indications of impending compartment syndrome. Changes in sensation (tingling) also suggest compartment syndrome. Because elevation of the extremity may lower venous pressure and slow arterial perfusion, the extremity should not be elevated above heart level. Similarly, the application of cold compresses may result in vasoconstriction and exacerbate compartment syndrome. Administration of morphine may be warranted, but it is not the first priority.

Which guideline for the assessment of intimate partner violence (IPV) should the emergency nurse follow? Patients should be routinely screened for family and IPV. Patients whom the nurse deems high risk should be assessed for IPV. All female patients and patients under 18 should be assessed for IPV. Patients should be assessed for IPV provided corroborating evidence exists.

Patients should be routinely screened for family and IPV. In the emergency department, the nurse needs to screen for family and IPV. Routine screening for this risk factor is required. Such assessment should not be limited to female, high-risk, or young patients, and evidence need not be present to screen for the problem.

Which interventions should the nurse perform before using an open-suctioning technique for a patient with an endotracheal (ET) tube (select all that apply.)? Select all that apply. Put on clean gloves. Administer a bronchodilator. Perform a cardiopulmonary assessment. Hyperoxygenate the patient for 30 seconds. Perform hand hygiene before performing the procedure. Insert a few drops of normal saline into the ET to break up secretions.

Perform a cardiopulmonary assessment. Hyperoxygenate the patient for 30 seconds. Suctioning is preceded by a thorough assessment and hyperoxygenation for 30 seconds. Sterile, not clean, gloves are necessary, and it is not necessary to administer a bronchodilator. Instillation of normal saline into the ET tube is not an accepted standard practice. (Yeaaaaah, I will argue to the death that hand hygiene is definitely also necessary...)

A patient informs the nurse of experiencing syncope. Which nursing action should the nurse prioritize in the patient's subsequent diagnostic workup? Preparing to assist with a head-up tilt-test Preparing an IV dose of a β-adrenergic blocker Assessing the patient's knowledge of pacemakers Teaching the patient about the role of antiplatelet aggregators

Preparing to assist with a head-up tilt-test In patients without structural heart disease, the head-up tilt-test is a common component of the diagnostic workup after episodes of syncope. IV β-blockers are not indicated, although an IV infusion of low-dose isoproterenol may be started in an attempt to provoke a response if the head-up tilt-test did not have a response. Addressing pacemakers is premature and inappropriate at this stage of diagnosis. Patient teaching surrounding antiplatelet aggregators is not directly relevant to the patient's syncope at this time.

The nurse is caring for a 76-yr-old man who has undergone left total knee arthroplasty to relieve the pain of severe osteoarthritis. What care would be expected postoperatively? Progressive leg exercises to obtain 90-degree flexion Early ambulation with full weight bearing on the left leg Bed rest for 3 days with the left leg immobilized in extension Immobilization of the left knee in 30-degree flexion to prevent dislocation

Progressive leg exercises to obtain 90-degree flexion The patient is encouraged to engage in progressive leg exercises until 90-degree flexion is possible; continuous passive motion also may be used based on surgeon preference. Early ambulation is implemented, sometimes the day of surgery, but orders are likely to indicate weight bearing as tolerated rather than full weight bearing. Immobilization and bed rest are not indicated. The patient's knee is unlikely to dislocate.

Students are having an end of the semester party, which includes drinking alcohol, having snacks, and swimming. A student was found floating in the pool. Which action by first responders is most important? Immobilizing the cervical spine Evacuating water from the lungs Securing the airway and providing ventilation Establishing IV access and infusing warmed fluids

Securing the airway and providing ventilation Aggressive resuscitation efforts (e.g., airway and ventilation management), especially in the prehospital phase, improve survival of drowning victims. Initial evaluation involves assessment of airway, cervical spine, breathing, and circulation. Treatment of submersion injuries focuses on correcting hypoxia and fluid imbalances, supporting basic physiologic functions, and rewarming when hypothermia is present. Most drowning victims do not aspirate any liquid due to laryngospasm.

A 64-yr-old male patient admitted to the critical care unit for gastrointestinal hemorrhage complains of feeling tense and nervous. He appears restless with increased blood pressure and pulse. If the physical assessment shows no other changes, it is most important for the critical care nurse to take which action? Administer prescribed IV dose of lorazepam (Ativan). Stay with the patient and encourage expression of concerns. Ask a family member to remain at the bedside with the patient. Teach the patient how to use guided imagery to reduce anxiety.

Stay with the patient and encourage expression of concerns. Anxiety is a common problem for critically ill patients. The nurse should first stay with the patient and encourage the patient to express concerns and needs. After expression of feelings, the nurse should determine the appropriate intervention if needed (e.g., lorazepam, guided imagery, family presence). In addition, staying with the patient will allow the nurse to continue to assess for physiologic changes associated with recurrent gastrointestinal bleeding.

Cardioversion is attempted for a patient with atrial flutter and a rapid ventricular response. After the delivering 50 joules by synchronized cardioversion, the patient develops ventricular fibrillation. Which action should the nurse take immediately? Administer 250 mL of 0.9% saline solution IV by rapid bolus. Assess the apical pulse, blood pressure, and bilateral neck vein distention. Turn the synchronizer switch to the "off" position and recharge the device. Tell the patient to report any chest pain or discomfort and administer morphine sulfate.

Turn the synchronizer switch to the "off" position and recharge the device. Ventricular fibrillation produces no effective cardiac contractions or cardiac output. If during synchronized cardioversion the patient becomes pulseless or the rhythm deteriorates to ventricular fibrillation, the nurse should turn the synchronizer switch off and initiate defibrillation. Fluids, additional assessment, or treatment of pain alone will not restore an effective heart rhythm.

1. Which actions should the nurse initiate to reduce the risk for ventilator-associated pneumonia (VAP) (select all that apply)? a. Obtain arterial blood gases daily. b. Provide a "sedation holiday" daily. c. Elevate the head of the bed to at least 30°. d. Give prescribed pantoprazole (Protonix). e. Provide oral care with chlorhexidine (0.12%) solution daily.

b, c, d, e

Prevention of AKI is important because of the high mortality rate. Which patients are at increased risk for AKI (select all that apply)? a. An 86-year-old woman scheduled for a cardiac catheterization b. A 48-year-old man with multiple injuries from a motor vehicle accident c. A 32-year-old woman following a C-section delivery for abruptio placentae d. A 64-year-old woman with chronic heart failure admitted with bloody stools e. A 58-year-old man with prostate cancer undergoing preoperative workup for prostatectomy

a, b, c, d, e. High-risk patients include those exposed to nephrotoxic agents and advanced age (a), massive trauma (b), prolonged hypovolemia or hypotension (possibly b and c), obstetric complications (c), cardiac failure (d), preexisting chronic kidney disease, extensive burns, or sepsis. Patients with prostate cancer may have obstruction of the outflow tract, which increases risk of postrenal AKI (e).

Patients with acute respiratory failure will have drug therapy to meet their individual needs. Which drugs will meet the goal of reducing pulmonary congestion (select all that apply)? a. Morphine b. Furosemide (Lasix) c. Nitroglycerin (Tridil) d. Albuterol (Ventolin) e. Ceftriaxone (Rocephin) f. Methylprednisolone (Solu-Medrol)

a, b, c. Morphine and nitroglycerin (e.g., Tridil) will decrease pulmonary congestion caused by heart failure; IV diuretics (e.g., furosemide [Lasix]) are also used. Inhaled albuterol (Ventolin) or metaproterenol (Alupent) will relieve bronchospasms. Ceftriaxone (Rocephin) and azithromycin (Zithromax) are used to treat pulmonary infections. Methylprednisolone (Solu-Medrol), an IV corticosteroid, will reduce airway inflammation. Morphine is also used to decrease anxiety, agitation, and pain.

Which drugs will be used to treat the patient with CKD for mineral and bone disorder (select all that apply)? a. Cinacalcet (Sensipar) b. Sevelamer (Renagel) c. IV glucose and insulin d. Calcium acetate (PhosLo) e. IV 10% calcium gluconate

a, b, d. Cinacalcet (Sensipar), a calcimimetic agent to control secondary hyperparathyroidism; sevelamer (Renagel), a noncalcium phosphate binder; and calcium acetate (PhosLo), a calcium-based phosphate binder are used to treat mineral and bone disorder in CKD. IV glucose and insulin and IV 10% calcium gluconate along with sodium polystyrene sulfonate (Kayexalate) are used to treat the hyperkalemia of CKD.

23. What emergency considerations must be included with facial fractures (select all that apply)? a. Airway patency b. Oral examination c. Cervical spine injury d. Cranial nerve assessment e. Immobilization of the jaw

a, c. Airway patency and cervical spinal cord injury are the emergency considerations with facial fractures. Oral examination and cranial nerve assessment will be done after the patient is stabilized. Immobilization of the jaw is done surgically for a mandibular fracture.

Which changes of aging contribute to the increased risk for respiratory failure in older adults (select all that apply)? a. Alveolar dilation b. Increased delirium c. Changes in vital signs d. Increased infection risk e. Decreased respiratory muscle strength f. Diminished elastic recoil within the airways

a, d, e, f. Changes from aging that increase the older adult's risk for respiratory failure include alveolar dilation, increased risk for infection, decreased respiratory muscle strength, and diminished elastic recoil in the airways. Although delirium can complicate ventilator management, it does not increase the older patient's risk for respiratory failure. The older adult's blood pressure (BP) and heart rate (HR) increase but this does not affect the risk for respiratory failure. The ventilatory capacity is decreased and the larger air spaces decrease the surface area for gas exchange, which increases the risk.

Nursing management of the patient with acute pancreatitis includes (select all that apply) a. checking for signs of hypocalcemia. b. providing a diet low in carbohydrates. c. giving insulin based on a sliding scale. d. observing stools for signs of steatorrhea. e. monitoring for infection, particularly respiratory tract infection. (Lewis 1042)

a, e Rationale: During the acute phase, it is important to monitor vital signs. Hemodynamic stability may be compromised by hypotension, fever, and tachypnea. Intravenous fluids are ordered, and the response to therapy is monitored. Fluid and electrolyte balances are closely monitored. Frequent vomiting, along with gastric suction, may result in decreased levels of chloride, sodium, and potassium. Because hypocalcemia can occur in acute pancreatitis, the nurse should observe for symptoms of tetany, such as jerking, irritability, and muscular twitching. Numbness or tingling around the lips and in the fingers is an early indicator of hypocalcemia. The patient should be assessed for Chvostek's sign or Trousseau's sign. A patient with acute pancreatitis should be observed for fever and other manifestations of infection. Respiratory infections are common because the retroperitoneal fluid raises the diaphragm, which causes the patient to take shallow, guarded abdominal breaths.

Which action will the nurse include in the plan of care for a 72-year-old woman admitted with multiple myeloma? a. Monitor fluid intake and output. b. Administer calcium supplements. c. Assess lymph nodes for enlargement. d. Limit weight bearing and ambulation.

a. A high fluid intake and urine output helps prevent the complications of kidney stones caused by hypercalcemia and renal failure caused by deposition of Bence-Jones protein in the renal tubules. Weight bearing and ambulation are encouraged to help bone retain calcium. Lymph nodes are not enlarged with multiple myeloma. Calcium supplements will further increase the patient's calcium level and are not used

The patient progressed from acute lung injury to acute respiratory distress syndrome (ARDS). He is on the ventilator and receiving propofol (Diprivan) for sedation and fentanyl (Sublimaze) to decrease anxiety, agitation, and pain in order to decrease his work of breathing, O2 consumption, carbon dioxide production, and risk of injury. What intervention is recommended in caring for this patient? a. A sedation holiday b. Monitoring for hypermetabolism c. Keeping his legs still to avoid dislodging the airway d. Repositioning him every 4 hours to decrease agitation

a. A sedation holiday is needed to assess the patient's condition and readiness to extubate. A hypermetabolic state occurs with critical illness. With malnourished patients, enteral or parenteral nutrition is started within 24 hours; with well-nourished patients it is started within 3 days. With these medications, the patient will be assessed for cardiopulmonary depression. Venous thromboembolism prophylaxis will be used but there is no reason to keep the legs still. Repositioning the patient every 2 hours may help to decrease discomfort and agitation

The best patient response to treatment of ARDS occurs when initial management includes what? a. Treatment of the underlying condition b. Administration of prophylactic antibiotics c. Treatment with diuretics and mild fluid restriction d. Endotracheal intubation and mechanical ventilation

a. Because ARDS is precipitated by a physiologic insult, a critical factor in its prevention and early management is treatment of the underlying condition. Prophylactic antibiotics, treatment with diuretics and fluid restriction, and mechanical ventilation are also used as ARDS progresses.

Which assessment finding should cause the nurse to suspect the early onset of hypoxemia? a. Restlessness b. Hypotension c. Central cyanosis d. Cardiac dysrhythmias

a. Because the brain is very sensitive to a decrease in oxygen delivery, restlessness, agitation, disorientation, and confusion are early signs of hypoxemia, for which the nurse should be alert. Mild hypertension is also an early sign, accompanied by tachycardia. Central cyanosis is an unreliable, late sign of hypoxemia. Cardiac dysrhythmias also occur later early RAT is late to BED

Priority Decision: A patient on a medical unit has a potassium level of 6.8 mEq/L. What is the priority action that the nurse should take? a. Place the patient on a cardiac monitor. b. Check the patient's blood pressure (BP). c. Instruct the patient to avoid high-potassium foods. d. Call the lab and request a redraw of the lab to verify results.

a. Dysrhythmias may occur with an elevated potassium level and are potentially lethal. Monitor the rhythm while contacting the physician or calling the rapid response team. Vital signs should be checked. Depending on the patient's history and cause of increased potassium, instruct the patient about dietary sources of potassium; however, this would not help at this point. The nurse may want to recheck the value but until then the heart rhythm needs to be monitored.

The nurse is preparing to care for a burn client scheduled for an escharotomy procedure being performed for a third degree circumferential arm burn. The nurse understands that the anticipated therapeutic outcome of the escharotomy is: a. return of distal pulses b. brisk bleeding from the site c. decreasing edema formation d. formation of granulation tissue

a. Escharotomies arepreformed to relieve the compartment syndrome that can occur when edema forms under nondistensible eschar in a circumferential third degree burn.

Priority Decision: During the immediate postoperative care of a recipient of a kidney transplant, what should the nurse expect to do? a. Regulate fluid intake hourly based on urine output. b. Monitor urine-tinged drainage on abdominal dressing. c. Medicate the patient frequently for incisional flank pain. d. Remove the urinary catheter to evaluate the ureteral implant.

a. Fluid and electrolyte balance is critical in the transplant recipient patient, especially because diuresis often begins soon after surgery. Fluid replacement is adjusted hourly based on kidney function and urine output. Urine-tinged drainage on the abdominal dressing may indicate leakage from the ureter implanted into the bladder and the health care provider should be notified. The donor patient may have a flank or laparoscopic incision(s) where the kidney was removed. The recipient has an abdominal incision where the kidney was placed in the iliac fossa. The urinary catheter is usually used for 2 to 3 days to monitor urine output and kidney function.

Identify two ways the following three-volume components of intracranial pressure (ICP) can be changed to adapt to small increases in intracranial pressure. a. Cerebrospinal fluid (CSF) b. Brain Tissue c. Blood tissue

a. Increased absorption, decreased production, displacement into spinal canal b. herniation, lesion, edema, collapse of veins and dural sinuses, increased venous outflow and decreased blood flow c. distention of dura, slight compression of tissue

A patient received a kidney transplant last month. Because of the effects of immunosuppressive drugs and CKD, what complication of transplantation should the nurse be assessing the patient for to decrease the risk of mortality? a. Infection b. Rejection c. Malignancy d. Cardiovascular disease

a. Infection is a significant cause of morbidity and mortality after transplantation because the surgery, the immunosuppressive drugs, and the effects of CKD all suppress the body's normal defense mechanisms, thus increasing the risk of infection. The nurse must assess the patient as well as use aseptic technique to prevent infections. Rejection may occur but for other reasons. Malignancy occurrence increases later due to immunosuppressive therapy. Cardiovascular disease is the leading cause of death after renal transplantation but this would not be expected to cause death within the first month after transplantation.

Priority Decision: After endotracheal intubation and mechanical ventilation have been started, a patient in respiratory failure becomes very agitated and is breathing asynchronously with the ventilator. What is it most important for the nurse to do first? a. Evaluate the patient's pain level, ABGs, and electrolyte values b. Sedate the patient to unconsciousness to eliminate patient awareness c. Administer the PRN vecuronium (Norcuron) to promote synchronous ventilations d. Slow the rate of ventilations provided by the ventilator to allow for spontaneous breathing by the patient

a. It is most important to assess the patient for the cause of the restlessness and agitation (e.g., pain, hypoxemia, electrolyte imbalances) and treat the underlying cause before sedating the patient. Although sedation, analgesia, and neuromuscular blockade are often used to control agitation and pain, these treatments may contribute to prolonged ventilator support and hospital days. *nursing process - start by assessing*

A patient with newly discovered high BP has an average reading of 158/98 mmHg after 3 months of exercise and diet modifications. Which management strategy will be a priority for this patient? a. Medication will be required because the BP is still not at goal b. BP monitoring should continue for another 3 months to confirm a diagnosis of hypertension c. Lifestyle changes are less important, since they were not effective, and medications will be started d. More vigorous changes in the patient's lifestyle are needed for a longer time before starting medications

a. Medication will be required because the BP is still not at goal

When mechanical ventilation is used for the patient with ARDS, what is the rationale for applying positive end-expiratory pressure (PEEP)? a. Prevent alveolar collapse and open up collapsed alveoli b. Permit smaller tidal volumes with permissive hypercapnia c. Promote complete emptying of the lungs during exhalation d. Permit extracorporeal oxygenation and carbon dioxide removal outside the body

a. Positive end-expiratory pressure (PEEP) used with mechanical ventilation applies positive pressure to the airway and lungs at the end of exhalation, keeping the lung partially expanded and preventing collapse of the alveoli and helping to open up collapsed alveoli. Permissive hypercapnia is allowed when the patient with ARDS is ventilated with smaller tidal volumes to prevent barotrauma. Extracorporeal membrane oxygenation and extracorporeal CO2 removal involve passing blood across a gas-exchanging membrane outside the body and then returning oxygenated blood to the body.

In caring for the patient with ARDS, what is the most characteristic sign the nurse would expect the patient to exhibit? a. Refractory hypoxemia b. Bronchial breath sounds c. Progressive hypercapnia d. Increased pulmonary artery wedge pressure (PAWP)

a. Refractory hypoxemia - hypoxemia that does not respond to increasing concentrations of oxygenation by any route - is a hallmark of ARDS and is *always* present. Bronchial breath sounds *may* be associated with the progression of ARDS. PaCO2 levels *may be normal* until the patient is no longer able to compensate in response to the hypoxemia. Pulmonary artery wedge pressure (PAWP) that is normally elevated in cardiogenic pulmonary edema is *normal* in the pulmonary edema of ARDS.

A 36-year-old male patient in the outpatient clinic is diagnosed with acute hepatitis C (HCV) infection. Which action by the nurse is appropriate? a. Schedule the patient for HCV genotype testing. b. Administer the HCV vaccine and immune globulin. c. Teach the patient about ribavirin (Rebetol) treatment. d. Explain that the infection will resolve over a few months.

a. Schedule the patient for HCV genotype testing. Genotyping of HCV has an important role in managing treatment and is done before drug therapy is initiated. Because most patients with acute HCV infection convert to the chronic state, the nurse should not teach the patient that the HCV will resolve in a few months. Immune globulin or vaccine is not available for HCV. Ribavirin is used for chronic HCV infection.

Which finding indicates to the nurse that lactulose (Cephulac) is effective for a 72-year-old man who has advanced cirrhosis? a. The patient is alert and oriented. b. The patient denies nausea or anorexia. c. The patient's bilirubin level decreases. d. The patient has at least one stool daily.

a. The patient is alert and oriented. The purpose of lactulose in the patient with cirrhosis is to lower ammonia levels and prevent encephalopathy. Although lactulose may be used to treat constipation, that is not the purpose for this patient. Lactulose will not decrease nausea and vomiting or lower bilirubin levels.

A man with end-stage kidney disease is scheduled for hemodialysis following healing of an arteriovenous fistula (AVF). What should the nurse explain to him that will occur during dialysis? a. He will be able to visit, read, sleep, or watch TV while reclining in a chair. b. He will be placed on a cardiac monitor to detect any adverse effects that might occur. c. The dialyzer will remove and hold part of his blood for 20 to 30 minutes to remove the waste products. d. A large catheter with two lumens will be inserted into the fistula to send blood to and return it from the dialyzer.

a. While patients are undergoing hemodialysis, they can perform quiet activities that do not require the limb that has the vascular access. Blood pressure is monitored frequently and the dialyzer monitors dialysis function but cardiac monitoring is not usually indicated. The hemodialysis machine continuously circulates both the blood and the dialysate past the semipermeable membrane in the machine. Graft and fistula access involve the insertion of two needles into the site: one to remove blood from and the other to return blood to the dialyzer.

The nurse reviews the electronic medical record for a patient scheduled for a total hip replacement. Which assessment data shown in the accompanying figure increase the patient's risk for respiratory complications after surgery? a. Albumin level and recent weight loss b. Mild confusion and recent weight loss c. Age and recent arthroscopic procedure. d. Anemia and recent arthroscopic procedure

a. albumin level and recent weight loss

An older man arrives in triage disoriented and dyspneic. His skin is hot and dry. His wife states that he was fine earlier today. The nurse's next priority would be to a. assess his vital signs. b. obtain a brief medical history from his wife. c. start supplemental O2 and have the ED physician see him. d. determine the kind of insurance he has before treating him.

a. assess his vital signs.

When caring for a patient with a left arm arteriovenous fistula, which action will the nurse include in the plan of care to maintain the patency of the fistula? a. Auscultate for a bruit at the fistula site. b. Assess the quality of the left radial pulse. c. Compare blood pressures in the left and right arms. d. Irrigate the fistula site with saline every 8 to 12 hours

a. auscultate for a bruit at the fistula site The presence of a thrill and bruit indicates adequate blood flow through the fistula. Pulse rate and quality are not good indicators of fistula patency. Blood pressures should never be obtained on the arm with a fistula. Irrigation of the fistula might damage the fistula, and typically only dialysis staff would access the fistula.

A pt c pelvic Fx should be monitored for a. changes in UO b. petechiae on the abd c. a palpable lump in the buttock d. sudden increase in BP

a. changes in UO (could indicate damage to urinary structures)

A child was admitted to the ED with a thermal burn to the right arm and leg. Which assessment by the nurse requires immediate action? a. coughing and wheezing b. bright red skin with small blister on the burn sites c. thirst d. singed hair

a. coughing and wheezing may indicate that the child has inhaled smoke or toxic fumes. Maintaining airway patency is the highest nursing priority in this situation.

The MOST common early clinical manifestations of ARDS that the nurse may observe are a. dyspnea and tachypnea b. cyanosis and apprehension c. hypotension and tachycardia d. respiratory distress and frothy sputum

a. dyspnea and tachypnea

A client is admitted to a burn intensive care unit with extensive full thickness burns. What should be the nurse's initial concern? a. fluid status b. risk for infection c. body image d. level of pain

a. in early burn care, the client's greatest need has to do with fluid resuscitation because of large volume fluid loss through the damaged skin.

The purpose of adding PEEP to positive pressure ventilation is to a. increase functional residual capacity and improve oxygenation b. increased FIO2 in an attempt to wean the patient and avoid O2 toxicity c. determine if the patient is in synchrony with the ventilator or needs to be paralyzed d. determine if the patient is able to be weaned and avoid the risk of pneumomediastinum

a. increase functional residual capacity and improve oxygenation

A nurse is caring for an obese patient with right lower lobe pneumonia. Which position will be best to improve gas exchange? a. On the left side b. On the right side c. In the tripod position d. In the high-Fowler's position

a. on the left side ("good" lung dependent for more perfusion of working cells)

A 55-year-old patient admitted with an abrupt onset of jaundice and nausea has abnormal liver function studies but serologic testing is negative for viral causes of hepatitis. Which question by the nurse is most appropriate? a. "Is there any history of IV drug use?" b. "Do you use any over-the-counter drugs?" c. "Are you taking corticosteroids for any reason?" d. "Have you recently traveled to a foreign country?"

b. "Do you use any over-the-counter drugs?" The patient's symptoms, lack of antibodies for hepatitis, and the abrupt onset of symptoms suggest toxic hepatitis, which can be caused by commonly used over-the-counter drugs such as acetaminophen (Tylenol). Travel to a foreign country and a history of IV drug use are risk factors for viral hepatitis. Corticosteroid use does not cause the symptoms listed.

The nurse is caring for a 45-year-old male patient during a water deprivation test. Which finding is most important for the nurse to communicate to the health care provider? a. The patient complains of intense thirst. b. The patient has a 5-lb (2.3 kg) weight loss. c. The patient's urine osmolality does not increase. d. The patient feels dizzy when sitting on the edge of the bed.

b. A drop in the weight of more than 2 kg indicates severe dehydration, and the test should be discontinued. The other assessment data are not unusual with this test.

What is the primary way that a nurse will evaluate the patency of an AVF? a. Palpate for pulses distal to the graft site. b. Auscultate for the presence of a bruit at the site. c. Evaluate the color and temperature of the extremity. d. Assess for the presence of numbness and tingling distal to the site.

b. A patent arteriovenous fistula (AVF) creates turbulent blood flow that can be assessed by listening for a bruit or palpated for a thrill as the blood passes through the graft. Assessment of neurovascular status in the extremity distal to the graft site is important to determine that the graft does not impair circulation to the extremity but the neurovascular status does not indicate whether the graft is open.

In a patient with AKI, which laboratory urinalysis result indicates tubular damage? a. Hematuria b. Specific gravity fixed at 1.010 c. Urine sodium of 12 mEq/L (12 mmol/L) d. Osmolality of 1000 mOsm/kg (1000 mmol/kg)

b. A urine specific gravity that is consistently 1.010 and a urine osmolality of about 300 mOsm/kg is the same specific gravity and osmolality as plasma. This indicates that tubules are damaged and unable to concentrate urine. Hematuria is more common with postrenal damage. Tubular damage is associated with a high sodium concentration (greater than 40 mEq/L).

A serum potassium level of 3.2 mEq/L (3.2 mmol/L) is reported for a patient with cirrhosis who has scheduled doses of spironolactone (Aldactone) and furosemide (Lasix). due. Which action should the nurse take? a. Administer both drugs. b. Administer the spironolactone. c. Withhold the spironolactone and administer the furosemide. d. Withhold both drugs until discussed with the health care provider.

b. Administer the spironolactone. Spironolactone is a potassium-sparing diuretic and will help increase the patient's potassium level. The nurse does not need to talk with the doctor before giving the spironolactone, although the health care provider should be notified about the low potassium value. The furosemide will further decrease the patient's potassium level and should be held until the nurse talks with the health care provider.

Which action should the nurse take to evaluate treatment effectiveness for a patient who has hepatic encephalopathy? a. Request that the patient stand on one foot. b. Ask the patient to extend both arms forward. c. Request that the patient walk with eyes closed. d. Ask the patient to perform the Valsalva maneuver.

b. Ask the patient to extend both arms forward. Extending the arms allows the nurse to check for asterixis, a classic sign of hepatic encephalopathy. The other tests might also be done as part of the neurologic assessment but would not be diagnostic for hepatic encephalopathy.

The patient with CKD asks why she is receiving nifedipine (Procardia) and furosemide (Lasix). The nurse understands that these drugs are being used to treat the patient's a. anemia. b. hypertension. c. hyperkalemia. d. mineral and bone disorder.

b. Nifedipine (Procardia) is a calcium channel blocker and furosemide (Lasix) is a loop diuretic. Both are used to treat hypertension.

A patient in hypercapnic respiratory failure has a nursing diagnosis of ineffective airway clearance related to increasing exhaustion. What is an appropriate nursing intervention for this patient? a. Inserting an oral airway b. Performing augmented coughing c. Teaching the patient huff coughing d. Teaching the patient slow pursed lip breathing

b. Augmented coughing is done by applying pressure on the abdominal muscles at the beginning of expiration. This type of coughing helps to increase abdominal pressure and expiratory flow to assist the cough to remove secretions in the patient who is exhausted. An oral airway is used only if there is a possibility that the tongue will obstruct the airway. Huff coughing prevents the glottis from closing during the cough and works well for patients with chronic obstructive pulmonary disease (COPD) to clear central airways. Slow pursed lip breathing allows more time for expiration and prevents small bronchioles from collapsing.

In which type of dialysis does the patient dialyze during sleep and leave the fluid in the abdomen during the day? a. Long nocturnal hemodialysis b. Automated peritoneal dialysis (APD) c. Continuous venovenous hemofiltration (CVVH) d. Continuous ambulatory peritoneal dialysis (CAPD)

b. Automated peritoneal dialysis (APD) is the type of dialysis in which the patient dialyzes during sleep and leaves the fluid in the abdomen during the day. Long nocturnal hemodialysis occurs while the patient is sleeping and is done up to six times per week. Continuous venovenous hemofiltration (CVVH) is a type of continuous renal replacement therapy used to treat AKI. Continuous ambulatory peritoneal dialysis (CAPD) is dialysis that is done with exchanges of 1.5 to 3 L of dialysate at least four times daily.

Which topic is most important to include in patient teaching for a 41-year-old patient diagnosed with early alcoholic cirrhosis? a. Maintaining good nutrition b. Avoiding alcohol ingestion c. Taking lactulose (Cephulac) d. Using vitamin B supplements

b. Avoiding alcohol ingestion The disease progression can be stopped or reversed by alcohol abstinence. The other interventions may be used when cirrhosis becomes more severe to decrease symptoms or complications, but the priority for this patient is to stop the progression of the disease.

A patient with AKI has a serum potassium level of 6.7 mEq/L (6.7 mmol/L) and the following arterial blood gas results: pH 7.28, PaCO2 30 mm Hg, PaO2 86 mm Hg, HCO3 18 mEq/L (18 mmol/L). The nurse recognizes that treatment of the acid-base problem with sodium bicarbonate would cause a decrease in which value? a. pH b. Potassium level c. Bicarbonate level d. Carbon dioxide level

b. During acidosis, potassium moves out of the cell in exchange for H+ ions, increasing the serum potassium level. Correction of the acidosis with sodium bicarbonate will help to shift the potassium back into the cells. A decrease in pH and the bicarbonate and PaCO2 levels would indicate worsening acidosis.

A nurse is caring for a patient with second- and third-degree burns to 50% of the body. The nurse prepares fluid resuscitation based on knowledge of the Parkland (Baxter) formula that includes which of the following recommendations? A. The total 24-hour fluid requirement should be administered in the first 8 hours. B. One half of the total 24-hour fluid requirement should be administered in the first 8 hours. C. One third of the total 24-hour fluid requirement should be administered in the first 4 hours. D. One half of the total 24-hour fluid requirement should be administered in the first 4 hours.

b. Fluid resuscitation with the Parkland (Baxter) formula recommends that one half of the total fluid requirement should be administered in the first 8 hours, one quarter of total fluid requirement should be administered in the second 8 hours, and one quarter of total fluid requirement should be administered in the third 8 hours.

A patient with acute respiratory distress syndrome (ARDS) and acute kidney injury has the following medications ordered. Which medication should the nurse discuss with the health care provider before giving? a. Pantoprazole (Protonix) 40 mg IV b. Gentamicin (Garamycin) 60 mg IV c. Sucralfate (Carafate) 1 g per nasogastric tube d. Methylprednisolone (Solu-Medrol) 60 mg IV

b. Gentamicin 60 mg IV

What is the most serious electrolyte disorder associated with kidney disease? a. Hypocalcemia b. Hyperkalemia c. Hyponatremia d. Hypermagnesemia

b. Hyperkalemia can lead to life-threatening dysrhythmias. Hypocalcemia leads to an accelerated rate of bone remodeling and potentially to tetany. Hyponatremia may lead to confusion. Elevated sodium levels lead to edema, hypertension, and heart failure. Hypermagnesemia may decrease reflexes, mental status, and blood pressure.

Priority Decision: A dehydrated patient is in the Injury stage of the RIFLE staging of AKI. What would the nurse first anticipate in the treatment of this patient? a. Assess daily weight b. IV administration of fluid and furosemide (Lasix) c. IV administration of insulin and sodium bicarbonate d. Urinalysis to check for sediment, osmolality, sodium, and specific gravity

b. Injury is the stage of RIFLE classification when urine output is less than 0.5 mL/kg/hr for 12 hours, the serum creatinine is increased times two or the glomerular filtration rate (GFR) is decreased by 50%. This stage may be reversible by treating the cause or, in this patient, the dehydration by administering IV fluid and a low dose of a loop diuretic, furosemide (Lasix). Assessing the daily weight will be done to monitor fluid changes but it is not the first treatment the nurse should anticipate. IV administration of insulin and sodium bicarbonate would be used for hyperkalemia. Checking the urinalysis will help to determine if the AKI has a prerenal, intrarenal, or postrenal cause by what is seen in the urine but with this patient's dehydration, it is thought to be prerenal to begin treatment.

A 53-year-old patient is being treated for bleeding esophageal varices with balloon tamponade. Which nursing action will be included in the plan of care? a. Instruct the patient to cough every hour. b. Monitor the patient for shortness of breath. c. Verify the position of the balloon every 4 hours. d. Deflate the gastric balloon if the patient reports nausea.

b. Monitor the patient for shortness of breath. The most common complication of balloon tamponade is aspiration pneumonia. In addition, if the gastric balloon ruptures, the esophageal balloon may slip upward and occlude the airway. Coughing increases the pressure on the varices and increases the risk for bleeding. Balloon position is verified after insertion and does not require further verification. The esophageal balloon is deflated every 8 to 12 hours to avoid necrosis, but if the gastric balloon is deflated, the esophageal balloon may occlude the airway.

While obtaining subjective assessment date from a patient with hypertension, the nurse recognizes that a modifiable risk factor for the development of hypertension is: a. a low-calcium diet b. excessive alcohol consumption c. a family history of hypertension d. consumption of a high-protein diet

b. excessive alcohol consumption

To prevent the most common serious complication of PD, what is important for the nurse to do? a. Infuse the dialysate slowly. b. Use strict aseptic technique in the dialysis procedures. c. Have the patient empty the bowel before the inflow phase. d. Reposition the patient frequently and promote deep breathing.

b. Peritonitis is a common complication of peritoneal dialysis (PD) and may require catheter removal and termination of dialysis. Infection occurs from contamination of the dialysate or tubing or from progression of exit-site or tunnel infections and strict sterile technique must be used by health professionals as well as the patient to prevent contamination. Too-rapid infusion may cause shoulder pain and pain may be caused if the catheter tip touches the bowel. Difficulty breathing, atelectasis, and pneumonia may occur from pressure of the fluid on the diaphragm, which may be prevented by elevating the head of the bed and promoting repositioning and deep breathing.

Which action will the nurse include in the plan of care for a patient who has been diagnosed with chronic hepatitis B? a. Advise limiting alcohol intake to 1 drink daily. b. Schedule for liver cancer screening every 6 months. c. Initiate administration of the hepatitis C vaccine series. d. Monitor anti-hepatitis B surface antigen (anti-HBs) levels annually.

b. Schedule for liver cancer screening every 6 months. Patients with chronic hepatitis are at higher risk for development of liver cancer, and should be screened for liver cancer every 6 to 12 months. Patients with chronic hepatitis are advised to completely avoid alcohol. There is no hepatitis C vaccine. Because anti-HBs is present whenever there has been a past hepatitis B infection or vaccination, there is no need to regularly monitor for this antibody.

In replying to a patient's questions about the seriousness of her chronic kidney disease (CKD), the nurse knows that the stage of CKD is based on what? a. Total daily urine output b. Glomerular filtration rate c. Degree of altered mental status d. Serum creatinine and urea levels

b. Stages of chronic kidney disease are based on the GFR. No specific markers of urinary output, mental status, or azotemia classify the degree of chronic kidney disease (CKD).

Which information given by a 70-year-old patient during a health history indicates to the nurse that the patient should be screened for hepatitis C? a. The patient had a blood transfusion in 2005. b. The patient used IV drugs about 20 years ago. c. The patient frequently eats in fast-food restaurants. d. The patient traveled to a country with poor sanitation.

b. The patient used IV drugs about 20 years ago. Any patient with a history of IV drug use should be tested for hepatitis C. Blood transfusions given after 1992 (when an antibody test for hepatitis C became available) do not pose a risk for hepatitis C. Hepatitis C is not spread by the oral-fecal route and therefore is not caused by contaminated food or by traveling in underdeveloped countries.

A 68-year-old man with a history of heart failure resulting from hypertension has AKI as a result of the effects of nephrotoxic diuretics. Currently his serum potassium is 6.2 mEq/L (6.2 mmol/L) with cardiac changes, his BUN is 108 mg/dL (38.6 mmol/L), his serum creatinine is 4.1 mg/dL (362 mmol/L), and his serum HCO3 − is 14 mEq/L (14 mmol/L). He is somnolent and disoriented. Which treatment should the nurse expect to be used for him? a. Loop diuretics b. Renal replacement therapy c. Insulin and sodium bicarbonate d. Sodium polystyrene sulfonate (Kayexalate)

b. This patient has at least three of the six common indications for renal replacement therapy (RRT), including (1) high potassium level, (2) metabolic acidosis, and (3) changed mental status. The other indications are (4) volume overload, resulting in compromised cardiac status (this patient has a history of hypertension), (5) BUN greater than 120 mg/dL, and (6) pericarditis, pericardial effusion, or cardiac tamponade. Although the other treatments may be used, they will not be as effective as RRT for this older patient. Loop diuretics and increased fluid are used if the patient is dehydrated. Insulin and sodium bicarbonate can be used to temporarily drive the potassium into the cells. Sodium polystyrene sulfonate (Kayexalate) is used to actually decrease the amount of potassium in the body.

A patient with ARDS has a nursing diagnosis of risk for infection. To detect the presence of infections commonly associated with ARDS, what should the nurse monitor? a. Gastric aspirate for pH and blood b. Quality, quantity, and consistency of sputum c. Subcutaneous emphysema of the face, neck, and chest d. Mucous membranes of the oral cavity for open lesions

b. Ventilator-associated pneumonia (VAP) is one of the most common complications of ARDS. Early detection requires frequent monitoring of sputum smears and cultures and assessment of the quality, quantity, and consistency of sputum. Prevention of VAP is done with strict infection control measures, ventilator bundle protocol, and subglottal secretion drainage. Blood in gastric aspirate may indicate a stress ulcer and subcutaneous emphysema of the face, neck, and chest occurs with barotrauma during mechanical ventilation. Oral infections may result from prophylactic antibiotics and impaired host defenses but are not common.

To detect possible complications in a patient with severe cirrhosis who has bleeding esophageal varices, it is most important for the nurse to monitor a. bilirubin levels. b. ammonia levels. c. potassium levels. d. prothrombin time.

b. ammonia levels. The protein in the blood in the gastrointestinal (GI) tract will be absorbed and may result in an increase in the ammonia level because the liver cannot metabolize protein very well. The prothrombin time, bilirubin, and potassium levels should also be monitored, but they will not be affected by the bleeding episode.

A patient with respiratory failure has a respiratory rate of 6 breaths/minute and an O2 sat of 88%. The patient is increasingly lethargic. Which intervention will the nurse anticipate? a. administration of 100% oxygen by non-rebreather mask b. endotracheal intubation and positive pressure ventilation c. insertion of mini tracheostomy with frequent suctioning d. initiation of CPAP ventilation

b. endotracheal intubation and positive pressure ventilation

The nurse is planning care for a patient with severe heart failure who has developed elevated blood urea nitrogen (BUN) and creatinine levels. The primary collaborative treatment goal in the plan will be a. augmenting fluid volume. b. maintaining cardiac output. c. diluting nephrotoxic substances. d. preventing systemic hypertension.

b. maintaining cardiac output

The nurse will plan to teach the patient diagnosed with acute hepatitis B about a. side effects of nucleotide analogs. b. measures for improving the appetite. c. ways to increase activity and exercise. d. administering -interferon (Intron A).

b. measures for improving the appetite. Maintaining adequate nutritional intake is important for regeneration of hepatocytes. Interferon and antivirals may be used for chronic hepatitis B, but they are not prescribed for acute hepatitis B infection. Rest is recommended.

A patient admitted with acute respiratory failure has a nursing diagnosis of ineffective airway clearance related to thick secretions. Which action is a priority for the nurse to include in the plan of care? a. Encourage use of the incentive spirometer. b. Offer the patient fluids at frequent intervals. c. Teach the patient the importance of ambulation. d. Titrate oxygen level to keep O2 saturation >93%.

b. offer the patient fluids at frequent intervals

Which statement by the nurse when explaining the purpose of positive end-expiratory pressure (PEEP) to the family members of a patient with ARDS is accurate? a. "PEEP will push more air into the lungs during inhalation." b. "PEEP prevents the lung air sacs from collapsing during exhalation." c. "PEEP will prevent lung damage while the patient is on the ventilator." d. "PEEP allows the breathing machine to deliver 100% oxygen to the lungs."

b. peep prevents the lung air sacs from collapsing during exhalation

A pt c OA is scheduled for a THA. The nurse explains the purpose of this procedure is to (SATA) a. fuse the joint b. replace the joint c. prevent further damage d. improve or maintain ROM e. decrease the amount of destruction in the joint

b. replace the joint d. improve or maintain ROM

A patient with pancreatic cancer is admitted to the hospital for evaluation of possible treatment options. The patient asks the nurse to explain the Whipple procedure that the surgeon has described. The explanation includes the information that a Whipple procedure involves a. creating a bypass around the obstruction caused by the tumor by joining the gallbladder to the jejunum. b. resection of the entire pancreas and the distal portion of the stomach, with anastomosis of the common bile duct and the stomach into the duodenum. c. removal of part of the pancreas, part of the stomach, the duodenum, and the gallbladder, with joining of the pancreatic duct, the common bile duct, and the stomach into the jejunum. d. radical removal of the pancreas, the duodenum, and the spleen, and attachment of the stomach to the jejunum, which requires oral supplementation of pancreatic digestive enzymes and insulin replacement therapy. (Lewis 1042)

c Rationale: The classic operation for pancreatic cancer is a radical pancreaticoduodenectomy, or Whipple procedure. This entails resection of the proximal pancreas (i.e., proximal pancreatectomy), the adjoining duodenum (i.e., duodenectomy), the distal portion of the stomach (i.e., partial gastrectomy), and the distal segment of the common bile duct. The pancreatic duct, common bile duct, and stomach are anastomosed to the jejunum.

The patient is being admitted to the intensive care unit (ICU) with hypercapnic respiratory failure. Which manifestations should the nurse expect to assess in the patient (select all that apply)? a. Cyanosis b. Metabolic acidosis c. Morning headache d. Respiratory acidosis e. Use of tripod position f. Rapid, shallow respirations

c, d, e, f. Morning headache, respiratory acidosis, the use of tripod position, and rapid, shallow respirations would be expected. The other manifestations are characteristic of hypoxemic respiratory failure.

The nurse is monitoring the electrocardiograms of several patients on a cardiac telemetry unit. The patients are directly visible to the nurse, and all of the patients are observed to be sitting up and talking with visitors. Which patient's rhythm would require the nurse to take immediate action? a. A 62-yr-old man with a fever and sinus tachycardia with a rate of 110 beats/min b. A 72-yr-old woman with atrial fibrillation with 60 to 80 QRS complexes per minute c. A 52-yr-old man with premature ventricular contractions (PVCs) at a rate of 12 per minute d. A 42-yr-old woman with first-degree AV block and sinus bradycardia at a rate of 56 beats/min

c. A 52-yr-old man with premature ventricular contractions (PVCs) at a rate of 12 per minute Frequent premature ventricular contractions (PVCs) (>1 every 10 beats) may reduce the cardiac output and precipitate angina and heart failure, depending on their frequency. Because PVCs in CAD or acute myocardial infarction indicate ventricular irritability, the patient's physiologic response to PVCs must be monitored. Frequent PVCs most likely must be treated with oxygen therapy, electrolyte replacement, or antidysrhythmic agents.

A client with burn injury asks the nurse what the term full thickness means. The nurse should respond that burns classified as full thickness involve tissue destruction down to which level? a. epidermis b. dermis c. subcutaneous tissue d. internal organs

c. A full thickness burn involves all skin layers, including the epidermis and dermis, and may extend into the subcutaneous tissue and fat.

A patient on hemodialysis develops a thrombus of a subcutaneous arteriovenous (AV) graft, requiring its removal. While waiting for a replacement graft or fistula, the patient is most likely to have what done for treatment? a. Peritoneal dialysis b. Peripheral vascular access using radial artery c. Silastic catheter tunneled subcutaneously to the jugular vein d. Peripherally inserted central catheter (PICC) line inserted into subclavian vein

c. A more permanent, soft, flexible Silastic double-lumen catheter is used for long-term access when other forms of vascular access have failed. These catheters are tunneled subcutaneously and have Dacron cuffs that prevent infection from tracking along the catheter.

The nurse is caring for a 36-year-old patient with pancreatic cancer. Which nursing action is the highest priority? a. Offer psychologic support for depression. b. Offer high-calorie, high-protein dietary choices. c. Administer prescribed opioids to relieve pain as needed. d. Teach about the need to avoid scratching any pruritic areas.

c. Administer prescribed opioids to relieve pain as needed. Effective pain management will be necessary in order for the patient to improve nutrition, be receptive to teaching, or manage anxiety or depression.

A patient with AKI is a candidate for continuous renal replacement therapy (CRRT). What is the most common indication for use of CRRT? a. Azotemia b. Pericarditis c. Fluid overload d. Hyperkalemia

c. Continuous renal replacement therapy (CRRT) is indicated for the patient with AKI as an alternative or adjunct to hemodialysis to slowly remove solutes and fluid in the hemodynamically unstable patient. It is especially useful for treatment of fluid overload, but hemodialysis is indicated for treatment of hyperkalemia, pericarditis, or other serious effects of uremia.

What does the dialysate for PD routinely contain? a. Calcium in a lower concentration than in the blood b. Sodium in a higher concentration than in the blood c. Dextrose in a higher concentration than in the blood d. Electrolytes in an equal concentration to that of the blood

c. Dextrose or icodextrin or amino acid is added to dialysate fluid to create an osmotic gradient across the membrane to remove excess fluid from the blood. The dialysate fluid has no potassium so that potassium will diffuse into the dialysate from the blood. Dialysate also usually contains higher calcium to promote its movement into the blood. Dialysate sodium is usually less than or equal to that of blood to prevent sodium and fluid retention.

The nurse suspects the early stage of ARDS in any seriously ill patient who manifests what? a. Develops respiratory acidosis b. Has diffuse crackles and rhonchi c. Exhibits dyspnea and restlessness d. Has a decreased PaO2 and an increased PaCO2

c. Early signs of ARDS are insidious and difficult to detect but the nurse should be alert for any early signs of hypoxemia, such as dyspnea, restlessness, tachypnea, cough, and decreased mentation, in patients at risk for ARDS. Abnormal findings on physical examination or diagnostic studies, such as adventitious lung sounds, signs of respiratory distress, respiratory alkalosis, or decreasing PaO2, are usually indications that ARDS has progressed beyond the initial stages.

When caring for a patient with an electrical burn injury, the nurse should question a health care provider's order for A. Mannitol 75 gm IV. B. Urine for myoglobulin. C. Lactated Ringer's at 25 ml/hr. D. Sodium bicarbonate 24 mEq every 4 hours.

c. Electrical injury puts the patient at risk for myoglobinuria, which can lead to acute renal tubular necrosis (ATN). Treatment consists of infusing lactated Ringer's at a rate sufficient to maintain urinary output at 75 to 100 ml/hr. Mannitol can also be used to maintain urine output. Sodium bicarbonate may be given to alkalinize the urine. The urine would also be monitored for the presence of myoglobin. An infusion rate of 25 ml/hr is not sufficient to maintain adequate urine output in prevention and treatment of ATN.

The patient with a history of heart failure and acute respiratory failure has thick secretions that she is having difficulty coughing up. Which intervention would best help to mobilize her secretions? a. Administer more IV fluid b. Perform postural drainage c. Provide O2 by aerosol mask d. Suction airways nasopharyngeally

c. For the patient with a history of heart failure, current acute respiratory failure, and thick secretions, the best intervention is to liquefy the secretions with either aerosol mask or using normal saline administered by a nebulizer. Excess IV fluid may cause cardiovascular distress and the patient probably would not tolerate postural drainage with her history. Suctioning thick secretions without thinning them is difficult and increases the patient's difficulty in maintaining oxygenation. With copious secretions, this could be done after thinning the secretions.

In patients with ARDS who survive the acute phase of lung injury, what manifestations are seen when they progress to the fibrotic phase? a. Chronic pulmonary edema and atelectasis b. Resolution of edema and healing of lung tissue c. Continued hypoxemia because of diffusion limitation d. Increased lung compliance caused by the breakdown of fibrotic tissue

c. In the fibrotic phase of ARDS, diffuse scarring and fibrosis of the lungs occur, resulting in decreased surface area for gas exchange and continued hypoxemia caused by diffusion limitation. Although edema is resolved, lung compliance is decreased because of interstitial fibrosis. Long-term mechanical ventilation is required. The patient has a poor prognosis for survival.

When teaching the patient about what was happening when experiencing an *intrapulmonary* shunt, which explanation is accurate? a. This occurs when an obstruction impairs the flow of blood to the ventilated areas of the lung. b. This occurs when blood passes through an anatomic channel in the heart and bypasses the lungs. c. This occurs when blood flows through the capillaries in the lungs without participating in gas exchange. d. Gas exchange across the alveolar capillary interface is compromised by thickened or damaged alveolar membranes.

c. Intrapulmonary shunt occurs when blood flows through the capillaries in the lungs without participating in gas exchange (e.g., acute respiratory distress syndrome [ARDS], pneumonia). Obstruction impairs the flow of blood to the ventilated areas of the lung in a V/Q mismatch ratio greater than 1 (e.g., pulmonary embolus). Blood passes through an anatomic channel in the heart and bypasses the lungs with anatomic shunt (e.g., ventricular septal defect). Gas exchange across the alveolar capillary interface is compromised by thickened or damaged alveolar membranes in diffusion limitation (e.g., pulmonary fibrosis, ARDS).

The patient with CKD is brought to the emergency department with Kussmaul respirations. What does the nurse know about CKD that could cause this patient's Kussmaul respirations? a. Uremic pleuritis is occurring. b. There is decreased pulmonary macrophage activity. c. They are caused by respiratory compensation for metabolic acidosis. d. Pulmonary edema from heart failure and fluid overload is occurring.

c. Kussmaul respirations occur with severe metabolic acidosis when the respiratory system is attempting to compensate by removing carbon dioxide with exhalations. Uremic pleuritis would cause a pleural friction rub. Decreased pulmonary macrophage activity increases the risk of pulmonary infection. Dyspnea would occur with pulmonary edema.

When explaining respiratory failure to the patient's family, what should the nurse use as an accurate description? a. The absence of ventilation b. Any episode in which part of the airway is obstructed c. Inadequate gas exchange to meet the metabolic needs of the body d. An episode of acute hypoxemia caused by a pulmonary dysfunction

c. Respiratory failure results when the transfer of oxygen or carbon dioxide function of the respiratory system is impaired and, although the definition is determined by PaO2 and PaCO2 levels, the major factor in respiratory failure is inadequate gas exchange to meet tissue oxygen (O2 ) needs. Absence of ventilation is respiratory arrest and partial airway obstruction may not necessarily cause respiratory failure. Acute hypoxemia may be caused by factors other than pulmonary dysfunction

Which finding is most important for the nurse to communicate to the health care provider about a patient who received a liver transplant 1 week ago? a. Dry palpebral and oral mucosa b. Crackles at bilateral lung bases c. Temperature 100.8° F (38.2° C) d. No bowel movement for 4 days

c. Temperature 100.8° F (38.2° C) The risk of infection is high in the first few months after liver transplant and fever is frequently the only sign of infection. The other patient data indicate the need for further assessment or nursing actions and might be communicated to the health care provider, but they do not indicate a need for urgent action.

For a patient with CKD the nurse identifies a nursing diagnosis of risk for injury: fracture related to alterations in calcium and phosphorus metabolism. What is the pathologic process directly related to the increased risk for fractures? a. Loss of aluminum through the impaired kidneys b. Deposition of calcium phosphate in soft tissues of the body c. Impaired vitamin D activation resulting in decreased GI absorption of calcium d. Increased release of parathyroid hormone in response to decreased calcium levels

c. The calcium-phosphorus imbalances that occur in CKD result in hypocalcemia, from a deficiency of active vitamin D and increased phosphorus levels. This leads to an increased rate of bone remodeling with a weakened bone matrix. Aluminum accumulation is also believed to contribute to the osteomalacia. Osteitis fibrosa involves replacement of calcium in the bone with fibrous tissue and is primarily a result of elevated levels of parathyroid hormone resulting from hypocalcemia.

During the nursing assessment of the patient with renal insufficiency, the nurse asks the patient specifically about a history of a. angina. b. asthma. c. hypertension. d. rheumatoid arthritis.

c. The most common causes of CKD in the United States are diabetes mellitus and hypertension. The nurse should obtain information on long-term health problems that are related to kidney disease. The other disorders are not closely associated with renal disease.

When the V/Q lung scan result returns with a mismatch ratio that is greater than 1, which condition should be suspected? a. Pain b. Atelectasis c. Pulmonary embolus d. Ventricular septal defect

c. There will be more ventilation than perfusion (V/Q ratio greater than 1) with a pulmonary embolus. Pain and atelectasis will cause a V/Q ratio less than 1. A ventricular septal defect causes an anatomic shunt as the blood bypasses the lungs.

A child has just been admitted to the pediatric burn unit. Currently, the child is being evaluated for burns to his chest and upper legs. He complains of thirst and asks for a drink. What is the most appropriate nursing action? a. give a small glass of clear liquid b. give a small glass of a full liquid c. keep the child NPO d. order a pediatric meal tray with extra liquids

c. Until a complete assessment and treatment plan are initiated, the child should be kept NPO. A complication of major burns is paralytic ileus, so until that has been ruled out, oral fluids should not be provided.

A female pt in the acute phase of burn care has electrical burns on the left side of her body, type 2 diabetes mellitus, and a serum glucose of 485 mg/dL. What is the nurse's priority for preventing a life threatening complication of hyperglycemia for the burn patient? a. replace the blood lost b. maintain a neutral pH c. Maintain fluid balance d. Replace serum potassium

c. this pt is most likely experiencing hyperglycemic hyperosmolar nonketotic syndrome (HHNKS) which dehydrates a patient rapidly. This increases the pt's risk for hypovolemia and hypotension.

What causes the gastrointestinal (GI) manifestation of stomatitis in the patient with CKD? a. High serum sodium levels b. Irritation of the GI tract from creatinine c. Increased ammonia from bacterial breakdown of urea d. Iron salts, calcium-containing phosphate binders, and limited fluid intake

c. Uremic fetor, or the urine odor of the breath, is caused by high urea content in the blood. Increased ammonia from bacterial breakdown of urea leads to stomatitis and mucosal ulcerations. Irritation of the gastrointestinal (GI) tract from urea in CKD contributes to anorexia, nausea, and vomiting. Ingestion of iron salts and calcium-containing phosphate binders, limited fluid intake, and limited activity cause constipation.

A patient is admitted with first- and second-degree burns covering the face, neck, entire right upper extremity, and the right anterior trunk area. Using the rule of nines, the nurse would calculate the extent of these burns as being A. 9%. B. 18%. C. 22.5%. D. 36%.

c. Using the rule of nines, the face and neck together encompass 4.5% of the body area; the right upper arm encompasses 9% of the body area; and the entire anterior trunk encompasses 18% of the body area. Since the patient has burns on only the right side of the anterior trunk, the nurse would assess that burn as encompassing half of the 18%, or 9%. Therefore adding the three areas together, the nurse would correctly calculate the extent of this patient's burns to cover approximately 22.5% of the total body surface area.

2. The nurse teaches individuals that one of the best ways to prevent musculoskeletal injuries during physical exercise is by doing what? a. Increase muscle strength with daily isometric exercise. b. Avoid exercising on concrete or hard pavement surfaces. c. Perform stretching and warm-up exercises before exercise. d. Wrap susceptible joints with elastic bandages or adhesive tape before exercise.

c. Warm-up exercises "prelengthen" potentially strained tissues by avoiding the quick stretch often encountered in sports and also increase the temperature of muscle, resulting in increased speed of cell metabolism, increased speed of nerve impulses, and improved oxygenation of muscle fibers. Stretching is also thought to improve kinesthetic awareness, lessening the chance of uncoordinated movement. Muscle strength is not a key factor in soft tissue injuries and taping or wrapping joints may actually predispose a person to injury by weakening the joint, unless a previous injury is being treated.

A nurse is caring for a client with a new donor site that was harvested to treat a burn. The nurse should position the client to: a. allow ventilation of the site b. make the site dependent c. avoid pressure on the site d. keep the site fully covered

c. a universal concern in the care of donor sites for burn care is to keep the site away from sources of pressure.

The adult client was burned as a result of an explosion. The burn initially affected the client's entire face (anterior half of the head) and the upper half of the anterior torso, and there were circumferential burns to the lower half of both arms. The client's clothes caught on fire, and the client ran, causing subsequent burn injuries to the posterior surface of the head and the upper half of the posterior torso. Using the rule of nines, what would be the extent of the burn injury? a. 18% b. 24% c. 36% d. 48%

c. anterior head = 4.5%, upper half of anterior torso = 9%, lower half of both arms is 9%, posterior head 4.5%, upper half of posterior torso 9%, total 36%

To prepare a 56-year-old male patient with ascites for paracentesis, the nurse a. places the patient on NPO status. b. assists the patient to lie flat in bed. c. asks the patient to empty the bladder. d. positions the patient on the right side.

c. asks the patient to empty the bladder. The patient should empty the bladder to decrease the risk of bladder perforation during the procedure. The patient would be positioned in Fowler's position and would not be able to lie flat without compromising breathing. Because no sedation is required for paracentesis, the patient does not need to be NPO.

The condition of a client with extensive third degree burns begins to deteriorate. The nurse is aware that which type of shock may occur as a result of inadequate circulating blood volume that occurs with a burn injury? a. cardiogenic b. distributive c. hypovolemic d. septic

c. burns and the resulting low circulating fluid volume can cause hypovolemic shock.

An older woman arrives in the ED complaining of severe pain in her right shoulder. The nurse notes that her clothes are soiled with urine and feces. She tells the nurse that she lives with her son and that she "fell." She is tearful and asks you if she can be admitted. What possibility should the nurse consider? a. dementia b. possible cancer c. family violence d. orthostatic hypotension

c. family violence

A client is undergoing fluid replacement after being burned on 20% of her body 12 hours ago. The nursing assessment reveals a blood pressure of 90/50, a pulse of 110, and urine output of 20 mL over the past hour. The nurse reports the findings to the physician and anticipates which of the following prescriptions? a. transfusing 1 unit of packed red blood cells b. administering a diuretic to increase urine output c. increasing the amount of IV lactated Ringers solution administered per hour d. changing the IV lactated Ringer's solution to one that contains dextrose in water.

c. fluid management during the first 24 hours following a burn injury generally includes the infusion of LR solution. Fluid resuscitation is determined by urine output and hourly urine output should be at least 30mL/hr. The client's urine output is indicative of insufficient fluid resuscitation, which places the client at risk for inadequate perfusion of the brain, heart, kidneys, and other body organs. Therefore, should expect ↑ of LR's.

Maintenance of fluid balance in the pt c ARDS involves a. hydration using colloids b. administration of surfactant c. fluid restriction and diuretics PRN d. keeping the Hgb at/above 9

c. fluid restriction and diuretics PRN

A 37-year-old female patient is hospitalized with acute kidney injury (AKI). Which information will be most useful to the nurse in evaluating improvement in kidney function? a. Urine volume b. Creatinine level c. Glomerular filtration rate (GFR) d. Blood urea nitrogen (BUN) level

c. glomerular filtration rate

A patient develops increasing dyspnea and hypoxemia 2 days after heart surgery. To determine whether the patient has acute respiratory distress syndrome (ARDS) or pulmonary edema caused by heart failure, the nurse will plan to assist with a. obtaining a ventilation-perfusion scan. b. drawing blood for arterial blood gases. c. insertion of a pulmonary artery catheter. d. positioning the patient for a chest x-ray.

c. insertion of a pulmonary artery catheter

During change-of-shift report on a medical unit, the nurse learns that a patient with aspiration pneumonia who was admitted with respiratory distress has become increasingly agitated. Which action should the nurse take first? a. Give the prescribed PRN sedative drug. b. Offer reassurance and reorient the patient. c. Use pulse oximetry to check the oxygen saturation. d. Notify the health care provider about the patient's status.

c. use pulse oximetry to check the O2 Sat

A 56-year-old woman with type 2 diabetes mellitus and chronic kidney disease has a serum potassium level of 6.8 mEq/L. The nurse should assess the patient for A.fatigue. B. flank tenderness. C.cardiac dysrhythmias. D.elevated triglycerides.

cardiac dysrhythmias. Hyperkalemia is the most serious electrolyte disorder associated with kidney disease. Fatal dysrhythmias can occur when the serum potassium level reaches 7 to 8 mEq/L. Fatigue and hypertriglyceridemia may be present but do not require urgent intervention. Tenderness or pain over the kidneys is not expected in CKD.

During the emergent phase of burn injury, the nurse assesses for the presence of hypovolemia. In burn patients, hypovolemia occurs primarily as a result of a. Blood loss from injured tissue. b. Third spacing of fluid into fluid-filled vesicles. c. Evaporation of fluid from denuded body surfaces. d. Capillary permeability with fluid shift to the interstitium.

d Rationale: Hypovolemic shock is caused by a massive shift of fluids out of the blood vessels as a result of increased capillary permeability. Water, sodium, and plasma proteins move into interstitial spaces and other surrounding tissue.

What are intrarenal causes of acute kidney injury (AKI) (select all that apply)? a. Anaphylaxis b. Renal stones c. Bladder cancer d. Nephrotoxic drugs e. Acute glomerulonephritis f. Tubular obstruction by myoglobin

d, e, f. Intrarenal causes of acute kidney injury (AKI) include conditions that cause direct damage to the kidney tissue, including nephrotoxic drugs, acute glomerulonephritis, and tubular obstruction by myoglobin, or prolonged ischemia. Anaphylaxis and other prerenal problems are frequently the initial cause of AKI. Renal stones and bladder cancer are among the postrenal causes of AKI.

A patient with acute respiratory distress syndrome (ARDS) who is intubated and receiving mechanical ventilation develops a right pneumothorax. Which action will the nurse anticipate taking next? a. Increase the tidal volume and respiratory rate. b. Increase the fraction of inspired oxygen (FIO2). c. Perform endotracheal suctioning more frequently. d. Lower the positive end-expiratory pressure (PEEP).

d. lower the PEEP

Acute tubular necrosis (ATN) is the most common cause of intrarenal AKI. Which patient is most likely to develop ATN? a. Patient with diabetes mellitus b. Patient with hypertensive crisis c. Patient who tried to overdose on acetaminophen d. Patient with major surgery who required a blood transfusion

d. Acute tubular necrosis (ATN) is primarily the result of ischemia, nephrotoxins, or sepsis. Major surgery is most likely to cause severe kidney ischemia in the patient requiring a blood transfusion. A blood transfusion hemolytic reaction produces nephrotoxic injury if it occurs. Diabetes mellitus, hypertension, and acetaminophen overdose will not contribute to ATN.

Which data will the nurse monitor in relation to the 4+ pitting edema assessed in a patient with cirrhosis? a. Hemoglobin b. Temperature c. Activity level d. Albumin level

d. Albumin level The low oncotic pressure caused by hypoalbuminemia is a major pathophysiologic factor in the development of edema. The other parameters should also be monitored, but they are not directly associated with the patient's current symptoms.

A 72-year-old man tells the nurse that he cannot perform most of the physical activities he could do 5 years ago because of overall joint aches and pains. What can the nurse do to assist the patient to prevent further deconditioning and decrease the risk for developing musculoskeletal problems? a. Limit weight-bearing exercise to prevent stress on fragile bones and possible hip fractures. b. Advise the patient to avoid the use of canes and walkers because they increase dependence on ambulation aids. c. Advise the patient to increase his activity by more frequently climbing stairs in buildings and other environments with steps. d. Discuss use of over-the-counter (OTC) medications to decrease inflammation and pain so that exercise can be maintained.

d. Almost all older adults have some degree of decreased muscle strength, joint stiffness, and pain with motion. The use of mild antiinflammatory agents decreases inflammation and pain and can help the patient to maintain activity and prevent further deconditioning but other prescribed drugs and potential abdominal problems must be considered by the patient. Musculoskeletal problems in the older adult can be prevented with appropriate strategies, especially exercise. Stair walking can create enough stress on fragile bones to cause a hip fracture and use of ramps may help to prevent falls. Walkers and canes should be used as necessary to decrease stress on joints so that activity can be maintained.

Which serum laboratory value indicates to the nurse that the patient's CKD is getting worse? a. Decreased BUN b. Decreased sodium c. Decreased creatinine d. Decreased calculated glomerular filtration rate (GFR)

d. As GFR decreases, BUN and serum creatinine levels increase. Although elevated BUN and creatinine indicate that waste products are accumulating, the calculated GFR is considered a more accurate indicator of kidney function than BUN or serum creatinine.

A patient rapidly progressing toward end-stage kidney disease asks about the possibility of a kidney transplant. In responding to the patient, the nurse knows that what is a contraindication to kidney transplantation? a. Hepatitis C infection b. Coronary artery disease c. Refractory hypertension d. Extensive vascular disease

d. Extensive vascular disease is a contraindication for renal transplantation, primarily because adequate blood supply is essential for the health of the new kidney. Other contraindications include disseminated malignancies, refractory or untreated cardiac disease, chronic respiratory failure, chronic infection, or unresolved psychosocial disorders. Coronary artery disease (CAD) may be treated with bypass surgery before transplantation and transplantation can relieve hypertension. Hepatitis B or C infection is not a contraindication.

What is the primary reason that hemodynamic monitoring is instituted in severe respiratory failure? a. To detect V/Q mismatches b. To continuously measure the arterial BP c. To evaluate oxygenation and ventilation status d. To evaluate cardiac status and blood flow to tissues

d. Hemodynamic monitoring with a pulmonary artery catheter is instituted in severe respiratory failure to determine the amount of blood flow to tissues and the response of the lungs and heart to hypoxemia. Continuous BP monitoring may be performed but BP is a reflection of cardiac activity, which can be determined by the pulmonary artery catheter findings. Arterial blood gases (ABGs) are important to evaluate oxygenation and ventilation status and V/Q mismatches.

While caring for the patient in the oliguric phase of AKI, the nurse monitors the patient for associated collaborative problems. When should the nurse notify the health care provider? a. Urine output is 300 mL/day. b. Edema occurs in the feet, legs, and sacral area. c. Cardiac monitor reveals a depressed T wave and elevated ST segment. d. The patient experiences increasing muscle weakness and abdominal cramping.

d. Hyperkalemia is a potentially life-threatening complication of AKI in the oliguric phase. Muscle weakness and abdominal cramping are signs of the neuromuscular impairment that occurs with hyperkalemia. In addition, hyperkalemia can cause the cardiac conduction abnormalities of peaked T wave, prolonged PR interval, prolonged QRS interval, and depressed ST segment. Urine output of 300 mL/day is expected during the oliguric phase, as is the development of peripheral edema.

A patient has been treated for second- and third-degree burns over 30% of his body and is now ready for discharge. You provide discharge instructions related to wound care. Which statement indicates that the patient understands the instructions? a. "I can expect occasional periods of low grade fever and can take Tylenol every 4 hours" b. "I must wear my jobst elastic garment all day and can only remove it when I'm going to bed." c. "I will need to take sponge baths at home to avoid exposing the wounds to unsterile bath water." d. "If any healed areas break open, I should cover them with a sterile dressing and then immediately report it."

d. If any healed areas break open, I should cover them with a sterile dressing and then immediately report it."

Which arterial blood gas (ABG) results would most likely indicate acute respiratory failure in a patient with chronic lung disease? a. PaO2 52 mm Hg, PaCO2 56 mm Hg, pH 7.4 b. PaO2 46 mm Hg, PaCO2 52 mm Hg, pH 7.36 c. PaO2 48 mm Hg, PaCO2 54 mm Hg, pH 7.38 d. PaO2 50 mm Hg, PaCO2 54 mm Hg, pH 7.28

d. In a patient with normal lung function, respiratory failure is commonly defined as a PaO2 ≤60 mm Hg or a PaCO2>45 mm Hg or both. However, because the patient with chronic pulmonary disease normally maintains low PaO2 and high PaCO2, acute respiratory failure in these patients can be defined as an acute decrease in PaO2 or an increase in PaCO2 from the patient's baseline parameters, accompanied by an acidic pH. The pH of 7.28 reflects an acidemia and a loss of compensation in the patient with chronic lung disease. *look for ones that are acidotic, to start with*

What accurately describes the care of the patient with CKD? a. A nutrient that is commonly supplemented for the patient on dialysis because it is dialyzable is iron. b. The syndrome that includes all of the signs and symptoms seen in the various body systems in CKD is azotemia. c. The use of morphine is contraindicated in the patient with CKD because accumulation of its metabolites may cause seizures. d. The use of calcium-based phosphate binders in the patient with CKD is contraindicated when serum calcium levels are increased.

d. In the patient with CKD, when serum calcium levels are increased, calcium-based phosphate binders are not used. The nutrient supplemented for patients on dialysis is *folic acid.* The various body system manifestations occur with *uremia*, which includes azotemia. *Meperidine* is contraindicated in patients with CKD related to possible seizures (honestly it's pretty out of favor generally d/t the problem c metabolites).

In caring for a patient in acute respiratory failure, the nurse recognizes that noninvasive positive pressure ventilation (NIPPV) may be indicated for which patient? a. Is comatose and has high oxygen requirements b. Has copious secretions that require frequent suctioning c. Responds to hourly bronchodilator nebulization treatments d. Is alert and cooperative but has increasing respiratory exhaustion

d. Noninvasive positive pressure ventilation (NIPPV) involves the application of a face mask and delivery of a volume of air under inspiratory pressure. Because the device is worn externally, the patient must be able to cooperate in its use and frequent access to the airway for suctioning or inhaled medications must not be necessary. It is not indicated when high levels of oxygen are needed or respirations are absent.

The nurse assesses that a patient in respiratory distress is developing respiratory fatigue and the risk of respiratory arrest when the patient displays which behavior? a. Cannot breathe unless he is sitting upright b. Uses the abdominal muscles during expiration c. Has an increased inspiratory-expiratory (I/E) ratio d. Has a change in respiratory rate from rapid to slow

d. The increase in respiratory rate required to blow off accumulated CO2 predisposes to respiratory muscle fatigue. The slowing of a rapid rate in a patient in acute distress indicates tiring and the possibility of respiratory arrest unless ventilatory assistance is provided. A decreased inspiratory-expiratory (I/E) ratio, orthopnea, and accessory muscle use are common findings in respiratory distress but do not necessarily signal respiratory fatigue or arrest.

A victim of an industrial accident has chemical spilled on his face and body. The chemical, which has a pH of 7.51, is flushed with water by paramedics. What is the most important information for the receiving nurse to obtain about the pt from the paramedics? a. containment of chemical b. duration of water flushing c. other injuries of the victim d. specific location of accident

d. The nurse must know where the accident occurred to determine if the pt was rescued from an enclosed space. If so, the pt is at high risk for an inhalation injury because the enclosure concentrates the noxious fumes making an inhalation injury more likely.

Prone positioning is considered for a patient with ARDS who has not responded to other measures to increase PaO2. The nurse knows that this strategy will a. increase the mobilization of pulmonary secretions. b. decrease the workload of the diaphragm and intercostal muscles. c. promote opening of atelectatic alveoli in the upper portion of the lung. d. promote perfusion of nonatelectatic alveoli in the anterior portion of the lung.

d. When a patient with ARDS is supine, alveoli in the posterior areas of the lung are dependent and fluid-filled and the heart and mediastinal contents place more pressure on the lungs, predisposing to atelectasis. If the patient is turned prone, air-filled nonatelectatic alveoli in the anterior portion of the lung receive more blood and perfusion may be better matched to ventilation, causing less V/Q mismatch. Lateral rotation therapy is used to stimulate postural drainage and help mobilize pulmonary secretions.

The nurse is caring for a 78-year-old patient who was hospitalized 2 days earlier with community-acquired pneumonia. Which assessment information is most important to communicate to the health care provider? a. Scattered crackles bilaterally in the posterior lung bases. b. Persistent cough that is productive of blood-tinged sputum. c. Temperature of 101.5° F (38.6° C) after 2 days of IV antibiotic therapy. d. Decreased oxygen saturation to 90% with 100% O2 by non-rebreather mask.

d. decreased O2 Sat to 90% with 100% O2 by non-rebreather mask

Which nursing interventions included in the care of a mechanically ventilated patient with acute respiratory failure can the registered nurse (RN) delegate to an experienced licensed practical/vocational nurse (LPN/LVN) working in the intensive care unit? a. Assess breath sounds every hour. b. Monitor central venous pressures. c. Place patient in the prone position. d. Insert an indwelling urinary catheter.

d. insert an indwelling urinary catheter

In teaching a patient with hypertension about controlling the condition, the nurse recognizes that: a. all patients with elevated BP require medication b. obese persons must achieve a normal weight to lower BP c. It is not necessary to limit salt in the diet if taking a diuretic d. lifestyle modifications are indicated for all persons with elevated BP

d. lifestyle modifications are indicated for all persons with elevated BP

The O2 delivery system chosen for the pt in acute respiratory failure should a. always be a low-flow device, such as NC or face mask b. administer CPAP centilation to prevent CO2 narcosis c. correct the PaO2 to a normal level as quickly as possible using mechanical ventilation d. maintain the PaO2 at greater than or equal to 60 mmHg at the lowest O2 concentration possible

d. maintain the PaO2 at greater than or equal to 60 mmHg at the lowest O2 concentration possible

A pt c stable, closed humeral Fx has a temporary splint c bulky padding applied c elastic bandage. The nurse notifies the surgeon of possible early compartment syndrome with the pt experiences a. increasing edema of the limb b. muscle spasms of the lower arm c. bounding pulse at the Fx site d. pain when passively extending fingers

d. pain when passively extending fingers

A 51-year-old woman had an incisional cholecystectomy 6 hours ago. The nurse will place the highest priority on assisting the patient to a. choose low-fat foods from the menu. b. perform leg exercises hourly while awake. c. ambulate the evening of the operative day. d. turn, cough, and deep breathe every 2 hours.

d. turn, cough, and deep breathe every 2 hours. Postoperative nursing care after a cholecystectomy focuses on prevention of respiratory complications because the surgical incision is high in the abdomen and impairs coughing and deep breathing. The other nursing actions are also important to implement but are not as high a priority as ensuring adequate ventilation.

The nurse suspects an ankle sprain when a patient at an urgent care center describes a. being hit by another soccer player during a game b. having ankle pain after sprinting around the track c. dropping a 10-lb weight on his lower leg at the gym d. twisting his ankle while running bases during a baseball game

d. twisting his ankle while running bases during a baseball game

A major consideration in the management of the other adult with hypertension is to: a. prevent primary hypertension from converting to secondary hypertension b. recognize that the older adult is less likely to adhere to the drug therapy regimen than a younger adult c. ensure that the patient receives larger initial doses of antihypertensive drugs because of impaired absorption d. use careful technique in assessing the BP of the patient because of the possible presence of an auscultatory gap

d. use careful technique in assessing the BP of the patient because of the possible presence of an auscultatory gap *a is not how it works, b old people often more conscientious about meds in my anecdotal opinion, c excretion is impaired, which means smaller doses often needed*


Ensembles d'études connexes

ESYS 57C Chapter 2 Electrical Drawings

View Set

ECO315 Money & Banking Final Exam

View Set

Span 2 Vocab list Cap 5 A Part 1

View Set

Unit 7 Disclosures in Real Estate: Seller's Statutory Disclosures

View Set

IB Geography Option D- Hazards and disasters - risk assessment and response

View Set